Art Iv

You might also like

Download as docx, pdf, or txt
Download as docx, pdf, or txt
You are on page 1of 261

ART IV – CITIZENSHIP

G.R. No. 221697

MARY GRACE NATIVIDAD S. POE-LLAMANZARES, Petitioners,


vs.
COMELEC AND ESTRELLA C. ELAMPARO Respondents.

x-----------------------x

G.R. No. 221698-700

MARY GRACE NATIVIDAD S. POE-LLAMANZARES, Petitioners,


vs.
COMELEC, FRANCISCO S. TATAD, ANTONIO P. CONTRERAS AND AMADO D.
VALDEZ Respondents.

DECISION

PEREZ, J.:

Before the Court are two consolidated petitions under Rule 64 in relation to Rule 65 of the
Rules of Court with extremely urgent application for an ex parte issuance of temporary
restraining order/status quo ante order and/or writ of preliminary injunction assailing the
following: (1) 1 December 2015 Resolution of the Commission on Elections (COMELEC)
Second Division; (2) 23 December 2015 Resolution of the COMELEC En Banc, in SPA No. 15-
001 (DC); (3) 11 December 2015 Resolution of the COMELEC First Division; and ( 4) 23
December 2015 Resolution of the COMELEC En Banc, in SPA No. 15-002 (DC), SPA No. 15-
007 (DC) and SPA No. 15-139 (DC) for having been issued without jurisdiction or with grave
abuse of discretion amounting to lack or excess of jurisdiction.

The Facts

Mary Grace Natividad S. Poe-Llamanzares (petitioner) was found abandoned as a newborn


infant in the Parish Church of Jaro, Iloilo by a certain Edgardo Militar (Edgardo) on 3
September 1968. Parental care and custody over petitioner was passed on by Edgardo to his
relatives, Emiliano Militar (Emiliano) and his wife. Three days after, 6 September 1968,
Emiliano reported and registered petitioner as a foundling with the Office of the Civil Registrar
of Iloilo City (OCR-Iloilo). In her Foundling Certificate and Certificate of Live Birth, the
petitioner was given the name "Mary Grace Natividad Contreras Militar." 1

When petitioner was five (5) years old, celebrity spouses Ronald Allan Kelley Poe (a.k.a.
Fenando Poe, Jr.) and Jesusa Sonora Poe (a.k.a. Susan Roces) filed a petition for her adoption
with the Municipal Trial Court (MTC) of San Juan City. On 13 May 1974, the trial court granted
their petition and ordered that petitioner's name be changed from "Mary Grace Natividad
Contreras Militar" to "Mary Grace Natividad Sonora Poe." Although necessary notations were
made by OCR-Iloilo on petitioner's foundling certificate reflecting the court decreed
adoption,2 the petitioner's adoptive mother discovered only sometime in the second half of
2005 that the lawyer who handled petitioner's adoption failed to secure from the OCR-Iloilo a
new Certificate of Live Birth indicating petitioner's new name and the name of her adoptive
parents. 3 Without delay, petitioner's mother executed an affidavit attesting to the lawyer's
omission which she submitted to the OCR-Iloilo. On 4 May 2006, OCR-Iloilo issued a new
Certificate of Live Birth in the name of Mary Grace Natividad Sonora Poe.4

Having reached the age of eighteen (18) years in 1986, petitioner registered as a voter with
the local COMELEC Office in San Juan City. On 13 December 1986, she received her COMELEC
Voter's Identification Card for Precinct No. 196 in Greenhills, San Juan, Metro Manila. 5

On 4 April 1988, petitioner applied for and was issued Philippine Passport No. F927287 6 by
the Department of Foreign Affairs (DFA). Subsequently, on 5 April 1993 and 19 May 1998,
she renewed her Philippine passport and respectively secured Philippine Passport Nos.
L881511 and DD156616.7

Initially, the petitioner enrolled and pursued a degree in Development Studies at the
University of the Philippines8 but she opted to continue her studies abroad and left for the
United States of America (U.S.) in 1988. Petitioner graduated in 1991 from Boston College in
Chestnuts Hill, Massachusetts where she earned her Bachelor of Arts degree in Political
Studies.9

On 27 July 1991, petitioner married Teodoro Misael Daniel V. Llamanzares (Llamanzares), a


citizen of both the Philippines and the U.S., at Sanctuario de San Jose Parish in San Juan
City. 10 Desirous of being with her husband who was then based in the U.S., the couple flew
back to the U.S. two days after the wedding ceremony or on 29 July 1991. 11

While in the U.S., the petitioner gave birth to her eldest child Brian Daniel (Brian) on 16 April
1992.12 Her two daughters Hanna MacKenzie (Hanna) and Jesusa Anika (Anika) were both
born in the Philippines on 10 July 1998 and 5 June 2004, respectively. 13

On 18 October 2001, petitioner became a naturalized American citizen. 14


She obtained U.S.
Passport No. 017037793 on 19 December 2001. 15

On 8 April 2004, the petitioner came back to the Philippines together with Hanna to support
her father's candidacy for President in the May 2004 elections. It was during this time that
she gave birth to her youngest daughter Anika. She returned to the U.S. with her two
daughters on 8 July 2004. 16

After a few months, specifically on 13 December 2004, petitioner rushed back to the
Philippines upon learning of her father's deteriorating medical condition. 17 Her father slipped
into a coma and eventually expired. The petitioner stayed in the country until 3 February 2005
to take care of her father's funeral arrangements as well as to assist in the settlement of his
estate.18

According to the petitioner, the untimely demise of her father was a severe blow to her entire
family. In her earnest desire to be with her grieving mother, the petitioner and her husband
decided to move and reside permanently in the Philippines sometime in the first quarter of
2005.19 The couple began preparing for their resettlement including notification of their
children's schools that they will be transferring to Philippine schools for the next
semester;20coordination with property movers for the relocation of their household goods,
furniture and cars from the U.S. to the Philippines;21 and inquiry with Philippine authorities as
to the proper procedure to be followed in bringing their pet dog into the country.22 As early
as 2004, the petitioner already quit her job in the U.S. 23
Finally, petitioner came home to the Philippines on 24 May 2005 24 and without delay, secured
a Tax Identification Number from the Bureau of Internal Revenue. Her three (3) children
immediately followed25 while her husband was forced to stay in the U.S. to complete pending
projects as well as to arrange the sale of their family home there.26

The petitioner and her children briefly stayed at her mother's place until she and her husband
purchased a condominium unit with a parking slot at One Wilson Place Condominium in San
Juan City in the second half of 2005.27 The corresponding Condominium Certificates of Title
covering the unit and parking slot were issued by the Register of Deeds of San Juan City to
petitioner and her husband on 20 February 2006.28 Meanwhile, her children of school age
began attending Philippine private schools.

On 14 February 2006, the petitioner made a quick trip to the U.S. to supervise the disposal
of some of the family's remaining household belongings. 29 She travelled back to the
Philippines on 11 March 2006.30

In late March 2006, petitioner's husband officially informed the U.S. Postal Service of the
family's change and abandonment of their address in the U.S. 31 The family home was
eventually sold on 27 April 2006.32 Petitioner's husband resigned from his job in the U.S. in
April 2006, arrived in the country on 4 May 2006 and started working for a major Philippine
company in July 2006.33

In early 2006, petitioner and her husband acquired a 509-square meter lot in Corinthian Hills,
Quezon City where they built their family home34 and to this day, is where the couple and
their children have been residing.35 A Transfer Certificate of Title covering said property was
issued in the couple's name by the Register of Deeds of Quezon City on 1June 2006.

On 7 July 2006, petitioner took her Oath of Allegiance to the Republic of the Philippines
pursuant to Republic Act (R.A.) No. 9225 or the Citizenship Retention and Re-acquisition Act
of 2003.36 Under the same Act, she filed with the Bureau of Immigration (BI) a sworn petition
to reacquire Philippine citizenship together with petitions for derivative citizenship on behalf
of her three minor children on 10 July 2006.37 As can be gathered from its 18 July 2006 Order,
the BI acted favorably on petitioner's petitions and declared that she is deemed to have
reacquired her Philippine citizenship while her children are considered as citizens of the
Philippines.38 Consequently, the BI issued Identification Certificates (ICs) in petitioner's name
and in the names of her three (3) children. 39

Again, petitioner registered as a voter of Barangay Santa Lucia, San Juan City on 31 August
2006.40 She also secured from the DFA a new Philippine Passport bearing the No.
XX4731999.41 This passport was renewed on 18 March 2014 and she was issued Philippine
Passport No. EC0588861 by the DFA.42

On 6 October 2010, President Benigno S. Aquino III appointed petitioner as Chairperson of


the Movie and Television Review and Classification Board (MTRCB).43 Before assuming her
post, petitioner executed an "Affidavit of Renunciation of Allegiance to the United States of
America and Renunciation of American Citizenship" before a notary public in Pasig City on 20
October 2010,44 in satisfaction of the legal requisites stated in Section 5 of R.A. No.
9225.45 The following day, 21 October 2010 petitioner submitted the said affidavit to the
BI46 and took her oath of office as Chairperson of the MTRCB. 47 From then on, petitioner
stopped using her American passport.48
On 12 July 2011, the petitioner executed before the Vice Consul of the U.S. Embassy in Manila
an "Oath/Affirmation of Renunciation of Nationality of the United States." 49 On that day, she
accomplished a sworn questionnaire before the U.S. Vice Consul wherein she stated that she
had taken her oath as MTRCB Chairperson on 21 October 2010 with the intent, among others,
of relinquishing her American citizenship.50 In the same questionnaire, the petitioner stated
that she had resided outside of the U.S., specifically in the Philippines, from 3 September
1968 to 29 July 1991 and from May 2005 to present.51

On 9 December 2011, the U.S. Vice Consul issued to petitioner a "Certificate of Loss of
Nationality of the United States" effective 21 October 2010. 52

On 2 October 2012, the petitioner filed with the COMELEC her Certificate of Candidacy (COC)
for Senator for the 2013 Elections wherein she answered "6 years and 6 months" to the
question "Period of residence in the Philippines before May 13, 2013." 53 Petitioner obtained
the highest number of votes and was proclaimed Senator on 16 May 2013. 54

On 19 December 2013, petitioner obtained Philippine Diplomatic Passport No. DE0004530. 55

On 15 October 2015, petitioner filed her COC for the Presidency for the May 2016
Elections. 56 In her COC, the petitioner declared that she is a natural-born citizen and that her
residence in the Philippines up to the day before 9 May 2016 would be ten (10) years and
eleven (11) months counted from 24 May 2005.57 The petitioner attached to her COC an
"Affidavit Affirming Renunciation of U.S.A. Citizenship" subscribed and sworn to before a
notary public in Quezon City on 14 October 2015. 58

Petitioner's filing of her COC for President in the upcoming elections triggered the filing of
several COMELEC cases against her which were the subject of these consolidated cases.

Origin of Petition for Certiorari in G.R. No. 221697

A day after petitioner filed her COC for President, Estrella Elamparo (Elamparo) filed a petition
to deny due course or cancel said COC which was docketed as SPA No. 15-001 (DC) and
raffled to the COMELEC Second Division.59She is convinced that the COMELEC has jurisdiction
over her petition.60 Essentially, Elamparo's contention is that petitioner committed material
misrepresentation when she stated in her COC that she is a natural-born Filipino citizen and
that she is a resident of the Philippines for at least ten (10) years and eleven (11) months up
to the day before the 9 May 2016 Elections.61

On the issue of citizenship, Elamparo argued that petitioner cannot be considered as a natural-
born Filipino on account of the fact that she was a foundling. 62 Elamparo claimed that
international law does not confer natural-born status and Filipino citizenship on
foundlings.63 Following this line of reasoning, petitioner is not qualified to apply for
reacquisition of Filipino citizenship under R.A. No. 9225 for she is not a natural-born Filipino
citizen to begin with.64Even assuming arguendo that petitioner was a natural-born Filipino,
she is deemed to have lost that status when she became a naturalized American
citizen.65 According to Elamparo, natural-born citizenship must be continuous from birth.66

On the matter of petitioner's residency, Elamparo pointed out that petitioner was bound by
the sworn declaration she made in her 2012 COC for Senator wherein she indicated that she
had resided in the country for only six ( 6) years and six ( 6) months as of May 2013 Elections.
Elamparo likewise insisted that assuming arguendo that petitioner is qualified to regain her
natural-born status under R.A. No. 9225, she still fell short of the ten-year residency
requirement of the Constitution as her residence could only be counted at the earliest from
July 2006, when she reacquired Philippine citizenship under the said Act. Also on the
assumption that petitioner is qualified to reacquire lost Philippine Citizenship, Elamparo is of
the belief that she failed to reestablish her domicile in the Philippines.67

Petitioner seasonably filed her Answer wherein she countered that:

(1) the COMELEC did not have jurisdiction over Elamparo's petition as it was actually
a petition for quo warranto which could only be filed if Grace Poe wins in the
Presidential elections, and that the Department of Justice (DOJ) has primary
jurisdiction to revoke the BI's July 18, 2006 Order;

(2) the petition failed to state a cause of action because it did not contain allegations
which, if hypothetically admitted, would make false the statement in her COC that she
is a natural-born Filipino citizen nor was there any allegation that there was a willful
or deliberate intent to misrepresent on her part;

(3) she did not make any material misrepresentation in the COC regarding her
citizenship and residency qualifications for:

a. the 1934 Constitutional Convention deliberations show that foundlings were


considered citizens;

b. foundlings are presumed under international law to have been born of


citizens of the place where they are found;

c. she reacquired her natural-born Philippine citizenship under the provisions


of R.A. No. 9225;

d. she executed a sworn renunciation of her American citizenship prior to the


filing of her COC for President in the May 9, 2016 Elections and that the same
is in full force and effect and has not been withdrawn or recanted;

e. the burden was on Elamparo in proving that she did not possess natural-
born status;

f. residence is a matter of evidence and that she reestablished her domicile in


the Philippines as early as May 24, 2005;

g. she could reestablish residence even before she reacquired natural-born


citizenship under R.A. No. 9225;

h. statement regarding the period of residence in her 2012 COC for Senator
was an honest mistake, not binding and should give way to evidence on her
true date of reacquisition of domicile;

i. Elamparo's petition is merely an action to usurp the sovereign right of the


Filipino people to decide a purely political question, that is, should she serve as
the country's next leader.68
After the parties submitted their respective Memoranda, the petition was deemed submitted
for resolution.

On 1 December 2015, the COMELEC Second Division promulgated a Resolution finding that
petitioner's COC, filed for the purpose of running for the President of the Republic of the
Philippines in the 9 May 2016 National and Local Elections, contained material representations
which are false. The fallo of the aforesaid Resolution reads:

WHEREFORE, in view of all the foregoing considerations, the instant Petition to Deny Due
Course to or Cancel Certificate of Candidacy is hereby GRANTED. Accordingly, the Certificate
of Candidacy for President of the Republic of the Philippines in the May 9, 2016 National and
Local Elections filed by respondent Mary Grace Natividad Sonora Poe Llamanzares is
hereby CANCELLED.69

Motion for Reconsideration of the 1 December 2015 Resolution was filed by petitioner which
the COMELEC En Banc resolved in its 23 December 2015 Resolution by denying the same. 70

Origin of Petition for Certiorari in G.R. Nos. 221698-700

This case stemmed from three (3) separate petitions filed by Francisco S. Tatad (Tatad),
Antonio P. Contreras (Contreras) and Amado D. Valdez (Valdez) against petitioner before the
COMELEC which were consolidated and raffled to its First Division.

In his petition to disqualify petitioner under Rule 25 of the COMELEC Rules of


Procedure,71 docketed as SPA No. 15-002 (DC), Tatad alleged that petitioner lacks the
requisite residency and citizenship to qualify her for the Presidency. 72

Tatad theorized that since the Philippines adheres to the principle of jus sanguinis, persons of
unknown parentage, particularly foundlings, cannot be considered natural-born Filipino
citizens since blood relationship is determinative of natural-born status.73 Tatad invoked the
rule of statutory construction that what is not included is excluded. He averred that the fact
that foundlings were not expressly included in the categories of citizens in the 193 5
Constitution is indicative of the framers' intent to exclude them. 74 Therefore, the burden lies
on petitioner to prove that she is a natural-born citizen.75

Neither can petitioner seek refuge under international conventions or treaties to support her
claim that foundlings have a nationality.76 According to Tatad, international conventions and
treaties are not self-executory and that local legislations are necessary in order to give effect
to treaty obligations assumed by the Philippines.77 He also stressed that there is no standard
state practice that automatically confers natural-born status to foundlings.78

Similar to Elamparo's argument, Tatad claimed that petitioner cannot avail of the option to
reacquire Philippine citizenship under R.A. No. 9225 because it only applies to former natural-
born citizens and petitioner was not as she was a foundling.79

Referring to petitioner's COC for Senator, Tatad concluded that she did not comply with the
ten (10) year residency requirement.80 Tatad opined that petitioner acquired her domicile in
Quezon City only from the time she renounced her American citizenship which was sometime
in 2010 or 2011.81 Additionally, Tatad questioned petitioner's lack of intention to abandon her
U.S. domicile as evinced by the fact that her husband stayed thereat and her frequent trips
to the U.S.82
In support of his petition to deny due course or cancel the COC of petitioner, docketed as SPA
No. 15-139 (DC), Valdez alleged that her repatriation under R.A. No. 9225 did not bestow
upon her the status of a natural-born citizen.83 He advanced the view that former natural-
born citizens who are repatriated under the said Act reacquires only their Philippine citizenship
and will not revert to their original status as natural-born citizens.84

He further argued that petitioner's own admission in her COC for Senator that she had only
been a resident of the Philippines for at least six (6) years and six (6) months prior to the 13
May 2013 Elections operates against her. Valdez rejected petitioner's claim that she could
have validly reestablished her domicile in the Philippines prior to her reacquisition of Philippine
citizenship. In effect, his position was that petitioner did not meet the ten (10) year residency
requirement for President.

Unlike the previous COMELEC cases filed against petitioner, Contreras' petition, 85 docketed as
SPA No. 15-007 (DC), limited the attack to the residency issue. He claimed that petitioner's
2015 COC for President should be cancelled on the ground that she did not possess the ten-
year period of residency required for said candidacy and that she made false entry in her COC
when she stated that she is a legal resident of the Philippines for ten (10) years and eleven
(11) months by 9 May 2016.86 Contreras contended that the reckoning period for computing
petitioner's residency in the Philippines should be from 18 July 2006, the date when her
petition to reacquire Philippine citizenship was approved by the BI. 87 He asserted that
petitioner's physical presence in the country before 18 July 2006 could not be valid evidence
of reacquisition of her Philippine domicile since she was then living here as an American citizen
and as such, she was governed by the Philippine immigration laws.88

In her defense, petitioner raised the following arguments:

First, Tatad's petition should be dismissed outright for failure to state a cause of action. His
petition did not invoke grounds proper for a disqualification case as enumerated under
Sections 12 and 68 of the Omnibus Election Code.89 Instead, Tatad completely relied on the
alleged lack of residency and natural-born status of petitioner which are not among the
recognized grounds for the disqualification of a candidate to an elective office. 90

Second, the petitions filed against her are basically petitions for quo warranto as they focus
on establishing her ineligibility for the Presidency. 91 A petition for quo warranto falls within
the exclusive jurisdiction of the Presidential Electoral Tribunal (PET) and not the COMELEC. 92

Third, the burden to prove that she is not a natural-born Filipino citizen is on the
respondents.93 Otherwise stated, she has a presumption in her favor that she is a natural-
born citizen of this country.

Fourth, customary international law dictates that foundlings are entitled to a nationality and
are presumed to be citizens of the country where they are found. 94 Consequently, the
petitioner is considered as a natural-born citizen of the Philippines.95

Fifth, she claimed that as a natural-born citizen, she has every right to be repatriated under
R.A. No. 9225 or the right to reacquire her natural-born status.96 Moreover, the official acts
of the Philippine Government enjoy the presumption of regularity, to wit: the issuance of the
18 July 2006 Order of the BI declaring her as natural-born citizen, her appointment as MTRCB
Chair and the issuance of the decree of adoption of San Juan RTC. 97 She believed that all
these acts reinforced her position that she is a natural-born citizen of the Philippines.98
Sixth, she maintained that as early as the first quarter of 2005, she started reestablishing her
domicile of choice in the Philippines as demonstrated by her children's resettlement and
schooling in the country, purchase of a condominium unit in San Juan City and the
construction of their family home in Corinthian Hills.99

Seventh, she insisted that she could legally reestablish her domicile of choice in the Philippines
even before she renounced her American citizenship as long as the three determinants for a
change of domicile are complied with.100She reasoned out that there was no requirement that
renunciation of foreign citizenship is a prerequisite for the acquisition of a new domicile of
choice.101

Eighth, she reiterated that the period appearing in the residency portion of her COC for
Senator was a mistake made in good faith.102

In a Resolution103 promulgated on 11 December 2015, the COMELEC First Division ruled that
petitioner is not a natural-born citizen, that she failed to complete the ten (10) year residency
requirement, and that she committed material misrepresentation in her COC when she
declared therein that she has been a resident of the Philippines for a period of ten (10) years
and eleven (11) months as of the day of the elections on 9 May 2016. The COMELEC First
Division concluded that she is not qualified for the elective position of President of the Republic
of the Philippines. The dispositive portion of said Resolution reads:

WHEREFORE, premises considered, the Commission RESOLVED, as it hereby RESOLVES,


to GRANT the Petitions and cancel the Certificate of Candidacy of MARY GRACE
NATIVIDAD SONORA POE-LLAMANZARES for the elective position of President of the
Republic of the Philippines in connection with the 9 May 2016 Synchronized Local and National
Elections.

Petitioner filed a motion for reconsideration seeking a reversal of the COMELEC First Division's
Resolution. On 23 December 2015, the COMELEC En Banc issued a Resolution denying
petitioner's motion for reconsideration.

Alarmed by the adverse rulings of the COMELEC, petitioner instituted the present petitions
for certiorari with urgent prayer for the issuance of an ex parte temporary
restraining order/status quo ante order and/or writ of preliminary injunction. On 28 December
2015, temporary restraining orders were issued by the Court enjoining the COMELEC and its
representatives from implementing the assailed COMELEC Resolutions until further orders
from the Court. The Court also ordered the consolidation of the two petitions filed by petitioner
in its Resolution of 12 January 2016. Thereafter, oral arguments were held in these cases.

The Court GRANTS the petition of Mary Grace Natividad S. Poe-Llamanzares and to ANNUL
and SET ASIDE the:

1. Resolution dated 1 December 2015 rendered through its Second Division, in SPA
No. 15-001 (DC), entitled Estrella C. Elamparo, petitioner, vs. Mary Grace Natividad
Sonora Poe-Llamanzares.

2. Resolution dated 11 December 2015, rendered through its First Division, in the
consolidated cases SPA No. 15-002 (DC) entitled Francisco S. Tatad, petitioner, vs.
Mary Grace Natividad Sonora Poe-Llamanzares, respondent; SPA No. 15-007 (DC)
entitled Antonio P. Contreras, petitioner, vs. Mary Grace Natividad Sonora Poe-
Llamanzares, respondent; and SPA No. 15-139 (DC) entitled Amado D. Valdez,
petitioner, v. Mary Grace Natividad Sonora Poe-Llamanzares, respondent.

3. Resolution dated 23 December 2015 of the Commission En Banc, upholding the 1


December 2015 Resolution of the Second Division.

4. Resolution dated 23 December 2015 of the Commission En Banc, upholding the 11


December 2015 Resolution of the First Division.

The procedure and the conclusions from which the questioned Resolutions emanated are
tainted with grave abuse of discretion amounting to lack of jurisdiction. The petitioner is a
QUALIFIED CANDIDATE for President in the 9 May 2016 National Elections.

The issue before the COMELEC is whether or not the COC of petitioner should be denied due
course or cancelled "on the exclusive ground" that she made in the certificate a false material
representation. The exclusivity of the ground should hedge in the discretion of the COMELEC
and restrain it from going into the issue of the qualifications of the candidate for the position,
if, as in this case, such issue is yet undecided or undetermined by the proper authority. The
COMELEC cannot itself, in the same cancellation case, decide the qualification or lack thereof
of the candidate.

We rely, first of all, on the Constitution of our Republic, particularly its provisions in Article
IX, C, Section 2:

Section 2. The Commission on Elections shall exercise the following powers and functions:

(1) Enforce and administer all laws and regulations relative to the conduct of
an election, plebiscite, initiative, referendum, and recall.

(2) Exercise exclusive original jurisdiction over all contests relating to the
elections, returns, and qualifications of all elective regional, provincial, and city
officials, and appellate jurisdiction over all contests involving elective municipal
officials decided by trial courts of general jurisdiction, or involving elective
barangay officials decided by trial courts of limited jurisdiction.

Decisions, final orders, or rulings of the Commission on election contests


involving elective municipal and barangay offices shall be final, executory, and
not appealable.

(3) Decide, except those involving the right to vote, all questions affecting
elections, including determination of the number and location of polling places,
appointment of election officials and inspectors, and registration of voters.

(4) Deputize, with the concurrence of the President, law enforcement agencies
and instrumentalities of the Government, including the Armed Forces of the
Philippines, for the exclusive purpose of ensuring free, orderly, honest,
peaceful, and credible elections.

(5) Register, after sufficient publication, political parties, organizations, or


coalitions which, in addition to other requirements, must present their platform
or program of government; and accredit citizens' arms of the Commission on
Elections. Religious denominations and sects shall not be registered. Those
which seek to achieve their goals through violence or unlawful means, or refuse
to uphold and adhere to this Constitution, or which are supported by any foreign
government shall likewise be refused registration.

Financial contributions from foreign governments and their agencies to political


parties, organizations, coalitions, or candidates related to elections constitute
interference in national affairs, and, when accepted, shall be an additional
ground for the cancellation of their registration with the Commission, in addition
to other penalties that may be prescribed by law.

(6) File, upon a verified complaint, or on its own initiative, petitions in court for
inclusion or exclusion of voters; investigate and, where appropriate, prosecute
cases of violations of election laws, including acts or omissions constituting
election frauds, offenses, and malpractices.

(7) Recommend to the Congress effective measures to minimize election


spending, including limitation of places where propaganda materials shall be
posted, and to prevent and penalize all forms of election frauds, offenses,
malpractices, and nuisance candidacies.

(8) Recommend to the President the removal of any officer or employee it has
deputized, or the imposition of any other disciplinary action, for violation or
disregard of, or disobedience to its directive, order, or decision.

(9) Submit to the President and the Congress a comprehensive report on the
conduct of each election, plebiscite, initiative, referendum, or recall.

Not any one of the enumerated powers approximate the exactitude of the provisions of Article
VI, Section 17 of the same basic law stating that:

The Senate and the House of Representatives shall each have an Electoral Tribunal
which shall be the sole judge of all contests relating to the election, returns, and
qualifications of their respective Members. Each Electoral Tribunal shall be composed
of nine Members, three of whom shall be Justices of the Supreme Court to be
designated by the Chief Justice, and the remaining six shall be Members of the Senate
or the House of Representatives, as the case may be, who shall be chosen on the basis
of proportional representation from the political parties and the parties or
organizations registered under the party-list system represented therein. The senior
Justice in the Electoral Tribunal shall be its Chairman.

or of the last paragraph of Article VII, Section 4 which provides that:

The Supreme Court, sitting en banc, shall be the sole judge of all contests relating to
the election, returns, and qualifications of the President or Vice-President, and may
promulgate its rules for the purpose.

The tribunals which have jurisdiction over the question of the qualifications of the President,
the Vice-President, Senators and the Members of the House of Representatives was made
clear by the Constitution. There is no such provision for candidates for these positions.
Can the COMELEC be such judge?

The opinion of Justice Vicente V. Mendoza in Romualdez-Marcos v. Commission on


Elections,104 which was affirmatively cited in the En Banc decision in Fermin v. COMELEC105 is
our guide. The citation in Fermin reads:

Apparently realizing the lack of an authorized proceeding for declaring the ineligibility of
candidates, the COMELEC amended its rules on February 15, 1993 so as to provide in Rule
25 § 1, the following:

Grounds for disqualification. - Any candidate who does not possess all the
qualifications of a candidate as provided for by the Constitution or by existing
law or who commits any act declared by law to be grounds for disqualification
may be disqualified from continuing as a candidate.

The lack of provision for declaring the ineligibility of candidates, however, cannot be supplied
by a mere rule. Such an act is equivalent to the creation of a cause of action which is a
substantive matter which the COMELEC, in the exercise of its rule-making power under Art.
IX, A, §6 of the Constitution, cannot do it. It is noteworthy that the Constitution withholds
from the COMELEC even the power to decide cases involving the right to vote, which
essentially involves an inquiry into qualifications based on age, residence and citizenship of
voters. [Art. IX, C, §2(3)]

The assimilation in Rule 25 of the COMELEC rules of grounds for ineligibility into grounds for
disqualification is contrary to the evident intention of the law. For not only in their grounds
but also in their consequences are proceedings for "disqualification" different from those for
a declaration of "ineligibility." "Disqualification" proceedings, as already stated, are based on
grounds specified in § 12 and §68 of the Omnibus Election Code and in §40 of the Local
Government Code and are for the purpose of barring an individual from becoming a candidate
or from continuing as a candidate for public office. In a word, their purpose is to eliminate a
candidate from the race either from the start or during its progress. "Ineligibility," on the
other hand, refers to the lack of the qualifications prescribed in the Constitution or the statutes
for holding public office and the purpose of the proceedings for declaration of ineligibility is
to remove the incumbent from office.

Consequently, that an individual possesses the qualifications for a public office does not imply
that he is not disqualified from becoming a candidate or continuing as a candidate for a public
office and vice versa. We have this sort of dichotomy in our Naturalization Law. (C.A. No.
473) That an alien has the qualifications prescribed in §2 of the Law does not imply that he
does not suffer from any of [the] disqualifications provided in §4.

Before we get derailed by the distinction as to grounds and the consequences of the respective
proceedings, the importance of the opinion is in its statement that "the lack of provision for
declaring the ineligibility of candidates, however, cannot be supplied by a mere rule". Justice
Mendoza lectured in Romualdez-Marcos that:

Three reasons may be cited to explain the absence of an authorized proceeding for
determining before election the qualifications of a candidate.

First is the fact that unless a candidate wins and is proclaimed elected, there is no necessity
for determining his eligibility for the office. In contrast, whether an individual should be
disqualified as a candidate for acts constituting election offenses (e.g., vote buying, over
spending, commission of prohibited acts) is a prejudicial question which should be determined
lest he wins because of the very acts for which his disqualification is being sought. That is
why it is provided that if the grounds for disqualification are established, a candidate will not
be voted for; if he has been voted for, the votes in his favor will not be counted; and if for
some reason he has been voted for and he has won, either he will not be proclaimed or his
proclamation will be set aside.

Second is the fact that the determination of a candidates' eligibility, e.g., his citizenship or,
as in this case, his domicile, may take a long time to make, extending beyond the beginning
of the term of the office. This is amply demonstrated in the companion case (G.R. No.
120265, Agapito A. Aquino v. COMELEC) where the determination of Aquino's residence was
still pending in the COMELEC even after the elections of May 8, 1995. This is contrary to the
summary character proceedings relating to certificates of candidacy. That is why the law
makes the receipt of certificates of candidacy a ministerial duty of the COMELEC and its
officers. The law is satisfied if candidates state in their certificates of candidacy that they are
eligible for the position which they seek to fill, leaving the determination of their qualifications
to be made after the election and only in the event they are elected. Only in cases involving
charges of false representations made in certificates of candidacy is the COMELEC given
jurisdiction.

Third is the policy underlying the prohibition against pre-proclamation cases in elections for
President, Vice President, Senators and members of the House of Representatives. (R.A. No.
7166, § 15) The purpose is to preserve the prerogatives of the House of Representatives
Electoral Tribunal and the other Tribunals as "sole judges" under the Constitution of
the election, returns and qualifications of members of Congress of the President and Vice
President, as the case may be.106

To be sure, the authoritativeness of the Romualdez pronouncements as reiterated


in Fermin, led to the amendment through COMELEC Resolution No. 9523, on 25 September
2012 of its Rule 25. This, the 15 February1993 version of Rule 25, which states that:

Grounds for disqualification. -Any candidate who does not possess all the qualifications of a
candidate as provided for by the Constitution or by existing law or who commits any act
declared by law to be grounds for disqualification may be disqualified from continuing as a
candidate.107

was in the 2012 rendition, drastically changed to:

Grounds. - Any candidate who, in action or protest in which he is a party, is declared by final
decision of a competent court, guilty of, or found by the Commission to be suffering from any
disqualification provided by law or the Constitution.

A Petition to Disqualify a Candidate invoking grounds for a Petition to Deny to or Cancel a


Certificate of Candidacy or Petition to Declare a Candidate as a Nuisance Candidate, or a
combination thereof, shall be summarily dismissed.

Clearly, the amendment done in 2012 is an acceptance of the reality of absence of an


authorized proceeding for determining before election the qualifications of candidate. Such
that, as presently required, to disqualify a candidate there must be a declaration by a final
judgment of a competent court that the candidate sought to be disqualified "is guilty of or
found by the Commission to be suffering from any disqualification provided by law or the
Constitution."

Insofar as the qualification of a candidate is concerned, Rule 25 and Rule 23 are flipsides of
one to the other. Both do not allow, are not authorizations, are not vestment of jurisdiction,
for the COMELEC to determine the qualification of a candidate. The facts of qualification must
beforehand be established in a prior proceeding before an authority properly vested with
jurisdiction. The prior determination of qualification may be by statute, by executive order or
by a judgment of a competent court or tribunal.

If a candidate cannot be disqualified without a prior finding that he or she is suffering from a
disqualification "provided by law or the Constitution," neither can the certificate of candidacy
be cancelled or denied due course on grounds of false representations regarding his or her
qualifications, without a prior authoritative finding that he or she is not qualified, such prior
authority being the necessary measure by which the falsity of the representation can be found.
The only exception that can be conceded are self-evident facts of unquestioned or
unquestionable veracity and judicial confessions. Such are, anyway, bases equivalent to prior
decisions against which the falsity of representation can be determined.

The need for a predicate finding or final pronouncement in a proceeding under Rule 23 that
deals with, as in this case, alleged false representations regarding the candidate's citizenship
and residence, forced the COMELEC to rule essentially that since foundlings 108 are not
mentioned in the enumeration of citizens under the 1935 Constitution, 109 they then cannot be
citizens. As the COMELEC stated in oral arguments, when petitioner admitted that she is a
foundling, she said it all. This borders on bigotry. Oddly, in an effort at tolerance, the
COMELEC, after saying that it cannot rule that herein petitioner possesses blood relationship
with a Filipino citizen when "it is certain that such relationship is indemonstrable," proceeded
to say that "she now has the burden to present evidence to prove her natural filiation with a
Filipino parent."

The fact is that petitioner's blood relationship with a Filipino citizen is DEMONSTRABLE.

At the outset, it must be noted that presumptions regarding paternity is neither unknown nor
unaccepted in Philippine Law. The Family Code of the Philippines has a whole chapter on
Paternity and Filiation.110 That said, there is more than sufficient evider1ce that petitioner has
Filipino parents and is therefore a natural-born Filipino. Parenthetically, the burden of proof
was on private respondents to show that petitioner is not a Filipino citizen. The private
respondents should have shown that both of petitioner's parents were aliens. Her admission
that she is a foundling did not shift the burden to her because such status did not exclude the
possibility that her parents were Filipinos, especially as in this case where there is a high
probability, if not certainty, that her parents are Filipinos.

The factual issue is not who the parents of petitioner are, as their identities are unknown, but
whether such parents are Filipinos. Under Section 4, Rule 128:

Sect. 4. Relevancy, collateral matters - Evidence must have such a relation to the fact in issue
as to induce belief in its existence or no-existence. Evidence on collateral matters shall not
be allowed, except when it tends in any reasonable degree to establish the probability of
improbability of the fact in issue.

The Solicitor General offered official statistics from the Philippine Statistics Authority
(PSA)111 that from 1965 to 1975, the total number of foreigners born in the Philippines was
15,986 while the total number of Filipinos born in the country was 10,558,278. The statistical
probability that any child born in the Philippines in that decade is natural-born Filipino
was 99.83%. For her part, petitioner presented census statistics for Iloilo Province for 1960
and 1970, also from the PSA. In 1960, there were 962,532 Filipinos and 4,734 foreigners in
the province; 99.62% of the population were Filipinos. In 1970, the figures were 1,162,669
Filipinos and 5,304 foreigners, or 99.55%. Also presented were figures for the child
producing ages (15-49). In 1960, there were 230,528 female Filipinos as against 730 female
foreigners or 99.68%. In the same year, there were 210,349 Filipino males and 886 male
aliens, or 99.58%. In 1970, there were 270,299 Filipino females versus 1, 190 female aliens,
or 99.56%. That same year, there were 245,740 Filipino males as against only 1,165 male
aliens or 99.53%. COMELEC did not dispute these figures. Notably, Commissioner Arthur Lim
admitted, during the oral arguments, that at the time petitioner was found in 1968, the
majority of the population in Iloilo was Filipino.112

Other circumstantial evidence of the nationality of petitioner's parents are the fact that she
was abandoned as an infant in a Roman Catholic Church in Iloilo City.1âwphi1 She also has
typical Filipino features: height, flat nasal bridge, straight black hair, almond shaped eyes and
an oval face.

There is a disputable presumption that things have happened according to the ordinary course
of nature and the ordinary habits of life.113 All of the foregoing evidence, that a person with
typical Filipino features is abandoned in Catholic Church in a municipality where the population
of the Philippines is overwhelmingly Filipinos such that there would be more than a 99%
chance that a child born in the province would be a Filipino, would indicate more than ample
probability if not statistical certainty, that petitioner's parents are Filipinos. That probability
and the evidence on which it is based are admissible under Rule 128, Section 4 of the Revised
Rules on Evidence.

To assume otherwise is to accept the absurd, if not the virtually impossible, as the norm. In
the words of the Solicitor General:

Second. It is contrary to common sense because foreigners do not come to the Philippines so
they can get pregnant and leave their newborn babies behind. We do not face a situation
where the probability is such that every foundling would have a 50% chance of being a Filipino
and a 50% chance of being a foreigner. We need to frame our questions properly. What are
the chances that the parents of anyone born in the Philippines would be foreigners? Almost
zero. What are the chances that the parents of anyone born in the Philippines would be
Filipinos? 99.9%.

According to the Philippine Statistics Authority, from 2010 to 2014, on a yearly average, there
were 1,766,046 children born in the Philippines to Filipino parents, as opposed to 1,301
children in the Philippines of foreign parents. Thus, for that sample period, the ratio of non-
Filipino children to natural born Filipino children is 1:1357. This means that the statistical
probability that any child born in the Philippines would be a natural born Filipino is 99.93%.

From 1965 to 1975, the total number of foreigners born in the Philippines is 15,986 while the
total number of Filipinos born in the Philippines is 15,558,278. For this period, the ratio of
non-Filipino children is 1:661. This means that the statistical probability that any child born
in the Philippines on that decade would be a natural born Filipino is 99.83%.

We can invite statisticians and social anthropologists to crunch the numbers for us, but I am
confident that the statistical probability that a child born in the Philippines would be a natural
born Filipino will not be affected by whether or not the parents are known. If at all, the
likelihood that a foundling would have a Filipino parent might even be higher than 99.9%.
Filipinos abandon their children out of poverty or perhaps, shame. We do not imagine
foreigners abandoning their children here in the Philippines thinking those infants would have
better economic opportunities or believing that this country is a tropical paradise suitable for
raising abandoned children. I certainly doubt whether a foreign couple has ever considered
their child excess baggage that is best left behind.

To deny full Filipino citizenship to all foundlings and render them stateless just because there
may be a theoretical chance that one among the thousands of these foundlings might be the
child of not just one, but two, foreigners is downright discriminatory, irrational, and unjust.
It just doesn't make any sense. Given the statistical certainty - 99.9% - that any child born
in the Philippines would be a natural born citizen, a decision denying foundlings such status
is effectively a denial of their birthright. There is no reason why this Honorable Court should
use an improbable hypothetical to sacrifice the fundamental political rights of an entire class
of human beings. Your Honor, constitutional interpretation and the use of common sense are
not separate disciplines.

As a matter of law, foundlings are as a class, natural-born citizens. While the 1935
Constitution's enumeration is silent as to foundlings, there is no restrictive language which
would definitely exclude foundlings either. Because of silence and ambiguity in the
enumeration with respect to foundlings, there is a need to examine the intent of the framers.
In Nitafan v. Commissioner of Internal Revenue,114 this Court held that:

The ascertainment of that intent is but in keeping with the fundamental principle of
constitutional construction that the intent of the framers of the organic law and of the
people adopting it should be given effect. The primary task in constitutional
construction is to ascertain and thereafter assure the realization of the purpose of the
framers and of the people in the adoption of the Constitution. It may also be safely
assumed that the people in ratifying the Constitution were guided mainly by the
explanation offered by the framers.115

As pointed out by petitioner as well as the Solicitor General, the deliberations of the 1934
Constitutional Convention show that the framers intended foundlings to be covered by the
enumeration. The following exchange is recorded:

Sr. Rafols: For an amendment. I propose that after subsection 2, the following is inserted:
"The natural children of a foreign father and a Filipino mother not recognized by the father.

xxxx

President:
[We] would like to request a clarification from the proponent of the amendment. The
gentleman refers to natural children or to any kind of illegitimate children?

Sr. Rafols:
To all kinds of illegitimate children. It also includes natural children of unknown
parentage, natural or illegitimate children of unknown parents.

Sr. Montinola:
For clarification. The gentleman said "of unknown parents." Current codes consider them
Filipino, that is, I refer to the Spanish Code wherein all children of unknown parentage born
in Spanish territory are considered Spaniards, because the presumption is that a child of
unknown parentage is the son of a Spaniard. This may be applied in the Philippines in that a
child of unknown parentage born in the Philippines is deemed to be Filipino, and there is no
need ...

Sr. Rafols:
There is a need, because we are relating the conditions that are [required] to be Filipino.

Sr. Montinola:
But that is the interpretation of the law, therefore, there is no [more] need for amendment.

Sr. Rafols:
The amendment should read thus:
"Natural or illegitimate of a foreign father and a Filipino mother recognized by one, or the
children of unknown parentage."

Sr. Briones:
The amendment [should] mean children born in the Philippines of unknown parentage.

Sr. Rafols:
The son of a Filipina to a Foreigner, although this [person] does not recognize the child, is not
unknown.

President:
Does the gentleman accept the amendment or not?

Sr. Rafols:
I do not accept the amendment because the amendment would exclude the children of a
Filipina with a foreigner who does not recognize the child. Their parentage is not unknown
and I think those of overseas Filipino mother and father [whom the latter] does not recognize,
should also be considered as Filipinos.

President:
The question in order is the amendment to the amendment from the Gentleman from Cebu,
Mr. Briones.

Sr. Busion:
Mr. President, don't you think it would be better to leave this matter in the hands of the
Legislature?

Sr. Roxas:
Mr. President, my humble opinion is that these cases are few and far in between, that the
constitution need [not] refer to them. By international law the principle that children or people
born in a country of unknown parents are citizens in this nation is recognized, and it is not
necessary to include a provision on the subject exhaustively.116

Though the Rafols amendment was not carried out, it was not because there was any objection
to the notion that persons of "unknown parentage" are not citizens but only because their
number was not enough to merit specific mention. Such was the account, 117 cited by
petitioner, of delegate and constitution law author Jose Aruego who said:
During the debates on this provision, Delegate Rafols presented an amendment to
include as Filipino citizens the illegitimate children with a foreign father of a mother
who was a citizen of the Philippines, and also foundlings; but this amendment was
defeated primarily because the Convention believed that the cases, being too few to
warrant the inclusion of a provision in the Constitution to apply to them, should be
governed by statutory legislation. Moreover, it was believed that the rules of
international law were already clear to the effect that illegitimate children followed the
citizenship of the mother, and that foundlings followed the nationality of the place
where they were found, thereby making unnecessary the inclusion in the Constitution
of the proposed amendment.

This explanation was likewise the position of the Solicitor General during the 16 February
2016 Oral Arguments:

We all know that the Rafols proposal was rejected. But note that what was declined was the
proposal for a textual and explicit recognition of foundlings as Filipinos. And so, the way to
explain the constitutional silence is by saying that it was the view of Montinola and Roxas
which prevailed that there is no more need to expressly declare foundlings as Filipinos.

Obviously, it doesn't matter whether Montinola's or Roxas' views were legally correct. Framers
of a constitution can constitutionalize rules based on assumptions that are imperfect or even
wrong. They can even overturn existing rules. This is basic. What matters here is that
Montinola and Roxas were able to convince their colleagues in the convention that there is no
more need to expressly declare foundlings as Filipinos because they are already impliedly so
recognized.

In other words, the constitutional silence is fully explained in terms of linguistic efficiency and
the avoidance of redundancy. The policy is clear: it is to recognize foundlings, as a class, as
Filipinos under Art. IV, Section 1 (3) of the 1935 Constitution. This inclusive policy is carried
over into the 1973 and 1987 Constitution. It is appropriate to invoke a famous scholar as he
was paraphrased by Chief Justice Fernando: the constitution is not silently silent, it is silently
vocal. 118

The Solicitor General makes the further point that the framers "worked to create a just and
humane society," that "they were reasonable patriots and that it would be unfair to impute
upon them a discriminatory intent against foundlings." He exhorts that, given the grave
implications of the argument that foundlings are not natural-born Filipinos, the Court must
search the records of the 1935, 1973 and 1987 Constitutions "for an express intention to deny
foundlings the status of Filipinos. The burden is on those who wish to use the constitution to
discriminate against foundlings to show that the constitution really intended to take this path
to the dark side and inflict this across the board marginalization."

We find no such intent or language permitting discrimination against foundlings. On the


contrary, all three Constitutions guarantee the basic right to equal protection of the laws. All
exhort the State to render social justice. Of special consideration are several provisions in the
present charter: Article II, Section 11 which provides that the "State values the dignity of
every human person and guarantees full respect for human rights," Article XIII, Section 1
which mandates Congress to "give highest priority to the enactment of measures that protect
and enhance the right of all the people to human dignity, reduce social, economic, and political
inequalities x x x" and Article XV, Section 3 which requires the State to defend the "right of
children to assistance, including proper care and nutrition, and special protection from all
forms of neglect, abuse, cruelty, exploitation, and other conditions prejudicial to their
development." Certainly, these provisions contradict an intent to discriminate against
foundlings on account of their unfortunate status.

Domestic laws on adoption also support the principle that foundlings are Filipinos. These laws
do not provide that adoption confers citizenship upon the adoptee. Rather, the adoptee must
be a Filipino in the first place to be adopted. The most basic of such laws is Article 15 of the
Civil Code which provides that "[l]aws relating to family rights, duties, status, conditions, legal
capacity of persons are binding on citizens of the Philippines even though living abroad."
Adoption deals with status, and a Philippine adoption court will have jurisdiction only if the
adoptee is a Filipino. In Ellis and Ellis v. Republic,119 a child left by an unidentified mother was
sought to be adopted by aliens. This Court said:

In this connection, it should be noted that this is a proceedings in rem, which no court may
entertain unless it has jurisdiction, not only over the subject matter of the case and over the
parties, but also over the res, which is the personal status of Baby Rose as well as that of
petitioners herein. Our Civil Code (Art. 15) adheres to the theory that jurisdiction over the
status of a natural person is determined by the latter's nationality. Pursuant to this theory,
we have jurisdiction over the status of Baby Rose, she being a citizen of the Philippines, but
not over the status of the petitioners, who are foreigners.120 (Underlining supplied)

Recent legislation is more direct. R.A. No. 8043 entitled "An Act Establishing the Rules to
Govern the Inter-Country Adoption of Filipino Children and For Other Purposes" (otherwise
known as the "Inter-Country Adoption Act of 1995"), R.A. No. 8552, entitled "An Act
Establishing the Rules and Policies on the Adoption of Filipino Children and For Other
Purposes" (otherwise known as the Domestic Adoption Act of 1998) and this Court's A.M. No.
02-6-02-SC or the "Rule on Adoption," all expressly refer to "Filipino children" and include
foundlings as among Filipino children who may be adopted.

It has been argued that the process to determine that the child is a foundling leading to the
issuance of a foundling certificate under these laws and the issuance of said certificate are
acts to acquire or perfect Philippine citizenship which make the foundling a naturalized Filipino
at best. This is erroneous. Under Article IV, Section 2 "Natural-born citizens are those who
are citizens of the Philippines from birth without having to perform any act to acquire or
perfect their Philippine citizenship." In the first place, "having to perform an act" means that
the act must be personally done by the citizen. In this instance, the determination of foundling
status is done not by the child but by the authorities.121 Secondly, the object of the process
is the determination of the whereabouts of the parents, not the citizenship of the child. Lastly,
the process is certainly not analogous to naturalization proceedings to acquire Philippine
citizenship, or the election of such citizenship by one born of an alien father and a Filipino
mother under the 1935 Constitution, which is an act to perfect it.

In this instance, such issue is moot because there is no dispute that petitioner is a foundling,
as evidenced by a Foundling Certificate issued in her favor. 122 The Decree of Adoption issued
on 13 May 1974, which approved petitioner's adoption by Jesusa Sonora Poe and Ronald Allan
Kelley Poe, expressly refers to Emiliano and his wife, Rosario Militar, as her "foundling
parents," hence effectively affirming petitioner's status as a foundling.123

Foundlings are likewise citizens under international law. Under the 1987 Constitution, an
international law can become part of the sphere of domestic law either by transformation or
incorporation. The transformation method requires that an international law be transformed
into a domestic law through a constitutional mechanism such as local legislation. 124 On the
other hand, generally accepted principles of international law, by virtue of the incorporation
clause of the Constitution, form part of the laws of the land even if they do not derive from
treaty obligations. Generally accepted principles of international law include international
custom as evidence of a general practice accepted as law, and general principles of law
recognized by civilized nations.125 International customary rules are accepted as binding as a
result from the combination of two elements: the established, widespread, and consistent
practice on the part of States; and a psychological element known as the opinionjuris sive
necessitates (opinion as to law or necessity). Implicit in the latter element is a belief that the
practice in question is rendered obligatory by the existence of a rule of law requiring
it.126 "General principles of law recognized by civilized nations" are principles "established by
a process of reasoning" or judicial logic, based on principles which are "basic to legal systems
generally,"127 such as "general principles of equity, i.e., the general principles of fairness and
justice," and the "general principle against discrimination" which is embodied in the "Universal
Declaration of Human Rights, the International Covenant on Economic, Social and Cultural
Rights, the International Convention on the Elimination of All Forms of Racial Discrimination,
the Convention Against Discrimination in Education, the Convention (No. 111) Concerning
Discrimination in Respect of Employment and Occupation."128 These are the same core
principles which underlie the Philippine Constitution itself, as embodied in the due process
and equal protection clauses of the Bill of Rights.129

Universal Declaration of Human Rights ("UDHR") has been interpreted by this Court as part
of the generally accepted principles of international law and binding on the State. 130 Article
15 thereof states:

1. Everyone has the right to a nationality.

2. No one shall be arbitrarily deprived of his nationality nor denied the right to change
his nationality.

The Philippines has also ratified the UN Convention on the Rights of the Child (UNCRC). Article
7 of the UNCRC imposes the following obligations on our country:

Article 7

1. The child shall be registered immediately after birth and shall have the right from birth to
a name, the right to acquire a nationality and as far as possible, the right to know and be
cared for by his or her parents.

2. States Parties shall ensure the implementation of these rights in accordance with their
national law and their obligations under the relevant international instruments in this field, in
particular where the child would otherwise be stateless.

In 1986, the country also ratified the 1966 International Covenant on Civil and Political Rights
(ICCPR). Article 24 thereof provide for the right of every child "to acquire a nationality:"

Article 24

1. Every child shall have, without any discrimination as to race, colour, sex, language, religion,
national or social origin, property or birth, the right, to such measures of protection as are
required by his status as a minor, on the part of his family, society and the State.

2. Every child shall be registered immediately after birth and shall have a name.
3. Every child has the right to acquire a nationality.

The common thread of the UDHR, UNCRC and ICCPR is to obligate the Philippines to grant
nationality from birth and ensure that no child is stateless. This grant of nationality must be
at the time of birth, and it cannot be accomplished by the application of our present
naturalization laws, Commonwealth Act No. 473, as amended, and R.A. No. 9139, both of
which require the applicant to be at least eighteen (18) years old.

The principles found in two conventions, while yet unratified by the Philippines, are generally
accepted principles of international law. The first is Article 14 of the 1930 Hague Convention
on Certain Questions Relating to the Conflict of Nationality Laws under which a foundling is
presumed to have the "nationality of the country of birth," to wit:

Article 14

A child whose parents are both unknown shall have the nationality of the country of birth. If
the child's parentage is established, its nationality shall be determined by the rules applicable
in cases where the parentage is known.

A foundling is, until the contrary is proved, presumed to have been born on the territory of
the State in which it was found. (Underlining supplied)

The second is the principle that a foundling is presumed born of citizens of the country where
he is found, contained in Article 2 of the 1961 United Nations Convention on the Reduction of
Statelessness:

Article 2

A foundling found in the territory of a Contracting State shall, in the absence of proof to the
contrary, be considered to have been born within the territory of parents possessing the
nationality of that State.

That the Philippines is not a party to the 1930 Hague Convention nor to the 1961 Convention
on the Reduction of Statelessness does not mean that their principles are not binding. While
the Philippines is not a party to the 1930 Hague Convention, it is a signatory to the Universal
Declaration on Human Rights, Article 15(1) ofwhich131effectively affirms Article 14 of the 1930
Hague Convention. Article 2 of the 1961 "United Nations Convention on the Reduction of
Statelessness" merely "gives effect" to Article 15(1) of the UDHR. 132 In Razon v.
Tagitis, 133 this Court noted that the Philippines had not signed or ratified the "International
Convention for the Protection of All Persons from Enforced Disappearance." Yet, we ruled that
the proscription against enforced disappearances in the said convention was nonetheless
binding as a "generally accepted principle of international law." Razon v. Tagitis is likewise
notable for declaring the ban as a generally accepted principle of international law although
the convention had been ratified by only sixteen states and had not even come into force and
which needed the ratification of a minimum of twenty states. Additionally, as petitioner points
out, the Court was content with the practice of international and regional state organs,
regional state practice in Latin America, and State Practice in the United States.

Another case where the number of ratifying countries was not determinative is Mijares v.
Ranada, 134 where only four countries had "either ratified or acceded to" 135 the 1966
"Convention on the Recognition and Enforcement of Foreign Judgments in Civil and
Commercial Matters" when the case was decided in 2005. The Court also pointed out that
that nine member countries of the European Common Market had acceded to the Judgments
Convention. The Court also cited U.S. laws and jurisprudence on recognition of foreign
judgments. In all, only the practices of fourteen countries were considered and yet, there was
pronouncement that recognition of foreign judgments was widespread practice.

Our approach in Razon and Mijares effectively takes into account the fact that "generally
accepted principles of international law" are based not only on international custom, but also
on "general principles of law recognized by civilized nations," as the phrase is understood in
Article 38.1 paragraph (c) of the ICJ Statute. Justice, fairness, equity and the policy against
discrimination, which are fundamental principles underlying the Bill of Rights and which are
"basic to legal systems generally,"136 support the notion that the right against enforced
disappearances and the recognition of foreign judgments, were correctly considered as
"generally accepted principles of international law" under the incorporation clause.

Petitioner's evidence137 shows that at least sixty countries in Asia, North and South America,
and Europe have passed legislation recognizing foundlings as its citizen. Forty-two (42) of
those countries follow the jus sanguinis regime. Of the sixty, only thirty-three (33) are parties
to the 1961 Convention on Statelessness; twenty-six (26) are not signatories to the
Convention. Also, the Chief Justice, at the 2 February 2016 Oral Arguments pointed out that
in 166 out of 189 countries surveyed (or 87.83%), foundlings are recognized as citizens.
These circumstances, including the practice of jus sanguinis countries, show that it is a
generally accepted principle of international law to presume foundlings as having been born
of nationals of the country in which the foundling is found.

Current legislation reveals the adherence of the Philippines to this generally accepted principle
of international law. In particular, R.A. No. 8552, R.A. No. 8042 and this Court's Rules on
Adoption, expressly refer to "Filipino children." In all of them, foundlings are among the
Filipino children who could be adopted. Likewise, it has been pointed that the DFA issues
passports to foundlings. Passports are by law, issued only to citizens. This shows that even
the executive department, acting through the DFA, considers foundlings as Philippine citizens.

Adopting these legal principles from the 1930 Hague Convention and the 1961 Convention on
Statelessness is rational and reasonable and consistent with the jus sanguinis regime in our
Constitution. The presumption of natural-born citizenship of foundlings stems from the
presumption that their parents are nationals of the Philippines. As the empirical data provided
by the PSA show, that presumption is at more than 99% and is a virtual certainty.

In sum, all of the international law conventions and instruments on the matter of nationality
of foundlings were designed to address the plight of a defenseless class which suffers from a
misfortune not of their own making. We cannot be restrictive as to their application if we are
a country which calls itself civilized and a member of the community of nations. The Solicitor
General's warning in his opening statement is relevant:

.... the total effect of those documents is to signify to this Honorable Court that those treaties
and conventions were drafted because the world community is concerned that the situation
of foundlings renders them legally invisible. It would be tragically ironic if this Honorable Court
ended up using the international instruments which seek to protect and uplift foundlings a
tool to deny them political status or to accord them second-class citizenship.138

The COMELEC also ruled139 that petitioner's repatriation in July 2006 under the provisions of
R.A. No. 9225 did not result in the reacquisition of natural-born citizenship. The COMELEC
reasoned that since the applicant must perform an act, what is reacquired is not "natural-
born" citizenship but only plain "Philippine citizenship."

The COMELEC's rule arrogantly disregards consistent jurisprudence on the matter of


repatriation statutes in general and of R.A. No. 9225 in particular.

In the seminal case of Bengson Ill v. HRET, 140


repatriation was explained as follows:

Moreover, repatriation results in the recovery of the original nationality. This means that a
naturalized Filipino who lost his citizenship will be restored to his prior status as a naturalized
Filipino citizen. On the other hand, if he was originally a natural-born citizen before he lost
his Philippine citizenship, he will be restored to his former status as a natural-born Filipino.

R.A. No. 9225 is a repatriation statute and has been described as such in several cases. They
include Sobejana-Condon v. COMELEC141 where we described it as an
"abbreviated repatriation process that restores one's Filipino citizenship x x x." Also included
is Parreno v. Commission on Audit,142 which cited Tabasa v. Court of Appeals,143where we
said that "[t]he repatriation of the former Filipino will allow him to recover his natural-born
citizenship. Parreno v. Commission on Audit144 is categorical that "if petitioner reacquires his
Filipino citizenship (under R.A. No. 9225), he will ... recover his natural-born citizenship."

The COMELEC construed the phrase "from birth" in the definition of natural citizens as
implying "that natural-born citizenship must begin at birth and remain uninterrupted and
continuous from birth." R.A. No. 9225 was obviously passed in line with Congress' sole
prerogative to determine how citizenship may be lost or reacquired. Congress saw it fit to
decree that natural-born citizenship may be reacquired even if it had been once lost. It is not
for the COMELEC to disagree with the Congress' determination.

More importantly, COMELEC's position that natural-born status must be continuous was
already rejected in Bengson III v. HRET145 where the phrase "from birth" was clarified to mean
at the time of birth: "A person who at the time of his birth, is a citizen of a particular country,
is a natural-born citizen thereof." Neither is "repatriation" an act to "acquire or perfect" one's
citizenship. In Bengson III v. HRET, this Court pointed out that there are only two types of
citizens under the 1987 Constitution: natural-born citizen and naturalized, and that there is
no third category for repatriated citizens:

It is apparent from the enumeration of who are citizens under the present Constitution that
there are only two classes of citizens: (1) those who are natural-born and (2) those who are
naturalized in accordance with law. A citizen who is not a naturalized Filipino, ie., did not have
to undergo the process of naturalization to obtain Philippine citizenship, necessarily is a
natural-born Filipino. Noteworthy is the absence in said enumeration of a separate category
for persons who, after losing Philippine citizenship, subsequently reacquire it. The reason
therefor is clear: as to such persons, they would either be natural-born or naturalized
depending on the reasons for the loss of their citizenship and the mode prescribed by the
applicable law for the reacquisition thereof. As respondent Cruz was not required by law to
go through naturalization proceedings in order to reacquire his citizenship, he is perforce a
natural-born Filipino. As such, he possessed all the necessary qualifications to be elected as
member of the House of Representatives.146

The COMELEC cannot reverse a judicial precedent. That is reserved to this Court. And while
we may always revisit a doctrine, a new rule reversing standing doctrine cannot be
retroactively applied. In Morales v. Court of Appeals and Jejomar Erwin S. Binay, Jr.,147 where
we decreed reversed the condonation doctrine, we cautioned that it "should be prospective in
application for the reason that judicial decisions applying or interpreting the laws of the
Constitution, until reversed, shall form part of the legal system of the Philippines." This Court
also said that "while the future may ultimately uncover a doctrine's error, it should be, as a
general rule, recognized as good law prior to its abandonment. Consequently, the people's
reliance thereupon should be respected."148

Lastly, it was repeatedly pointed out during the oral arguments that petitioner committed a
falsehood when she put in the spaces for "born to" in her application for repatriation under
R.A. No. 9225 the names of her adoptive parents, and this misled the BI to presume that she
was a natural-born Filipino. It has been contended that the data required were the names of
her biological parents which are precisely unknown.

This position disregards one important fact - petitioner was legally adopted. One of the effects
of adoption is "to sever all legal ties between the biological parents and the adoptee, except
when the biological parent is the spouse of the adoptee." 149 Under R.A. No. 8552, petitioner
was also entitled to an amended birth certificate "attesting to the fact that the adoptee is the
child of the adopter(s)" and which certificate "shall not bear any notation that it is an amended
issue."150 That law also requires that "[a]ll records, books, and papers relating to the adoption
cases in the files of the court, the Department [of Social Welfare and Development], or any
other agency or institution participating in the adoption proceedings shall be kept strictly
confidential."151 The law therefore allows petitioner to state that her adoptive parents were
her birth parents as that was what would be stated in her birth certificate anyway. And given
the policy of strict confidentiality of adoption records, petitioner was not obligated to disclose
that she was an adoptee.

Clearly, to avoid a direct ruling on the qualifications of petitioner, which it cannot make in the
same case for cancellation of COC, it resorted to opinionatedness which is,
moreover, erroneous. The whole process undertaken by COMELEC is wrapped in grave abuse
of discretion.

On Residence

The tainted process was repeated in disposing of the issue of whether or not petitioner
committed false material representation when she stated in her COC that she has before and
until 9 May 2016 been a resident of the Philippines for ten (10) years and eleven (11) months.

Petitioner's claim that she will have been a resident for ten (10) years and eleven (11) months
on the day before the 2016 elections, is true.

The Constitution requires presidential candidates to have ten (10) years' residence in the
Philippines before the day of the elections. Since the forthcoming elections will be held on 9
May 2016, petitioner must have been a resident of the Philippines prior to 9 May 2016 for ten
(10) years. In answer to the requested information of "Period of Residence in the Philippines
up to the day before May 09, 2016," she put in "10 years 11 months" which according to her
pleadings in these cases corresponds to a beginning date of 25 May 2005 when she returned
for good from the U.S.

When petitioner immigrated to the U.S. in 1991, she lost her original domicile, which is the
Philippines. There are three requisites to acquire a new domicile: 1. Residence or bodily
presence in a new locality; 2. an intention to remain there; and 3. an intention to abandon
the old domicile.152 To successfully effect a change of domicile, one must demonstrate an
actual removal or an actual change of domicile; a bona fide intention of abandoning the
former place of residence and establishing a new one and definite acts which correspond with
the purpose. In other words, there must basically be animus manendi coupled with animus
non revertendi. The purpose to remain in or at the domicile of choice must be for an indefinite
period of time; the change of residence must be voluntary; and the residence at the place
chosen for the new domicile must be actual.153

Petitioner presented voluminous evidence showing that she and her family abandoned their
U.S. domicile and relocated to the Philippines for good. These evidence include petitioner's
former U.S. passport showing her arrival on 24 May 2005 and her return to the Philippines
every time she travelled abroad; e-mail correspondences starting in March 2005 to September
2006 with a freight company to arrange for the shipment of their household items weighing
about 28,000 pounds to the Philippines; e-mail with the Philippine Bureau of Animal Industry
inquiring how to ship their dog to the Philippines; school records of her children showing
enrollment in Philippine schools starting June 2005 and for succeeding years; tax identification
card for petitioner issued on July 2005; titles for condominium and parking slot issued in
February 2006 and their corresponding tax declarations issued in April 2006; receipts dated
23 February 2005 from the Salvation Army in the U.S. acknowledging donation of items from
petitioner's family; March 2006 e-mail to the U.S. Postal Service confirming request for
change of address; final statement from the First American Title Insurance Company showing
sale of their U.S. home on 27 April 2006; 12 July 2011 filled-up questionnaire submitted to
the U.S. Embassy where petitioner indicated that she had been a Philippine resident since
May 2005; affidavit from Jesusa Sonora Poe (attesting to the return of petitioner on 24 May
2005 and that she and her family stayed with affiant until the condominium was purchased);
and Affidavit from petitioner's husband (confirming that the spouses jointly decided to
relocate to the Philippines in 2005 and that he stayed behind in the U.S. only to finish some
work and to sell the family home).

The foregoing evidence were undisputed and the facts were even listed by the COMELEC,
particularly in its Resolution in the Tatad, Contreras and Valdez cases.

However, the COMELEC refused to consider that petitioner's domicile had been timely changed
as of 24 May 2005. At the oral arguments, COMELEC Commissioner Arthur Lim conceded the
presence of the first two requisites, namely, physical presence and animus manendi, but
maintained there was no animus non-revertendi.154 The COMELEC disregarded the import of
all the evidence presented by petitioner on the basis of the position that the earliest date that
petitioner could have started residence in the Philippines was in July 2006 when her
application under R.A. No. 9225 was approved by the BI. In this regard, COMELEC relied
on Coquilla v. COMELEC,155 Japzon v. COMELEC156 and Caballero v. COMELEC. 157 During the
oral arguments, the private respondents also added Reyes v. COMELEC.158 Respondents
contend that these cases decree that the stay of an alien former Filipino cannot be counted
until he/she obtains a permanent resident visa or reacquires Philippine citizenship, a visa-free
entry under a balikbayan stamp being insufficient. Since petitioner was still an American
(without any resident visa) until her reacquisition of citizenship under R.A. No. 9225, her stay
from 24 May 2005 to 7 July 2006 cannot be counted.

But as the petitioner pointed out, the facts in these four cases are very different from her
situation. In Coquilla v. COMELEC,159 the only evidence presented was a community tax
certificate secured by the candidate and his declaration that he would be running in the
elections. Japzon v. COMELEC160 did not involve a candidate who wanted to count residence
prior to his reacquisition of Philippine citizenship. With the Court decreeing that residence is
distinct from citizenship, the issue there was whether the candidate's acts after reacquisition
sufficed to establish residence. In Caballero v. COMELEC, 161 the candidate admitted that his
place of work was abroad and that he only visited during his frequent vacations. In Reyes v.
COMELEC,162 the candidate was found to be an American citizen who had not even reacquired
Philippine citizenship under R.A. No. 9225 or had renounced her U.S. citizenship. She was
disqualified on the citizenship issue. On residence, the only proof she offered was a seven-
month stint as provincial officer. The COMELEC, quoted with approval by this Court, said that
"such fact alone is not sufficient to prove her one-year residency."

It is obvious that because of the sparse evidence on residence in the four cases cited by the
respondents, the Court had no choice but to hold that residence could be counted only from
acquisition of a permanent resident visa or from reacquisition of Philippine citizenship. In
contrast, the evidence of petitioner is overwhelming and taken together leads to no other
conclusion that she decided to permanently abandon her U.S. residence (selling the house,
taking the children from U.S. schools, getting quotes from the freight company, notifying the
U.S. Post Office of the abandonment of their address in the U.S., donating excess items to
the Salvation Army, her husband resigning from U.S. employment right after selling the U.S.
house) and permanently relocate to the Philippines and actually re-established her residence
here on 24 May 2005 (securing T.I.N, enrolling her children in Philippine schools, buying
property here, constructing a residence here, returning to the Philippines after all trips abroad,
her husband getting employed here). Indeed, coupled with her eventual application to
reacquire Philippine citizenship and her family's actual continuous stay in the Philippines over
the years, it is clear that when petitioner returned on 24 May 2005 it was for good.

In this connection, the COMELEC also took it against petitioner that she had entered the
Philippines visa-free as a balikbayan. A closer look at R.A. No. 6768 as amended, otherwise
known as the "An Act Instituting a Balikbayan Program," shows that there is no overriding
intent to treat balikbayans as temporary visitors who must leave after one year. Included in
the law is a former Filipino who has been naturalized abroad and "comes or returns to the
Philippines." 163 The law institutes a balikbayan program "providing the opportunity to avail
of the necessary training to enable the balikbayan to become economically self-reliant
members of society upon their return to the country" 164in line with the government's
"reintegration program."165 Obviously, balikbayans are not ordinary transients.

Given the law's express policy to facilitate the return of a balikbayan and help him reintegrate
into society, it would be an unduly harsh conclusion to say in absolute terms that
the balikbayan must leave after one year. That visa-free period is obviously granted him to
allow him to re-establish his life and reintegrate himself into the community before he attends
to the necessary formal and legal requirements of repatriation. And that is exactly what
petitioner did - she reestablished life here by enrolling her children and buying property while
awaiting the return of her husband and then applying for repatriation shortly thereafter.

No case similar to petitioner's, where the former Filipino's evidence of change in domicile is
extensive and overwhelming, has as yet been decided by the Court. Petitioner's evidence of
residence is unprecedented. There is no judicial precedent that comes close to the facts of
residence of petitioner. There is no indication in Coquilla v. COMELEC,166 and the other cases
cited by the respondents that the Court intended to have its rulings there apply to a situation
where the facts are different. Surely, the issue of residence has been decided particularly on
the facts-of-the case basis.

To avoid the logical conclusion pointed out by the evidence of residence of petitioner, the
COMELEC ruled that petitioner's claim of residence of ten (10) years and eleven (11) months
by 9 May 2016 in her 2015 COC was false because she put six ( 6) years and six ( 6) months
as "period of residence before May 13, 2013" in her 2012 COC for Senator. Thus, according
to the COMELEC, she started being a Philippine resident only in November 2006. In doing so,
the COMELEC automatically assumed as true the statement in the 2012 COC and the 2015
COC as false.

As explained by petitioner in her verified pleadings, she misunderstood the date required in
the 2013 COC as the period of residence as of the day she submitted that COC in 2012. She
said that she reckoned residency from April-May 2006 which was the period when the U.S.
house was sold and her husband returned to the Philippines. In that regard, she was advised
by her lawyers in 2015 that residence could be counted from 25 May 2005.

Petitioner's explanation that she misunderstood the query in 2012 (period of residence before
13 May 2013) as inquiring about residence as of the time she submitted the COC, is bolstered
by the change which the COMELEC itself introduced in the 2015 COC which is now "period of
residence in the Philippines up to the day before May 09, 2016." The COMELEC would not
have revised the query if it did not acknowledge that the first version was vague.

That petitioner could have reckoned residence from a date earlier than the sale of her U.S.
house and the return of her husband is plausible given the evidence that she had returned a
year before. Such evidence, to repeat, would include her passport and the school records of
her children.

It was grave abuse of discretion for the COMELEC to treat the 2012 COC as a binding and
conclusive admission against petitioner. It could be given in evidence against her, yes, but it
was by no means conclusive. There is precedent after all where a candidate's mistake as to
period of residence made in a COC was overcome by evidence. In Romualdez-Marcos v.
COMELEC,167 the candidate mistakenly put seven (7) months as her period of residence where
the required period was a minimum of one year. We said that "[i]t is the fact of residence,
not a statement in a certificate of candidacy which ought to be decisive in determining whether
or not an individual has satisfied the constitutions residency qualification requirement." The
COMELEC ought to have looked at the evidence presented and see if petitioner was telling the
truth that she was in the Philippines from 24 May 2005. Had the COMELEC done its duty, it
would have seen that the 2012 COC and the 2015 COC both correctly stated
the pertinent period of residency.

The COMELEC, by its own admission, disregarded the evidence that petitioner actually and
physically returned here on 24 May 2005 not because it was false, but only because COMELEC
took the position that domicile could be established only from petitioner's repatriation under
R.A. No. 9225 in July 2006. However, it does not take away the fact that in reality, petitioner
had returned from the U.S. and was here to stay permanently, on 24 May 2005. When she
claimed to have been a resident for ten (10) years and eleven (11) months, she could do so
in good faith.

For another, it could not be said that petitioner was attempting to hide anything. As already
stated, a petition for quo warranto had been filed against her with the SET as early as August
2015. The event from which the COMELEC pegged the commencement of residence,
petitioner's repatriation in July 2006 under R.A. No. 9225, was an established fact to repeat,
for purposes of her senatorial candidacy.

Notably, on the statement of residence of six (6) years and six (6) months in the 2012 COC,
petitioner recounted that this was first brought up in the media on 2 June 2015 by Rep. Tobias
Tiangco of the United Nationalist Alliance. Petitioner appears to have answered the issue
immediately, also in the press. Respondents have not disputed petitioner's evidence on this
point. From that time therefore when Rep. Tiangco discussed it in the media, the stated period
of residence in the 2012 COC and the circumstances that surrounded the statement were
already matters of public record and were not hidden.

Petitioner likewise proved that the 2012 COC was also brought up in the SET petition for quo
warranto. Her Verified Answer, which was filed on 1 September 2015, admitted that she made
a mistake in the 2012 COC when she put in six ( 6) years and six ( 6) months as she
misunderstood the question and could have truthfully indicated a longer period. Her answer
in the SET case was a matter of public record. Therefore, when petitioner accomplished her
COC for President on 15 October 2015, she could not be said to have been attempting to hide
her erroneous statement in her 2012 COC for Senator which was expressly mentioned in her
Verified Answer.

The facts now, if not stretched to distortion, do not show or even hint at an intention to hide
the 2012 statement and have it covered by the 2015 representation. Petitioner, moreover,
has on her side this Court's pronouncement that:

Concededly, a candidate's disqualification to run for public office does not necessarily
constitute material misrepresentation which is the sole ground for denying due course to, and
for the cancellation of, a COC. Further, as already discussed, the candidate's
misrepresentation in his COC must not only refer to a material fact (eligibility and
qualifications for elective office), but should evince a deliberate intent to mislead, misinform
or hide a fact which would otherwise render a candidate ineligible. It must be made with an
intention to deceive the electorate as to one's qualifications to run for public office. 168

In sum, the COMELEC, with the same posture of infallibilism, virtually ignored a good number
of evidenced dates all of which can evince animus manendi to the Philippines and animus non
revertedi to the United States of America. The veracity of the events of coming and staying
home was as much as dismissed as inconsequential, the focus having been fixed at the
petitioner's "sworn declaration in her COC for Senator" which the COMELEC said "amounts to
a declaration and therefore an admission that her residence in the Philippines only commence
sometime in November 2006"; such that "based on this declaration, [petitioner] fails to meet
the residency requirement for President." This conclusion, as already shown, ignores the
standing jurisprudence that it is the fact of residence, not the statement of the person that
determines residence for purposes of compliance with the constitutional requirement of
residency for election as President. It ignores the easily researched matter that cases on
questions of residency have been decided favorably for the candidate on the basis of facts of
residence far less in number, weight and substance than that presented by petitioner. 169 It
ignores, above all else, what we consider as a primary reason why petitioner cannot be bound
by her declaration in her COC for Senator which declaration was not even considered by the
SET as an issue against her eligibility for Senator. When petitioner made the declaration in
her COC for Senator that she has been a resident for a period of six (6) years and six (6)
months counted up to the 13 May 2013 Elections, she naturally had as reference the residency
requirements for election as Senator which was satisfied by her declared years of residence.
It was uncontested during the oral arguments before us that at the time the declaration for
Senator was made, petitioner did not have as yet any intention to vie for the Presidency in
2016 and that the general public was never made aware by petitioner, by word or action, that
she would run for President in 2016. Presidential candidacy has a length-of-residence different
from that of a senatorial candidacy. There are facts of residence other than that which was
mentioned in the COC for Senator. Such other facts of residence have never been proven to
be false, and these, to repeat include:
[Petitioner] returned to the Philippines on 24 May 2005. (petitioner's] husband however
stayed in the USA to finish pending projects and arrange the sale of their family home.

Meanwhile [petitioner] and her children lived with her mother in San Juan City. [Petitioner]
enrolled Brian in Beacon School in Taguig City in 2005 and Hanna in Assumption College in
Makati City in 2005. Anika was enrolled in Learning Connection in San Juan in 2007, when
she was already old enough to go to school.

In the second half of 2005, [petitioner] and her husband acquired Unit 7F of One Wilson Place
Condominium in San Juan. [Petitioner] and her family lived in Unit 7F until the construction
of their family home in Corinthian Hills was completed.

Sometime in the second half of 2005, [petitioner's] mother discovered that her former lawyer
who handled [petitioner's] adoption in 1974 failed to secure from the Office of the Civil
Registrar of Iloilo a new Certificate of Live Birth indicating [petitioner's] new name and stating
that her parents are "Ronald Allan K. Poe" and "Jesusa L. Sonora."

In February 2006, [petitioner] travelled briefly to the US in order to supervise the disposal of
some of the family's remaining household belongings.1a\^/phi1 [Petitioner] returned to the
Philippines on 11 March 2006.

In late March 2006, [petitioner's] husband informed the United States Postal Service of the
family's abandonment of their address in the US.

The family home in the US was sole on 27 April 2006.

In April 2006, [petitioner's] husband resigned from his work in the US. He returned to the
Philippines on 4 May 2006 and began working for a Philippine company in July 2006.

In early 2006, [petitioner] and her husband acquired a vacant lot in Corinthian Hills, where
they eventually built their family home.170

In light of all these, it was arbitrary for the COMELEC to satisfy its intention to let the case
fall under the exclusive ground of false representation, to consider no other date than that
mentioned by petitioner in her COC for Senator.

All put together, in the matter of the citizenship and residence of petitioner for her candidacy
as President of the Republic, the questioned Resolutions of the COMELEC in Division and En
Banc are, one and all, deadly diseased with grave abuse of discretion from root to fruits.

WHEREFORE, the petition is GRANTED. The Resolutions, to wit:

1. dated 1 December 2015 rendered through the COMELEC Second Division, in SPA No. 15-
001 (DC), entitled Estrella C. Elamparo, petitioner, vs. Mary Grace Natividad Sonora Poe-
Llamanzares, respondent, stating that:

[T]he Certificate of Candidacy for President of the Republic of the Philippines in the May 9,
2016 National and Local Elections filed by respondent Mary Grace Natividad Sonora Poe-
Llamanzares is hereby GRANTED.
2. dated 11 December 2015, rendered through the COMELEC First Division, in the consolidated
cases SPA No. 15-002 (DC) entitled Francisco S. Tatad, petitioner, vs. Mary Grace Natividad
Sonora Poe-Llamanzares, respondent; SPA No. 15-007 (DC) entitled Antonio P. Contreras,
petitioner, vs. Mary Grace Natividad Sonora Poe-Llamanzares, respondent; and SPA No. 15-
139 (DC) entitled Amado D. Valdez, petitioner, v. Mary Grace Natividad Sonora Poe-
Llamanzares, respondent; stating that:

WHEREFORE, premises considered, the Commission RESOLVED, as it hereby RESOLVES, to


GRANT the petitions and cancel the Certificate of Candidacy of MARY GRACE NATIVIDAD
SONORA POE-LLAMANZARES for the elective position of President of the Republic of the
Philippines in connection with the 9 May 2016 Synchronized Local and National Elections.

3. dated 23 December 2015 of the COMELEC En Banc, upholding the 1 December 2015
Resolution of the Second Division stating that:

WHEREFORE, premises considered, the Commission RESOLVED, as it hereby RESOLVES, to


DENY the Verified Motion for Reconsideration of SENATOR MARY GRACE NATIVIDAD SONORA
POE-LLAMANZARES. The Resolution dated 11 December 2015 of the Commission First
Division is AFFIRMED.

4. dated 23 December 2015 of the COMELEC En Banc, upholding the 11 December 2015
Resolution of the First Division.

are hereby ANNULED and SET ASIDE. Petitioner MARY GRACE NATIVIDAD SONORA POE-
LLAMANZARES is DECLARED QUALIFIED to be a candidate for President in the National and
Local Elections of 9 May 2016.

SO ORDERED.
G.R. No. 221538, September 20, 2016

RIZALITO Y. DAVID, Petitioner, v. SENATE ELECTORAL TRIBUNAL AND MARY GRACE


POE-LLAMANZARES, Respondents.

DECISION

LEONEN, J.:

The words of our most fundamental law cannot be read so as to callously exclude all foundlings
from public service.

When the names of the parents of a foundling cannot be discovered despite a diligent search,
but sufficient evidence is presented to sustain a reasonable inference that satisfies the
quantum of proof required to conclude that at least one or both of his or her parents is Filipino,
then this should be sufficient to establish that he or she is a natural-born citizen. When these
inferences are made by the Senate Electoral Tribunal in the exercise of its sole and exclusive
prerogative to decide the qualifications of the members of the Senate, then there is no grave
abuse of discretion remediable by either Rule 65 of the Rules of Court or Article VIII, Section
I of the Constitution.

This case certainly does not decide with finality the citizenship of every single foundling as
natural-born. The circumstances of each case are unique, and substantial proof may exist to
show that a foundling is not natural-born. The nature of the Senate Electoral Tribunal and its
place in the scheme of political powers, as devised by the Constitution, are likewise different
from the other ways to raise questions of citizenship.

Before this Court is a Petition for Certiorari 1 filed by petitioner Rizalito Y. David (David). He
prays for the nullification of the assailed November 17, 2015 Decision and December 3, 2015
Resolution of public respondent Senate Electoral Tribunal in SET Case No. 001-15.2 The
assailed November 17, 2015 Decision3 dismissed the Petition for Quo Warranto filed by David,
which sought to unseat private respondent Mary Grace Poe-Llamanzares as a Senator for
allegedly not being a natural-born citizen of the Philippines and, therefore, not being qualified
to hold such office under Article VI, Section 34 of the 1987 Constitution. The assailed
December 3, 2015 Resolution5 denied David's Motion for Reconsideration.

Senator Mary Grace Poe-Llamanzares (Senator Poe) is a foundling whose biological parents
are unknown. As an infant, she was abandoned at the Parish Church of Jaro, Iloilo. 6 Edgardo
Militar found her outside the church on September 3, 1968 at about 9:30 a.m. 7 He later turned
her over to Mr. and Mrs. Emiliano Militar. 8 Emiliano Militar reported to the Office of the Local
Civil Registrar that the infant was found on September 6, 1968. 9 She was given the name
Mary Grace Natividad Contreras Militar.10 Local Civil Registrar issued a Certificate of Live
Birth/Foundling Certificate stating:ChanRoblesVirtualawlibrary

Circumstances: THE SUBJECT CHILD WAS FOUND IN THE PARISH CHURCHD [sic] OF JARO,
ON SEPTEMBER 3, 1968 AT ABOUT 9:30 A.M. BY EDGARDO MILITAR AND THE SAID CHILD
IS PRESENTLY IN THE CUSTODY OF MR. AND MRS. EMILIANO MILITAR AT STA. ISABEL
STREET, JARO . . .11chanroblesvirtuallawlibrary
On May 13, 1974, the Municipal Court of San Juan, Rizal promulgated the Decision granting
the Petition for Adoption of Senator Poe by Spouses Ronald Allan Poe (more popularly known
as Fernando Poe, Jr.) and Jesusa Sonora Poe (more popularly known as Susan Roces).12 The
Decision also ordered the change in Senator Poe's name from Mary Grace Natividad Contreras
Militar to Mary Grace Natividad Sonora Poe. 13 October 27, 2005, Clerk of Court III Eleanor A.
Sorio certified that the Decision had become final in a Certificate of Finality.14chanrobleslaw

On April 11, 1980, the Office of Civil Registrar-Iloilo received the Decision of the San Juan
Court Municipal Court and noted on Senator Poe's foundling certificate that she was adopted
by Spouses Ronald Allan and Jesusa Poe.15 This hand-written notation appears on Senator
Poe's foundling certificate:ChanRoblesVirtualawlibrary
NOTE: Adopted child by the Spouses Ronald Allan Poe and Jesusa Sonora Poe as per Court
Order, Mun. Court, San Juan, Rizal, by Hon. Judge Alfredo M. Gorgonio dated May 13, 1974,
under Sp. Proc. No. 138.16chanroblesvirtuallawlibrary
Senator Poe became a registered voter in Greenhills, San Juan, Metro Manila when she turned
18 years old.17 The Commission on Elections issued her a Voter's Identification Card for
Precinct No. 196, Greenhills, San Juan, Metro Manila on December 13, 1986. 18chanrobleslaw

On April 4, 1988, the Department of Foreign Affairs issued her a Philippine passport. 19 Her
passport was renewed on April 5, 1993, May 19, 1998, October 13, 2009, December 19,
2013, and March 18, 2014.20Having become Senator, she was also issued a Philippine
diplomatic passport on December 19, 2013.21chanrobleslaw

Senator Poe took Development Studies at the University of the Philippines, Manila, but
eventually went to the United States in 1988 to obtain her college degree. 22 In 1991, she
earned a bachelor's degree in Political Science from Boston College, Chestnut Hill,
Massachusetts.23chanrobleslaw

On July 27, 1991, Senator Poe married Teodoro Misael Daniel V. Llamanzares, both an
American and Filipino national since birth.24 The marriage took place in Sanctuario de San
Jose Parish, San Juan, Manila.25cralawred On July 29, 1991, Senator Poe returned to the
United States with her husband.26 For some time, she lived with her husband and children in
the United States.27chanrobleslaw

Senator Poe and her husband had three (3) children: Brian Daniel (Brian), Hanna MacKenzie
(Hanna), and Jesusa Anika (Anika).28 Brian was born in the United States on April 16, 1992.
Hanna was born on July 10, 1998, and Anika on June 5, 2004. Both Hanna and Anika were
born in the Philippines.29chanrobleslaw

Senator Poe was naturalized and granted American citizenship on October 18, 2001. 30 She
was subsequently given a United States passport.31chanrobleslaw

Senator Poe's adoptive father, Fernando Poe, Jr., ran for President of the Republic of the
Philippines in the 2004 National Elections.32 To support her father's candidacy, Senator Poe
and her daughter Hanna returned to the Philippines on April 8, 2004.33 After the Elections,
she returned to the United States on July 8, 2004. 34 It was during her stay in the Philippines
that she gave birth to her youngest daughter, Anika. 35chanrobleslaw

Fernando Poe, Jr. was hospitalized on December 11, 2004 and eventually "slipped into a
coma."36Senator Poe returned to the Philippines on December 13, 2004. 37 On December 14,
2004, her father died.38 She stayed in the country until February 3, 2005 to attend her father's
funeral and to attend to the settling of his estate. 39chanrobleslaw

In 2004, Senator Poe resigned from work in the United States. She never looked for work
again in the United States.40chanrobleslaw
Senator Poe decided to return home in 2005.41 After consulting her children, they all agreed
to return to the Philippines to support the grieving Susan Roces. 42 In early 2005, they notified
Brian and Hanna's schools Virginia, United States that they would be transferring to the
Philippines the following semester.43She came back on May 24, 2005.44 Her children also
arrived in the first half of 2005.45 However, her husband stayed in the United States to "finish
pending projects, and to arrange for the sale of the family home there." 46chanrobleslaw

Following her return, Senator Poe was issued by the Bureau of Internal Revenue a Tax
Identification Number (TIN) on July 22, 2005.47chanrobleslaw

On July 7, 2006, Senator Poe took the Oath of Allegiance to Republic of the Philippines: 48
I, Mary Grace Poe Llamanzares, solemnly swear that I will support and defend the Constitution
of the Republic of the Philippines and obey the laws and legal orders promulgated by the duly
constituted authorities of the Philippines; and I hereby declare that I recognize and accept
the supreme authority of the Philippines and will maintain true faith and allegiance thereto;
and that I impose this obligation upon myself voluntarily without mental reservation or
purpose of evasion.49chanroblesvirtuallawlibrary
On July 10, 2006, Senator Poe filed a Petition for Retention and or Re-acquisition of Philippine
Citizenship through Republic Act No. 9225.50 She also "filed applications for derivative
citizenship on behalf of her three children who were all below eighteen (18) years of age at
that time."51chanrobleslaw

The Petition was granted by the Bureau of Immigration and Deportation on July 18, 2006
through an Order signed by Associate Commissioner Roy M. Almoro for Commissioner Alipio
F. Fernandez, Jr:52
A careful review of the documents submitted in support of the instant petition indicate that
David was a former citizen of the Republic of the Philippines being born to Filipino parents
and is presumed to be a natural born Philippine citizen; thereafter, became an American
citizen and is now a holder of an American passport; was issued an ACT and ICR and has
taken her oath of allegiance to the Republic of the Philippines on July 7, 2006 and so is
thereby deemed to have re-acquired her Philippine Citizenship.53 (Emphasis in the original)
In the same Order, Senator Poe's children were "deemed Citizens of the Philippines in
accordance with Section 4 of R[epublic] A[ct] No. 9225." 54 Until now, the Order "has not been
set aside by the Department of Justice or any other agency of Government." 55chanrobleslaw

On July 31, 2006, the Bureau of Immigration issued Identification Certificates in the name of
Senator Poe and her children.56 It stated that Senator Poe is a "citizen of the Philippines
pursuant to the Citizenship Retention and Re-acquisition Act of 2003 . . . in relation to
Administrative Order No. 91, Series of 2004 and Memorandum Circular No. AFF-2-005 per
Office Order No. AFF-06-9133 signed Associate Commissioner Roy M. Almoro dated July 18,
2006."57chanrobleslaw

Senator Poe became a registered voter of Barangay Santa Lucia, San Juan City on August 31,
2006.58chanrobleslaw

Senator Poe made several trips to the United States of America between 2006 and 2009 using
her United States Passport No. 170377935. 59 She used her passport "after having taken her
Oath of Allegiance to the Republic on 07 July 2006, but not after she has formally renounced
her American citizenship on 20 October 2010."60 The following are the flight records given by
the Bureau of Immigration:ChanRoblesVirtualawlibrary
Departures Flight No.
November 1, 2006 SQ071

July 20, 2007 PR730

October 31, 2007 PR300

October 2, 2008 PR358

April 20, 2009 PR104

July 31, 2009 PR730

October 19, 2009 PR102

November 15, 2009 PR103

December 27, 2009 PR112

March 27, 2010 PR102

Arrivals Flight No.

November 4, 2006 SQ076

July 23, 2007 PR731

November 5, 2007 PR337

May 8, 2008 PR103

October 5, 2008 PR359

May 21, 2009 PR105

August 3, 2009 PR733

November 15, 2009 PR10361


On October 6, 2010, President Benigno Simeon Aquino III appointed Senator Poe as
Chairperson of the Movie and Television Review and Classification Board (MTRCB). 62 On
October 20, 2010, Senator Poe executed an Affidavit of Renunciation of Allegiance to the
United States of America and Renunciation of American Citizenship,63 stating:

chanRoblesvirtualLawlibraryI, MARY GRACE POE-LLAMANZARES, Filipino, of legal age, and


presently residing at No. 107 Rodeo Drive, Corinthian Hills, Quezon City, Philippines, after
having been duly sworn to in accordance with the law, do hereby depose and state that with
this affidavit, I hereby expressly and voluntarily renounce my United States
nationality/American citizenship, together with all rights and privileges and all duties and
allegiance and fidelity thereunto pertaining. I make this renunciation intentionally, voluntarily,
and of my own free will, free of any duress or undue influence. 64 (Emphasis in the original)

The affidavit was submitted to the Bureau of Immigration on October 21, 2010. 65 On October
21, 2010, she took her Oath of Office as MTRCB Chairperson and assumed office on October
26, 2010.66 Her oath of office stated:ChanRoblesVirtualawlibrary
PANUNUMPA SA KATUNGKULAN

Ako, si MARY GRACE POE LLAMANZARES, na itinalaga sa katungkulan bilang Chairperson,


Movie and Television Review and Classification Board, ay taimtim na nanunumpa na tutuparin
ko nang buong husay at katapatan, sa abot ng aking kakayahan, ang mga tungkulin ng aking
kasalukuyang katungkulan at ng mga iba pang pagkaraan nito'y gagampanan ko sa ilalim ng
Republika ng Pilipinas; na aking itataguyod at ipagtatanggol ang Saligan Batas ng Pilipinas;
na tunay na mananalig at tatalima ako rito; na susundin ko ang mga batas, mga kautusang
lega, at mga dekretong pinaiiral ng mga sadyang itinakdang may kapangyarihan ng Republika
ng Pilipinas; at kusa kong babalikatin ang pananagutang ito, nang walang ano mang pasubali
o hangaring umiwas.

Kasihan nawa ako ng Diyos.

NILAGDAAN AT PINANUMPAAN sa harap ko ngayong ika-21 ng Oktubre 2010, Lungsod ng


Maynila, Pilipinas.67 (Emphasis in the original)
Senator Poe executed an Oath/Affirmation of Renunciation of Nationality of the United
States68 in the presence of Vice-Consul Somer E. Bessire-Briers on July 12, 2011.69 On this
occasion, she also filled out the Questionnaire Information for Determining Possible Loss of
U.S. Citizenship.70 On December 9, 2011, Vice Consul Jason Galian executed a Certificate of
Loss of Nationality for Senator Poe.71 The certificate was approved by the Overseas Citizen
Service, Department of State, on February 3, 2012. 72chanrobleslaw

Senator Poe decided to run as Senator in the 2013 Elections. 73 On September 27, 2012, she
executed a Certificate of Candidacy, which was submitted to the Commission on Elections on
October 2, 2012.74 She won and was declared as Senator-elect on May 16,
2013.75chanrobleslaw

David, a losing candidate in the 2013 Senatorial Elections, filed before the Senate Electoral
Tribunal a Petition for Quo Warranto on August 6, 2015. 76 He contested the election of Senator
Poe for failing to "comply with the citizenship and residency requirements mandated by the
1987 Constitution."77chanrobleslaw

Thereafter, the Senate Electoral Tribunal issued Resolution No. 15-01 requiring David "to
correct the formal defects of his petition."78 David filed his amended Petition on August 17,
2015.79chanrobleslaw

On August 18, 2015, Resolution No. 15-02 was issued by the Senate Electoral Tribunal,
through its Executive Committee, ordering the Secretary of the Senate Electoral Tribunal to
summon Senator Poe to file an answer to the amended Petition. 80chanrobleslaw

Pending the filing of Senator Poe's answer, David filed a Motion Subpoena the Record of
Application of Citizenship Re-acquisition and related documents from the Bureau of
Immigration on August 25, 2015.81The documents requested included Senator Poe's record
of travels and NSO kept Birth Certificate.82 On August 26, 2015, the Senate Electoral Tribunal
issued Resolution No. 15-04 granting the Motion.83 The same Resolution directed the
Secretary of the Tribunal to issue a subpoena to the concerned officials of the Bureau of
Immigration and the National Statistics Office.84 The subpoenas ordered the officials to appear
on September 1, 2015 at 10:00 a.m. before the Office of the Secretary of the Senate bearing
three (3) sets of the requested documents.85 The subpoenas were complied with by both the
Bureau of Immigration and the National Statistics Office on September 1,
2015.86chanrobleslaw

On September 1, 2015, Senator Poe submitted her Verified Answer with (1) Prayer for
Summary Dismissal; (2) Motion for Preliminary Hearing on Grounds for Immediate
Dismissal/Affirmative Defenses; (3) Motion to Cite David for Direct Contempt of Court; and
(4) Counterclaim for Indirect Contempt of Court.87chanrobleslaw

On September 2, 2015, the Senate Electoral Tribunal issued Resolution No. 15-05 requiring
the parties to file a preliminary conference brief on or before September 9, 2015. 88 The
Resolution also set the Preliminary Conference on September 11, 2015. 89 During the
Preliminary Conference, the parties "agreed to drop the issue of residency on the ground of
prescription."90chanrobleslaw

Oral arguments were held by the Senate Electoral Tribunal on September 21, 2015. 91 The
parties were then "required to submit their respective [memoranda], without prejudice to the
submission of DNA evidence by [Senator Poe] within thirty (30) days from the said
date."92chanrobleslaw

On October 21, 2015, Senator Poe moved to extend for 15 days the submission of DNA test
results.93The Senate Electoral Tribunal granted the Motion on October 27, 2015 through
Resolution No. 15-08.94On November 5, 2015, Senator Poe filed a Manifestation regarding the
results of DNA Testing,95 which stated that "none of the tests that [Senator Poe] took provided
results that would shed light to the real identity of her biological parents." 96 The Manifestation
also stated that Senator Poe was to continue to find closure regarding the issue and submit
any development to the Senate Electoral Tribunal. Later, Senator Poe submitted "the issue of
her natural-born Filipino citizenship as a foundling for resolution upon the legal arguments
set forth in her submissions to the Tribunal."97 On November 6, 2015, through Resolution No.
15-10, the Senate Electoral Tribunal "noted the [M]anifestation and considered the case
submitted for resolution."98chanrobleslaw

On November 17, 2015, the Senate Electoral Tribunal promulgated its assailed Decision
finding Senator Poe to be a natural-born citizen and, therefore, qualified to hold office as
Senator.99 The Decision stated:ChanRoblesVirtualawlibrary
We rule that Respondent is a natural-born citizen under the 1935 Constitution and continue
to be a natural-born citizen as defined under the 1987 Constitution, as she is a citizen of the
Philippines from birth, without having to perform any act to acquire or perfect (her) Philippine
citizenship.

. . . .

In light of our earlier pronouncement that Respondent is a natural-born Filipino citizen,


Respondent validly reacquired her natural-born Filipino citizenship upon taking her Oath of
Allegiance to the Republic of the Philippines, as required under Section 3 of R.A. No. 9225.

Under Section 11 of B.I. Memorandum Circular No. AFF 05-002 (the Revised Rules
Implementing R.A. No. 9225), the foregoing Oath of Allegiance is the "final act" to reacquire
natural-born Philippine citizenship.

. . . .

To repeat, Respondent never used her USA passport from the moment she renounced her
American citizenship on 20 October 2010. She remained solely a natural-born Filipino citizen
from that time on until today.

WHEREFORE, in view of the foregoing, the petition for quo warranto is DISMISSED.

No pronouncement as to costs.

SO ORDERED.100 (Citations omitted)


On November 23, 2015, David moved for reconsideration.101 The Senate Electoral Tribunal
issued Resolution No. 15-11 on November 24, 2015, giving Senator Poe five (5) days to
comment on the Motion for Reconsideration.102chanrobleslaw

Senator Poe filed her Comment/Opposition to the Motion for Reconsideration on December 1,
2015.103David's Motion for Reconsideration was denied by the Senate Electoral Tribunal on
December 3, 2015:104
WHEREFORE, the Tribunal resolves to DENY the Verified Motion for Reconsideration (of the
Decision promulgated on 17 November 2015) of David Rizalito Y. David dated 23 November
2015.

The Tribunal further resolves to CONFIRM Resolution No. 15-11 dated 24 November 2015
issued by the Executive Committee of the Tribunal; to NOTE the Comment/Opposition filed
by counsel for Respondent on 01 December 2015; to GRANT the motion for leave to appear
and submit memorandum as amici curiae filed by Dean Arturo de Castro [and to] NOTE the
Memorandum (for Volunteer Amicus Curiae) earlier submitted by Dean de Castro before the
Commission on Elections in SPA No. 15-139 (DC), entitled "Amado D. Valdez, Petitoner,
versus Mary Grace Natividad Sonora Poe Llaman[z]ares, Respondent."

SO ORDERED.105 (Emphasis in the original)


On December 8, 2015, the Senate Electoral Tribunal's Resolution was received by David.106 On
December 9, 2015, David filed the pre Petition for Certiorari before this
Court.107chanrobleslaw

On December 16, 2015, this Court required the Senate Electoral Tribunal and Senator Poe to
comment on the Petition "within a non-extendible period of fifteen (15) days from
notice."108 The Resolution also set oral arguments on January 19, 2016. 109 The Senate
Electoral Tribunal, through the Office of the Solicitor General, submitted its Comment on
December 30, 2015.110 Senator Poe submitted her Comment on January 4,
2016.111chanrobleslaw

This case was held in abeyance pending the resolution of the Commission on Elections case
on the issue of private respondent's citizenship.

For resolution is the sole issue of whether the Senate Electoral Tribunal committed grave
abuse of discretion amounting to lack or excess of jurisdiction in dismissing petitioner's
Petition for Quo Warranto based on its finding that private respondent is a natural-born Filipino
citizen, qualified to hold a seat as Senator under Article VI, Section 3 of the 1987 Constitution.

Petitioner comes to this Court invoking our power of judicial review through a petition for
certiorari under Rule 65 of the 1997 Rules of Civil Procedure. He seeks to annul the assailed
Decision and Resolution of the Senate Electoral Tribunal, which state its findings and
conclusions on private respondent's citizenship.
Ruling on petitioner's plea for post-judgment relief calls for a consideration of two (2) factors:
first, the breadth of this Court's competence relative to that of the Senate Electoral Tribunal;
and second, the nature of the remedial vehicle—a petition for certiorari—through which one
who is aggrieved by a judgment of the Senate Electoral Tribunal may seek relief from this
Court.

I. A

The Senate Electoral Tribunal, along with the House of Representatives Electoral Tribunal, is
a creation of Article VI, Section 17 of the 1987 Constitution: 112
ARTICLE VI
The Legislative Department

. . . .

SECTION 17. The Senate and the House of Representatives shall each have an Electoral
Tribunal which shall be the sole judge of all contests relating to the election, returns, and
qualifications of their respective Members. Each Electoral Tribunal shall be composed of nine
Members, three of whom shall be Justices of the Supreme Court to be designated by the Chief
Justice, and the remaining six shall be Members of the Senate or the House of
Representatives, as the case may be, who shall be chosen on the basis of proportional
representation from the political parties and the parties or organizations registered under the
party-list system represented therein. The senior Justice in the Electoral Tribunal shall be its
Chairman. (Emphasis supplied)
Through Article VI, Section 17, the Constitution segregates from all other judicial and quasi-
judicial bodies (particularly, courts and the Commission on Elections 113) the power to rule on
contests114 relating to the election, returns, and qualifications of members of the Senate (as
well as of the House of Representatives). These powers are granted to a separate and distinct
constitutional organ. There are two (2) aspects to the exclusivity of the Senate Electoral
Tribunal's power. The power to resolve such contests is exclusive to any other body. The
resolution of such contests is its only task; it performs no other function.

The 1987 Constitution is not the first fundamental law to introduce into our legal system an
"independent, impartial and non-partisan body attached to the legislature and specially
created for that singular purpose."115 The 1935 Constitution similarly created an Electoral
Commission, independent from the National Assembly, to be the sole judge of all contests
relating to members of the National Assembly.116 This was a departure from the system
introduced by prior organic acts enforced under American colonial rule—namely: the Philippine
Bill of 1902 and the Jones Law of 1916—which vested the power to resolve such contests in
the legislature itself. When the 1935 Constitution was amended to make room for a bicameral
legislature, a corresponding amendment was made for there to be separate electoral tribunals
for each chamber of Congress.117 The 1973 Constitution did away with these electoral
tribunals, but they have since been restored by the 1987 Constitution.

All constitutional provisions—under the 1935 and 1987 Constitutions—which provide for the
creation of electoral tribunals (or their predecessor, the Electoral Commission), have been
unequivocal in their language. The electoral tribunal shall be the "sole" judge.

In Lazatin v. House Electoral Tribunal:118


The use of the word "sole" emphasizes the exclusive character of the jurisdiction conferred. .
. . The exercise of the power by the Electoral Commission under the 1935 Constitution has
been described as "intended to be as complete and unimpaired as if it had remained originally
in the legislature[.]" Earlier, this grant of power to the legislature was characterized by Justice
Malcohn as "full, clear and complete." . . . Under the amended 1935 Constitution, the power
was unqualifiedly reposed upon the Electoral Tribunal . . . and it remained as full, clear and
complete as that previously granted the legislature and the Electoral Commission. . . . The
same may be said with regard to the jurisdiction of the Electoral Tribunals under the 1987
Constitution.119chanroblesvirtuallawlibrary
Exclusive, original jurisdiction over contests relating to the election, returns, and
qualifications of the elective officials falling within the scope of their powers is, thus, vested
in these electoral tribunals. It is only before them that post-election challenges against the
election, returns, and qualifications of Senators and Representatives (as well as of the
President and the Vice-President, in the case of the Presidential Electoral Tribunal) may be
initiated.

The judgments of these tribunals are not beyond the scope of any review. Article VI, Section
17's stipulation of electoral tribunals' being the "sole" judge must be read in harmony with
Article VIII, Section 1's express statement that "[j]udicial power includes the duty of the
courts of justice . . . to determine whether or not there has been a grave abuse of discretion
amounting to lack or excess of jurisdiction on the part of any branch or instrumentality of
the Government." Judicial review is, therefore, still possible. In Libanan v. House of
Representatives Electoral Tribunal:120
The Court has stressed that ". . . so long as the Constitution grants the [House of
Representatives Electoral Tribunal] the power to be the sole judge of all contests relating to
the election, returns and qualifications of members of the House of Representatives, any final
action taken by the [House of Representatives Electoral Tribunal] on a matter within its
jurisdiction shall, as a rule, not be reviewed by this Court . . . the power granted to the
Electoral Tribunal . . . excludes the exercise of any authority on the part of this Court that
would in any wise restrict it or curtail it or even affect the same."

The Court did recognize, of course, its power of judicial review in exceptional cases. In Robles
vs. [House of Representatives Electoral Tribunal], the Court has explained that while the
judgments of the Tribunal are beyond judicial interference, the Court may do so, however,
but only "in the exercise of this Court's so-called extraordinary jurisdiction, . . . upon a
determination that the Tribunal's decision or resolution was rendered without or in excess of
its jurisdiction, or with grave abuse of discretion or paraphrasing Morrero, upon a clear
showing of such arbitrary and improvident use by the Tribunal of its power as constitutes a
denial of due process of law, or upon a demonstration of a very clear unmitigated error,
manifestly constituting such grave abuse of discretion that there has to be a remedy for such
abuse."

In the old, but still relevant, case of Morrero vs. Bocar, the Court has ruled that the power of
the Electoral Commission "is beyond judicial interference except, in any event, upon a clear
showing of such arbitrary and improvident use of power as will constitute a denial of due
process." The Court does not, to paraphrase it in Co vs. [House of Representatives Electoral
Tribunal], venture into the perilous area of correcting perceived errors of independent
branches of the Government; it comes in only when it has to vindicate a denial of due process
or correct an abuse of discretion so grave or glaring that no less than the Constitution itself
calls for remedial action.121 (Emphasis supplied, citations omitted)
This Court reviews judgments of the House and Senate Electoral Tribunals not in the exercise
of its appellate jurisdiction. Our review is limited to a determination of whether there has been
an error in jurisdiction, not an error in judgment.
I. B

A party aggrieved by the rulings of the Senate or House Electoral Tribunal invokes the
jurisdiction of this Court through the vehicle of a petition for certiorari under Rule 65 of the
1997 Rules of Civil Procedure. An appeal is a continuation of the proceedings in the tribunal
from which the appeal is taken. A petition for certiorari is allowed in Article VIII, Section 1 of
the Constitution and described in the 1997 Rules of Civil Procedure as an independent civil
action.122 The viability of such a petition is premised on an allegation of "grave abuse of
discretion."123chanrobleslaw

The term "grave abuse of discretion" has been generally held to refer to such arbitrary,
capricious, or whimsical exercise of judgment as is tantamount to lack of
jurisdiction:ChanRoblesVirtualawlibrary
[T]he abuse of discretion must be patent and gross as to amount to an evasion of a positive
duty or a virtual refusal to perform a duty enjoined by law, or to act at all in contemplation
of law, as where the power is exercised in an arbitrary and despotic manner by reason of
passion and hostility. Mere abuse of discretion is not enough: it must be
grave.124chanroblesvirtuallawlibrary
There is grave abuse of discretion when a constitutional organ such as the Senate Electoral
Tribunal or the Commission on Elections, makes manifestly gross errors in its factual
inferences such that critical pieces of evidence, which have been nevertheless properly
introduced by a party, or admitted, or which were the subject of stipulation, are ignored or
not accounted for.125chanrobleslaw

A glaring misinterpretation of the constitutional text or of statutory provisions, as well as a


misreading or misapplication of the current state of jurisprudence, is also considered grave
abuse of discretion.126 The arbitrariness consists in the disregard of the current state of our
law.

Adjudication that fails to consider the facts and evidence or frivolously departs from settled
principles engenders a strong suspicion of partiality. This can be a badge of hostile intent
against a party.

Writs of certiorari have, therefore, been issued: (a) where the tribunal's approach to an issue
is premised on wrong considerations and its conclusions founded on a gross misreading, if not
misrepresentation, of the evidence;127 (b) where a tribunal's assessment of a case is "far from
reasonable[,] [and] based solely on very personal and subjective assessment standards when
the law is replete with standards that can be used";128 "(c) where the tribunal's action on the
appreciation and evaluation of evidence oversteps the limits of its discretion to the point of
being grossly unreasonable";129 and (d) where the tribunal invokes erroneous or irrelevant
considerations in resolving an issue.130chanrobleslaw

I. C

We find no basis for concluding that the Senate Electoral Tribunal acted without or in excess
of jurisdiction, or with grave abuse of discretion amounting to lack or excess of jurisdiction.

The Senate Electoral Tribunal's conclusions are in keeping with a faithful and exhaustive
reading of the Constitution, one that proceeds from an intent to give life to all the aspirations
of all its provisions.

Ruling on the Petition for Quo Warranto initiated by petitioner, the Senate Electoral Tribunal
was confronted with a novel legal question: the citizenship status of children whose biological
parents are unknown, considering that the Constitution, in Article IV, Section 1(2) explicitly
makes reference to one's father or mother. It was compelled to exercise its original jurisdiction
in the face of a constitutional ambiguity that, at that point, was without judicial precedent.

Acting within this void, the Senate Electoral Tribunal was only asked to make a reasonable
interpretation of the law while needfully considering the established personal circumstances
of private respondent. It could not have asked the impossible of private respondent, sending
her on a proverbial fool's errand to establish her parentage, when the controversy before it
arose because private respondent's parentage was unknown and has remained so throughout
her life.

The Senate Electoral Tribunal knew the limits of human capacity. It did not insist on burdening
private respondent with conclusively proving, within the course of the few short months, the
one thing that she has never been in a position to know throughout her lifetime. Instead, it
conscientiously appreciated the implications of all other facts known about her finding.
Therefore, it arrived at conclusions in a manner in keeping with the degree of proof required
in proceedings before a quasi-judicial body: not absolute certainty, not proof beyond
reasonable doubt or preponderance of evidence, but "substantial evidence, or that amount of
relevant evidence which a reasonable mind might accept as adequate to justify a
conclusion."131chanrobleslaw

In the process, it avoided setting a damning precedent for all children with the misfortune of
having been abandoned by their biological parents. Far from reducing them to inferior,
second-class citizens, the Senate Electoral Tribunal did justice to the Constitution's aims of
promoting and defending the well-being of children, advancing human rights, and
guaranteeing equal protection of the laws and equal access to opportunities for public service.

II

Article VI, Section 3 of the 1987 Constitution spells out the requirement that "[n]o person
shall be a Senator unless he [or she] is a natural-born citizen of the Philippines."

Petitioner asserts that private respondent is not a natural-born citizen and, therefore, not
qualified to sit as Senator of the Republic, chiefly on two (2) grounds. First, he argues that as
a foundling whose parents are unknown, private respondent fails to satisfy the jus
sanguinis principle: that is, that she failed to establish her Filipino "blood line," which is
supposedly the essence of the Constitution's determination of who are natural-born citizens
of the Philippines. Proceeding from this first assertion, petitioner insists that as private
respondent was never a natural-born citizen, she could never leave reverted to natural-born
status despite the performance of acts that ostensibly comply with Republic Act No. 9225,
otherwise known as the Citizenship Retention and Re-acquisition Act of 2003.

Petitioner's case hinges on the primacy he places over Article IV, Section 1 of the 1987
Constitution and its enumeration of who are Filipino citizens, more specifically on Section
1(2), which identifies as citizens "[t]hose whose fathers or mothers are citizens of the
Philippines." Petitioner similarly claims that, as private respondent's foundling status is
settled, the burden to prove Filipino parentage was upon her. With private respondent having
supposedly failed to discharge this burden, the supposed inevitable conclusion is that she is
not a natural-born Filipino.

III
At the heart of this controversy is a constitutional ambiguity. Definitely, foundlings have
biological parents, either or both of whom can be Filipinos. Yet, by the nature of their being
foundlings, they may, at critical times, not know their parents. Thus, this controversy must
consider possibilities where parentage may be Filipino but, due to no fault of the foundling,
remains unknown.132 Resolving this controversy hinges on constitutional interpretation.

Discerning constitutional meaning is an exercise in discovering the sovereign's purpose so as


to identify which among competing interpretations of the same text is the more
contemporarily viable construction. Primarily, the actual words—text—and how they are
situated within the whole document—context—govern. Secondarily, when discerning meaning
from the plain text (i.e., verba legis) fails, contemporaneous construction may settle what is
more viable. Nevertheless, even when a reading of the plain text is already sufficient,
contemporaneous construction may still be resorted to as a means for verifying or validating
the clear textual or contextual meaning of the Constitution.

III. A

The entire exercise of interpreting a constitutional provision must necessarily begin with the
text itself. The language of the provision being interpreted is the principal source from which
this Court determines constitutional intent.133chanrobleslaw

To the extent possible, words must be given their ordinary meaning; this is consistent with
the basic precept of verba legis.134 The Constitution is truly a public document in that it was
ratified and approved by a direct act of the People exercising their right of suffrage, they
approved of it through a plebiscite. The preeminent consideration in reading the Constitution,
therefore, is the People's consciousness: that is, popular, rather than technical-legal,
understanding. Thus:ChanRoblesVirtualawlibrary
We look to the language of the document itself in our search for its meaning. We do not of
course stop there, but that is where we begin. It is to be assumed that the words in which
constitutional provisions are couched express the objective sought to be attained. They are
to be given their ordinary meaning except where technical terms are employed in which case
the significance thus attached to them prevails. As the Constitution is not primarily a lawyer's
document, it being essential for the rule of law to obtain that it should ever be present in the
people's consciousness, its language as much as possible should be understood in the sense
they have in common use. What it says according to the text of the provision to be construed
compels acceptance and negates the power of the courts to alter it, based on the postulate
that the framers and the people mean what they say. Thus, these are the cases where the
need for construction is reduced to a minimum.135(Emphasis supplied)
Reading a constitutional provision requires awareness of its relation with the whole of the
Constitution. A constitutional provision is but a constituent of a greater whole. It is the
framework of the Constitution that animates each of its components through the dynamism
of these components' interrelations. What is called into operation is the entire document, not
simply a peripheral item. The Constitution should, therefore, be appreciated and read as a
singular, whole unit—ut magis valeat quam pereat.136 Each provision must be understood and
effected in a way that gives life to all that the Constitution contains, from its foundational
principles to its finest fixings.137chanrobleslaw

The words and phrases that establish its framework and its values color each provision at the
heart of a controversy in an actual case. In Civil Liberties Union v. Executive Secretary:138
It is a well-established rule in constitutional construction that no one provision of the
Constitution is to be separated from all the others, to be considered alone, but that all the
provisions bearing upon a particular subject are to be brought into view and to be so
interpreted as to effectuate the great purposes of the instrument. Sections bearing on a
particular subject should be considered and interpreted together as to effectuate the whole
purpose of the Constitution and one section is not to be allowed to defeat another, if by any
reasonable construction, the two can be made to stand together.

In other words, the court must harmonize them, if practicable, and must lean in favor of
construction which will render every word operative, rather than one which may make the
words idle and nugatory.139 (Citations omitted)
Reading a certain text includes a consideration of jurisprudence that has previously considered
that exact same text, if any. Our legal system is founded on the basic principle that "judicial
decisions applying or interpreting the laws or the Constitution shall form part of [our] legal
system."140 Jurisprudence is not an independent source of law. Nevertheless, judicial
interpretation is deemed part of or written into the text itself as of the date that it was
originally passed. This is because judicial construction articulates the contemporaneous intent
that the text brings to effect.141 Nevertheless, one must not fall into the temptation of
considering prior interpretation as immutable.

Interpretation grounded on textual primacy likewise looks into how the text has evolved.
Unless completely novel, legal provisions are the result of the re-adoption—often with
accompanying re-calibration—of previously existing rules. Even when seemingly novel,
provisions are often introduced as a means of addressing the inadequacies and excesses of
previously existing rules.

One may trace the historical development of text by comparing its current iteration with prior
counterpart provisions, keenly taking note of changes in syntax, along with accounting for
more conspicuous substantive changes such as the addition and deletion of provisos or items
in enumerations, shifting terminologies, the use of more emphatic or more moderate
qualifiers, and the imposition of heavier penalties. The tension between consistency and
change galvanizes meaning.

Article IV, Section 1 of the 1987 Constitution, which enumerates who are citizens of the
Philippines, may be compared with counterpart provisions, not only in earlier Constitutions
but even in organic laws142and in similar mechanisms143 introduced by colonial rulers whose
precepts nevertheless still resonate today.

Even as ordinary meaning is preeminent, a realistic appreciation of legal interpretation must


grapple with the truth that meaning is not always singular and uniform. In Social Weather
Stations, Inc. v. Commission on Elections,144 this Court explained the place of a holistic
approach in legal interpretation:ChanRoblesVirtualawlibrary
Interestingly, both COMELEC and petitioners appeal to what they (respectively) construe to
be plainly evident from Section 5.2(a)'s text on the part of COMELEC, that the use of the
words "paid for" evinces no distinction between direct purchasers and those who purchase via
subscription schemes; and, on the part of petitioners, that Section 5.2(a)'s desistance from
actually using the word "subscriber" means that subscribers are beyond its contemplation.
The variance in the parties' positions, considering that they are both banking on what they
claim to be the Fair Election Act's plain meaning, is the best evidence of an extant ambiguity.

Second, statutory construction cannot lend itself to pedantic rigor that foments absurdity. The
dangers of inordinate insistence on literal interpretation are commonsensical and need not be
belabored. These dangers are by no means endemic to legal interpretation. Even in everyday
conversations, misplaced literal interpretations are fodder for humor. A fixation on technical
rules of grammar is no less innocuous. A pompously doctrinaire approach to text can stifle,
rather than facilitate, the legislative wisdom that unbridled textualism purports to bolster.

Third, the assumption that there is, in all cases, a universal plain language is erroneous. In
reality, universality and uniformity in meaning is a rarity. A contrary belief wrongly assumes
that language is static.

The more appropriate and more effective approach is, thus, holistic rather than
parochial: to consider context and the interplay of the historical, the contemporary,
and even the envisioned. Judicial interpretation entails the convergence of social realities
and social ideals. The latter are meant to be effected by the legal apparatus, chief of which is
the bedrock of the prevailing legal order: the Constitution. Indeed, the word in the vernacular
that describes the Constitution — saligan — demonstrates this imperative of constitutional
primacy.

Thus, we refuse to read Section 5.2(a) of the Fair Election Act in isolation. Here, we consider
not an abstruse provision but a stipulation that is part of the whole, i.e., the statute of which
it is a part, that is aimed at realizing the ideal of fair elections. We consider not a cloistered
provision but a norm that should have a present authoritative effect to achieve the ideals of
those who currently read, depend on, and demand fealty from the Constitution. 145 (Emphasis
supplied)
III. B

Contemporaneous construction and aids that are external to the text may be resorted to when
the text is capable of multiple, viable meanings.146 It is only then that one can go beyond the
strict boundaries of the document. Nevertheless, even when meaning has already been
ascertained from a reading of the plain text, contemporaneous construction may serve to
verify or validate the meaning yielded by such reading.

Limited resort to contemporaneous construction is justified by the realization that the business
of understanding the Constitution is not exclusive to this Court. The basic democratic
foundation of our constitutional order necessarily means that all organs of government, and
even the People, read the fundamental law and are guided by it. When competing viable
interpretations arise, a justiciable controversy may ensue requiring judicial intervention in
order to arrive with finality at which interpretation shall be sustained. To remain true to its
democratic moorings, however, judicial involvement must remain guided by a framework or
deference and constitutional avoidance. This same principle underlies the basic doctrine that
courts are to refrain from issuing advisory opinions. Specifically as regards this Court, only
constitutional issues that are narrowly framed, sufficient to resolve an actual case, may be
entertained.147chanrobleslaw

When permissible then, one may consider analogous jurisprudence (that is, judicial decisions
on similar, but not the very same, matters or concerns), 148 as well as thematically similar
statutes and international norms that form part of our legal system. This includes discerning
the purpose and aims of the text in light of the specific facts under consideration. It is also
only at this juncture—when external aids may be consulted—that the supposedly underlying
notions of the framers, as articulated through records of deliberations and other similar
accounts, can be illuminating.

III. C

In the hierarchy of the means for constitutional interpretation, inferring meaning from the
supposed intent of the framers or fathoming the original understanding of the individuals who
adopted the basic document is the weakest approach.

These methods leave the greatest room for subjective interpretation. Moreover, they allow
for the greatest errors. The alleged intent of the framers is not necessarily encompassed or
exhaustively articulated in the records of deliberations. Those that have been otherwise silent
and have not actively engaged in interpellation and debate may have voted for or against a
proposition for reasons entirely their own and not necessarily in complete agreement with
those articulated by the more vocal. It is even possible that the beliefs that motivated them
were based on entirely erroneous premises. Fathoming original understanding can also
misrepresent history as it compels a comprehension of actions made within specific historical
episodes through detached, and not necessarily better-guided, modern lenses.

Moreover, the original intent of the framers of the Constitution is not always uniform with the
original understanding of the People who ratified it. In Civil Liberties
Union:ChanRoblesVirtualawlibrary
While it is permissible in this jurisdiction to consult the debates and proceedings of the
constitutional convention in order to arrive at the reason and purpose of the resulting
Constitution, resort thereto may be had only when other guides fail as said proceedings are
powerless to vary the terms of the Constitution when the meaning is clear. Debates in the
constitutional convention "are of value as showing the views of the individual members, and
as indicating the reasons for their votes, but they give us no light as to the views of the large
majority who did not talk, much less of the mass of our fellow citizens whose votes at the
polls gave the instrument the force of fundamental law. We think it safer to construe the
constitution from what appears upon its face." The proper interpretation therefore depends
more on how it was understood by the people adopting it than in the framer's understanding
thereof.149 (Emphasis supplied)
IV

Though her parents are unknown, private respondent is a Philippine citizen without the need
for an express statement in the Constitution making her so. Her status as such is but the
logical consequence of a reasonable reading of the Constitution within its plain text. The
Constitution provides its own cues; there is not even a need to delve into the deliberations of
its framers and the implications of international legal instruments. This reading proceeds from
several levels.

On an initial level, a plain textual reading readily identifies the specific provision, which
principally governs: the Constitution's actual definition, in Article IV, Section 2, of "natural-
born citizens." This definition must be harmonized with Section 1's enumeration, which
includes a reference to parentage. These provisions must then be appreciated in relation to
the factual milieu of this case. The pieces of evidence before the Senate Electoral Tribunal,
admitted facts, and uncontroverted circumstances adequately justify the conclusion of private
respondent's Filipino parentage.

On another level, the assumption should be that foundlings are natural-born unless there is
substantial evidence to the contrary. This is necessarily engendered by a complete
consideration of the whole Constitution, not just its provisions on citizenship. This includes its
mandate of defending the well-being of children, guaranteeing equal protection of the law,
equal access to opportunities for public service, and respecting human rights, as well as its
reasons for requiring natural-born status for select public offices. Moreover, this is a reading
validated by contemporaneous construction that considers related legislative enactments,
executive and administrative actions, and international instruments.
V

Private respondent was a Filipino citizen at birth. This status' commencement from birth
means that private respondent never had to do anything to consummate this status. By
definition, she is natural-born. Though subsequently naturalized, she reacquired her natural-
born status upon satisfying the requirement of Republic Act No. 9225. Accordingly, she is
qualified to hold office as Senator of the Republic.

V. A

Article IV, Section 1 of the 1987 Constitution enumerates who are citizens of the
Philippines:ChanRoblesVirtualawlibrary
Section 1. The following are citizens of the Philippines:

chanRoblesvirtualLawlibrary

(1) Those who are citizens of the Philippines at the time of the adoption of this
Constitution;

(2) Those whose fathers or mothers are citizens of the Philippines;

(3) Those born before January 17, 1973, of Filipino mothers, who elect Philippine
citizenship upon reaching the age of majority; and

(4) Those who are naturalized in accordance with law.150


Article IV, Section 2 identifies who are natural-born citizens:ChanRoblesVirtualawlibrary
Sec. 2. Natural-born citizens are those who are citizens of the Philippines from birth
without having to perform any act to acquire or perfect their Philippine citizenship.
Those who elect Philippine citizenship in accordance with paragraph (3), Section 1 hereof shall
be deemed natural-born citizens. (Emphasis supplied)
Section 2's significance is self-evident. It provides a definition of the term "natural-born
citizens." This is distinct from Section 1's enumeration of who are citizens. As against Section
1's generic listing, Section 2 specifically articulates those who may count themselves as
natural-born.

The weight and implications of this categorical definition are better appreciated when
supplemented with an understanding of how our concepts of citizenship and natural-born
citizenship have evolved. As will be seen, the term "natural-born citizen" was a transplanted,
but tardily defined, foreign concept.

V. B

Citizenship is a legal device denoting political affiliation. It is the "right to have rights."151 It
is one's personal and . . . permanent membership in a political community. . . The core of
citizenship is the capacity to enjoy political rights, that is, the right to participate in
government principally through the right to vote, the right to hold public office[,] and the
right to petition the government for redress of grievance. 152chanrobleslaw

Citizenship also entails obligations to the political community of which one is


part.153 Citizenship, therefore, is intimately tied with the notion that loyalty is owed to the
state, considering the benefits and protection provided by it. This is particularly so if these
benefits and protection have been enjoyed from the moment of the citizen's birth.
Tecson v. Commission on Elections154 reckoned with the historical development of our concept
of citizenship, beginning under Spanish colonial rule. 155 Under the Spanish, the native
inhabitants of the Philippine Islands were identified not as citizens but as "Spanish
subjects."156 Church records show that native inhabitants were referred to as "indios." The
alternative identification of native inhabitants as subjects or as indios demonstrated the
colonial master's regard for native inhabitants as inferior. 157Natives were, thus, reduced to
subservience in their own land.

Under the Spanish Constitution of 1876, persons born within Spanish territory, not just
peninsular Spain, were considered Spaniards, classification, however, did not extend to the
Philippine Islands, as Article 89 expressly mandated that the archipelago was to be governed
by special laws.158 It was only on December 18, 1889, upon the effectivity in this jurisdiction
of the Civil Code of Spain, that there existed a categorical enumeration of who were Spanish
citizens,159 thus:ChanRoblesVirtualawlibrary
(a) Persons born in Spanish territory,

(b) Children of a Spanish father or mother, even if they were born outside of Spain,

(c) Foreigners who have obtained naturalization papers,

(d) Those who, without such papers, may have become domiciled inhabitants of any town
of the Monarchy.160
1898 marked the end of Spanish colonial rule. The Philippine Islands were ceded by Spain to
the United States of America under the Treaty of Paris, which was entered into on December
10, 1898. The Treaty of Paris did not automatically convert the native inhabitants to American
citizens.161 Instead, it left the determination of the native inhabitants' status to the Congress
of the United States:ChanRoblesVirtualawlibrary
Spanish subjects, natives of the Peninsula, residing in the territory over which Spain by the
present treaty relinquishes or cedes her sovereignty may remain in such territory or may
remove therefrom. . . . In case they remain in the territory they may preserve their allegiance
to the Crown of Spain by making . . . a declaration of their decision to preserve such
allegiance; in default of which declaration they shall be held to have renounced it and to have
adopted the nationality of the territory in which they may reside.

Thus -

The civil rights and political status of the native inhabitants of the territories hereby ceded to
the United States shall be determined by Congress.162chanroblesvirtuallawlibrary
Pending legislation by the United States Congress, the native inhabitants who had ceased to
be Spanish subjects were "issued passports describing them to be citizens of the Philippines
entitled to the protection of the United States." 163chanrobleslaw

The term "citizens of the Philippine Islands" first appeared in legislation in the Philippine
Organic Act, otherwise known as the Philippine Bill of 1902: 164
Section 4. That all inhabitants of the Philippine Islands continuing to reside therein, who were
Spanish subjects on the eleventh day of April, eighteen hundred and ninety-nine, and then
resided in said Islands, and their children born subsequent thereto, shall be deemed and
held to be citizens of the Philippine Islands and as such entitled to the protection of the
United States, except such as shall have elected to preserve their allegiance to the Crown of
Spain in accordance with the provisions of the treaty of peace between the United States and
Spain signed at Paris December tenth, eighteen hundred and ninety-eight. (Emphasis
supplied)
The Philippine Bill of 1902 explicitly covered the status of children born in the Philippine
Islands to its inhabitants who were Spanish subjects as of April 11, 1899. However, it did not
account for the status of children born in the Islands to parents who were not Spanish
subjects. A view was expressed that the common law concept of jus soli (or citizenship by
place of birth), which was operative in the United States, applied to the Philippine
Islands.165chanrobleslaw

On March 23, 1912, the United States Congress amended Section 4 of the Philippine Bill of
1902. It was made to include a proviso for the enactment by the legislature of a law on
acquiring citizenship. This proviso read:ChanRoblesVirtualawlibrary
Provided, That the Philippine Legislature, herein provided for, is hereby authorized to provide
by law for the acquisition of Philippine citizenship by those natives of the Philippine Islands
who do not come within the foregoing provisions, the natives of the insular possessions of the
United States, and such other persons residing in the Philippine Islands who are citizens of
the United States, or who could become citizens of the United States under the laws of the
United States if residing therein.166chanroblesvirtuallawlibrary
In 1916, the Philippine Autonomy Act, otherwise known as the Jones Law of 1916, replaced
the Philippine Bill of 1902. It restated the citizenship provision of the Philippine Bill of 1902,
as amended:167
Section 2.—Philippine Citizenship and Naturalization

That all inhabitants of the Philippine Islands who were Spanish subjects on the eleventh day
of April, eighteen hundred and ninety-nine, and then resided in said Islands, and their children
born subsequent thereto, shall be deemed and held to be citizens of the Philippine Islands,
except such as shall have elected to preserve their allegiance to the Crown of Spain in
accordance with the provisions of the treaty of peace between the United States and Spain,
signed at Paris December tenth, eighteen hundred and ninety-eight, and except such others
as have since become citizens of some other country: Provided, That the Philippine
Legislature, herein provided for, is hereby authorized to provide by law for the acquisition of
Philippine citizenship by those natives of the Philippine Islands who do not come within the
foregoing provisions, the natives of the insular possessions of the United States, and such
other persons residing in the Philippine Islands who are citizens of the United States, or who
could become citizens of the United States under the laws of the United States if residing
therein.
The Jones Law of 1916 provided that a native-born inhabitant of the Philippine Islands was
deemed to be a citizen of the Philippines as of April 11, 1899 if he or she was "(1) a subject
of Spain on April 11, 1899, (2) residing in the Philippines on said date, and (3) since that
date, not a citizen of some other country." 168chanrobleslaw

There was previously the view that jus soli may apply as a mode of acquiring citizenship. It
was the 1935 Constitution that made sole reference to parentage vis-a-vis the determination
of citizenship.169 Article III, Section 1 of the 1935 Constitution
provided:ChanRoblesVirtualawlibrary
SECTION 1. The following are citizens of the Philippines:

chanRoblesvirtualLawlibrary

(1) Those who are citizens of the Philippine Islands at the time of the adoption of this
Constitution.
(2) Those born in the Philippines Islands of foreign parents who, before the adoption of
this Constitution, had been elected to public office in the Philippine Islands.

(3) Those whose fathers are citizens of the Philippines.

(4) Those whose mothers are citizens of the Philippines and upon reaching the age of
majority, elect Philippine citizenship.

(5) Those who are naturalized in accordance with law.


The term "natural-born citizen" first appeared in this jurisdiction in the 1935 Constitution's
provision stipulating the qualifications for President and Vice-President of the Philippines.
Article VII, Section 3 read:ChanRoblesVirtualawlibrary
SECTION 3. No person may be elected to the office of President or Vice-President, unless he
be a natural-born citizen of the Philippines, a qualified voter, forty years of age or over, and
has been a resident of the Philippines for at least ten years immediately preceding the
election.
While it used the term "natural-born citizen," the 1935 Constitution did not define the term.

Article II, Section 1(4) of the 1935 Constitution—read with the then civil law provisions that
stipulated the automatic loss of Filipino citizens lip by women who marry alien husbands—
was discriminatory towards women.170 The 1973 Constitution rectified this problematic
situation:ChanRoblesVirtualawlibrary
SECTION 1. The following are citizens of the Philippines:

chanRoblesvirtualLawlibrary

(1) Those who are citizens of the Philippines at the time of the adoption of this
Constitution.

(2) Those whose fathers or mothers are citizens of the Philippines.

(3) Those who elect Philippine citizenship pursuant to the provisions of the Constitution
of nineteen hundred and thirty-five.

(4) Those who are naturalized in accordance with law.

SECTION 2. A female citizen of the Philippines who marries an alien shall retain her Philippine
citizenship, unless by her act or omission she is deemed, under the law, to have renounced
her citizenship.171chanroblesvirtuallawlibrary
The 1973 Constitution was the first instrument to actually define the term "natural-born
citizen." Article III, Section 4 of the 1973 Constitution provided:ChanRoblesVirtualawlibrary
SECTION 4. A natural-born citizen is one who is a citizen of the Philippines from birth without
having to perform any act to acquire or perfect his Philippine
citizenship.172chanroblesvirtuallawlibrary
The present Constitution adopted most of the provisions of the 1973 Constitution on
citizenship, "except for subsection (3) thereof that aimed to correct the irregular situation
generated by the questionable proviso in the 1935 Constitution." 173chanrobleslaw

Article IV, Section 1 of the 1987 Constitution now reads:ChanRoblesVirtualawlibrary


Section 1. The following are citizens of the Philippines:

chanRoblesvirtualLawlibrary

(1) Those who are citizens of the Philippines at the time of the adoption of this
Constitution;

(2) Those whose fathers or mothers are citizens of the Philippines;

(3) Those born before January 17, 1973, of Filipino mothers, who elect Philippine
citizenship upon reaching the age of majority; and

(4) Those who are naturalized in accordance with law.174


Article IV, Section 2 also calibrated the 1973 Constitution's previous definition of natural-born
citizens, as follows:ChanRoblesVirtualawlibrary
Sec. 2. Natural-born citizens are those who are citizens of the Philippines from birth
without having to perform any act to acquire or perfect their Philippine citizenship.
Those who elect Philippine citizenship in accordance with paragraph (3), Section 1 hereof shall
be deemed natural-born citizens. (Emphasis supplied)
Ironically, the concept of "natural-born" citizenship is a "foreign" concept that was
transplanted into this jurisdiction as part of the 1935 Constitution's eligibility requirements
for President and Vice-President of the Philippines.

In the United States Constitution, from which this concept originated, the term "natural-born
citizen" appears in only a single instance: as an eligibility requirement for the presidency. 175 It
is not defined in that Constitution or in American laws. Its origins and rationale for inclusion
as a requirement for the presidency are not even found in the records of constitutional
deliberations.176 However, it has been suggested that, as the United States was under British
colonial rule before its independence, the requirement of being natural-born was introduced
as a safeguard against foreign infiltration in the administration of national
government:ChanRoblesVirtualawlibrary
It has been suggested, quite plausibly, that this language was inserted in response to a letter
sent by John Jay to George Washington, and probably to other delegates, on July 25, 1787,
which stated:ChanRoblesVirtualawlibrary
Permit me to hint, whether it would be wise and seasonable to provide a strong check to the
admission of Foreigners into the administration of our national Government; and to declare
expressly that the Command in Chief of the American army shall not be given to nor devolve
on, any but a natural born Citizen.
Possibly this letter was motivated by distrust of Baron Von Steuben, who had served valiantly
in the Revolutionary forces, but whose subsequent loyalty was suspected by Jay. Another
theory is that the Jay letter, and the resulting constitutional provision, responded to rumors
that the Convention was concocting a monarchy to be ruled by a foreign
monarch.177chanroblesvirtuallawlibrary
In the United States, however, citizenship is based on jus soli, not jus sanguinis.

V. C

Today, there are only two (2) categories of Filipino citizens: natural-born and naturalized.

A natural-born citizen is defined in Article IV, Section 2 as one who is a citizen of the
Philippines "from birth without having to perform any act to acquire or perfect Philippine
citizenship." By necessary implication, a naturalized citizen is one who is not natural-
born. Bengson v. House of Representatives Electoral Tribunal 178 articulates this definition by
dichotomy:ChanRoblesVirtualawlibrary
[O]nly naturalized Filipinos are considered not natural-born citizens. It is apparent from the
enumeration of who are citizens under the present Constitution that there are only two classes
of citizens: . . . A citizen who is not a naturalized Filipino, i.e., did not have to undergo the
process of naturalization to obtain Philippine citizenship, necessarily is a natural-born
Filipino.179chanroblesvirtuallawlibrary
Former Associate Justice Artemio Panganiban further shed light on the concept of naturalized
citizens in his Concurring Opinion in Bengson: naturalized citizens, he stated, are "former
aliens or foreigners who had to undergo a rigid procedure, in which they had to adduce
sufficient evidence to prove that they possessed all the qualifications and none of the
disqualifications provided by law in order to become Filipino citizens." 180chanrobleslaw

One who desires to acquire Filipino citizenship by naturalization is generally required to file a
verified petition.181 He or she must establish. among others, that he or she is of legal age, is
of good moral character, and has the capacity to adapt to Filipino culture, tradition, and
principles, or otherwise has resided in the Philippines for a significant period of
time.182 Further, the applicant must show that he or she will not be a threat to the state, to
the public, and to the Filipinos' core beliefs.183chanrobleslaw

V. D

Article IV, Section 1 of the 1987 Constitution merely gives an enumeration. Section 2
categorically defines "natural-born citizens." This constitutional definition is further clarified
in jurisprudence, which delineates natural-born citizenship from naturalized citizenship.
Consistent with Article 8 of the Civil Code, this jurisprudential clarification is deemed written
into the interpreted text, thus establishing its contemporaneous intent.

Therefore, petitioner's restrictive reliance on Section 1 and the need to establish bloodline is
misplaced. It is inordinately selective and myopic. It divines Section 1's mere enumeration
but blatantly turns a blind eye to the succeeding Section's unequivocal definition.

Between Article IV, Section 1(2), which petitioner harps on, and Section 2, it is Section 2 that
is on point. To determine whether private respondent is a natural-born citizen, we must look
into whether she had to do anything to perfect her citizenship. In view of Bengson, this calls
for an inquiry into whether she underwent the naturalization process to become a Filipino.

She did not.

At no point has it been substantiated that private respondent went through the actual
naturalization process. There is no more straightforward and more effective way to terminate
this inquiry than this realization of total and utter lack of proof.

At most, there have been suggestions likening a preferential approach to foundlings, as well
as compliance with Republic Act No. 9225, with naturalization. These attempts at analogies
are misplaced. The statutory mechanisms for naturalization are clear, specific, and narrowly
devised. The investiture of citizenship on foundlings benefits children, individuals whose
capacity to act is restricted.184 It is a glaring mistake to liken them to an adult filing before
the relevant authorities a sworn petition seeking to become a Filipino, the grant of which is
contingent on evidence that he or she must himself or herself adduce. As shall later be
discussed, Republic Act No. 9225 is premised on the immutability of natural-born status. It
privileges natural-born citizens and proceeds from an entirely different premise from the
restrictive process of naturalization.

So too, the jurisprudential treatment of naturalization vis-a-vis natural-born status is clear.


It should be with the actual process of naturalization that natural-born status is to be
contrasted, not against other procedures relating to citizenship. Otherwise, the door may be
thrown open for the unbridled diminution of the status of citizens.

V. E

Natural-born citizenship is not concerned with being a human thoroughbred.

Section 2 defines "natural-born citizens." Section 1(2) stipulates that to be a citizen, either
one's father or one's mother must be a Filipino citizen.

That is all there is to Section 1(2). Physical features, genetics, pedigree, and ethnicity are not
determinative of citizenship.

Section 1(2) does not require one's parents to be natural-born Filipino citizens. It does not
even require them to conform to traditional conceptions of what is indigenously or ethnically
Filipino. One or both parents can, therefore, be ethnically foreign.

Section 1(2) requires nothing more than one ascendant degree: parentage. The citizenship of
everyone else in one's ancestry is irrelevant. There is no need, as petitioner insists, for a pure
Filipino bloodline.

Section 1(2) requires citizenship, not identity. A conclusion of Filipino citizenship may be
sustained by evidence adduced in a proper proceeding, which substantially proves that either
or both of one's parents is a Filipino citizen.

V. F

Private respondent has done this. The evidence she adduced in these proceedings attests to
how at least one—if not both—of her biological parents were Filipino citizens.

Proving private respondent's biological parentage is now practically impossible. To begin with,
she was abandoned as a newborn infant. She was abandoned almost half a century ago. By
now, there are only a handful of those who, in 1968, were able-minded adults who can still
lucidly render testimonies on the circumstances of her birth and finding. Even the
identification of individuals against whom DNA evidence may be tested is improbable, and by
sheer economic cost, prohibitive.

However, our evidentiary rules admit of alternative means for private respondent to establish
her parentage.

In lieu of direct evidence, facts may be proven through circumstantial evidence. In Suerte-
Felipe v. People:185
Direct evidence is that which proves the fact in dispute without the aid of any inference or
presumption; while circumstantial evidence is the proof of fact or facts from which, taken
either singly or collectively, the existence of a particular fact in dispute may be inferred as a
necessary or probable consequence.186chanroblesvirtuallawlibrary
People v. Raganas187 further defines circumstantial evidence:ChanRoblesVirtualawlibrary
Circumstantial evidence is that which relates to a series of facts other than the fact in issue,
which by experience have been found so associated with such fact that in a relation of cause
and effect, they lead us to a satisfactory conclusion.188 (Citation omitted)
Rule 133, Section 4 of the Revised Rules on Evidence, for instance, stipulates when
circumstantial evidence is sufficient to justify a conviction in criminal
proceedings:ChanRoblesVirtualawlibrary
Section 4. Circumstantial evidence, when sufficient. — Circumstantial evidence is sufficient
for conviction if:

chanRoblesvirtualLawlibrary(a) There is more than one circumstances;

(b) The facts from which the inferences are derived are proven; and cralawlawlibrary

(c) The combination of all the circumstances is such as to produce a conviction beyond
reasonable doubt.
Although the Revised Rules on Evidence's sole mention of circumstantial evidence is in
reference to criminal proceedings, this Court has nevertheless sustained the use of
circumstantial evidence in other proceedings.189 There is no rational basis for making the use
of circumstantial evidence exclusive to criminal proceedings and for not considering
circumstantial facts as valid means for proof in civil and/or administrative proceedings.

In criminal proceedings, circumstantial evidence suffices to sustain a conviction (which may


result in deprivation of life, liberty, and property) anchored on the highest standard or proof
that our legal system would require, i.e., proof beyond reasonable doubt. If circumstantial
evidence suffices for such a high standard, so too may it suffice to satisfy the less stringent
standard of proof in administrative and quasi-judicial proceedings such as those before the
Senate Electoral Tribunal, i.e., substantial evidence.190chanrobleslaw

Private respondent was found as a newborn infant outside the Parish Church of Jaro, Iloilo on
September 3, 1968.191 In 1968, Iloilo, as did most—if not all—Philippine provinces, had a
predominantly Filipino population.192 Private respondent is described as having "brown
almond-shaped eyes, a low nasal bridge, straight black hair and an oval-shaped face."193 She
stands at 5 feet and 2 inches tall.194 Further, in 1968, there was no international airport in
Jaro, Iloilo.

These circumstances are substantial evidence justifying an inference that her biological
parents were Filipino. Her abandonment at a Catholic Church is more or less consistent with
how a Filipino who, in 1968, lived in a predominantly religious and Catholic environment,
would have behaved. The absence of an international airport in Jaro, Iloilo precludes the
possibility of a foreigner mother, along with a foreigner father, swiftly and surreptitiously
coming in and out of Jaro, Iloilo just to give birth and leave her offspring there. Though proof
of ethnicity is unnecessary, her physical features nonetheless attest to it.

In the other related case of Poe-Llamanzares v. Commission on Elections,195 the Solicitor


General underscored how it is statistically more probable that private respondent was born a
Filipino citizen rather than as a foreigner. He submitted the following table is support of his
statistical inference:196
NUMBER OF FOREIGN AND FILIPINO CHILDREN BORN IN THE PHILIPPINES: 1965-1975 and
2010-2014
FOREIGN CHILDREN BORN IN THE FILIPINO CHILDREN BORN IN THE
YEAR
PHILIPPINES PHILIPPINES

1965 1,479 795,415

1966 1,437 823,342

1967 1,440 840,302

1968 1,595 898,570

1969 1,728 946,753

1970 1,521 966,762

1971 1,401 963,749

1972 1,784 968,385

1973 1,212 1,045,290

1974 1,496 1,081,873

1975 1,493 1,223,837

2010 1,244 1,782,877

2011 1,140 1,746,685

2012 1,454 1,790,367

2013 1,315 1,751,523

2014 1,351 1,748,782

Source: Philippine Statistics Authority [illegible] 197chanroblesvirtuallawlibrary


Thus, out of the 900,165 recorded births in the Philippines in 1968, only 1,595 or 0.18%
newborns were foreigners. This translates to roughly 99.8% probability that private
respondent was born a Filipino citizen.

Given the sheer difficulty, if not outright impossibility, of identifying her parents after half a
century, a range of substantive proof is available to sustain a reasonable conclusion as to
private respondent's parentage.

VI

Before a discussion on how private respondent's natural-born status is sustained by a general


assumption on foundlings arising from a comprehensive reading and validated by a
contemporaneous construction of the Constitution, and considering that we have just
discussed the evidence pertaining to the circumstances of private respondent's birth, it is
opportune to consider petitioner's allegations that private respondent bore the burden of
proving—through proof of her bloodline—her natural-born status.
Petitioner's claim that the burden of evidence shifted to private respondent upon a mere
showing that she is a foundling is a serious error.

Petitioner invites this Court to establish a jurisprudential presumption that all newborns who
have been abandoned in rural areas in the Philippines are not Filipinos. His emphasis on
private respondent's supposed burden to prove the circumstances of her birth places upon
her an impossible condition. To require proof from private respondent borders on the absurd
when there is no dispute that the crux of the controversy—the identity of her biological
parents—is simply not known.

"Burden of proof is the duty of a party to present evidence on the facts in issue necessary to
establish his claim or defense by the amount of evidence required by law." Burden of proof
lies on the party making the allegations;198 that is, the party who "alleges the affirmative of
the issue"199 Burden of proof never shifts from one party to another. What shifts is the burden
of evidence. This shift happens when a party makes a prima facie case in his or her
favor.200 The other party then bears the "burden of going forward" 201 with the evidence
considering that which has ostensibly been established against him or her.

In an action for quo warranto, the burden of proof necessarily falls on the party who brings
the action and who alleges that the respondent is ineligible for the office involved in the
controversy. In proceedings before quasi-judicial bodies such as the Senate Electoral Tribunal,
the requisite quantum of proof is substantial evidence. 202 This burden was petitioner's to
discharge. Once the petitioner makes a prima facie case, the burden of evidence shifts to the
respondent.

Private respondent's admitted status as a foundling does not establish a prima facie case in
favor of petitioner. While it does establish that the identities of private respondent's biological
parents are not known, it does not automatically mean that neither her father nor her mother
is a Filipino.

The most that petitioner had in his favor was doubt. A taint of doubt, however, is by no means
substantial evidence establishing a prima facie case and shifting the burden of evidence to
private respondent.

Isolating the fact of private respondent's being a foundling, petitioner trivializes other
uncontroverted circumstances that we have previously established as substantive evidence
of private respondent's parentage:ChanRoblesVirtualawlibrary
(1) Petitioner was found in front of a church in Jaro, Iloilo;

(2) She was only an infant when she was found, practically a newborn;

(3) She was-found sometime in September 1968;

(4) Immediately after she was found, private respondent was registered as a foundling;
(5) There was no international airport in Jaro, Iloilo; and

(6) Private respondent's physical features are consistent with those of typical Filipinos.
Petitioner's refusal to account for these facts demonstrates an imperceptive bias. As against
petitioner's suggested conclusions, the more reasonable inference from these facts is that at
least one of private respondent's parents is a Filipino.

VII

Apart from how private respondent is a natural-born Filipino citizen consistent with a reading
that harmonizes Article IV, Section 2's definition of natural-born citizens and Section 1(2)'s
reference to parentage, the Constitution sustains a presumption that all foundlings found in
the Philippines are born to at least either a Filipino father or a Filipino mother and are thus
natural-born, unless there is substantial proof otherwise. Consistent with Article IV, Section
1(2), any such countervailing proof must show that both—not just one—of a foundling's
biological parents are not Filipino citizens.

VII. A

Quoting heavily from Associate Justice Teresita Leonardo-De Castro's Dissenting Opinion to
the assailed November 17, 2015 Decision, petitioner intimates that no inference or
presumption in favor of natural-born citizenship may be indulged in resolving this case.203 He
insists that it is private respondent's duty to present incontrovertible proof of her Filipino
parentage.

Relying on presumptions is concededly less than ideal. Common sense dictates that actual
proof is preferable. Nevertheless, resolving citizenship issues based on presumptions is firmly
established in jurisprudence.

In 2004, this Court resolved Tecson on the basis of presumptions. Ruling on the allegations
that former presidential candidate Ronald Allan Poe (more popularly known as Fernando Poe,
Jr.) was not a natural-born Filipino citizen, this Court proceeded from the presumptions that:
first, Fernando Poe Jr.'s grandfather, Lorenzo Pou, was born sometime in 1870, while the
country was still under Spanish colonial rule;204 and second, that Lorenzo Pou's place of
residence, as indicated in his dearth certificate, must have also been his place of residence
before death, which subjected him to the "en masse Filipinization," or sweeping investiture of
Filipino citizenship effected by the Philippine Bill of 1902. 205 This Court then noted that Lorenzo
Pou's citizenship would have extended to his son and Fernando Poe Jr.'s father, Allan F. Poe.
Based on these, Fernando Poe. Jr. would then have been a natural-born Filipino as he was
born while the 1935 Constitution, which conferred Filipino citizenship to those born to Filipino
fathers, was in effect:ChanRoblesVirtualawlibrary
In ascertaining, in G.R. No. 161824, whether grave abuse of discretion has been committed
by the COMELEC, it is necessary to take on the matter of whether or not respondent FPJ is a
natural-born citizen, which, in turn, depended on whether or not the father of respondent,
Allan F. Poe, would have himself been a Filipino citizen and, in the affirmative, whether or not
the alleged illegitimacy of respondent prevents him from taking after the Filipino citizenship
of his putative father. Any conclusion on the Filipino citizenship of Lorenzo Pou could only be
drawn from the presumption that having died in 1954 at 84 years old, when the Philippines
was under Spanish rule, and that San Carlos, Pangasinan, his place of residence upon his
death in 1954, in the absence of any other evidence, could have well been his place of
residence before death, such that Lorenzo Pou would have benefited from the "en masse
Filipinization" that the Philippine Bill had effected in 1902. That citizenship (of Lorenzo Pou),
if acquired, would thereby extend to his son, Allan F. Poe, father of respondent FPJ. The 1935
Constitution, during which regime respondent FPJ has seen first light, confers citizenship to
all persons whose fathers are Filipino citizens regardless of whether such children are
legitimate or illegitimate.206chanroblesvirtuallawlibrary
It is true that there is jurisprudence—Paa v. Chan207 and Go v. Ramos208 (which merely
cites Paa)—to the effect that presumptions cannot be entertained in citizenship cases.

Paa, decided in 1967, stated:ChanRoblesVirtualawlibrary


It is incumbent upon the respondent, who claims Philippine citizenship, to prove to the
satisfaction of the court that he is really a Filipino. No presumption can be indulged in favor
of the claimant, of Philippine citizenship, and any doubt regarding citizenship must be
resolved in favor of the State.209 (Emphasis supplied)
These pronouncements are no longer controlling in light of this Court's more recent ruling
in Tecson.

Moreover, what this Court stated in Paa was that "no presumption can be indulged in favor
of theclaimant of Philippine citizenship." This reference to "the claimant" was preceded by a
sentence specifically referencing the duty of "the respondent." The syntax of this Court's
pronouncement—using the definitive article "the"—reveals that its conclusion was specific
only to Chan and to his circumstances. Otherwise, this Court would have used generic
language. Instead of the definite article "the," it could have used the indefinite article "a" in
that same sentence: "no presumption can be indulged in favor of aclaimant of Philippine
citizenship." In the alternative, it could have used other words that would show absolute or
sweeping application, for instance: "no presumption can be indulged in favor
of any/everyclaimant of Philippine citizenship;" or, "no presumption can be indulged in favor
of all claimants of Philippine citizenship."

The factual backdrop of Paa is markedly different from those of this case. Its statements,
therefore, are inappropriate precedents for this case. In Paa, clear evidence was adduced
showing that respondent Quintin Chan was registered as an alien with the Bureau of
Immigration. His father was likewise registered as an alien. These pieces of evidence already
indubitably establish foreign citizenship and shut the door to any presumption. In contrast,
petitioner in this case presents no proof, direct or circumstantial, of private respondent's or
of both of her parents' foreign citizenship.

Go cited Paa, taking the same quoted portion but revising it to make it appear that the same
pronouncement was generally applicable:ChanRoblesVirtualawlibrary
It is incumbent upon one who claims Philippine citizenship to prove to the satisfaction of the
court that he is really a Filipino. No presumption can be indulged hi favor of the claimant of
Philippine citizenship, and any doubt regarding citizenship must be resolved in favor of the
state.210 (Emphasis supplied)
Thus, Paa's essential and pivotal nuance was lost in proverbial translation. In any case, Go
was decided by this Court sitting in Division. It cannot overturn Tecson, which was decided
by this Court sitting En Banc. Likewise, Go's factual and even procedural backdrops are
different from those of this case. Goinvolved the deportation of an allegedly illegal and
undesirable alien, not an election controversy. In Go, copies of birth certificates unequivocally
showing the Chinese citizenship of Go and of his siblings were adduced.
VII. B

The presumption that all foundlings found in the Philippines are born to at least either a
Filipino father or a Filipino mother (and are thus natural-born, unless there is substantial proof
otherwise) arises when one reads the Constitution as a whole, so as to "effectuate [its] whole
purpose."211chanrobleslaw

As much as we have previously harmonized Article IV, Section 2 with Article IV, Section 1(2),
constitutional provisions on citizenship must not be taken in isolation. They must be read in
light of the constitutional mandate to defend the well-being of children, to guarantee equal
protection of the law and equal access to opportunities for public service, and to respect
human rights. They must also be read in conjunction with the Constitution's reasons for
requiring natural-born status for select public offices. Further, this presumption is validated
by contemporaneous construction that considers related legislative enactments, executive
and administrative actions, and international instruments.

Article II, Section 13 and Article XV, Section 3 of the 1987 Constitution require the state to
enhance children's well-being and to project them from conditions prejudicial to or that may
undermine their development. Fulfilling this mandate includes preventing discriminatory
conditions and, especially, dismantling mechanisms for discrimination that hide behind the
veneer of the legal apparatus:ChanRoblesVirtualawlibrary
ARTICLE II

. . . .

State Policies

. . . .

SECTION 13. The State recognizes the vital role of the youth in nation-building and shall
promote and protect their physical, moral, spiritual, intellectual, and social well-
being. It shall inculcate in the youth patriotism and nationalism, and encourage their
involvement in public and civic affairs.

....

ARTICLE XV
The Family

. . . .

SECTION 3. The State shall defend:

chanRoblesvirtualLawlibrary. . . .

(2) The right of children to assistance, including proper care and nutrition, and special
protection from all forms of neglect, abuse, cruelty, exploitation, and other
conditions prejudicial to their development[.] (Emphasis supplied)
Certain crucial government offices are exclusive to natural-born citizens of the Philippines.
The 1987 Constitution makes the following offices exclusive to natural-born
citizens:ChanRoblesVirtualawlibrary
(1) President;212

(2) Vice-President;213

(3) Senator;214

(4) Member of the House of Representatives;215

(5) Member of the Supreme Court or any lower collegiate court; 216

(6) Chairperson and Commissioners of the Civil Service Commission; 217

(7) Chairperson and Commissioners of the Commission on Elections;218

(8) Chairperson and Commissioners of the Commission on Audit; 219

(9) Ombudsman and his or her deputies;220

(10) Board of Governors of the Bangko Sentral ng Pilipinas;221 and

(11) Chairperson and Members of the Commission on Human Rights.222


Apart from these, other positions that are limited to natural-born citizens include, among
others, city fiscals,223 assistant city fiscals,224 Presiding Judges and Associate Judges of the
Sandiganbayan, and other public offices.225 Certain professions are also limited to natural-
born citizens,226 as are other legally established benefits and incentives. 227chanrobleslaw

Concluding that foundlings are not natural-born Filipino citizens is tantamount to permanently
discriminating against our foundling citizens. They can then never be of service to the country
in the highest possible capacities. It is also tantamount to excluding them from certain means
such as professions and state scholarships, which will enable the actualization of their
aspirations. These consequences cannot be tolerated by the Constitution, not least of all
through the present politically charged proceedings, the direct objective of which is merely to
exclude a singular politician from office. Concluding that foundlings are not natural-born
citizens creates an inferior class of citizens who are made to suffer that inferiority through no
fault of their own.

If that is not discrimination, we do not know what is.

The Constitution guarantees equal protection of the laws and equal access to opportunities
for public service:ChanRoblesVirtualawlibrary
ARTICLE II

. . . .

State Policies

. . . .

SECTION 26. The State shall guarantee equal access to opportunities for public service,
and prohibit political dynasties as may be defined by law.

....
ARTICLE III
Bill of Rights

SECTION 1. No person shall be deprived of life, liberty, or property without due process of
law, nor shall any person be denied the equal protection of the laws.

....

ARTICLE XIII
Social Justice and Human Rights

SECTION 1. The Congress shall give highest priority to the enactment of measures
that protect and enhance the right of all the people to human dignity, reduce social,
economic, and political inequalities, and remove cultural inequities by equitably
diffusing wealth and political power for the common good. (Emphasis supplied)
The equal protection clause serves as a guarantee that "persons under like circumstances and
falling within the same class are treated alike, in terms of 'privileges conferred and liabilities
enforced.' It is a guarantee against 'undue favor and individual or class privilege, as well as
hostile discrimination or oppression of inequality.'" 228chanrobleslaw

Other than the anonymity of their biological parents, no substantial


distinction229 differentiates foundlings from children with known Filipino parents. They are
both entitled to the full extent of the state's protection from the moment of their birth.
Foundlings' misfortune in failing to identify the parents who abandoned them—an inability
arising from no fault of their own—cannot be the foundation of a rule that reduces them to
statelessness or, at best, as inferior, second-class citizens who are not entitled to as much
benefits and protection from the state as those who know their parents. Sustaining this
classification is not only inequitable; it is dehumanizing. It condemns those who, from the
very beginning of their lives, were abandoned to a life of desolation and deprivation.

This Court does not exist in a vacuum. It is a constitutional organ, mandated to effect the
Constitution's dictum of defending and promoting the well-being and development of children.
It is not our business to reify discriminatory classes based on circumstances of birth.

Even more basic than their being citizens of the Philippines, foundlings are human persons
whose dignity we value and rights we, as a civilized nation, respect.
Thus:ChanRoblesVirtualawlibrary
ARTICLE II

. . . .

State Policies

. . . .

SECTION 11. The State values the dignity of every human person and guarantees full
respect for human rights. (Emphasis supplied)
VII. C

Though the matter is settled by interpretation exclusively within the confines of constitutional
text, the presumption that foundlings are natural-born citizens of the Philippines (unless
substantial evidence of the foreign citizenship of both of the foundling's parents is presented)
is validated by a parallel consideration or contemporaneous construction of the Constitution
with acts of Congress, international instruments in force in the Philippines, as well as acts of
executive organs such as the Bureau of Immigration, Civil Registrars, and the President of
the Philippines.

Congress has enacted statutes founded on the premise that foundlings are Filipino citizens at
birth. It has adopted mechanisms to effect the constitutional mandate to protect children.
Likewise, the Senate has ratified treaties that put this mandate into effect.

Republic Act No. 9344, otherwise known as the Juvenile Justice and Welfare Act of 2006,
provides:ChanRoblesVirtualawlibrary
SEC. 2. Declaration of State Policy. - The following State policies shall be observed at all
times:

chanRoblesvirtualLawlibrary. . . .

(b) The State shall protect the best interests of the child through measures that will
ensure the observance of international standards of child protection, especially
those to which the Philippines is a party. Proceedings before any authority shall be
conducted in the best interest of the child and in a manner which allows the child to participate
and to express himself/herself freely. The participation of children in the program and policy
formulation and implementation related to juvenile justice and welfare shall be ensured by
the concerned government agency. (Emphasis supplied)
Section 4(b) of the Republic Act No. 9344 defines the "best interest of the child" as the "totality
of the circumstances and conditions which are most congenial to the survival, protection and
feelings of security of the child and most encouraging to the child's physical, psychological
and emotional development."

Consistent with this statute is our ratification230 of the United Nations Convention on the
Rights of the Child. This specifically requires the states-parties' protection of: first, children's
rights to immediate registration and nationality after birth; second, against statelessness;
and third, against discrimination on account of their birth status. 231 Pertinent portions of the
Convention read:ChanRoblesVirtualawlibrary
Preamble

The State Parties to the present Convention,

Considering that, in accordance with the principles proclaimed in the Charter of the United
Nations, recognition of the inherent dignity and of the equal and inalienable rights of
all members of the human family is the foundation of freedom, justice and peace in the
world,

Bearing in mind that the peoples of the United Nations have, in the Charter, reaffirmed their
faith in fundamental human rights and in the dignity and worth of the human
person, and have determined to promote social progress and better standards of life in larger
freedom,

Recognizing that the United Nations has, in the Universal Declaration of Human Rights and in
the International Covenants on Human Rights, proclaimed and agreed that everyone is
entitled to all the rights and freedoms set forth therein, without distinction of any
kind, such as race, colour, sex, language, religion, political or other opinion, national or
social origin, property, birth or other status,
Recalling that, in the Universal Declaration of Human Rights, the United Nations has
proclaimed that childhood is entitled to special care and assistance,

. . . .

Have agreed as follows:

chanRoblesvirtualLawlibrary. . . .

Article 2

1. State parties shall respect and ensure the rights set forth in the present
Convention to each child within their jurisdiction without discrimination of
any kind, irrespective of the child's or his or her parent's or legal
guardian's race, colour, sex, language, religion, political or other opinion,
national, ethnic or social origin, property, disability, birth or other
status.

2. States Parties shall take appropriate measures to ensure that the child
is protected against all forms of discrimination or punishment on the
basis of the status, activities, expressed opinions, or beliefs of the child's
parents, legal guardians, or family members.

Article 3

1. In all actions concerning children, whether undertaken by public or private


social welfare institutions, courts of law, administrative authorities or
legislative bodies, the best interests of the child shall be a primary
consideration.

2. States Parties undertake to ensure the child such protection and care
as is necessary for his or her well-being, taking into account the rights and
duties of his or her parents, legal guardians, or other individuals legally
responsible for him or her, and, to this end, shall take all appropriate legislative
and administrative measures.

. . . .

Article 7

1. The child, shall be registered immediately after birth and shall have the
right from birth to a name, the right to acquire a nationality and as far as
possible, the right to know and be cared for by his or her parents.

2. States Parties shall ensure the implementation of these rights in


accordance with their national law and their obligations under the relevant
international instruments in this field, in particular where the child would
otherwise be stateless. (Emphasis supplied)

The Philippines likewise ratified232 the 1966 International Covenant on Civil and Political
Rights. As with the Convention on the Rights of the Child, this treaty requires that children be
allowed immediate registration after birth and to acquire a nationality. It similarly defends
them against discrimination:ChanRoblesVirtualawlibrary
Article 24. . . .

1. Every child shall have, without any discrimination as to race, colour, sex, language,
religion, national or social origin, property or birth, the right to such measures of protection
as are required by his status as a minor, on the part of his family, society and the State.

2. Every child shall be registered immediately after birth and shall have a name.

3. Every child has the right to acquire a nationality.

. . . .

Article 26. All persons are equal before the law and are entitled without any
discrimination to the equal protection of the law. In this respect, the law shall prohibit
any discrimination and guarantee to all persons equal and effective protection
against discrimination on any ground such as race, colour, sex, language, religion,
political or other opinion, national or social origin, property, birth or other status. (Emphasis
supplied)
Treaties are "international agreements] concluded between state| in written form and
governed by international law, whether embodied in a single instrument or in two or more
related instruments and whatever its particular designation."233 Under Article VII, Section 21
of the 1987 Constitution, treaties require concurrence by the Senate before they became
binding:ChanRoblesVirtualawlibrary
SECTION 21. No treaty or international agreement shall be valid and effective unless
concurred in by at least two-thirds of all the Members of the Senate.
The Senate's ratification of a treaty makes it legally effective and binding by transformation.
It then has the force and effect of a statute enacted by Congress. In Pharmaceutical and
Health Care Association of the Philippines v. Duque III, et al.:234
Under the 1987 Constitution, international law can become part of the sphere of domestic law
either by transformation or incorporation. The transformation method requires that an
international law be transformed into a domestic law through a constitutional mechanism such
as local legislation. The incorporation method applies when, by mere constitutional
declaration, international law is deemed to have the force of domestic law.

Treaties become part of the law of the land through transformation pursuant to Article VII,
Section 21 of the Constitution which provides that "[n]o treaty or international agreement
shall be valid and effective unless concurred in by at least two-thirds of all the members of
the Senate." Thus, treaties or conventional international law must go through a process
prescribed by the Constitution for it to be transformed into municipal law that can be applied
to domestic conflicts.235 (Emphasis supplied)
Following ratification by the Senate, no further action, legislative or otherwise, is necessary.
Thereafter, the whole of government—including the judiciary—is duty-bound to abide by the
treaty, consistent with the maxim pacta sunt servanda.

Accordingly, by the Constitution and by statute, foundlings cannot be the object of


discrimination. They are vested with the rights to be registered and granted nationality upon
birth. To deny them these rights, deprive them of citizenship, and render them stateless is to
unduly burden them, discriminate them, and undermine their development.

Not only Republic Act No. 9344, the Convention on the Rights of the Child, and the
International Covenant on Civil and Political Rights effect the constitutional dictum of
promoting the well-being of children and protecting them from discrimination. Other
legislative enactments demonstrate the intent to treat foundlings as Filipino citizens from
birth.

Republic Act No. 8552, though briefly referred to as the Domestic Adoption Act of 1998, is
formally entitled An Act Establishing the Rules and Policies on Domestic Adoption of Filipino
Children and for Other Purposes. It was enacted as a mechanism to "provide alternative
protection and assistance through foster care or adoption of every child who is neglected,
orphaned, or abandoned."236chanrobleslaw

Foundlings are explicitly among the "Filipino children" covered by Republic Act No. 8552:237
SECTION 5. Location of Unknown Parent(s). — It shall be the duty of the Department or the
child-placing or child-caring agency which has custody of the child to exert all efforts to locate
his/her unknown biological parent(s). If such efforts fail, the child shall be registered as
a foundling and subsequently be the subject of legal proceedings where he/she
shall be declared abandoned. (Emphasis supplied)
Similarly, Republic Act No. 8043, though briefly referred to as the Inter-Country Adoption Act
of 1995, is formally entitled An Act Establishing the Rules to Govern Inter-Country Adoption
of Filipino Children, and for Other Purposes. As with Republic Act No. 8552, it expressly
includes foundlings among "Filipino children" who may be
adopted:ChanRoblesVirtualawlibrary
SECTION 8. Who May Be Adopted. — Only a legally free child may be the subject of inter-
country adoption, hi order that such child may be considered for placement, the following
documents must be submitted: to the Board:

chanRoblesvirtualLawlibrary a) Child study;

b) Birth certificate/foundling certificate;

c) Deed of voluntary commitment/decree of abandonment/death certificate of parents;

d) Medical evaluation/history;

e) Psychological evaluation, as necessary; and cralawlawlibrary

f) Recent photo of the child. (Emphasis supplied)


In the case of foundlings, foundling certificates may be presented in lieu of authenticated birth
certificates to satisfy the requirement for the issuance of passports, which will then facilitate
their adoption by foreigners:ChanRoblesVirtualawlibrary
SECTION 5. If the applicant is an adopted person, he must present a certified true copy of
the Court Order of Adoption, certified true copy of his original and amended birth certificates
as issued by the OCRG. If the applicant is a minor, a Clearance from the DSWD shall be
required. In case the applicant is for adoption by foreign parents under R.A. No. 8043, the
following, shall be required:

chanRoblesvirtualLawlibrary

a) Certified true copy of the Court Decree of Abandonment of Child, the Death Certificate
of the child's parents, or the Deed of Voluntary Commitment executed after the birth
of the child.
b) Endorsement of child to the Intercountry Adoption Board by the DSWD.

c) Authenticated Birth or Foundling Certificate.238 (Emphasis supplied)


Our statutes on adoption allow for the recognition of foundlings' Filipino citizenship on account
of their birth. They benefit from this without having to do any act to perfect their citizenship
or without having to complete the naturalization process. Thus, by definition, they are natural-
born citizens.

Specifically regarding private respondent, several acts of executive organs have recognized
her natural-born status. This status was never questioned throughout her life; that is, until
circumstances made it appear that she was a viable candidate for President of the Philippines.
Until this, as well as the proceedings in the related case of Poe-Llamanzares, private
respondent's natural-born status has been affirmed and reaffirmed through various official
public acts.

First, private respondent was issued a foundling certificate and benefitted from the domestic
adoption process. Second, on July 18, 2006, she was granted an order of reacquisition of
natural-born citizenship under Republic Act No. 9225 by the Bureau of Immigration. Third, on
October 6, 2010, the President of the Philippines appointed her as MTRCB Chairperson—an
office that requires natural-born citizenship.239chanrobleslaw

VIII

As it is settled that private respondent's being a foundling is not a bar to natural-born


citizenship, petitioner's proposition as to her inability to benefit from Republic Act No. 9225
crumbles. Private respondent, a natural-born Filipino citizen, re-acquired natural-born Filipino
citizenship when, following her naturalization as a citizen of the United States, she complied
with the requisites of Republic Act No. 9225.

VIII. A

"Philippine citizenship may be lost or reacquired in the manner provided by


law."240 Commonwealth Act No. 63, which was in effect when private respondent was
naturalized an American citizen on October 18, 2001, provided in Section 1(1) that "[a]
Filipino citizen may lose his citizenship . . . [b]y naturalization in a foreign country." Thus,
private respondent lost her Philippine citizenship when she was naturalized an American
citizen. However, on July 7, 2006, she took her Oath of Allegiance to the Republic of the
Philippines under Section 3 of Republic Act No. 9225. Three (3) days later, July 10, 2006, she
filed before the Bureau of Immigration and Deportation a Petition for Reacquisition of her
Philippine citizenship. Shortly after, this Petition was granted. 241chanrobleslaw

Republic Act No. 9225 superseded Commonwealth Act No. 63242 and Republic Act No.
8171243specifically "to do away with the provision in Commonwealth Act No. 63 which takes
away Philippine citizenship from natural-born Filipinos who become naturalized citizens of
other countries."244chanrobleslaw

The citizenship regime put in place by Republic Act No. 9225 is designed, in its own words, to
ensure "that all Philippine citizens who become citizens of another country shall be
deemed not to have lost their Philippine citizenship."245 This Court shed light on this
in Calilung v. Commission on Elections:246 "[w]hat Rep. Act No. 9225 does is allow dual
citizenship to natural-born Filipino citizens who have lost Philippine citizenship by reason of
their naturalization as citizens of a foreign country."247chanrobleslaw

Republic Act No. 9225 made natural-born Filipinos' status permanent and immutable despite
naturalization as citizens of other countries. To effect this, Section 3 of Republic Act No. 9225
provides:ChanRoblesVirtualawlibrary
SEC. 3. Retention of Philippine Citizenship. — Any provision of law to the contrary
notwithstanding, natural-born citizens of the Philippines who have lost their Philippine
citizenship by reason of their naturalization as citizens of a foreign country are hereby deemed
to have reacquired Philippine citizenship upon taking the following oath of allegiance to the
Republic:ChanRoblesVirtualawlibrary
"I _________________________, solemnly swear (or affirm) that I will support and defend
the Constitution of the Republic of the Philippines and obey the laws and legal orders
promulgated by the duly constituted authorities of the Philippines; and I hereby declare that
I recognize and accept the supreme authority of the Philippines and will maintain true faith
and allegiance thereto; and that I impose this obligation upon myself voluntarily without
mental reservation or purpose of evasion."
Natural-born citizens of the Philippines who, after the effectivity of this Act, become citizens
of a foreign country shall retain their Philippine citizenship upon taking the aforesaid oath.
Section 3's implications are clear. Natural-born Philippine citizens who, after Republic Act
9225 took effect, are naturalized in foreign countries "retain," that is, keep, their Philippine
citizenship, although the effectivity of this retention and the ability to exercise the rights and
capacities attendant to this status are subject to certain solemnities (i.e., oath of allegiance
and other requirements for specific rights and/or acts, as enumerated in Section 5). On the
other hand, those who became citizens of another country before the effectivity of Republic
Act No. 9225 "reacquire" their Philippine citizenship and may exercise attendant rights and
capacities, also upon compliance with certain solemnities. Read in conjunction with Section
2's declaration of a policy of immutability, this reacquisition is not a mere restoration that
leaves a vacuum in the intervening period. Rather, this reacquisition works to restore natural-
born status as though it was never lost at all.

VIII. B

Taking the Oath of Allegiance effects the retention or reacquisition of natural-born citizenship.
It also facilitates the enjoyment of civil and political rights, "subject to all attendant liabilities
and responsibilities."248 However, other conditions must be met for the exercise of other
faculties:ChanRoblesVirtualawlibrary
Sec. 5. Civil and Political Rights and Liabilities. - Those who retain or re-acquire Philippine
citizenship under this Act shall enjoy full civil and political rights and be subject to all attendant
liabilities and responsibilities under existing laws of the Philippines and the following
conditions:

chanRoblesvirtualLawlibrary

(1) Those intending to exercise their right of suffrage must meet the requirements
under Section 1, Article V of the Constitution, Republic Act No. 9189, otherwise
known as "the Overseas Absentee Voting Act of 2003" and other existing laws;
(2) Those seeking elective public office in the Philippines shall meet the qualifications
for holding such public office as required by the Constitution and existing laws
and, at the time of the filing of the certificate of candidacy, make a
personal and sworn renunciation of any and all foreign citizenship before
any public officer authorized to administer an oath;

(3) Those appointed to any public office shall subscribe and swear to an oath of
allegiance to the Republic of the Philippines and its duly constituted
authorities prior to their assumption of office; Provided, That they renounce
their oath of allegiance to the country where they took that oath;

(4) Those intending to practice their profession in the Philippines shall apply with the
proper authority for a license or permit to engage in such practice; and

(5) That the right to vote or be elected or appointed to any public office in the
Philippines cannot be exercised by, or extended to, those who:

a. are candidates for or are occupying any public office in the country of which they
are naturalized citizens; and/or

b. are in active service as commissioned or noncommissioned officers in the armed


forces of the country which they are naturalized citizens. (Emphasis supplied)
Thus, natural-born Filipinos who have been naturalized elsewhere and wish to run for elective
public office must comply with all of the following requirements:

chanRoblesvirtualLawlibraryFirst, taking the oath of allegiance to the Republic. This effects


the retention or reacquisition of one's status as a natural-born Filipino.249 This also enables
the enjoyment of full civil and political rights, subject to all attendant liabilities and
responsibilities under existing laws, provided the solemnities recited in Section 5 of Republic
Act No. 9225 are satisfied.250chanrobleslaw

Second, compliance with Article V, Section 1 of the 1987 Constitution, 251 Republic Act No.
9189, otherwise known as the Overseas Absentee Voting Act of 2003, and other existing laws.
This is to facilitate the exercise of the right of suffrage; that is, to allow for voting in
elections.252chanrobleslaw

Third, "mak[ing] a personal and sworn renunciation of any and all foreign citizenship before
any public officer authorized to administer an oath."253 This, along with satisfying the other
qualification requirements under relevant laws, makes one eligible for elective public office.

As explained in Sobejana-Condon v. Commission on Elections,254 this required sworn


renunciation is intended to complement Article XI, Section 18 of the Constitution in that
"[p]ublic officers and employees owe the State and this Constitution allegiance at all times
and any public officer or employee who seeks to change his citizenship or acquire the status
of an immigrant of another country during his tenure shall be dealt with by law." 255 It is also
in view of this that Section 5(5) similarly bars those who seek or occupy public office
elsewhere and/or who are serving in the armed forces of other countries from being appointed
or elected to public office in the Philippines.

VIII. C

Private respondent has complied with all of these requirements. First, on July 7, 2006, she
took the Oath of Allegiance to the Republic of the Philippines.256 Second, on August 31, 2006,
she became a registered voter of Barangay Santa Lucia, San Juan. 257 This evidences her
compliance with Article V, Section 1 of the 1987 Constitution. Since she was to vote within
the country, this dispensed with the need to comply with the Overseas Absentee Voting Act
of 2003. Lastly, on October 20, 2010, she executed an Affidavit of Renunciation of Allegiance
to the United States of America and Renunciation of American Citizenship. 258This was
complemented by her execution of an Oath/Affirmation of Renunciation of Nationality of the
United States259 before Vice-Consul Somer E. Bessire-Briers on July 12, 2011,260 which was,
in turn, followed by Vice Consul Jason Galian's issuance of a Certificate of Loss of Nationality
on December 9, 2011261 and the approval of this certificate by the Overseas Citizen Service,
Department of State, on February 3, 2012.262chanrobleslaw

Private respondent has, therefore, not only fully reacquired natural-born citizenship; she has
also complied with all of the other requirements for eligibility to elective public office, as
stipulated in Republic Act No. 9225.

VIII. D

It is incorrect to intimate that private respondent's having had to comply with Republic Act
No. 9225 shows that she is a naturalized, rather than a natural-born, Filipino citizen. It is
wrong to postulate that compliance with Republic Act No. 9225 signifies the performance of
acts to perfect citizenship.

To do so is to completely disregard the unequivocal policy of permanence and immutability


as articulated in Section 2 of Republic Act No. 9225 and as illuminated in jurisprudence. It is
to erroneously assume that a natural-born Filipino citizen's naturalization elsewhere is an
irreversible termination of his or her natural-born status.

To belabor the point, those who take the Oath of Allegiance under Section 3 of Republic Act
No. 9225 reacquire natural-born citizenship. The prefix "re" signifies reference to the
preceding state of affairs. It is to this status quo ante that one returns. "Re"-acquiring can
only mean a reversion to "the way things were." Had Republic Act No. 9225 intended to mean
the investiture of an entirely new status, it should not have used a word such as "reacquire."
Republic Act No. 9225, therefore, does not operate to make new citizens whose citizenship
commences only from the moment of compliance with its requirements.

Bengson, speaking on the analogous situation of repatriation, ruled that repatriation involves
the restoration of former status or the recovery of one's original
nationality:ChanRoblesVirtualawlibrary
Moreover, repatriation results in the recovery of the original nationality. This means that a
naturalized Filipino who lost his citizenship will be restored to his prior status as a naturalized
Filipino citizen. On the other hand, if he was originally a natural-born citizen before he lost
his Philippine citizenship, he will be restored to his former status as a natural-born
Filipino.263 (Emphasis supplied)
Although Bengson was decided while Commonwealth Act No. 63 was in force, its ruling is in
keeping with Republic Act No. 9225 's policy of permanence and immutablity: "all Philippine
citizens of another country shall be deemed not to have lost their Philippine
citizenship."264 In Bengson's words, the once naturalized citizen is "restored" or brought back
to his or her natural-born status. There may have been an interruption in the recognition of
this status, as, in the interim, he or she was naturalized elsewhere, but the restoration of
natural-born status expurgates this intervening fact. Thus, he or she does not become a
Philippine citizen only from the point of restoration and moving forward. He or she is
recognized, de jure, as a Philippine citizen from birth, although the intervening fact may have
consequences de facto.

Republic Act No. 9225 may involve extended processes not limited to taking the Oath of
Allegiance and requiring compliance with additional solemnities, but these are for facilitating
the enjoyment of other incidents to citizenship, not for effecting the reacquisition of natural-
born citizenship itself. Therefore, it is markedly different from naturalization as there is no
singular, extended process with which the former natural-born citizen must comply.

IX

To hold, as petitioner suggests, that private respondent is stateless265 is not only to set a
dangerous and callous precedent. It is to make this Court an accomplice to injustice.

Equality, the recognition of the humanity of every individual, and social justice are the
bedrocks of our constitutional order. By the unfortunate fortuity of the inability or outright
irresponsibility of those gave them life, foundlings are compelled to begin their very existence
at a disadvantage. Theirs is a continuing destitution that can never be truly remedied by any
economic relief.

If we are to make the motives of our Constitution true, then we an never tolerate an
interpretation that condemns foundlings to an even greater misfortune because of their being
abandoned. The Constitution cannot be rendered inert and meaningless for them by
mechanical judicial fiat.

Dura lex sed lex is not a callous and unthinking maxim to be deployed against other
reasonable interpretations of our basic law. It does command us to consider legal text, but
always with justice in mind.

It is the empowering and ennobling interpretation of the Constitution that we must always
sustain. Not only will this manner of interpretation edify the less fortunate; it establishes us,
as Filipinos, as a humane and civilized people.

The Senate Electoral Tribunal acted well within the bounds of its constitutional competence
when it ruled that private respondent is a natural-born citizen qualified to sit as Senator of
the Republic. Contrary to petitioner's arguments, there is no basis for annulling its assailed
Decision and Resolution.

WHEREFORE, the Petition for Certiorari is DISMISSED. Public respondent Senate Electoral
Tribunal did not act without or in excess of its jurisdiction or with grave abuse of discretion
amounting to lack or excess of jurisdiction in rendering its assailed November 17, 2015
Decision and December 3, 2015 Resolution.

Private respondent Mary Grace Poe-Llamanzares is a natural-born Filipino citizen qualified to


hold office as Senator of the Republic.
SO ORDERED.chanRoblesvirtualLawlibrary

[G.R. No. 161434. March 3, 2004]

MARIA JEANETTE C. TECSON and FELIX B. DESIDERIO, JR., petitioners, vs. The
COMMISSION ON ELECTIONS, RONALD ALLAN KELLY POE (a.k.a. FERNANDO
POE, JR.) and VICTORINO X. FORNIER, respondents.

[G.R. No. 161634. March 3, 2004]

ZOILO ANTONIO VELEZ, petitioner, vs. RONALD ALLAN KELLEY


POE, a.k.a. FERNANDO POE, JR., respondent.

[G. R. No. 161824. March 3, 2004]

VICTORINO X. FORNIER, petitioner, vs. HON. COMMISSION ON ELECTIONS and


RONALD ALLAN KELLEY POE, ALSO KNOWN AS FERNANDO POE
JR., respondents.

DECISION
VITUG, J.:

Citizenship is a treasured right conferred on those whom the state believes are
deserving of the privilege. It is a precious heritage, as well as an inestimable
acquisition,[1]that cannot be taken lightly by anyone - either by those who enjoy it
or by those who dispute it.
Before the Court are three consolidated cases, all of which raise a single question of
profound importance to the nation. The issue of citizenship is brought up to challenge the
qualifications of a presidential candidate to hold the highest office of the land. Our people are
waiting for the judgment of the Court with bated breath. Is Fernando Poe, Jr., the hero of
silver screen, and now one of the main contenders for the presidency, a natural-born Filipino
or is he not?
The moment of introspection takes us face to face with Spanish and American colonial
roots and reminds us of the rich heritage of civil law and common law traditions, the fusion
resulting in a hybrid of laws and jurisprudence that could be no less than distinctly Filipino.

Antecedent Case Settings


On 31 December 2003, respondent Ronald Allan Kelly Poe, also known as Fernando Poe,
Jr. (hereinafter "FPJ"), filed his certificate of candidacy for the position of President of the
Republic of the Philippines under the Koalisyon ng Nagkakaisang Pilipino (KNP) Party, in the
forthcoming national elections. In his certificate of candidacy, FPJ, representing himself to be
a natural-born citizen of the Philippines, stated his name to be "Fernando Jr.," or "Ronald
Allan" Poe, his date of birth to be 20 August 1939 and his place of birth to be Manila.
Victorino X. Fornier, petitioner in G.R. No. 161824, entitled "Victorino X. Fornier,
Petitioner, versus Hon. Commission on Elections and Ronald Allan Kelley Poe, also known as
Fernando Poe, Jr., Respondents," initiated, on 09 January 2004, a petition docketed SPA No.
04-003 before the Commission on Elections ("COMELEC") to disqualify FPJ and to deny due
course or to cancel his certificate of candidacy upon the thesis that FPJ made a material
misrepresentation in his certificate of candidacy by claiming to be a natural-born Filipino
citizen when in truth, according to Fornier, his parents were foreigners; his mother, Bessie
Kelley Poe, was an American, and his father, Allan Poe, was a Spanish national, being the son
of Lorenzo Pou, a Spanish subject. Granting, petitioner asseverated, that Allan F. Poe was a
Filipino citizen, he could not have transmitted his Filipino citizenship to FPJ, the latter being
an illegitimate child of an alien mother. Petitioner based the allegation of the illegitimate birth
of respondent on two assertions - first, Allan F. Poe contracted a prior marriage to a certain
Paulita Gomez before his marriage to Bessie Kelley and, second, even if no such prior
marriage had existed, Allan F. Poe, married Bessie Kelly only a year after the birth of
respondent.
In the hearing before the Third Division of the COMELEC on 19 January 2004, petitioner,
in support of his claim, presented several documentary exhibits - 1) a copy of the certificate
of birth of FPJ, 2) a certified photocopy of an affidavit executed in Spanish by Paulita Poe y
Gomez attesting to her having filed a case for bigamy and concubinage against the father of
respondent, Allan F. Poe, after discovering his bigamous relationship with Bessie Kelley, 3)
an English translation of the affidavit aforesaid, 4) a certified photocopy of the certificate of
birth of Allan F. Poe, 5) a certification issued by the Director of the Records Management and
Archives Office, attesting to the fact that there was no record in the National Archives that a
Lorenzo Poe or Lorenzo Pou resided or entered the Philippines before 1907, and 6) a
certification from the Officer-In-Charge of the Archives Division of the National Archives to
the effect that no available information could be found in the files of the National Archives
regarding the birth of Allan F. Poe.
On his part, respondent, presented twenty-two documentary pieces of evidence, the more
significant ones being - a) a certification issued by Estrella M. Domingo of the Archives Division
of the National Archives that there appeared to be no available information regarding the birth
of Allan F. Poe in the registry of births for San Carlos, Pangasinan, b) a certification issued by
the Officer-In-Charge of the Archives Division of the National Archives that no available
information about the marriage of Allan F. Poe and Paulita Gomez could be found, c) a
certificate of birth of Ronald Allan Poe, d) Original Certificate of Title No. P-2247 of the Registry
of Deeds for the Province of Pangasinan, in the name of Lorenzo Pou, e) copies of Tax
Declaration No. 20844, No. 20643, No. 23477 and No. 23478 in the name of Lorenzo Pou, f)
a copy of the certificate of death of Lorenzo Pou, g) a copy of the purported marriage contract
between Fernando Pou and Bessie Kelley, and h) a certification issued by the City Civil
Registrar of San Carlos City, Pangasinan, stating that the records of birth in the said office
during the period of from 1900 until May 1946 were totally destroyed during World War II.
On 23 January 2004, the COMELEC dismissed SPA No. 04-003 for lack of merit. Three
days later, or on 26 January 2004, Fornier filed his motion for reconsideration. The motion
was denied on 06 February 2004 by the COMELEC en banc. On 10 February 2004, petitioner
assailed the decision of the COMELEC before this Court conformably with Rule 64, in relation
to Rule 65, of the Revised Rules of Civil Procedure. The petition, docketed G. R. No. 161824,
likewise prayed for a temporary restraining order, a writ of preliminary injunction or any other
resolution that would stay the finality and/or execution of the COMELEC resolutions.
The other petitions, later consolidated with G. R. No. 161824, would include G. R. No.
161434, entitled "Maria Jeanette C. Tecson, and Felix B. Desiderio, Jr., vs. The Commission
on Elections, Ronald Allan Kelley Poe (a.k.a. Fernando Poe, Jr.), and Victorino X. Fornier," and
the other, docketed G. R. No. 161634, entitled "Zoilo Antonio G. Velez, vs. Ronald Allan Kelley
Poe, a.k.a. Fernando Poe, Jr.," both challenging the jurisdiction of the COMELEC and asserting
that, under Article VII, Section 4, paragraph 7, of the 1987 Constitution, only the Supreme
Court had original and exclusive jurisdiction to resolve the basic issue on the case.

Jurisdiction of the Court

In G. R. No. 161824
In seeking the disqualification of the candidacy of FPJ and to have the COMELEC deny
due course to or cancel FPJs certificate of candidacy for alleged misrepresentation of a
material fact (i.e., that FPJ was a natural-born citizen) before the COMELEC, petitioner Fornier
invoked Section 78 of the Omnibus Election Code

Section 78. Petition to deny due course to or cancel a certificate of candidacy. --- A verified
petition seeking to deny due course or to cancel a certificate of candidacy may be filed by any
person exclusively on the ground that any material representation contained therein as
required under Section 74 hereof is false

in consonance with the general powers of COMELEC expressed in Section 52 of the Omnibus
Election Code -

Section 52. Powers and functions of the Commission on Elections. In addition to the powers
and functions conferred upon it by the Constitution, the Commission shall have exclusive
charge of the enforcement and administration of all laws relative to the conduct of elections
for the purpose of ensuring free, orderly and honest elections -

and in relation to Article 69 of the Omnibus Election Code which would authorize "any
interested party" to file a verified petition to deny or cancel the certificate of candidacy of any
nuisance candidate.
Decisions of the COMELEC on disqualification cases may be reviewed by the Supreme
Court per Rule 64[2] in an action for certiorari under Rule 65[3] of the Revised Rules of Civil
Procedure. Section 7, Article IX, of the 1987 Constitution also reads

"Each Commission shall decide by a majority vote of all its Members any case or matter
brought before it within sixty days from the date of its submission for decision or resolution. A
case or matter is deemed submitted for decision or resolution upon the filing of the last
pleading, brief, or memorandum, required by the rules of the Commission or by the
Commission itself. Unless otherwise provided by this Constitution or by law, any decision,
order, or ruling of each Commission may be brought to the Supreme Court on certiorari by
the aggrieved party within thirty days from receipt of a copy thereof."
Additionally, Section 1, Article VIII, of the same Constitution provides that judicial power
is vested in one Supreme Court and in such lower courts as may be established by law which
power includes the duty of the courts of justice to settle actual controversies involving rights
which are legally demandable and enforceable, and to determine whether or not there has
been a grave abuse of discretion amounting to lack or excess of jurisdiction on the part of
any branch or instrumentality of the Government.
It is sufficiently clear that the petition brought up in G. R. No. 161824 was aptly elevated
to, and could well be taken cognizance of by, this Court. A contrary view could be a gross
denial to our people of their fundamental right to be fully informed, and to make a proper
choice, on who could or should be elected to occupy the highest government post in the land.
In G. R. No. 161434 and G. R. No. 161634
Petitioners Tecson, et al., in G. R. No. 161434, and Velez, in G. R. No. 161634, invoke
the provisions of Article VII, Section 4, paragraph 7, of the 1987 Constitution in assailing the
jurisdiction of the COMELEC when it took cognizance of SPA No. 04-003 and in urging the
Supreme Court to instead take on the petitions they directly instituted before it. The
Constitutional provision cited reads:

"The Supreme Court, sitting en banc, shall be the sole judge of all contests relating to the
election, returns, and qualifications of the President or Vice-President, and may promulgate
its rules for the purpose."

The provision is an innovation of the 1987 Constitution. The omission in the 1935 and the
1973 Constitution to designate any tribunal to be the sole judge of presidential and vice-
presidential contests, has constrained this Court to declare, in Lopez vs. Roxas,[4] as not
(being) justiciable controversies or disputes involving contests on the elections, returns and
qualifications of the President or Vice-President. The constitutional lapse prompted Congress,
on 21 June 1957, to enact Republic Act No. 1793, "An Act Constituting an Independent
Presidential Electoral Tribunal to Try, Hear and Decide Protests Contesting the Election of the
President-Elect and the Vice-President-Elect of the Philippines and Providing for the Manner
of Hearing the Same." Republic Act 1793 designated the Chief Justice and the Associate
Justices of the Supreme Court to be the members of the tribunal. Although the subsequent
adoption of the parliamentary form of government under the 1973 Constitution might have
implicitly affected Republic Act No. 1793, the statutory set-up, nonetheless, would now be
deemed revived under the present Section 4, paragraph 7, of the 1987 Constitution.
Ordinary usage would characterize a "contest" in reference to a post-
election scenario. Election contests consist of either an election protest or a quo
warranto which, although two distinct remedies, would have one objective in view, i.e., to
dislodge the winning candidate from office. A perusal of the phraseology in Rule 12, Rule 13,
and Rule 14 of the "Rules of the Presidential Electoral Tribunal," promulgated by the Supreme
Court en banc on 18 April 1992, would support this premise -

Rule 12. Jurisdiction. - The Tribunal shall be the sole judge of all contests relating to the
election, returns, and qualifications of the President or Vice-President of the Philippines.

Rule 13. How Initiated. - An election contest is initiated by the filing of an election protest or
a petition for quo warranto against the President or Vice-President. An election protest shall
not include a petition for quo warranto. A petition for quo warranto shall not include an
election protest.
Rule 14. Election Protest. - Only the registered candidate for President or for Vice-President
of the Philippines who received the second or third highest number of votes may contest the
election of the President or the Vice-President, as the case may be, by filing a verified petition
with the Clerk of the Presidential Electoral Tribunal within thirty (30) days after the
proclamation of the winner.

The rules categorically speak of the jurisdiction of the tribunal over contests relating to
the election, returns and qualifications of the "President" or "Vice-President", of
the Philippines, and not of "candidates" for President or Vice-President. A quo
warranto proceeding is generally defined as being an action against a person who usurps,
intrudes into, or unlawfully holds or exercises a public office.[5] In such context, the election
contest can only contemplate a post-election scenario. In Rule 14, only a registered candidate
who would have received either the second or third highest number of votes could file an
election protest. This rule again presupposes a post-election scenario.
It is fair to conclude that the jurisdiction of the Supreme Court, defined by Section 4,
paragraph 7, of the 1987 Constitution, would not include cases directly brought before it,
questioning the qualifications of a candidate for the presidency or vice-presidency before the
elections are held.
Accordingly, G. R. No. 161434, entitled "Maria Jeanette C. Tecson, et al., vs. Commission
on Elections et al.," and G. R. No. 161634, entitled "Zoilo Antonio Velez vs. Ronald Allan Kelley
Poe a.k.a. Fernando Poe, Jr." would have to be dismissed for want of jurisdiction.
The Citizenship Issue
Now, to the basic issue; it should be helpful to first give a brief historical background on
the concept of citizenship.
Perhaps, the earliest understanding of citizenship was that given by Aristotle, who,
sometime in 384 to 322 B.C., described the "citizen" to refer to a man who shared in the
administration of justice and in the holding of an office.[6] Aristotle saw its significance if only
to determine the constituency of the "State," which he described as being composed of such
persons who would be adequate in number to achieve a self-sufficient existence.[7] The
concept grew to include one who would both govern and be governed, for which qualifications
like autonomy, judgment and loyalty could be expected. Citizenship was seen to deal with
rights and entitlements, on the one hand, and with concomitant obligations, on the other.[8] In
its ideal setting, a citizen was active in public life and fundamentally willing to submit his
private interests to the general interest of society.
The concept of citizenship had undergone changes over the centuries. In the 18th
century, the concept was limited, by and large, to civil citizenship, which established the rights
necessary for individual freedom, such as rights to property, personal liberty and justice.[9] Its
meaning expanded during the 19th century to include political citizenship, which
encompassed the right to participate in the exercise of political power.[10] The 20th century
saw the next stage of the development of social citizenship, which laid emphasis on the right
of the citizen to economic well-being and social security.[11] The idea of citizenship has gained
expression in the modern welfare state as it so developed in Western Europe. An ongoing and
final stage of development, in keeping with the rapidly shrinking global village, might well be
the internationalization of citizenship.[12]

The Local Setting - from Spanish


Times to the Present
There was no such term as "Philippine citizens" during the Spanish regime but "subjects
of Spain" or "Spanish subjects."[13] In church records, the natives were called 'indios',
denoting a low regard for the inhabitants of the archipelago. Spanish laws on citizenship
became highly codified during the 19th century but their sheer number made it difficult to
point to one comprehensive law. Not all of these citizenship laws of Spain however, were
made to apply to the Philippine Islands except for those explicitly extended by Royal
Decrees.[14]
Spanish laws on citizenship were traced back to the Novisima Recopilacion, promulgated
in Spain on 16 July 1805 but as to whether the law was extended to the Philippines remained
to be the subject of differing views among experts; [15] however, three royal decrees were
undisputably made applicable to Spaniards in the Philippines - the Order de la Regencia of 14
August 1841,[16] the Royal Decree of 23 August 1868 specifically defining the political status
of children born in the Philippine Islands,[17] and finally, the Ley Extranjera de Ultramar of 04
July 1870, which was expressly made applicable to the Philippines by the Royal Decree of 13
July 1870.[18]
The Spanish Constitution of 1876 was never extended to the Philippine Islands because
of the express mandate of its Article 89, according to which the provisions of
the Ultramaramong which this country was included, would be governed by special laws. [19]
It was only the Civil Code of Spain, made effective in this jurisdiction on 18 December
1889, which came out with the first categorical enumeration of who were Spanish citizens. -

(a) Persons born in Spanish territory,

(b) Children of a Spanish father or mother, even if they were born outside of Spain,

(c) Foreigners who have obtained naturalization papers,

(d) Those who, without such papers, may have become domiciled inhabitants of any
town of the Monarchy.[20]

The year 1898 was another turning point in Philippine history. Already in the state of
decline as a superpower, Spain was forced to so cede her sole colony in the East to an
upcoming world power, the United States. An accepted principle of international law dictated
that a change in sovereignty, while resulting in an abrogation of all political laws then in force,
would have no effect on civil laws, which would remain virtually intact.
The Treaty of Paris was entered into on 10 December 1898 between Spain and the United
States.[21] Under Article IX of the treaty, the civil rights and political status of the native
inhabitants of the territories ceded to the United States would be determined by its Congress
-

"Spanish subjects, natives of the Peninsula, residing in the territory over which Spain by the
present treaty relinquishes or cedes her sovereignty may remain in such territory or may
remove therefrom, retaining in either event all their rights of property, including the right to
sell or dispose of such property or of its proceeds; and they shall also have the right to carry
on their industry, commerce, and professions, being subject in respect thereof to such laws
as are applicable to foreigners. In case they remain in the territory they may preserve their
allegiance to the Crown of Spain by making, before a court of record, within a year from the
date of the exchange of ratifications of this treaty, a declaration of their decision to preserve
such allegiance; in default of which declaration they shall be held to have renounced it and to
have adopted the nationality of the territory in which they reside.
Thus

"The civil rights and political status of the native inhabitants of the territories hereby ceded
to the United States shall be determined by the Congress."[22]

Upon the ratification of the treaty, and pending legislation by the United States Congress on
the subject, the native inhabitants of the Philippines ceased to be Spanish subjects. Although
they did not become American citizens, they, however, also ceased to be "aliens" under
American laws and were thus issued passports describing them to be citizens of the Philippines
entitled to the protection of the United States.
The term "citizens of the Philippine Islands" appeared for the first time in the Philippine
Bill of 1902, also commonly referred to as the Philippine Organic Act of 1902, the first
comprehensive legislation of the Congress of the United States on the Philippines -

".... that all inhabitants of the Philippine Islands continuing to reside therein, who were
Spanish subjects on the 11th day of April, 1891, and then resided in said Islands, and their
children born subsequent thereto, shall be deemed and held to be citizens of the Philippine
Islands and as such entitled to the protection of the United States, except such as shall have
elected to preserve their allegiance to the Crown of Spain in accordance with the provisions
of the treaty of peace between the United States and Spain, signed at Paris, December tenth
eighteen hundred and ninety eight."[23]

Under the organic act, a citizen of the Philippines was one who was an inhabitant of the
Philippines, and a Spanish subject on the 11 th day of April 1899. The term inhabitant was
taken to include 1) a native-born inhabitant, 2) an inhabitant who was a native of Peninsular
Spain, and 3) an inhabitant who obtained Spanish papers on or before 11 April 1899. [24]
Controversy arose on to the status of children born in the Philippines from 11 April 1899
to 01 July 1902, during which period no citizenship law was extant in the Philippines. Weight
was given to the view, articulated in jurisprudential writing at the time, that the common law
principle of jus soli, otherwise also known as the principle of territoriality, operative in the
United States and England, governed those born in the Philippine Archipelago within that
period.[25] More about this later.
In 23 March 1912, the Congress of the United States made the following amendment to
the Philippine Bill of 1902 -

"Provided, That the Philippine Legislature is hereby authorized to provide by law for the
acquisition of Philippine citizenship by those natives of the Philippine Islands who do not come
within the foregoing provisions, the natives of other insular possession of the United States,
and such other persons residing in the Philippine Islands who would become citizens of the
United States, under the laws of the United States, if residing therein."[26]

With the adoption of the Philippine Bill of 1902, the concept of "Philippine citizens" had
for the first time crystallized. The word "Filipino" was used by William H. Taft, the first Civil
Governor General in the Philippines when he initially made mention of it in his slogan, "The
Philippines for the Filipinos." In 1916, the Philippine Autonomy Act, also known as the Jones
Law restated virtually the provisions of the Philippine Bill of 1902, as so amended by the Act
of Congress in 1912 -

That all inhabitants of the Philippine Islands who were Spanish subjects on the
eleventh day of April, eighteen hundred and ninety-nine, and then resided in said
Islands, and their children born subsequently thereto, shall be deemed and held to
be citizens of the Philippine Islands, except such as shall have elected to preserve their
allegiance to the Crown of Spain in accordance with the provisions of the treaty of peace
between the United States and Spain, signed at Paris December tenth, eighteen hundred and
ninety-eight and except such others as have since become citizens of some other country;
Provided, That the Philippine Legislature, herein provided for, is hereby authorized to provide
for the acquisition of Philippine citizenship by those natives of the Philippine Islands who do
not come within the foregoing provisions, the natives of the insular possessions of the United
States, and such other persons residing in the Philippine Islands who are citizens of the United
States, or who could become citizens of the United States under the laws of the United States,
if residing therein."

Under the Jones Law, a native-born inhabitant of the Philippines was deemed to be a
citizen of the Philippines as of 11 April 1899 if he was 1) a subject of Spain on 11 April 1899,
2) residing in the Philippines on said date, and, 3) since that date, not a citizen of some other
country.
While there was, at one brief time, divergent views on whether or not jus soli was a mode
of acquiring citizenship, the 1935 Constitution brought to an end to any such link with common
law, by adopting, once and for all, jus sanguinis or blood relationship as being the basis of
Filipino citizenship -

Section 1, Article III, 1935 Constitution. The following are citizens of the Philippines -

(1) Those who are citizens of the Philippine Islands at the time of the adoption of this
Constitution

(2) Those born in the Philippines Islands of foreign parents who, before the adoption of this
Constitution, had been elected to public office in the Philippine Islands.

(3) Those whose fathers are citizens of the Philippines.

(4) Those whose mothers are citizens of the Philippines and upon reaching the age of majority,
elect Philippine citizenship.

(5) Those who are naturalized in accordance with law.

Subsection (4), Article III, of the 1935 Constitution, taken together with existing civil law
provisions at the time, which provided that women would automatically lose their Filipino
citizenship and acquire that of their foreign husbands, resulted in discriminatory situations
that effectively incapacitated the women from transmitting their Filipino citizenship to their
legitimate children and required illegitimate children of Filipino mothers to still elect Filipino
citizenship upon reaching the age of majority. Seeking to correct this anomaly, as well as fully
cognizant of the newly found status of Filipino women as equals to men, the framers of the
1973 Constitution crafted the provisions of the new Constitution on citizenship to reflect such
concerns -

Section 1, Article III, 1973 Constitution - The following are citizens of the Philippines:

(1) Those who are citizens of the Philippines at the time of the adoption of this Constitution.

(2) Those whose fathers or mothers are citizens of the Philippines.


(3) Those who elect Philippine citizenship pursuant to the provisions of the Constitution of
nineteen hundred and thirty-five.

(4) Those who are naturalized in accordance with law.

For good measure, Section 2 of the same article also further provided that

"A female citizen of the Philippines who marries an alien retains her Philippine citizenship,
unless by her act or omission she is deemed, under the law to have renounced her
citizenship."

The 1987 Constitution generally adopted the provisions of the 1973 Constitution, except
for subsection (3) thereof that aimed to correct the irregular situation generated by the
questionable proviso in the 1935 Constitution.
Section I, Article IV, 1987 Constitution now provides:

The following are citizens of the Philippines:

(1) Those who are citizens of the Philippines at the time of the adoption of this Constitution.

(2) Those whose fathers or mothers are citizens of the Philippines.

(3) Those born before January 17, 1973 of Filipino mothers, who elect Philippine
citizenship upon reaching the age of majority; and

(4) Those who are naturalized in accordance with law.

The Case Of FPJ

Section 2, Article VII, of the 1987 Constitution expresses:

"No person may be elected President unless he is a natural-born citizen of the Philippines,
a registered voter, able to read and write, at least forty years of age on the day of the election,
and a resident of the Philippines for at least ten years immediately preceding such election."

The term "natural-born citizens," is defined to include "those who are citizens of the
Philippines from birth without having to perform any act to acquire or perfect their Philippine
citizenship."[27]
The date, month and year of birth of FPJ appeared to be 20 August 1939 during the
regime of the 1935 Constitution. Through its history, four modes of acquiring citizenship -
naturalization, jus soli, res judicata and jus sanguinis[28] had been in vogue. Only
two, i.e., jus soli and jus sanguinis, could qualify a person to being a natural-born citizen of
the Philippines.Jus soli, per Roa vs. Collector of Customs[29] (1912), did not last long. With
the adoption of the 1935 Constitution and the reversal of Roa in Tan Chong vs. Secretary of
Labor[30] (1947), jus sanguinis or blood relationship would now become the primary basis of
citizenship by birth.
Documentary evidence adduced by petitioner would tend to indicate that the earliest
established direct ascendant of FPJ was his paternal grandfather Lorenzo Pou, married to
Marta Reyes, the father of Allan F. Poe. While the record of birth of Lorenzo Pou had not been
presented in evidence, his death certificate, however, identified him to be a Filipino, a resident
of San Carlos, Pangasinan, and 84 years old at the time of his death on 11 September
1954. The certificate of birth of the father of FPJ, Allan F. Poe, showed that he was born on
17 May 1915 to an Espaol father, Lorenzo Pou, and a mestiza Espaol mother, Marta
Reyes. Introduced by petitioner was an uncertified copy of a supposed certificate of the
alleged marriage of Allan F. Poe and Paulita Gomez on 05 July 1936. The marriage certificate
of Allan F. Poe and Bessie Kelley reflected the date of their marriage to be on 16 September
1940. In the same certificate, Allan F. Poe was stated to be twenty-five years old, unmarried,
and a Filipino citizen, and Bessie Kelley to be twenty-two years old, unmarried, and an
American citizen. The birth certificate of FPJ, would disclose that he was born on 20 August
1939 to Allan F. Poe, a Filipino, twenty-four years old, married to Bessie Kelly, an American
citizen, twenty-one years old and married.
Considering the reservations made by the parties on the veracity of some of the entries
on the birth certificate of respondent and the marriage certificate of his parents, the only
conclusions that could be drawn with some degree of certainty from the documents would be
that -

1. The parents of FPJ were Allan F. Poe and Bessie Kelley;

2. FPJ was born to them on 20 August 1939;

3. Allan F. Poe and Bessie Kelley were married to each other on 16 September, 1940;

4. The father of Allan F. Poe was Lorenzo Poe; and

5. At the time of his death on 11 September 1954, Lorenzo Poe was 84 years old.

Would the above facts be sufficient or insufficient to establish the fact that FPJ is a natural-
born Filipino citizen? The marriage certificate of Allan F. Poe and Bessie Kelley, the birth
certificate of FPJ, and the death certificate of Lorenzo Pou are documents of public record in
the custody of a public officer. The documents have been submitted in evidence by both
contending parties during the proceedings before the COMELEC.
The birth certificate of FPJ was marked Exhibit "A" for petitioner and Exhibit "3" for
respondent. The marriage certificate of Allan F. Poe to Bessie Kelley was submitted as Exhibit
"21" for respondent. The death certificate of Lorenzo Pou was submitted by respondent as his
Exhibit "5." While the last two documents were submitted in evidence for respondent, the
admissibility thereof, particularly in reference to the facts which they purported to
show, i.e., the marriage certificate in relation to the date of marriage of Allan F. Poe to Bessie
Kelley and the death certificate relative to the death of Lorenzo Pou on 11 September 1954
in San Carlos, Pangasinan, were all admitted by petitioner, who had utilized those material
statements in his argument. All three documents were certified true copies of the originals.

Section 3, Rule 130, Rules of Court states that -

Original document must be produced; exceptions. - When the subject of inquiry is the
contents of a document, no evidence shall be admissible other than the original document
itself, except in the following cases:
xxxxxxxxx

(d) When the original is a public record in the custody of a public office or is recorded in a
public office.

Being public documents, the death certificate of Lorenzo Pou, the marriage certificate of Allan
F. Poe and Bessie Kelly, and the birth certificate of FPJ, constitute prima facie proof of their
contents. Section 44, Rule 130, of the Rules of Court provides:

Entries in official records. Entries in official records made in the performance of his duty by a
public officer of the Philippines, or by a person in the performance of a duty specially enjoined
by law, are prima facie evidence of the facts therein stated.

The trustworthiness of public documents and the value given to the entries made therein
could be grounded on 1) the sense of official duty in the preparation of the statement made,
2) the penalty which is usually affixed to a breach of that duty, 3) the routine and disinterested
origin of most such statements, and 4) the publicity of record which makes more likely the
prior exposure of such errors as might have occurred.[31]
The death certificate of Lorenzo Pou would indicate that he died on 11 September 1954,
at the age of 84 years, in San Carlos, Pangasinan. It could thus be assumed that Lorenzo Pou
was born sometime in the year 1870 when the Philippines was still a colony of
Spain. Petitioner would argue that Lorenzo Pou was not in the Philippines during the crucial
period of from 1898 to 1902 considering that there was no existing record about such fact in
the Records Management and Archives Office. Petitioner, however, likewise failed to show
that Lorenzo Pou was at any other place during the same period. In his death certificate, the
residence of Lorenzo Pou was stated to be San Carlos, Pangasinan. In the absence of any
evidence to the contrary, it should be sound to conclude, or at least to presume, that the
place of residence of a person at the time of his death was also his residence before death. It
would be extremely doubtful if the Records Management and Archives Office would have had
complete records of all residents of the Philippines from 1898 to 1902.

Proof of Paternity and Filiation


Under Civil Law.

Petitioner submits, in any case, that in establishing filiation (relationship or civil status of
the child to the father [or mother]) or paternity (relationship or civil status of the father to
the child) of an illegitimate child, FPJ evidently being an illegitimate son according to
petitioner, the mandatory rules under civil law must be used.
Under the Civil Code of Spain, which was in force in the Philippines from 08 December
1889 up until the day prior to 30 August 1950 when the Civil Code of the Philippines took
effect, acknowledgment was required to establish filiation or paternity. Acknowledgment was
either judicial (compulsory) or voluntary. Judicial or compulsory acknowledgment was
possible only if done during the lifetime of the putative parent; voluntary acknowledgment
could only be had in a record of birth, a will, or a public document.[32] Complementary to the
new code was Act No. 3753 or the Civil Registry Law expressing in Section 5 thereof, that -

In case of an illegitimate child, the birth certificate shall be signed and sworn to jointly by
the parents of the infant or only by the mother if the father refuses. In the latter case, it shall
not be permissible to state or reveal in the document the name of the father who refuses to
acknowledge the child, or to give therein any information by which such father could be
identified.

In order that the birth certificate could then be utilized to prove voluntary acknowledgment
of filiation or paternity, the certificate was required to be signed or sworn to by the father. The
failure of such requirement rendered the same useless as being an authoritative document of
recognition.[33] In Mendoza vs. Mella,[34] the Court ruled -

"Since Rodolfo was born in 1935, after the registry law was enacted, the question here really
is whether or not his birth certificate (Exhibit 1), which is merely a certified copy of the registry
record, may be relied upon as sufficient proof of his having been voluntarily recognized. No
such reliance, in our judgment, may be placed upon it. While it contains the names of both
parents, there is no showing that they signed the original, let alone swore to its contents as
required in Section 5 of Act No. 3753. For all that might have happened, it was not even they
or either of them who furnished the data to be entered in the civil register. Petitioners say
that in any event the birth certificate is in the nature of a public document wherein voluntary
recognition of a natural child may also be made, according to the same Article 131. True
enough, but in such a case, there must be a clear statement in the document that the parent
recognizes the child as his or her own."

In the birth certificate of respondent FPJ, presented by both parties, nowhere in the
document was the signature of Allan F. Poe found. There being no will apparently executed,
or at least shown to have been executed, by decedent Allan F. Poe, the only other proof of
voluntary recognition remained to be "some other public document." In Pareja vs.
Pareja,[35] this Court defined what could constitute such a document as proof of voluntary
acknowledgment:

"Under the Spanish Civil Code there are two classes of public documents, those executed by
private individuals which must be authenticated by notaries, and those issued by
competent public officials by reason of their office. The public document pointed out in Article
131 as one of the means by which recognition may be made belongs to the first class."

Let us leave it at that for the moment.


The 1950 Civil Code categorized the acknowledgment or recognition of illegitimate
children into voluntary, legal or compulsory. Voluntary recognition was required to be
expressedly made in a record of birth, a will, a statement before a court of record or in any
authentic writing. Legal acknowledgment took place in favor of full blood brothers and sisters
of an illegitimate child who was recognized or judicially declared as natural. Compulsory
acknowledgment could be demanded generally in cases when the child had in his favor any
evidence to prove filiation. Unlike an action to claim legitimacy which would last during the
lifetime of the child, and might pass exceptionally to the heirs of the child, an action to claim
acknowledgment, however, could only be brought during the lifetime of the presumed parent.
Amicus Curiae Ruben F. Balane defined, during the oral argument, "authentic writing," so
as to be an authentic writing for purposes of voluntary recognition, simply as being a genuine
or indubitable writing of the father. The term would include a public instrument (one duly
acknowledged before a notary public or other competent official) or a private writing admitted
by the father to be his.
The Family Code has further liberalized the rules; Article 172, Article 173, and Article 175
provide:
Art. 172. The filiation of legitimate children is established by any of the following:

(1) The record of birth appearing in the civil register or a final judgment; or

(2) An admission of legitimate filiation in a public document or a private handwritten


instrument and signed by the parent concerned.

In the absence of the foregoing evidence, the legitimate filiation shall be proved by:

(1) The open and continuous possession of the status of a legitimate child; or

(2) Any other means allowed by the Rules of Court and special laws.

Art. 173. The action to claim legitimacy may be brought by the child during his or her lifetime
and shall be transmitted to the heirs should the child die during minority or in a state of
insanity. In these cases, the heirs shall have a period of five years within which to institute
the action.

The action already commenced by the child shall survive notwithstanding the death of either
or both of the parties.

x x x x x x x x x.

Art. 175. Illegitimate children may establish their illegitimate filiation in the same way and on
the same, evidence as legitimate children.

The action must be brought within the same period specified in Article 173, except when the
action is based on the second paragraph of Article 172, in which case the action may be
brought during the lifetime of the alleged parent.

The provisions of the Family Code are retroactively applied; Article 256 of the code reads:

"Art. 256. This Code shall have retroactive effect insofar as it does not prejudice or impair
vested or acquired rights in accordance with the Civil Code or other laws.

Thus, in Vda. de Sy-Quia vs. Court of Appeals,[36] the Court has ruled:

"We hold that whether Jose was a voluntarily recognized natural child should be decided under
Article 278 of the Civil Code of the Philippines. Article 2260 of that Code provides that 'the
voluntary recognition of a natural child shall take place according to this Code, even if the
child was born before the effectivity of this body of laws' or before August 30, 1950. Hence,
Article 278 may be given retroactive effect."

It should be apparent that the growing trend to liberalize the acknowledgment or


recognition of illegitimate children is an attempt to break away from the traditional idea of
keeping well apart legitimate and non-legitimate relationships within the family in favor of the
greater interest and welfare of the child. The provisions are intended to merely govern the
private and personal affairs of the family. There is little, if any, to indicate that the legitimate
or illegitimate civil status of the individual would also affect his political rights or, in general,
his relationship to the State. While, indeed, provisions on "citizenship" could be found in the
Civil Code, such provisions must be taken in the context of private relations, the domain of
civil law; particularly -

"Civil Law is that branch of law which has for its double purpose the organization of the family
and the regulation of property. It has thus [been] defined as the mass of precepts which
determine and regulate the relations of assistance, authority and obedience among members
of a family, and those which exist among members of a society for the protection of private
interests."[37]

In Yaez de Barnuevo vs. Fuster,[38] the Court has held:

"In accordance with Article 9 of the Civil Code of Spain, x x x the laws relating to family rights
and duties, or to the status, condition and legal capacity of persons, govern Spaniards
although they reside in a foreign country; that, in consequence, 'all questions of a civil nature,
such as those dealing with the validity or nullity of the matrimonial bond, the domicile of the
husband and wife, their support, as between them, the separation of their properties, the
rules governing property, marital authority, division of conjugal property, the classification of
their property, legal causes for divorce, the extent of the latter, the authority to decree it,
and, in general, the civil effects of marriage and divorce upon the persons and properties of
the spouses, are questions that are governed exclusively by the national law of the husband
and wife."

The relevance of "citizenship" or "nationality" to Civil Law is best exemplified in Article 15


of the Civil Code, stating that -

"Laws relating to family rights and duties, or to the status, condition and legal capacity of
persons are binding upon citizens of the Philippines, even though living abroad" -

that explains the need to incorporate in the code a reiteration of the Constitutional provisions
on citizenship. Similarly, citizenship is significant in civil relationships found in different parts
of the Civil Code,[39] such as on successional rights and family relations.[40] In adoption, for
instance, an adopted child would be considered the child of his adoptive parents and accorded
the same rights as their legitimate child but such legal fiction extended only to define his
rights under civil law[41] and not his political status.
Civil law provisions point to an obvious bias against illegitimacy. This discriminatory
attitude may be traced to the Spanish family and property laws, which, while defining
proprietary and successional rights of members of the family, provided distinctions in the
rights of legitimate and illegitimate children. In the monarchial set-up of old Spain, the
distribution and inheritance of titles and wealth were strictly according to bloodlines and the
concern to keep these bloodlines uncontaminated by foreign blood was paramount.
These distinctions between legitimacy and illegitimacy were codified in the Spanish Civil
Code, and the invidious discrimination survived when the Spanish Civil Code became the
primary source of our own Civil Code. Such distinction, however, remains and should remain
only in the sphere of civil law and not unduly impede or impinge on the domain of political
law.
The proof of filiation or paternity for purposes of determining his citizenship status should
thus be deemed independent from and not inextricably tied up with that prescribed for civil
law purposes. The Civil Code or Family Code provisions on proof of filiation or paternity,
although good law, do not have preclusive effects on matters alien to personal and family
relations. The ordinary rules on evidence could well and should govern. For instance, the
matter about pedigree is not necessarily precluded from being applicable by the Civil Code or
Family Code provisions.
Section 39, Rule 130, of the Rules of Court provides -

Act or Declaration about pedigree. The act or declaration of a person deceased, or unable to
testify, in respect to the pedigree of another person related to him by birth or marriage, may
be received in evidence where it occurred before the controversy, and the relationship
between the two persons is shown by evidence other than such act or declaration. The word
`pedigree includes relationship, family genealogy, birth, marriage, death, the dates when and
the places where these facts occurred, and the names of the relatives. It embraces also facts
of family history intimately connected with pedigree.

For the above rule to apply, it would be necessary that (a) the declarant is already dead
or unable to testify, (b) the pedigree of a person must be at issue, (c) the declarant must be
a relative of the person whose pedigree is in question, (d) declaration must be made before
the controversy has occurred, and (e) the relationship between the declarant and the person
whose pedigree is in question must be shown by evidence other than such act or declaration.
Thus, the duly notarized declaration made by Ruby Kelley Mangahas, sister of Bessie
Kelley Poe submitted as Exhibit 20 before the COMELEC, might be accepted to prove the acts
of Allan F. Poe, recognizing his own paternal relationship with FPJ, i.e, living together with
Bessie Kelley and his children (including respondent FPJ) in one house, and as one family -

"I, Ruby Kelley Mangahas, of legal age and sound mind, presently residing in Stockton,
California, U.S.A., after being sworn in accordance with law do hereby declare that:

1. I am the sister of the late Bessie Kelley Poe.

2. Bessie Kelley Poe was the wife of Fernando Poe, Sr.

3. Fernando and Bessie Poe had a son by the name of Ronald Allan Poe, more
popularly known in the Philippines as `Fernando Poe, Jr., or `FPJ.

4. Ronald Allan Poe `FPJ was born on August 20, 1939 at St. Luke's Hospital,
Magdalena Street, Manila.

xxxxxxxxx

7. Fernando Poe Sr., and my sister Bessie, met and became engaged while they were
students at the University of the Philippines in 1936. I was also introduced to
Fernando Poe, Sr., by my sister that same year.

8. Fernando Poe, Sr., and my sister Bessie had their first child in 1938.

9. Fernando Poe, Sr., my sister Bessie and their first three children, Elizabeth,
Ronald, Allan and Fernando II, and myself lived together with our mother at
our family's house on Dakota St. (now Jorge Bocobo St.), Malate until the
liberation of Manila in 1945, except for some months between 1943-1944.

10. Fernando Poe, Sr., and my sister, Bessie, were blessed with four (4) more
children after Ronald Allan Poe.
xxxxxxxxx

18. I am executing this Declaration to attest to the fact that my nephew, Ronald Allan
Poe is a natural born Filipino, and that he is the legitimate child of Fernando
Poe, Sr.

Done in City of Stockton, California, U.S.A., this 12th day of January 2004.

Ruby Kelley Mangahas

Declarant

DNA Testing

In case proof of filiation or paternity would be unlikely to satisfactorily establish or would


be difficult to obtain, DNA testing, which examines genetic codes obtained from body cells of
the illegitimate child and any physical residue of the long dead parent could be resorted to. A
positive match would clear up filiation or paternity. In Tijing vs. Court of Appeals,[42] this Court
has acknowledged the strong weight of DNA testing -

"Parentage will still be resolved using conventional methods unless we adopt the modern and
scientific ways available. Fortunately, we have now the facility and expertise in using DNA
test for identification and parentage testing. The University of the Philippines Natural Science
Research Institute (UP-NSRI) DNA Analysis Laboratory has now the capability to conduct DNA
typing using short tandem repeat (STR) analysis. The analysis is based on the fact that the
DNA of a child/person has two (2) copies, one copy from the mother and the other from the
father. The DNA from the mother, the alleged father and the child are analyzed to establish
parentage. Of course, being a novel scientific technique, the use of DNA test as evidence is
still open to challenge. Eventually, as the appropriate case comes, courts should not hesitate
to rule on the admissibility of DNA evidence. For it was said, that courts should apply the
results of science when competently obtained in aid of situations presented, since to reject
said result is to deny progress."

Petitioners Argument For


Jurisprudential Conclusiveness

Petitioner would have it that even if Allan F. Poe were a Filipino citizen, he could not have
transmitted his citizenship to respondent FPJ, the latter being an illegitimate child. According
to petitioner, prior to his marriage to Bessie Kelley, Allan F. Poe, on July 5, 1936, contracted
marriage with a certain Paulita Gomez, making his subsequent marriage to Bessie Kelley
bigamous and respondent FPJ an illegitimate child. The veracity of the supposed certificate of
marriage between Allan F. Poe and Paulita Gomez could be most doubtful at best. But the
documentary evidence introduced by no less than respondent himself, consisting of a birth
certificate of respondent and a marriage certificate of his parents showed that FPJ was born
on 20 August 1939 to a Filipino father and an American mother who were married to each
other a year later, or on 16 September 1940. Birth to unmarried parents would make FPJ an
illegitimate child. Petitioner contended that as an illegitimate child, FPJ so followed the
citizenship of his mother, Bessie Kelley, an American citizen, basing his stand on the ruling of
this Court in Morano vs. Vivo,[43] citing Chiongbian vs. de Leon[44] and Serra vs. Republic.[45]
On the above score, the disquisition made by amicus curiae Joaquin G. Bernas, SJ, is
most convincing; he states -

"We must analyze these cases and ask what the lis mota was in each of them. If the
pronouncement of the Court on jus sanguinis was on the lis mota, the pronouncement would
be a decision constituting doctrine under the rule of stare decisis. But if the pronouncement
was irrelevant to the lis mota, the pronouncement would not be a decision but a mere obiter
dictum which did not establish doctrine. I therefore invite the Court to look closely into these
cases.

First, Morano vs. Vivo. The case was not about an illegitimate child of a Filipino father. It was
about a stepson of a Filipino, a stepson who was the child of a Chinese mother and a Chinese
father. The issue was whether the stepson followed the naturalization of the
stepfather. Nothing about jus sanguinis there. The stepson did not have the blood of the
naturalized stepfather.

Second, Chiongbian vs. de Leon. This case was not about the illegitimate son of a Filipino
father. It was about a legitimate son of a father who had become Filipino by election to public
office before the 1935 Constitution pursuant to Article IV, Section 1(2) of the 1935
Constitution. No one was illegitimate here.

Third, Serra vs. Republic. The case was not about the illegitimate son of a Filipino
father. Serra was an illegitimate child of a Chinese father and a Filipino mother. The issue was
whether one who was already a Filipino because of his mother who still needed to be
naturalized. There is nothing there about invidious jus sanguinis.

Finally, Paa vs. Chan.[46] This is a more complicated case. The case was about the citizenship
of Quintin Chan who was the son of Leoncio Chan. Quintin Chan claimed that his father,
Leoncio, was the illegitimate son of a Chinese father and a Filipino mother. Quintin therefore
argued that he got his citizenship from Leoncio, his father. But the Supreme Court said that
there was no valid proof that Leoncio was in fact the son of a Filipina mother. The Court
therefore concluded that Leoncio was not Filipino. If Leoncio was not Filipino, neither was his
son Quintin. Quintin therefore was not only not a natural-born Filipino but was not even a
Filipino.

The Court should have stopped there. But instead it followed with an obiter dictum. The Court
said obiter that even if Leoncio, Quintin's father, were Filipino, Quintin would not be Filipino
because Quintin was illegitimate. This statement about Quintin, based on a contrary to fact
assumption, was absolutely unnecessary for the case. x x x It was obiter dictum, pure and
simple, simply repeating the obiter dictum in Morano vs. Vivo.

xxxxxxxxx

"Aside from the fact that such a pronouncement would have no textual foundation in the
Constitution, it would also violate the equal protection clause of the Constitution not once but
twice. First, it would make an illegitimate distinction between a legitimate child and an
illegitimate child, and second, it would make an illegitimate distinction between the
illegitimate child of a Filipino father and the illegitimate child of a Filipino mother.
The doctrine on constitutionally allowable distinctions was established long ago by People vs.
Cayat.[47] I would grant that the distinction between legitimate children and illegitimate
children rests on real differences. x x x But real differences alone do not justify invidious
distinction. Real differences may justify distinction for one purpose but not for another
purpose.

x x x What is the relevance of legitimacy or illegitimacy to elective public service? What


possible state interest can there be for disqualifying an illegitimate child from becoming a
public officer. It was not the fault of the child that his parents had illicit liaison. Why deprive
the child of the fullness of political rights for no fault of his own? To disqualify an illegitimate
child from holding an important public office is to punish him for the indiscretion of his
parents. There is neither justice nor rationality in that. And if there is neither justice nor
rationality in the distinction, then the distinction transgresses the equal protection clause and
must be reprobated.

The other amici curiae, Mr. Justice Vicente Mendoza (a former member of this Court),
Professor Ruben Balane and Dean Martin Magallona, at bottom, have expressed similar
views.The thesis of petitioner, unfortunately hinging solely on pure obiter dicta, should indeed
fail.
Where jurisprudence regarded an illegitimate child as taking after the citizenship of its
mother, it did so for the benefit the child. It was to ensure a Filipino nationality for the
illegitimate child of an alien father in line with the assumption that the mother had custody,
would exercise parental authority and had the duty to support her illegitimate child. It was to
help the child, not to prejudice or discriminate against him.
The fact of the matter perhaps the most significant consideration is that the 1935
Constitution, the fundamental law prevailing on the day, month and year of birth of
respondent FPJ, can never be more explicit than it is. Providing neither conditions nor
distinctions, the Constitution states that among the citizens of the Philippines are those whose
fathers are citizens of the Philippines. There utterly is no cogent justification to prescribe
conditions or distinctions where there clearly are none provided.

In Sum

(1) The Court, in the exercise of its power of judicial review, possesses jurisdiction over
the petition in G. R. No. 161824, filed under Rule 64, in relation to Rule 65, of the Revised
Rules of Civil Procedure. G.R. No. 161824 assails the resolution of the COMELEC for alleged
grave abuse of discretion in dismissing, for lack of merit, the petition in SPA No. 04-003 which
has prayed for the disqualification of respondent FPJ from running for the position of President
in the 10th May 2004 national elections on the contention that FPJ has committed material
representation in his certificate of candidacy by representing himself to be a natural-born
citizen of the Philippines.
(2) The Court must dismiss, for lack of jurisdiction and prematurity, the petitions in G. R.
No. 161434 and No. 161634 both having been directly elevated to this Court in the latters
capacity as the only tribunal to resolve a presidential and vice-presidential election contest
under the Constitution. Evidently, the primary jurisdiction of the Court can directly be invoked
only after, not before, the elections are held.
(3) In ascertaining, in G.R. No. 161824, whether grave abuse of discretion has been
committed by the COMELEC, it is necessary to take on the matter of whether or not
respondent FPJ is a natural-born citizen, which, in turn, depended on whether or not the
father of respondent, Allan F. Poe, would have himself been a Filipino citizen and, in the
affirmative, whether or not the alleged illegitimacy of respondent prevents him from taking
after the Filipino citizenship of his putative father. Any conclusion on the Filipino citizenship
of Lorenzo Pou could only be drawn from the presumption that having died in 1954 at 84
years old, Lorenzo would have been born sometime in the year 1870, when the Philippines
was under Spanish rule, and that San Carlos, Pangasinan, his place of residence upon his
death in 1954, in the absence of any other evidence, could have well been his place of
residence before death, such that Lorenzo Pou would have benefited from the en
masse Filipinization that the Philippine Bill had effected in 1902. That citizenship (of Lorenzo
Pou), if acquired, would thereby extend to his son, Allan F. Poe, father of respondent FPJ. The
1935 Constitution, during which regime respondent FPJ has seen first light, confers citizenship
to all persons whose fathers are Filipino citizens regardless of whether such children are
legitimate or illegitimate.
(4) But while the totality of the evidence may not establish conclusively that respondent
FPJ is a natural-born citizen of the Philippines, the evidence on hand still would preponderate
in his favor enough to hold that he cannot be held guilty of having made a material
misrepresentation in his certificate of candidacy in violation of Section 78, in relation to
Section 74, of the Omnibus Election Code. Petitioner has utterly failed to substantiate his case
before the Court, notwithstanding the ample opportunity given to the parties to present their
position and evidence, and to prove whether or not there has been material
misrepresentation, which, as so ruled in Romualdez-Marcos vs. COMELEC,[48] must not only
be material, but also deliberate and willful.
WHEREFORE, the Court RESOLVES to DISMISS
1. G. R. No. 161434, entitled "Maria Jeanette C. Tecson and Felix B. Desiderio, Jr.,
Petitioners, versus Commission on Elections, Ronald Allan Kelley Poe (a.k.a. "Fernando Poe,
Jr.,) and Victorino X. Fornier, Respondents," and G. R. No. 161634, entitled "Zoilo Antonio
Velez, Petitioner, versus Ronald Allan Kelley Poe, a.k.a. Fernando Poe, Jr., Respondent," for
want of jurisdiction.
2. G. R. No. 161824, entitled Victorino X. Fornier, Petitioner, versus Hon. Commission on
Elections and Ronald Allan Kelley Poe, also known as Fernando Poe, Jr., for failure to show
grave abuse of discretion on the part of respondent Commission on Elections in dismissing
the petition in SPA No. 04-003.
No Costs.
SO ORDERED.
[G.R. No. 153883. January 13, 2004]

REPUBLIC OF THE PHILIPPINES, petitioner, vs. CHULE Y. LIM, respondent.

DECISION
YNARES-SANTIAGO, J.:

This petition for review on certiorari under Rule 45 of the Rules of Court stemmed from a
petition for correction of entries under Rule 108 of the Rules of Court filed by respondent
Chule Y. Lim with the Regional Trial Court of Lanao del Norte, Branch 4, docketed as Sp. Proc.
No. 4933.
In her petition, respondent claimed that she was born on October 29, 1954 in Buru-an,
Iligan City. Her birth was registered in Kauswagan, Lanao del Norte but the Municipal Civil
Registrar of Kauswagan transferred her record of birth to Iligan City. She alleged that both
her Kauswagan and Iligan City records of birth have four erroneous entries, and prays that
they be corrected.
The trial court then issued an Order,[1] which reads:

WHEREFORE, finding the petition to be sufficient in form and substance, let the hearing of
this case be set on December 27, 1999 before this Court, Hall of Justice, Rosario Heights,
Tubod, Iligan City at 8:30 oclock in the afternoon at which date, place and time any interested
person may appear and show cause why the petition should not be granted.

Let this order be published in a newspaper of general circulation in the City of Iligan and the
Province of Lanao del Norte once a week for three (3) consecutive weeks at the expense of
the petitioner.

Furnish copies of this order the Office of the Solicitor General at 134 Amorsolo St., Legaspi
Vill., Makati City and the Office of the Local Civil Registrar of Iligan City at Quezon Ave., Pala-
o, Iligan City.

SO ORDERED.

During the hearing, respondent testified thus:


First, she claims that her surname Yu was misspelled as Yo. She has been using Yu in all
her school records and in her marriage certificate.[2] She presented a clearance from the
National Bureau of Investigation (NBI)[3] to further show the consistency in her use of the
surname Yu.
Second, she claims that her fathers name in her birth record was written as Yo Diu To
(Co Tian) when it should have been Yu Dio To (Co Tian).
Third, her nationality was entered as Chinese when it should have been Filipino
considering that her father and mother never got married. Only her deceased father was
Chinese, while her mother is Filipina. She claims that her being a registered voter attests to
the fact that she is a Filipino citizen.
Finally, it was erroneously indicated in her birth certificate that she was a legitimate child
when she should have been described as illegitimate considering that her parents were never
married.
Placida Anto, respondents mother, testified that she is a Filipino citizen as her parents
were both Filipinos from Camiguin. She added that she and her daughters father were never
married because the latter had a prior subsisting marriage contracted in China.
In this connection, respondent presented a certification attested by officials of the local
civil registries of Iligan City and Kauswagan, Lanao del Norte that there is no record of
marriage between Placida Anto and Yu Dio To from 1948 to the present.
The Republic, through the City Prosecutor of Iligan City, did not present any evidence
although it actively participated in the proceedings by attending hearings and cross-examining
respondent and her witnesses.
On February 22, 2000, the trial court granted respondents petition and rendered
judgment as follows:

WHEREFORE, the foregoing premises considered, to set the records of the petitioner straight
and in their proper perspective, the petition is granted and the Civil Registrar of Iligan City is
directed to make the following corrections in the birth records of the petitioner, to wit:

1. Her family name from YO to YU;

2. Her fathers name from YO DIU TO (CO TIAN) to YU DIOTO (CO TIAN);

3. Her status from legitimate to illegitimate by changing YES to NO in answer to the


question LEGITIMATE?; and,

4. Her citizenship from Chinese to Filipino.

SO ORDERED.[4]

The Republic of the Philippines appealed the decision to the Court of Appeals which
affirmed the trial courts decision.[5]
Hence, this petition on the following assigned errors:
I

THE COURT OF APPEALS ERRED IN ORDERING THE CORRECTION OF THE CITIZENSHIP OF


RESPONDENT CHULE Y. LIM FROM CHINESE TO FILIPINO DESPITE THE FACT THAT
RESPONDENT NEVER DEMONSTRATED ANY COMPLIANCE WITH THE LEGAL REQUIREMENTS
FOR ELECTION OF CITIZENSHIP.

II

THE COURT OF APPEALS ERRED IN ALLOWING RESPONDENT TO CONTINUE USING HER


FATHERS SURNAME DESPITE ITS FINDING THAT RESPONDENT IS AN ILLEGITIMATE CHILD.[6]

To digress, it is just as well that the Republic did not cite as error respondents recourse
to Rule 108 of the Rules of Court to effect what indisputably are substantial corrections and
changes in entries in the civil register. To clarify, Rule 108 of the Revised Rules of Court
provides the procedure for cancellation or correction of entries in the civil registry. The
proceedings under said rule may either be summary or adversary in nature. If the correction
sought to be made in the civil register is clerical, then the procedure to be adopted is
summary. If the rectification affects the civil status, citizenship or nationality of a party, it is
deemed substantial, and the procedure to be adopted is adversary. This is our ruling
in Republic v. Valencia[7]where we held that even substantial errors in a civil registry may be
corrected and the true facts established under Rule 108 provided the parties aggrieved by the
error avail themselves of the appropriate adversary proceeding. An appropriate adversary suit
or proceeding is one where the trial court has conducted proceedings where all relevant facts
have been fully and properly developed, where opposing counsel have been given opportunity
to demolish the opposite partys case, and where the evidence has been thoroughly weighed
and considered.[8]
As likewise observed by the Court of Appeals, we take it that the Republics failure to cite
this error amounts to a recognition that this case properly falls under Rule 108 of the Revised
Rules of Court considering that the proceeding can be appropriately classified as adversarial.
Instead, in its first assignment of error, the Republic avers that respondent did not comply
with the constitutional requirement of electing Filipino citizenship when she reached the age
of majority. It cites Article IV, Section 1(3) of the 1935 Constitution, which provides that the
citizenship of a legitimate child born of a Filipino mother and an alien father followed the
citizenship of the father, unless, upon reaching the age of majority, the child elected Philippine
citizenship.[9] Likewise, the Republic invokes the provision in Section 1 of Commonwealth Act
No. 625, that legitimate children born of Filipino mothers may elect Philippine citizenship by
expressing such intention in a statement to be signed and sworn to by the party concerned
before any officer authorized to administer oaths, and shall be filed with the nearest civil
registry. The said party shall accompany the aforesaid statement with the oath of allegiance
to the Constitution and the Government of the Philippines.[10]
Plainly, the above constitutional and statutory requirements of electing Filipino citizenship
apply only to legitimate children. These do not apply in the case of respondent who was
concededly an illegitimate child, considering that her Chinese father and Filipino mother were
never married. As such, she was not required to comply with said constitutional and statutory
requirements to become a Filipino citizen. By being an illegitimate child of a Filipino mother,
respondent automatically became a Filipino upon birth. Stated differently, she is a Filipino
since birth without having to elect Filipino citizenship when she reached the age of majority.
In Ching, Re: Application for Admission to the Bar,[11] citing In re Florencio Mallare,[12] we
held:

Esteban Mallare, natural child of Ana Mallare, a Filipina, is therefore himself a Filipino, and no
other act would be necessary to confer on him all the rights and privileges attached to
Philippine citizenship (U.S. vs. Ong Tianse, 29 Phil. 332; Santos Co vs. Government of the
Philippine Islands, 42 Phil. 543; Serra vs. Republic, L-4223, May 12, 1952; Sy Quimsuan vs.
Republic, L-4693, Feb. 16, 1953; Pitallano vs. Republic, L-5111, June 28, 1954). Neither
could any act be taken on the erroneous belief that he is a non-Filipino divest him of the
citizenship privileges to which he is rightfully entitled.[13]

This notwithstanding, the records show that respondent elected Filipino citizenship when
she reached the age of majority. She registered as a voter in Misamis Oriental when she was
18 years old.[14] The exercise of the right of suffrage and the participation in election exercises
constitute a positive act of election of Philippine citizenship.[15]
In its second assignment of error, the Republic assails the Court of Appeals decision in
allowing respondent to use her fathers surname despite its finding that she is illegitimate.
The Republics submission is misleading. The Court of Appeals did not allow respondent
to use her fathers surname. What it did allow was the correction of her fathers misspelled
surname which she has been using ever since she can remember. In this regard, respondent
does not need a court pronouncement for her to use her fathers surname.
We agree with the Court of Appeals when it held:

Firstly, Petitioner-appellee is now 47 years old. To bar her at this time from using her fathers
surname which she has used for four decades without any known objection from anybody,
would only sow confusion. Concededly, one of the reasons allowed for changing ones name
or surname is to avoid confusion.

Secondly, under Sec. 1 of Commonwealth Act No. 142, the law regulating the use of aliases,
a person is allowed to use a name by which he has been known since childhood.

Thirdly, the Supreme Court has already addressed the same issue. In Pabellar v. Rep. of the
Phils.,[16] we held:

Section 1 of Commonwealth Act No. 142, which regulates the use of aliases, allows a person
to use a name by which he has been known since childhood (Lim Hok Albano v. Republic, 104
Phil. 795; People v. Uy Jui Pio, 102 Phil. 679; Republic v. Taada, infra). Even legitimate
children cannot enjoin the illegitimate children of their father from using his surname (De
Valencia v. Rodriguez, 84 Phil. 222).[17]

While judicial authority is required for a change of name or surname, [18] there is no such
requirement for the continued use of a surname which a person has already been using since
childhood.[19]
The doctrine that disallows such change of name as would give the false impression of
family relationship remains valid but only to the extent that the proposed change of name
would in great probability cause prejudice or future mischief to the family whose surname it
is that is involved or to the community in general.[20] In this case, the Republic has not shown
that the Yu family in China would probably be prejudiced or be the object of future mischief. In
respondents case, the change in the surname that she has been using for 40 years would
even avoid confusion to her community in general.
WHEREFORE, in view of the foregoing, the instant petition for review is DENIED. The
decision of the Court of Appeals in CA-G.R. CV No. 68893 dated May 29, 2002, is
AFFIRMED.Accordingly, the Civil Registrar of Iligan City is DIRECTED to make the following
corrections in the birth record of respondent Chule Y. Lim, to wit:

1. Her family name from YO to YU;

2. Her fathers name from YO DIU TO (CO TIAN) to YU DIOTO (CO TIAN);

3. Her status from legitimate to illegitimate by changing YES to NO in answer to the question
LEGITIMATE?; and,

4. Her citizenship from Chinese to Filipino.

SO ORDERED.
G.R. Nos. 92191-92 July 30, 1991

ANTONIO Y. CO, petitioner,


vs.
ELECTORAL TRIBUNAL OF THE HOUSE OF REPRESENTATIVES AND JOSE ONG,
JR., respondents.

G.R. Nos. 92202-03 July 30, 1991

SIXTO T. BALANQUIT, JR., petitioner,


vs.
ELECTORAL TRIBUNAL OF THE HOUSE OF REPRESENTATIVES AND JOSE ONG,
JR., respondents.

Hechanova & Associates for petitioner Co.


Brillantes, Nachura, Navarro and Arcilla Law Offices for respondent Ong, Jr.

GUTIERREZ, JR., J.:

The petitioners come to this Court asking for the setting aside and reversal of a decision of
the House of Representatives Electoral Tribunal (HRET).

The HRET declared that respondent Jose Ong, Jr. is a natural born Filipino citizen and a
resident of Laoang, Northern Samar for voting purposes. The sole issue before us is whether
or not, in making that determination, the HRET acted with grave abuse of discretion.

On May 11, 1987, the congressional election for the second district of Northern Samar was
held.

Among the candidates who vied for the position of representative in the second legislative
district of Northern Samar are the petitioners, Sixto Balinquit and Antonio Co and the private
respondent, Jose Ong, Jr.

Respondent Ong was proclaimed the duly elected representative of the second district of
Northern Samar.

The petitioners filed election protests against the private respondent premised on the
following grounds:

1) Jose Ong, Jr. is not a natural born citizen of the Philippines; and

2) Jose Ong, Jr. is not a resident of the second district of Northern Samar.

The HRET in its decision dated November 6, 1989, found for the private respondent.

A motion for reconsideration was filed by the petitioners on November 12, 1989. This was,
however, denied by the HRET in its resolution dated February 22, 1989.
Hence, these petitions for certiorari.

We treat the comments as answers and decide the issues raised in the petitions.

ON THE ISSUE OF JURISDICTION

The first question which arises refers to our jurisdiction.

The Constitution explicitly provides that the House of Representatives Electoral Tribunal
(HRET) and the Senate Electoral Tribunal (SET) shall be the sole judges of all contests relating
to the election, returns, and qualifications of their respective members. (See Article VI,
Section 17, Constitution)

The authority conferred upon the Electoral Tribunal is full, clear and complete. The use of the
word sole emphasizes the exclusivity of the jurisdiction of these Tribunals.

The Supreme Court in the case of Lazatin v. HRET (168 SCRA 391 [1988]) stated that under
the 1987 Constitution, the jurisdiction of the Electoral Tribunal is original and exclusive, viz:

The use of the word "sole" emphasizes the exclusive character of the jurisdiction
conferred (Angara v. Electoral Commission, supra at p. 162). The exercise of power
by the Electoral Commission under the 1935 Constitution has been described as
"intended to be as complete and unimpaired as if it had originally remained in the
legislature." (id., at p. 175) Earlier this grant of power to the legislature was
characterized by Justice Malcolm as "full, clear and complete; (Veloso v. Board of
Canvassers of Leyte and Samar, 39 Phil. 886 [1919]) Under the amended 1935
Constitution, the power was unqualifiedly reposed upon the Electoral Tribunal and it
remained as full, clear and complete as that previously granted the Legislature and
the Electoral Commission, (Lachica v. Yap, 25 SCRA 140 [1968]) The same may be
said with regard to the jurisdiction of the Electoral Tribunal under the 1987
Constitution. (p. 401)

The Court continued further, ". . . so long as the Constitution grants the HRET the power to
be the sole judge of all contests relating to election, returns and qualifications of members of
the House of Representatives, any final action taken by the HRET on a matter within its
jurisdiction shall, as a rule, not be reviewed by this Court . . . the power granted to the
Electoral Tribunal is full, clear and complete and excludes the exercise of any authority on the
part of this Court that would in any wise restrict it or curtail it or even affect the same." (pp.
403-404)

When may the Court inquire into acts of the Electoral Tribunals under our constitutional grants
of power?

In the later case of Robles v. HRET (181 SCRA 780 [1990]) the Supreme Court stated that
the judgments of the Tribunal are beyond judicial interference save only "in the exercise of
this Court's so-called extraordinary jurisdiction, . . . upon a determination that the Tribunal's
decision or resolution was rendered without or in excess of its jurisdiction, or with grave abuse
of discretion or paraphrasing Morrero, upon a clear showing of such arbitrary and improvident
use by the Tribunal of its power as constitutes a denial of due process of law, or upon a
demonstration of a very clear unmitigated ERROR, manifestly constituting such GRAVE ABUSE
OF DISCRETION that there has to be a remedy for such abuse." (at pp. 785-786)
In the leading case of Morrero v. Bocar (66 Phil. 429 [1938]) the Court ruled that the power
of the Electoral Commission "is beyond judicial interference except, in any event, upon a clear
showing of such arbitrary and improvident use of power as will constitute a denial of due
process." The Court does not venture into the perilous area of trying to correct perceived
errors of independent branches of the Government, It comes in only when it has to vindicate
a denial of due process or correct an abuse of discretion so grave or glaring that no less than
the Constitution calls for remedial action.

The Supreme Court under the 1987 Constitution, has been given an expanded jurisdiction, so
to speak, to review the decisions of the other branches and agencies of the government to
determine whether or not they have acted within the bounds of the Constitution. (See Article
VIII, Section 1, Constitution)

Yet, in the exercise thereof, the Court is to merely check whether or not the governmental
branch or agency has gone beyond the Constitutional limits of its jurisdiction, not that it erred
or has a different view. In the absence of a showing that the HRET has committed grave abuse
of discretion amounting to lack of jurisdiction, there is no occasion for the Court to exercise
its corrective power; it will not decide a matter which by its nature is for the HRET alone to
decide. (See Marcos v. Manglapus, 177 SCRA 668 [1989]) It has no power to look into what
it thinks is apparent error.

As constitutional creations invested with necessary power, the Electoral Tribunals, although
not powers in the tripartite scheme of the government, are, in the exercise of their functions
independent organs — independent of Congress and the Supreme Court. The power granted
to HRET by the Constitution is intended to be as complete and unimpaired as if it had remained
originally in the legislature. (Angara v. Electoral Commission, 63 Phil. 139 [1936])

In passing upon petitions, the Court with its traditional and careful regard for the balance of
powers, must permit this exclusive privilege of the Tribunals to remain where the Sovereign
authority has place it. (See Veloso v. Boards of Canvassers of Leyte and Samar, 39 Phil. 886
[1919])

It has been argued that under Article VI, Section 17 of the present Constitution, the situation
may exist as it exists today where there is an unhealthy one-sided political composition of the
two Electoral Tribunals. There is nothing in the Constitution, however, that makes the HRET
because of its composition any less independent from the Court or its constitutional functions
any less exclusive. The degree of judicial intervention should not be made to depend on how
many legislative members of the HRET belong to this party or that party. The test remains
the same-manifest grave abuse of discretion.

In the case at bar, the Court finds no improvident use of power, no denial of due process on
the part of the HRET which will necessitate the exercise of the power of judicial review by the
Supreme Court.

ON THE ISSUE OF CITIZENSHIP

The records show that in the year 1895, the private respondent's grandfather, Ong Te, arrived
in the Philippines from China. Ong Te established his residence in the municipality of Laoang,
Samar on land which he bought from the fruits of hard work.
As a resident of Laoang, Ong Te was able to obtain a certificate of residence from the then
Spanish colonial administration.

The father of the private respondent, Jose Ong Chuan was born in China in 1905. He was
brought by Ong Te to Samar in the year 1915.

Jose Ong Chuan spent his childhood in the province of Samar. In Laoang, he was able to
establish an enduring relationship with his neighbors, resulting in his easy assimilation into
the community.

As Jose Ong Chuan grew older in the rural and seaside community of Laoang, he absorbed
Filipino cultural values and practices. He was baptized into Christianity. As the years passed,
Jose Ong Chuan met a natural born-Filipino, Agripina Lao. The two fell in love and, thereafter,
got married in 1932 according to Catholic faith and practice.

The couple bore eight children, one of whom is the private respondent who was born in 1948.

The private respondent's father never emigrated from this country. He decided to put up a
hardware store and shared and survived the vicissitudes of life in Samar.

The business prospered. Expansion became inevitable. As a result, a branch was set-up in
Binondo, Manila. In the meantime, the father of the private respondent, unsure of his legal
status and in an unequivocal affirmation of where he cast his life and family, filed with the
Court of First Instance of Samar an application for naturalization on February 15, 1954.

On April 28, 1955, the CFI of Samar, after trial, declared Jose Ong Chuan a Filipino citizen.

On May 15, 1957, the Court of First Instance of Samar issued an order declaring the decision
of April 28, 1955 as final and executory and that Jose Ong Chuan may already take his Oath
of Allegiance.

Pursuant to said order, Jose Ong Chuan took his Oath of Allegiance; correspondingly, a
certificate of naturalization was issued to him.

At the time Jose Ong Chuan took his oath, the private respondent then a minor of nine years
was finishing his elementary education in the province of Samar. There is nothing in the
records to differentiate him from other Filipinos insofar as the customs and practices of the
local populace were concerned.

Fortunes changed. The house of the family of the private respondent in Laoang, Samar was
burned to the ground.

Undaunted by the catastrophe, the private respondent's family constructed another one in
place of their ruined house. Again, there is no showing other than that Laoang was their abode
and home.

After completing his elementary education, the private respondent, in search for better
education, went to Manila in order to acquire his secondary and college education.
In the meantime, another misfortune was suffered by the family in 1975 when a fire gutted
their second house in Laoang, Samar. The respondent's family constructed still another house,
this time a 16-door apartment building, two doors of which were reserved for the family.

The private respondent graduated from college, and thereafter took and passed the CPA Board
Examinations.

Since employment opportunities were better in Manila, the respondent looked for work here.
He found a job in the Central Bank of the Philippines as an examiner. Later, however, he
worked in the hardware business of his family in Manila. In 1971, his elder brother, Emil, was
elected as a delegate to the 1971 Constitutional Convention. His status as a natural born
citizen was challenged. Parenthetically, the Convention which in drafting the Constitution
removed the unequal treatment given to derived citizenship on the basis of the mother's
citizenship formally and solemnly declared Emil Ong, respondent's full brother, as a natural
born Filipino. The Constitutional Convention had to be aware of the meaning of natural born
citizenship since it was precisely amending the article on this subject.

The private respondent frequently went home to Laoang, Samar, where he grew up and spent
his childhood days.

In 1984, the private respondent married a Filipina named Desiree Lim.

For the elections of 1984 and 1986, Jose Ong, Jr. registered himself as a voter of Laoang,
Samar, and correspondingly, voted there during those elections.

The private respondent after being engaged for several years in the management of their
family business decided to be of greater service to his province and ran for public office.
Hence, when the opportunity came in 1987, he ran in the elections for representative in the
second district of Northern Samar.

Mr. Ong was overwhelmingly voted by the people of Northern Samar as their representative
in Congress. Even if the total votes of the two petitioners are combined, Ong would still lead
the two by more than 7,000 votes.

The pertinent portions of the Constitution found in Article IV read:

SECTION 1, the following are citizens of the Philippines:

1. Those who are citizens of the Philippines at the time of the adoption of the
Constitution;

2. Those whose fathers or mothers are citizens of the Philippines;

3. Those born before January 17, 1973, of Filipino mothers, who elect Philippine
citizenship upon reaching the age of majority; and

4. Those who are naturalized in accordance with law.

SECTION 2, Natural-born Citizens are those who are citizens of the Philippines from
birth without having to perform any act to acquire or perfect their citizenship. Those
who elect Philippine citizenship in accordance with paragraph 3 hereof shall be deemed
natural-born citizens.

The Court interprets Section 1, Paragraph 3 above as applying not only to those who elect
Philippine citizenship after February 2, 1987 but also to those who, having been born of
Filipino mothers, elected citizenship before that date.

The provision in Paragraph 3 was intended to correct an unfair position which discriminates
against Filipino women. There is no ambiguity in the deliberations of the Constitutional
Commission, viz:

Mr. Azcuna: With respect to the provision of section 4, would this refer only to those
who elect Philippine citizenship after the effectivity of the 1973 Constitution or would
it also cover those who elected it under the 1973 Constitution?

Fr. Bernas: It would apply to anybody who elected Philippine citizenship by virtue of
the provision of the 1935 Constitution whether the election was done before or after
January 17, 1973. (Records of the Constitutional Commission, Vol. 1, p. 228; Emphasis
supplied)

xxx xxx xxx

Mr. Trenas: The Committee on Citizenship, Bill of Rights, Political Rights and
Obligations and Human Rights has more or less decided to extend the interpretation
of who is a natural-born citizen as provided in section 4 of the 1973 Constitution by
adding that persons who have elected Philippine Citizenship under the 1935
Constitution shall be natural-born? Am I right Mr. Presiding Officer?

Fr. Bernas: yes.

xxx xxx xxx

Mr. Nolledo: And I remember very well that in the Reverend Father Bernas' well written
book, he said that the decision was designed merely to accommodate former delegate
Ernesto Ang and that the definition on natural-born has no retroactive effect. Now it
seems that the Reverend Father Bernas is going against this intention by supporting
the amendment?

Fr. Bernas: As the Commissioner can see, there has been an evolution in my thinking.
(Records of the Constitutional Commission, Vol. 1, p. 189)

xxx xxx xxx

Mr. Rodrigo: But this provision becomes very important because his election of
Philippine citizenship makes him not only a Filipino citizen but a natural-born Filipino
citizen entitling him to run for Congress. . .

Fr. Bernas: Correct. We are quite aware of that and for that reason we will leave it to
the body to approve that provision of section 4.
Mr. Rodrigo: I think there is a good basis for the provision because it strikes me as
unfair that the Filipino citizen who was born a day before January 17, 1973 cannot be
a Filipino citizen or a natural-born citizen. (Records of the Constitutional Commission,
Vol. 1, p. 231)

xxx xxx xxx

Mr. Rodrigo: The purpose of that provision is to remedy an inequitable


situation.1avvphi1 Between 1935 and 1973 when we were under the 1935
Constitution, those born of Filipino fathers but alien mothers were natural-born
Filipinos. However, those born of Filipino mothers but alien fathers would have to elect
Philippine citizenship upon reaching the age of majority; and if they do elect, they
become Filipino citizens but not natural-born Filipino citizens. (Records of the
Constitutional Commission, Vol. 1, p. 356)

The foregoing significantly reveals the intent of the framers. To make the provision
prospective from February 3, 1987 is to give a narrow interpretation resulting in an inequitable
situation. It must also be retroactive.

It should be noted that in construing the law, the Courts are not always to be hedged in by
the literal meaning of its language. The spirit and intendment thereof, must prevail over the
letter, especially where adherence to the latter would result in absurdity and injustice. (Casela
v. Court of Appeals, 35 SCRA 279 [1970])

A Constitutional provision should be construed so as to give it effective operation and suppress


the mischief at which it is aimed, hence, it is the spirit of the provision which should prevail
over the letter thereof. (Jarrolt v. Mabberly, 103 U.S. 580)

In the words of the Court in the case of J.M. Tuason v. LTA (31 SCRA 413 [1970]:

To that primordial intent, all else is subordinated. Our Constitution, any constitution is
not to be construed narrowly or pedantically for the prescriptions therein contained,
to paraphrase Justice Holmes, are not mathematical formulas having their essence in
their form but are organic living institutions, the significance of which is vital not
formal. . . . (p. 427)

The provision in question was enacted to correct the anomalous situation where one born of
a Filipino father and an alien mother was automatically granted the status of a natural-born
citizen while one born of a Filipino mother and an alien father would still have to elect
Philippine citizenship. If one so elected, he was not, under earlier laws, conferred the status
of a natural-born.

Under the 1973 Constitution, those born of Filipino fathers and those born of Filipino mothers
with an alien father were placed on equal footing. They were both considered as natural-born
citizens.

Hence, the bestowment of the status of "natural-born" cannot be made to depend on the
fleeting accident of time or result in two kinds of citizens made up of essentially the same
similarly situated members.
It is for this reason that the amendments were enacted, that is, in order to remedy this
accidental anomaly, and, therefore, treat equally all those born before the 1973 Constitution
and who elected Philippine citizenship either before or after the effectivity of that Constitution.

The Constitutional provision in question is, therefore curative in nature. The enactment was
meant to correct the inequitable and absurd situation which then prevailed, and thus, render
those acts valid which would have been nil at the time had it not been for the curative
provisions. (See Development Bank of the Philippines v. Court of Appeals, 96 SCRA 342
[1980])

There is no dispute that the respondent's mother was a natural born Filipina at the time of
her marriage. Crucial to this case is the issue of whether or not the respondent elected or
chose to be a Filipino citizen.

Election becomes material because Section 2 of Article IV of the Constitution accords natural
born status to children born of Filipino mothers before January 17, 1973, if
they elect citizenship upon reaching the age of majority.

To expect the respondent to have formally or in writing elected citizenship when he came of
age is to ask for the unnatural and unnecessary. The reason is obvious. He was already a
citizen. Not only was his mother a natural born citizen but his father had been naturalized
when the respondent was only nine (9) years old. He could not have divined when he came
of age that in 1973 and 1987 the Constitution would be amended to require him to have filed
a sworn statement in 1969 electing citizenship inspite of his already having been a citizen
since 1957. In 1969, election through a sworn statement would have been an unusual and
unnecessary procedure for one who had been a citizen since he was nine years old.

We have jurisprudence that defines "election" as both a formal and an informal process.

In the case of In Re: Florencio Mallare (59 SCRA 45 [1974]), the Court held that the exercise
of the right of suffrage and the participation in election exercises constitute a positive act of
election of Philippine citizenship. In the exact pronouncement of the Court, we held:

Esteban's exercise of the right of suffrage when he came of age, constitutes a positive
act of election of Philippine citizenship (p. 52; emphasis supplied)

The private respondent did more than merely exercise his right of suffrage. He has established
his life here in the Philippines.

For those in the peculiar situation of the respondent who cannot be expected to have elected
citizenship as they were already citizens, we apply the In Re Mallare rule.

The respondent was born in an outlying rural town of Samar where there are no alien enclaves
and no racial distinctions. The respondent has lived the life of a Filipino since birth. His father
applied for naturalization when the child was still a small boy. He is a Roman Catholic. He has
worked for a sensitive government agency. His profession requires citizenship for taking the
examinations and getting a license. He has participated in political exercises as a Filipino and
has always considered himself a Filipino citizen. There is nothing in the records to show that
he does not embrace Philippine customs and values, nothing to indicate any tinge of alien-
ness no acts to show that this country is not his natural homeland. The mass of voters of
Northern Samar are frilly aware of Mr. Ong's parentage. They should know him better than
any member of this Court will ever know him. They voted by overwhelming numbers to have
him represent them in Congress. Because of his acts since childhood, they have considered
him as a Filipino.

The filing of sworn statement or formal declaration is a requirement for those who still have
to elect citizenship. For those already Filipinos when the time to elect came up, there are acts
of deliberate choice which cannot be less binding. Entering a profession open only to Filipinos,
serving in public office where citizenship is a qualification, voting during election time, running
for public office, and other categorical acts of similar nature are themselves formal
manifestations of choice for these persons.

An election of Philippine citizenship presupposes that the person electing is an alien. Or his
status is doubtful because he is a national of two countries. There is no doubt in this case
about Mr. Ong's being a Filipino when he turned twenty-one (21).

We repeat that any election of Philippine citizenship on the part of the private respondent
would not only have been superfluous but it would also have resulted in an absurdity. How
can a Filipino citizen elect Philippine citizenship?

The respondent HRET has an interesting view as to how Mr. Ong elected citizenship. It
observed that "when protestee was only nine years of age, his father, Jose Ong Chuan became
a naturalized Filipino. Section 15 of the Revised Naturalization Act squarely applies its benefit
to him for he was then a minor residing in this country. Concededly, it was the law itself that
had already elected Philippine citizenship for protestee by declaring him as such." (Emphasis
supplied)

The petitioners argue that the respondent's father was not, validly, a naturalized citizen
because of his premature taking of the oath of citizenship.

The Court cannot go into the collateral procedure of stripping Mr. Ong's father of his
citizenship after his death and at this very late date just so we can go after the son.

The petitioners question the citizenship of the father through a collateral approach. This can
not be done. In our jurisdiction, an attack on a person's citizenship may only be done through
a direct action for its nullity. (See Queto v. Catolico, 31 SCRA 52 [1970])

To ask the Court to declare the grant of Philippine citizenship to Jose Ong Chuan as null and
void would run against the principle of due process. Jose Ong Chuan has already been laid to
rest. How can he be given a fair opportunity to defend himself. A dead man cannot speak. To
quote the words of the HRET "Ong Chuan's lips have long been muted to perpetuity by his
demise and obviously he could not use beyond where his mortal remains now lie to defend
himself were this matter to be made a central issue in this case."

The issue before us is not the nullification of the grant of citizenship to Jose Ong Chuan. Our
function is to determine whether or not the HRET committed abuse of authority in the exercise
of its powers. Moreover, the respondent traces his natural born citizenship through
his mother, not through the citizenship of his father. The citizenship of the father is relevant
only to determine whether or not the respondent "chose" to be a Filipino when he came of
age. At that time and up to the present, both mother and father were Filipinos. Respondent
Ong could not have elected any other citizenship unless he first formally renounced Philippine
citizenship in favor of a foreign nationality. Unlike other persons faced with a problem of
election, there was no foreign nationality of his father which he could possibly have chosen.

There is another reason why we cannot declare the HRET as having committed manifest grave
abuse of discretion. The same issue of natural-born citizenship has already been decided by
the Constitutional Convention of 1971 and by the Batasang Pambansa convened by authority
of the Constitution drafted by that Convention. Emil Ong, full blood brother of the respondent,
was declared and accepted as a natural born citizen by both bodies.

Assuming that our opinion is different from that of the Constitutional Convention, the
Batasang Pambansa, and the respondent HRET, such a difference could only be characterized
as error. There would be no basis to call the HRET decision so arbitrary and whimsical as to
amount to grave abuse of discretion.

What was the basis for the Constitutional Convention's declaring Emil Ong a natural born
citizen?

Under the Philippine Bill of 1902, inhabitants of the Philippines who were Spanish subjects on
the 11th day of April 1899 and then residing in said islands and their children born subsequent
thereto were conferred the status of a Filipino citizen.

Was the grandfather of the private respondent a Spanish subject?

Article 17 of the Civil Code of Spain enumerates those who were considered Spanish
Subjects, viz:

ARTICLE 17. The following are Spaniards:

1. Persons born in Spanish territory.

2. Children born of a Spanish father or mother, even though they were born out of
Spain.

3. Foreigners who may have obtained naturalization papers.

4. Those without such papers, who may have acquired domicile in any town in the
Monarchy. (Emphasis supplied)

The domicile of a natural person is the place of his habitual residence. This domicile, once
established is considered to continue and will not be deemed lost until a new one is
established. (Article 50, NCC; Article 40, Civil Code of Spain; Zuellig v. Republic, 83 Phil. 768
[1949])

As earlier stated, Ong Te became a permanent resident of Laoang, Samar around 1895.
Correspondingly, a certificate of residence was then issued to him by virtue of his being a
resident of Laoang, Samar. (Report of the Committee on Election Protests and Credentials of
the 1971 Constitutional Convention, September 7, 1972, p. 3)

The domicile that Ong Te established in 1895 continued until April 11, 1899; it even went
beyond the turn of the 19th century. It is also in this place were Ong Te set-up his business
and acquired his real property.
As concluded by the Constitutional Convention, Ong Te falls within the meaning of sub-
paragraph 4 of Article 17 of the Civil Code of Spain.

Although Ong Te made brief visits to China, he, nevertheless, always returned to the
Philippines. The fact that he died in China, during one of his visits in said country, was of no
moment. This will not change the fact that he already had his domicile fixed in the Philippines
and pursuant to the Civil Code of Spain, he had become a Spanish subject.

If Ong Te became a Spanish subject by virtue of having established his domicile in a town
under the Monarchy of Spain, necessarily, Ong Te was also an inhabitant of the Philippines
for an inhabitant has been defined as one who has actual fixed residence in a place; one who
has a domicile in a place. (Bouvier's Law Dictionary, Vol. II) A priori, there can be no other
logical conclusion but to educe that Ong Te qualified as a Filipino citizen under the provisions
of section 4 of the Philippine Bill of 1902.

The HRET itself found this fact of absolute verity in concluding that the private respondent
was a natural-born Filipino.

The petitioners' sole ground in disputing this fact is that document presented to prove it were
not in compliance with the best the evidence rule. The petitioners allege that the private
respondent failed to present the original of the documentary evidence, testimonial evidence
and of the transcript of the proceedings of the body which the aforesaid resolution of the 1971
Constitutional Convention was predicated.

On the contrary, the documents presented by the private respondent fall under the exceptions
to the best evidence rule.

It was established in the proceedings before the HRET that the originals of the Committee
Report No. 12, the minutes of the plenary session of 1971 Constitutional Convention held on
November 28, 1972 cannot be found.

This was affirmed by Atty. Ricafrente, Assistant Secretary of the 1971 Constitutional
Convention; by Atty. Nolledo, Delegate to the 1971 Constitutional Convention; and by Atty.
Antonio Santos, Chief Librarian of the U.P Law Center, in their respective testimonies given
before the HRET to the effect that there is no governmental agency which is the official
custodian of the records of the 1971 Constitutional Convention. (TSN, December 12, 1988,
pp. 30-31; TSN, January 17, 1989, pp. 34-35; TSN, February 1, 1989, p. 44; TSN, February
6, 1989, pp. 28-29)

The execution of the originals was established by Atty. Ricafrente, who as the Assistant
Secretary of the 1971 Constitutional Convention was the proper party to testify to such
execution. (TSN, December 12, 1989, pp. 11-24)

The inability to produce the originals before the HRET was also testified to as aforestated by
Atty. Ricafrente, Atty. Nolledo, and Atty. Santos. In proving the inability to produce, the law
does not require the degree of proof to be of sufficient certainty; it is enough that it be shown
that after a bona fide diligent search, the same cannot be found. (see Government of P.I. v.
Martinez, 44 Phil. 817 [1918])
Since the execution of the document and the inability to produce were adequately established,
the contents of the questioned documents can be proven by a copy thereof or by the
recollection of witnesses.

Moreover, to erase all doubts as to the authenticity of the documentary evidence cited in the
Committee Report, the former member of the 1971 Constitutional Convention, Atty. Nolledo,
when he was presented as a witness in the hearing of the protest against the private
respondent, categorically stated that he saw the disputed documents presented during the
hearing of the election protest against the brother of the private respondent. (TSN, February
1, 1989, pp. 8-9)

In his concurring opinion, Mr. Justice Sarmiento, a vice-president of the Constitutional


Convention, states that he was presiding officer of the plenary session which deliberated on
the report on the election protest against Delegate Emil Ong. He cites a long list of names of
delegates present. Among them are Mr. Chief Justice Fernan, and Mr. Justice Davide, Jr. The
petitioners could have presented any one of the long list of delegates to refute Mr. Ong's
having been declared a natural-born citizen. They did not do so. Nor did they demur to the
contents of the documents presented by the private respondent. They merely relied on the
procedural objections respecting the admissibility of the evidence presented.

The Constitutional Convention was the sole judge of the qualifications of Emil Ong to be a
member of that body. The HRET by explicit mandate of the Constitution, is the sole judge of
the qualifications of Jose Ong, Jr. to be a member of Congress. Both bodies deliberated at
length on the controversies over which they were sole judges. Decisions were arrived at only
after a full presentation of all relevant factors which the parties wished to present. Even
assuming that we disagree with their conclusions, we cannot declare their acts as committed
with grave abuse of discretion. We have to keep clear the line between error and grave abuse.

ON THE ISSUE OF RESIDENCE

The petitioners question the residence qualification of respondent Ong.

The petitioners lose sight of the meaning of "residence" under the Constitution. The term
"residence" has been understood as synonymous with domicile not only under the previous
Constitutions but also under the 1987 Constitution.

The deliberations of the Constitutional Commission reveal that the meaning of residence vis-
a-vis the qualifications of a candidate for Congress continues to remain the same as that of
domicile, to wit:

Mr. Nolledo: With respect to Section 5, I remember that in the 1971 Constitutional
Convention, there was an attempt to require residence in the place not less than one
year immediately preceding the day of the elections. So my question is: What is the
Committee's concept of residence of a candidate for the legislature? Is it actual
residence or is it the concept of domicile or constructive residence?

Mr. Davide: Madame President, in so far as the regular members of the National
Assembly are concerned, the proposed section merely provides, among others, and a
resident thereof, that is, in the district, for a period of not less than one year preceding
the day of the election. This was in effect lifted from the 1973 Constitution, the
interpretation given to it was domicile. (Records of the 1987 Constitutional Convention,
Vol. 11, July 22, 1986. p. 87)

xxx xxx xxx

Mrs. Rosario Braid: The next question is on Section 7, page 2. I think Commissioner
Nolledo has raised the same point that "resident" has been interpreted at times as a
matter of intention rather than actual residence.

Mr. De los Reyes: Domicile.

Ms. Rosario Braid: Yes, So, would the gentlemen consider at the proper time to go
back to actual residence rather than mere intention to reside?

Mr. De los Reyes: But we might encounter some difficulty especially considering that
a provision in the Constitution in the Article on Suffrage says that Filipinos living abroad
may vote as enacted by law. So, we have to stick to the original concept that it should
be by domicile and not physical and actual residence. (Records of the 1987
Constitutional Commission, Vol. 11, July 22, 1986, p. 110)

The framers of the Constitution adhered to the earlier definition given to the word "residence"
which regarded it as having the same meaning as domicile.

The term "domicile" denotes a fixed permanent residence to which when absent for business
or pleasure, one intends to return. (Ong Huan Tin v. Republic, 19 SCRA 966 [1967]) The
absence of a person from said permanent residence, no matter how long, notwithstanding, it
continues to be the domicile of that person. In other words, domicile is characterized
by animus revertendi (Ujano v. Republic, 17 SCRA 147 [1966])

The domicile of origin of the private respondent, which was the domicile of his parents, is
fixed at Laoang, Samar. Contrary to the petitioners' imputation, Jose Ong, Jr. never
abandoned said domicile; it remained fixed therein even up to the present.

The private respondent, in the proceedings before the HRET sufficiently established that after
the fire that gutted their house in 1961, another one was constructed.

Likewise, after the second fire which again destroyed their house in 1975, a sixteen-door
apartment was built by their family, two doors of which were reserved as their family
residence. (TSN, Jose Ong, Jr., November 18,1988, p. 8)

The petitioners' allegation that since the private respondent owns no property in Laoang,
Samar, he cannot, therefore, be a resident of said place is misplaced.

The properties owned by the Ong Family are in the name of the private respondent's parents.
Upon the demise of his parents, necessarily, the private respondent, pursuant to the laws of
succession, became the co-owner thereof (as a co- heir), notwithstanding the fact that these
were still in the names of his parents.

Even assuming that the private respondent does not own any property in Samar, the Supreme
Court in the case of De los Reyes v. Solidum (61 Phil. 893 [1935]) held that it is not required
that a person should have a house in order to establish his residence and domicile. It is
enough that he should live in the municipality or in a rented house or in that of a friend or
relative. (Emphasis supplied)

To require the private respondent to own property in order to be eligible to run for Congress
would be tantamount to a property qualification. The Constitution only requires that the
candidate meet the age, citizenship, voting and residence requirements. Nowhere is it
required by the Constitution that the candidate should also own property in order to be
qualified to run. (see Maquera v. Borra, 122 Phil. 412 [1965])

It has also been settled that absence from residence to pursue studies or practice a profession
or registration as a voter other than in the place where one is elected, does not constitute
loss of residence. (Faypon v. Quirino, 96 Phil. 294 [1954])

As previously stated, the private respondent stayed in Manila for the purpose of finishing his
studies and later to practice his profession, There was no intention to abandon the residence
in Laoang, Samar. On the contrary, the periodical journeys made to his home province reveal
that he always had the animus revertendi.

The Philippines is made up not only of a single race; it has, rather, undergone an interracial
evolution. Throughout our history, there has been a continuing influx of Malays, Chinese,
Americans, Japanese, Spaniards and other nationalities. This racial diversity gives strength to
our country.

Many great Filipinos have not been whole-blooded nationals, if there is such a person, for
there is none. To mention a few, the great Jose Rizal was part Chinese, the late Chief Justice
Claudio Teehankee was part Chinese, and of course our own President, Corazon Aquino is also
part Chinese. Verily, some Filipinos of whom we are proud were ethnically more Chinese than
the private respondent.

Our citizens no doubt constitute the country's greatest wealth. Citizenship is a special privilege
which one must forever cherish.

However, in order to truly revere this treasure of citizenship, we do not, on the basis of too
harsh an interpretation, have to unreasonably deny it to those who qualify to share in its
richness.

Under the overly strict jurisprudence surrounding our antiquated naturalization laws only the
very affluent backed by influential patrons, who were willing to suffer the indignities of a
lengthy, sometimes humiliating, and often corrupt process of clearances by minor bureaucrats
and whose lawyers knew how to overcome so many technical traps of the judicial process
were able to acquire citizenship. It is time for the naturalization law to be revised to enable a
more positive, affirmative, and meaningful examination of an applicant's suitability to be a
Filipino. A more humane, more indubitable and less technical approach to citizenship problems
is essential.

WHEREFORE, the petitions are hereby DISMISSED. The questioned decision of the House of
Representatives Electoral Tribunal is AFFIRMED. Respondent Jose Ong, Jr. is declared a
natural-born citizen of the Philippines and a resident of Laoang, Northern Samar.

SO ORDERED.
BAR MATTER No. 914 October 1, 1999

RE: APPLICATION FOR ADMISSION TO THE PHILIPPINE BAR,

vs.

VICENTE D. CHING, applicant.

RESOLUTION

KAPUNAN, J.:

Can a legitimate child born under the 1935 Constitution of a Filipino mother and an alien
father validly elect Philippine citizenship fourteen (14) years after he has reached the age of
majority? This is the question sought to be resolved in the present case involving the
application for admission to the Philippine Bar of Vicente D. Ching.

The facts of this case are as follows:

Vicente D. Ching, the legitimate son of the spouses Tat Ching, a Chinese citizen, and Prescila
A. Dulay, a Filipino, was born in Francia West, Tubao, La Union on 11 April 1964. Since his
birth, Ching has resided in the Philippines.

On 17 July 1998, Ching, after having completed a Bachelor of Laws course at the St. Louis
University in Baguio City, filed an application to take the 1998 Bar Examinations. In a
Resolution of this Court, dated 1 September 1998, he was allowed to take the Bar
Examinations, subject to the condition that he must submit to the Court proof of his Philippine
citizenship.

In compliance with the above resolution, Ching submitted on 18 November 1998, the following
documents:

1. Certification, dated 9 June 1986, issued by the Board of Accountancy of the


Professional Regulations Commission showing that Ching is a certified public
accountant;

2. Voter Certification, dated 14 June 1997, issued by Elizabeth B. Cerezo,


Election Officer of the Commission on Elections (COMELEC) in Tubao La Union
showing that Ching is a registered voter of the said place; and

3. Certification, dated 12 October 1998, also issued by Elizabeth B. Cerezo,


showing that Ching was elected as a member of the Sangguniang Bayan of
Tubao, La Union during the 12 May 1992 synchronized elections.

On 5 April 1999, the results of the 1998 Bar Examinations were released and Ching was one
of the successful Bar examinees. The oath-taking of the successful Bar examinees was
scheduled on 5 May 1999. However, because of the questionable status of Ching's citizenship,
he was not allowed to take his oath. Pursuant to the resolution of this Court, dated 20 April
1999, he was required to submit further proof of his citizenship. In the same resolution, the
Office of the Solicitor General (OSG) was required to file a comment on Ching's petition for
admission to the bar and on the documents evidencing his Philippine citizenship.

The OSG filed its comment on 8 July 1999, stating that Ching, being the "legitimate child of
a Chinese father and a Filipino mother born under the 1935 Constitution was a Chinese citizen
and continued to be so, unless upon reaching the age of majority he elected Philippine
citizenship" 1 in strict compliance with the provisions of Commonwealth Act No. 625 entitled
"An Act Providing for the Manner in which the Option to Elect Philippine Citizenship shall be
Declared by a Person Whose Mother is a Filipino Citizen." The OSG adds that "(w)hat he
acquired at best was only an inchoate Philippine citizenship which he could perfect by election
upon reaching the age of majority." 2 In this regard, the OSG clarifies that "two (2) conditions
must concur in order that the election of Philippine citizenship may be effective, namely: (a)
the mother of the person making the election must be a citizen of the Philippines; and (b)
said election must be made upon reaching the age of majority." 3 The OSG then explains the
meaning of the phrase "upon reaching the age of majority:"

The clause "upon reaching the age of majority" has been construed to mean a
reasonable time after reaching the age of majority which had been interpreted
by the Secretary of Justice to be three (3) years (VELAYO, supra at p.
51 citing Op., Sec. of Justice No. 70, s. 1940, Feb. 27, 1940). Said period may
be extended under certain circumstances, as when a (sic) person concerned
has always considered himself a Filipino (ibid., citing Op. Nos. 355 and 422, s.
1955; 3, 12, 46, 86 and 97, s. 1953). But in Cuenco, it was held that an election
done after over seven (7) years was not made within a reasonable time.

In conclusion, the OSG points out that Ching has not formally elected Philippine citizenship
and, if ever he does, it would already be beyond the "reasonable time" allowed by present
jurisprudence. However, due to the peculiar circumstances surrounding Ching's case, the OSG
recommends the relaxation of the standing rule on the construction of the phrase "reasonable
period" and the allowance of Ching to elect Philippine citizenship in accordance with C.A. No.
625 prior to taking his oath as a member of the Philippine Bar.

On 27 July 1999, Ching filed a Manifestation, attaching therewith his Affidavit of Election of
Philippine Citizenship and his Oath of Allegiance, both dated 15 July 1999. In his
Manifestation, Ching states:

1. I have always considered myself as a Filipino;

2. I was registered as a Filipino and consistently declared myself as one in my


school records and other official documents;

3. I am practicing a profession (Certified Public Accountant) reserved for Filipino


citizens;

4. I participated in electoral process[es] since the time I was eligible to vote;

5. I had served the people of Tubao, La Union as a member of the Sangguniang


Bayan from 1992 to 1995;

6. I elected Philippine citizenship on July 15, 1999 in accordance with


Commonwealth Act No. 625;
7. My election was expressed in a statement signed and sworn to by me before
a notary public;

8. I accompanied my election of Philippine citizenship with the oath of allegiance


to the Constitution and the Government of the Philippines;

9. I filed my election of Philippine citizenship and my oath of allegiance to (sic)


the Civil Registrar of Tubao La Union, and

10. I paid the amount of TEN PESOS (Ps. 10.00) as filing fees.

Since Ching has already elected Philippine citizenship on 15 July 1999, the question raised is
whether he has elected Philippine citizenship within a "reasonable time." In the affirmative,
whether his citizenship by election retroacted to the time he took the bar examination.

When Ching was born in 1964, the governing charter was the 1935 Constitution. Under Article
IV, Section 1(3) of the 1935 Constitution, the citizenship of a legitimate child born of a Filipino
mother and an alien father followed the citizenship of the father, unless, upon reaching the
age of majority, the child elected Philippine citizenship. 4 This right to elect Philippine
citizenship was recognized in the 1973 Constitution when it provided that "(t)hose who elect
Philippine citizenship pursuant to the provisions of the Constitution of nineteen hundred and
thirty-five" are citizens of the Philippines. 5 Likewise, this recognition by the 1973 Constitution
was carried over to the 1987 Constitution which states that "(t)hose born before January 17,
1973 of Filipino mothers, who elect Philippine citizenship upon reaching the age of majority"
are Philippine citizens. 6 It should be noted, however, that the 1973 and 1987 Constitutional
provisions on the election of Philippine citizenship should not be understood as having a
curative effect on any irregularity in the acquisition of citizenship for those covered by the
1935 Constitution. 7 If the citizenship of a person was subject to challenge under the old
charter, it remains subject to challenge under the new charter even if the judicial challenge
had not been commenced before the effectivity of the new Constitution. 8

C.A. No. 625 which was enacted pursuant to Section 1(3), Article IV of the 1935 Constitution,
prescribes the procedure that should be followed in order to make a valid election of Philippine
citizenship. Under Section 1 thereof, legitimate children born of Filipino mothers may elect
Philippine citizenship by expressing such intention "in a statement to be signed and sworn to
by the party concerned before any officer authorized to administer oaths, and shall be filed
with the nearest civil registry. The said party shall accompany the aforesaid statement with
the oath of allegiance to the Constitution and the Government of the Philippines."

However, the 1935 Constitution and C.A. No. 625 did not prescribe a time period within which
the election of Philippine citizenship should be made. The 1935 Charter only provides that the
election should be made "upon reaching the age of majority." The age of majority then
commenced upon reaching twenty-one (21) years. 9 In the opinions of the Secretary of Justice
on cases involving the validity of election of Philippine citizenship, this dilemma was resolved
by basing the time period on the decisions of this Court prior to the effectivity of the 1935
Constitution. In these decisions, the proper period for electing Philippine citizenship was, in
turn, based on the pronouncements of the Department of State of the United States
Government to the effect that the election should be made within a "reasonable time" after
attaining the age of majority. 10 The phrase "reasonable time" has been interpreted to mean
that the election should be made within three (3) years from reaching the age of
majority. 11 However, we held in Cuenco vs. Secretary of Justice, 12 that the three (3) year
period is not an inflexible rule. We said:
It is true that this clause has been construed to mean a reasonable period after
reaching the age of majority, and that the Secretary of Justice has ruled that
three (3) years is the reasonable time to elect Philippine citizenship under the
constitutional provision adverted to above, which period may be extended
under certain circumstances, as when the person concerned has always
considered himself a Filipino. 13

However, we cautioned in Cuenco that the extension of the option to elect Philippine
citizenship is not indefinite:

Regardless of the foregoing, petitioner was born on February 16, 1923. He


became of age on February 16, 1944. His election of citizenship was made on
May 15, 1951, when he was over twenty-eight (28) years of age, or over seven
(7) years after he had reached the age of majority. It is clear that said election
has not been made "upon reaching the age of majority." 14

In the present case, Ching, having been born on 11 April 1964, was already thirty-five (35)
years old when he complied with the requirements of C.A. No. 625 on 15 June 1999, or over
fourteen (14) years after he had reached the age of majority. Based on the interpretation of
the phrase "upon reaching the age of majority," Ching's election was clearly beyond, by any
reasonable yardstick, the allowable period within which to exercise the privilege. It should be
stated, in this connection, that the special circumstances invoked by Ching, i.e., his
continuous and uninterrupted stay in the Philippines and his being a certified public
accountant, a registered voter and a former elected public official, cannot vest in him
Philippine citizenship as the law specifically lays down the requirements for acquisition of
Philippine citizenship by election.

Definitely, the so-called special circumstances cannot constitute what Ching erroneously
labels as informal election of citizenship. Ching cannot find a refuge in the case of In
re: Florencio Mallare, 15 the pertinent portion of which reads:

And even assuming arguendo that Ana Mallare were (sic) legally married to an
alien, Esteban's exercise of the right of suffrage when he came of age,
constitutes a positive act of election of Philippine citizenship. It has been
established that Esteban Mallare was a registered voter as of April 14, 1928,
and that as early as 1925 (when he was about 22 years old), Esteban was
already participating in the elections and campaigning for certain candidate[s].
These acts are sufficient to show his preference for Philippine citizenship. 16

Ching's reliance on Mallare is misplaced. The facts and circumstances obtaining therein are
very different from those in the present case, thus, negating its applicability. First,
Esteban Mallare was born before the effectivity of the 1935 Constitution and the enactment
of C.A. No. 625. Hence, the requirements and procedures prescribed under the 1935
Constitution and C.A. No. 625 for electing Philippine citizenship would not be applicable to
him. Second, the ruling in Mallare was an obiter since, as correctly pointed out by the OSG,
it was not necessary for Esteban Mallare to elect Philippine citizenship because he was already
a Filipino, he being a natural child of a Filipino mother. In this regard, the Court stated:

Esteban Mallare, natural child of Ana Mallare, a Filipina, is therefore himself a


Filipino, and no other act would be necessary to confer on him all the rights
and privileges attached to Philippine citizenship (U.S. vs. Ong Tianse, 29 Phil.
332; Santos Co vs. Government of the Philippine Islands, 42 Phil. 543, Serra
vs. Republic, L-4223, May 12, 1952, Sy Quimsuan vs. Republic, L-4693, Feb.
16, 1953; Pitallano vs. Republic, L-5111, June 28, 1954). Neither could any act
be taken on the erroneous belief that he is a non-filipino divest him of the
citizenship privileges to which he is rightfully entitled. 17

The ruling in Mallare was reiterated and further elaborated in Co vs. Electoral Tribunal of the
House of Representatives, 18 where we held:

We have jurisprudence that defines "election" as both a formal and an informal


process.

In the case of In re: Florencio Mallare (59 SCRA 45 [1974]), the Court held that
the exercise of the right of suffrage and the participation in election exercises
constitute a positive act of election of Philippine citizenship. In the exact
pronouncement of the Court, we held:

Esteban's exercise of the right of suffrage when he came of age


constitutes a positive act of Philippine citizenship. (p. 52:
emphasis supplied)

The private respondent did more than merely exercise his right of suffrage. He has established
his life here in the Philippines.

For those in the peculiar situation of the respondent who cannot be excepted
to have elected Philippine citizenship as they were already citizens, we apply
the In Re Mallare rule.

xxx xxx xxx

The filing of sworn statement or formal declaration is a requirement for those


who still have to elect citizenship. For those already Filipinos when the time to
elect came up, there are acts of deliberate choice which cannot be less binding.
Entering a profession open only to Filipinos, serving in public office where
citizenship is a qualification, voting during election time, running for public
office, and other categorical acts of similar nature are themselves formal
manifestations for these persons.

An election of Philippine citizenship presupposes that the person electing is an


alien. Or his status is doubtful because he is a national of two countries. There
is no doubt in this case about Mr. Ong's being a Filipino when he turned twenty-
one (21).

We repeat that any election of Philippine citizenship on the part of the private
respondent would not only have been superfluous but it would also have
resulted in an absurdity. How can a Filipino citizen elect Philippine
citizenship? 19

The Court, like the OSG, is sympathetic with the plight of Ching. However, even if we consider
the special circumstances in the life of Ching like his having lived in the Philippines all his life
and his consistent belief that he is a Filipino, controlling statutes and jurisprudence constrain
us to disagree with the recommendation of the OSG. Consequently, we hold that Ching failed
to validly elect Philippine citizenship. The span of fourteen (14) years that lapsed from the
time he reached the age of majority until he finally expressed his intention to elect Philippine
citizenship is clearly way beyond the contemplation of the requirement of electing "upon
reaching the age of majority." Moreover, Ching has offered no reason why he delayed his
election of Philippine citizenship. The prescribed procedure in electing Philippine citizenship is
certainly not a tedious and painstaking process. All that is required of the elector is to execute
an affidavit of election of Philippine citizenship and, thereafter, file the same with the nearest
civil registry. Ching's unreasonable and unexplained delay in making his election cannot be
simply glossed over.

Philippine citizenship can never be treated like a commodity that can be claimed when needed
and suppressed when convenient. 20 One who is privileged to elect Philippine citizenship has
only an inchoate right to such citizenship. As such, he should avail of the right with fervor,
enthusiasm and promptitude. Sadly, in this case, Ching slept on his opportunity to elect
Philippine citizenship and, as a result. this golden privilege slipped away from his grasp.

IN VIEW OF THE FOREGOING, the Court Resolves to DENY Vicente D. Ching's application for
admission to the Philippine Bar.

SO ORDERED.
FIRST DIVISION

BALGAMELO CABILING MA, FELIX G.R. No. 183133


CABILING MA, JR., andVALERIANO
CABILING MA,
Petitioners,

Present:

CORONA,C.J.,
-versus- Chairperson,
VELASCO, JR.,
NACHURA,*
LEONARDO-DE CASTRO, and
PEREZ, JJ.

COMMISSIONER ALIPIO F.
FERNANDEZ, JR., ASSOCIATE
COMMISSIONER ARTHEL B.
CARONOGAN, ASSOCIATE
COMMISSIONER JOSE DL.
CABOCHAN, ASSOCIATE
COMMISSIONER TEODORO B.
DELARMENTE AND ASSOCIATE
COMMISSIONER FRANKLIN Z.
LITTAUA, in their capacities as
Chairman and Members of the Board
of Commissioners (Bureau of
Immigration), and MAT G. CATRAL,
Respondents. Promulgated:

July 26, 2010

x-----------------------------------------------------------------------------------------x

DECISION

PEREZ, J.:

Should children born under the 1935 Constitution of a Filipino mother and an alien
father, who executed an affidavit of election of Philippine citizenship and took their oath of
allegiance to the government upon reaching the age of majority, but who failed to immediately
file the documents of election with the nearest civil registry, be considered foreign nationals
subject to deportation as undocumented aliens for failure to obtain alien certificates of
registration?

Positioned upon the facts of this case, the question is translated into the inquiry
whether or not the omission negates their rights to Filipino citizenship as children of a Filipino
mother, and erase the years lived and spent as Filipinos.
The resolution of these questions would significantly mark a difference in the lives of
herein petitioners.

The Facts

Balgamelo Cabiling Ma (Balgamelo), Felix Cabiling Ma, Jr. (Felix, Jr.), Valeriano
Cabiling Ma (Valeriano), Lechi Ann Ma (Lechi Ann), Arceli Ma (Arceli), Nicolas Ma (Nicolas),
and Isidro Ma (Isidro) are the children of Felix (Yao Kong) Ma, [1] a Taiwanese, and Dolores
Sillona Cabiling, a Filipina.[2]
Records reveal that petitioners Felix, Jr., Balgamelo and Valeriano were all born under
aegis of the 1935 Philippine Constitution in the years 1948, 1951, and 1957, respectively. [3]

They were all raised in the Philippines and have resided in this country for almost sixty
(60) years; they spent their whole lives, studied and received their primary and secondary
education in the country; they do not speak nor understand the Chinese language, have not
set foot in Taiwan, and do not know any relative of their father; they have not even traveled
abroad; and they have already raised their respective families in the Philippines. [4]

During their age of minority, they secured from the Bureau of Immigration their Alien
Certificates of Registration (ACRs). [5]

Immediately upon reaching the age of twenty-one, they claimed Philippine citizenship
in accordance with Section 1(4), Article IV, of the 1935 Constitution, which provides that
(t)hose whose mothers are citizens of the Philippines and, upon reaching the age of majority,
elect Philippine citizenship are citizens of the Philippines. Thus, on 15 August 1969, Felix, Jr.
executed his affidavit of election of Philippine citizenship and took his oath of allegiance before
then Judge Jose L. Gonzalez, Municipal Judge, Surigao, Surigao del Norte. [6] On 14 January
1972, Balgamelo did the same before Atty. Patrocinio C. Filoteo, Notary Public, Surigao City,
Surigao del Norte.[7] In 1978, Valeriano took his oath of allegiance before then Judge Salvador
C. Sering, City Court of Surigao City, the fact of which the latter attested to in his Affidavit
of 7 March 2005.[8]
Having taken their oath of allegiance as Philippine citizens, petitioners, however, failed
to have the necessary documents registered in the civil registry as required under Section 1
of Commonwealth Act No. 625 (An Act Providing the Manner in which the Option to Elect
Philippine Citizenship shall be Declared by a Person whose Mother is a Filipino Citizen). It was
only on 27 July 2005 or more than thirty (30) years after they elected Philippine citizenship
that Balgamelo and Felix, Jr. did so.[9] On the other hand, there is no showing that Valeriano
complied with the registration requirement.

Individual certifications[10] all dated 3 January 2005 issued by the Office of the City
Election Officer, Commission on Elections, Surigao City, show that all of them are registered
voters of Barangay Washington, Precinct No. 0015A since June 1997, and that records on
previous registrations are no longer available because of the mandatory general registration
every ten (10) years. Moreover, aside from exercising their right of suffrage, Balgamelo is
one of the incumbent Barangay Kagawads in BarangayWashington, Surigao City.[11]
Records further reveal that Lechi Ann and Arceli were born also in Surigao City in 1953[12] and
1959,[13] respectively. The Office of the City Civil Registrar issued a Certification to the effect
that the documents showing that Arceli elected Philippine citizenship on 27 January 1986 were
registered in its Office on 4 February 1986. However, no other supporting documents appear
to show that Lechi Ann initially obtained an ACR nor that she subsequently elected Philippine
citizenship upon reaching the age of majority. Likewise, no document exists that will provide
information on the citizenship of Nicolas and Isidro.
The Complaint

On 16 February 2004, the Bureau of Immigration received the Complaint-


Affidavit[14] of a certain Mat G. Catral (Mr. Catral), alleging that Felix (Yao Kong) Ma and his
seven (7) children are undesirable and overstaying aliens. Mr. Catral, however, did not
participate in the proceedings, and the Ma family could not but believe that the complaint
against them was politically motivated because they strongly supported a candidate
in Surigao City in the 2004 National and Local Elections.[15]

On 9 November 2004, the Legal Department of the Bureau of Immigration charged


them for violation of Sections 37(a)(7)[16] and 45(e)[17] of Commonwealth Act No. 613,
otherwise known as the Philippine Immigration Act of 1940, as amended. The Charge
Sheet[18] docketed as BSI-D.C. No. AFF-04-574 (OC-STF-04-09/23-1416) reads, in part:

That Respondents x x x, all Chinese nationals, failed and continuously


failed to present any valid document to show their respective status in
the Philippines. They likewise failed to produce documents to show their election
of Philippines (sic) citizenship, hence, undocumented and overstaying foreign
nationals in the country.

That respondents, being aliens, misrepresent themselves as Philippine


citizens in order to evade the requirements of the immigration laws.

Ruling of the Board of Commissioners, Bureau of Immigration

After Felix Ma and his seven (7) children were afforded the opportunity to refute the
allegations, the Board of Commissioners (Board) of the Bureau of Immigration (BI), composed
of the public respondents, rendered a Judgment dated 2 February 2005 finding that Felix Ma
and his children violated Commonwealth Act No. 613, Sections 37(a)(7) and 45(e) in relation
to BI Memorandum Order Nos. ADD-01-031 and ADD-01-035 dated 6 and 22 August 2001,
respectively.[19]

The Board ruled that since they elected Philippine citizenship after the enactment of
Commonwealth Act No. 625, which was approved on 7 June 1941, they were governed by the
following rules and regulations:

1. Section 1 of Commonwealth Act No. 625, providing that the election of Philippine
citizenship embodied in a statement sworn before any officer authorized to administer oaths
and the oath of allegiance shall be filed with the nearest civil registry; [20] and Commission of
Immigration and Deportation (CID, now Bureau of Immigration [BI]) Circular dated 12 April
1954,[21] detailing the procedural requirements in the registration of the election of Philippine
citizenship.

2. Memorandum Order dated 18 August 1956[22] of the CID, requiring the filing of a
petition for the cancellation of their alien certificate of registration with the CID, in view of
their election of Philippine citizenship;

3. Department of Justice (DOJ) Opinion No. 182, 19 August 1982; and DOJ
Guidelines, 27 March 1985, requiring that the records of the proceedings be forwarded to the
Ministry (now the Department) of Justice for final determination and review.[23]
As regards the documentation of aliens in the Philippines, Administrative Order No. 1-
93 of the Bureau of Immigration[24] requires that ACR, E-series, be issued to foreign nationals
who apply for initial registration, finger printing and issuance of an ACR in accordance with
the Alien Registration Act of 1950.[25] According to public respondents, any foreign national
found in possession of an ACR other than the E-series shall be considered improperly
documented aliens and may be proceeded against in accordance with the Immigration Act of
1940 or the Alien Registration Act of 1950, as amended.[26]

Supposedly for failure to comply with the procedure to prove a valid claim to Philippine
citizenship via election proceedings, public respondents concluded that Felix, Jr. Balgamelo,
Arceli, Valeriano and Lechi Ann are undocumented and/or improperly documented
aliens.[27]

Nicolas and Isidro, on the other hand, did not submit any document to support their
claim that they are Philippine citizens. Neither did they present any evidence to show that
they are properly documented aliens. For these reasons, public respondents likewise deemed
them undocumented and/or improperly documented aliens.[28]

The dispositive portion[29] of the Judgment of 2 February 2005 reads:


1. Subject to the submission of appropriate clearances, summary
deportation of Felix (Yao Kong) Ma, Felix Ma, Jr., Balgamelo Ma, Valeriano
Ma, Lechi Ann Ma, Nicolas Ma, Arceli Ma and Isidro Ma, Taiwanese
[Chinese], under C.A. No. 613, Sections 37(a)(7), 45(e) and 38 in relation
to BI M.O. Nos. ADD-01-031 and ADD-01-035 dated 6 and 22 August 2001,
respectively;

2. Issuance of a warrant of deportation against Felix (Yao Kong) Ma,


Felix Ma, Jr., Balgamelo Ma, Valeriano Ma, Lechi Ann Ma, Nicolas Ma, Arceli
Ma and Isidro Ma under C.A. No. 613, Section 37(a);

3. Inclusion of the names of Felix (Yao Kong) Ma, Felix Ma, Jr., Balgamelo
Ma, Valeriano Ma, Lechi Ann Ma, Nicolas Ma, Arceli Ma and Isidro Ma in the
Immigration Blacklist; and

4. Exclusion from the Philippines of Felix (Yao Kong) Ma, Felix Ma, Jr.,
Balgamelo Ma, Valeriano Ma, Lechi Ann Ma, Nicolas Ma, Arceli Ma and Isidro
Ma under C.A. No. 613, Section 29(a)(15). (Emphasis supplied.)

In its Resolution[30] of 8 April 2005, public respondents partially reconsidered their


Judgment of 2 February 2005. They were convinced that Arceli is an immigrant under
Commonwealth Act No. 613, Section 13(g).[31] However, they denied the Motion for
Reconsideration with respect to Felix Ma and the rest of his children.[32]

Ruling of the Court of Appeals


On 3 May 2005, only Balgamelo, Felix, Jr., and Valeriano filed the Petition for
Certiorari under Rule 65 of the 1997 Rules of Civil Procedure before the Court of Appeals,
which was docketed as CA-G.R. SP No. 89532. They sought the nullification of the issuances
of the public respondents, to wit: (1) the Judgment dated 2 February 2005, ordering the
summary deportation of the petitioners, issuance of a warrant of deportation against them,
inclusion of their names in the Immigration Blacklist, and exclusion of the petitioners from
the Philippines; and (2) the Resolution dated 8 April 2005, denying the petitioners Motion for
Reconsideration.

On 29 August 2007, the Court of Appeals dismissed the petition [33] after finding that
the petitioners failed to comply with the exacting standards of the law providing for the
procedure and conditions for their continued stay in the Philippines either as aliens or as its
nationals.[34]

On 29 May 2008, it issued a Resolution [35] denying the petitioners Motion for
Reconsideration dated 20 September 2007.
To reiterate, a persons continued and uninterrupted stay in
the Philippines, his being a registered voter or an elected public official cannot
vest in him Philippine citizenship as the law specifically lays down the
requirements for acquisition of Philippine citizenship by election. The prescribed
procedure in electing Philippine citizenship is certainly not a tedious and
painstaking process. All that is required of the elector is to execute an affidavit
of election of Philippine citizenship and, thereafter, file the same with the
nearest civil registry. The constitutional mandate concerning citizenship must
be adhered to strictly. Philippine citizenship can never be treated like a
commodity that can be claimed when needed and suppressed when convenient.
One who is privileged to elect Philippine citizenship has only an inchoate right
to such citizenship. As such, he should avail of the right with fervor, enthusiasm
and promptitude.[36]

Our Ruling

The 1935 Constitution declares as citizens of the Philippines those whose mothers are
citizens of the Philippines and elect Philippine citizenship upon reaching the age of
majority. The mandate states:

Section 1. The following are citizens of the Philippines:


(1) xxx;

xxxx

(4) Those whose mothers are citizens of the Philippines and, upon
reaching the age of majority, elect Philippine citizenship.[37]

In 1941, Commonwealth Act No. 625 was enacted. It laid down the manner of electing
Philippine citizenship, to wit:

Section 1. The option to elect Philippine citizenship in accordance with


subsection (4), Section 1, Article IV, of the Constitution shall be expressed in a
statement to be signed and sworn to by the party concerned before any officer
authorized to administer oaths, and shall be filed with the nearest civil
registry. The said party shall accompany the aforesaid statement with the oath
of allegiance to the Constitution and the Government of the Philippines.

The statutory formalities of electing Philippine citizenship are: (1) a statement of


election under oath; (2) an oath of allegiance to the Constitution and Government of
the Philippines; and (3) registration of the statement of election and of the oath with the
nearest civil registry.

In Re:Application for Admission to the Philippine Bar, Vicente D. Ching,[38] we determined the
meaning of the period of election described by phrase upon reaching the age of majority. Our
references were the Civil Code of the Philippines, the opinions of the Secretary of Justice, and
the case of Cueco v. Secretary of Justice.[39] We pronounced:

x x x [T]he 1935 Constitution and C.A. No. 625 did not prescribe a time period
within which the election of Philippine citizenship should be made. The 1935
Charter only provides that the election should be made upon reaching the age
of majority. The age of majority then commenced upon reaching twenty-one
(21) years.[40] In the opinions of the Secretary of Justice on cases involving the
validity of election of Philippine citizenship, this dilemma was resolved by basing
the time period on the decisions of this Court prior to the effectivity of the 1935
Constitution. In these decisions, the proper period for electing Philippine
citizenship was, in turn, based on the pronouncements of the Department of
State of the United States Government to the effect that the election should be
made within a reasonable time after attaining the age of majority. [41] The
phrase reasonable time has been interpreted to mean that the elections should
be made within three (3) years from reaching the age of majority.[42] However,
we held in Cue[n]co vs. Secretary of Justice,[43] that the three (3) year period
is not an inflexible rule. We said:

It is true that this clause has been construed to mean a


reasonable time after reaching the age of majority, and that the
Secretary of Justice has ruled that three (3) years is the
reasonable time to elect Philippine citizenship under the
constitutional provision adverted to above, which period may be
extended under certain circumstances, as when the person
concerned has always considered himself a Filipino.

However, we cautioned in Cue[n]co that the extension of the option to elect


Philippine citizenship is not indefinite.

Regardless of the foregoing, petitioner was born


on February 16, 1923. He became of age on February 16,
1944. His election of citizenship was made on May 15, 1951,
when he was over twenty-eight (28) years of age, or over seven
(7) years after he had reached the age of majority. It is clear
that said election has not been made upon reaching the age of
majority.[44]

We reiterated the above ruling in Go, Sr. v. Ramos,[45] a case in which we adopted the
findings of the appellate court that the father of the petitioner, whose citizenship was in
question, failed to elect Philippine citizenship within the reasonable period of three (3) years
upon reaching the age of majority; and that the belated submission to the local civil registry
of the affidavit of election and oath of allegiance x x x was defective because the affidavit of
election was executed after the oath of allegiance, and the delay of several years before their
filing with the proper office was not satisfactorily explained.[46]
In both cases, we ruled against the petitioners because they belatedly complied with
all the requirements. The acts of election and their registration with the nearest civil registry
were all done beyond the reasonable period of three years upon reaching the age of majority.

The instant case presents a different factual setting. Petitioners complied with the first
and second requirements upon reaching the age of majority. It was only the registration of
the documents of election with the civil registry that was belatedly done.

We rule that under the facts peculiar to the petitioners, the right to elect Philippine
citizenship has not been lost and they should be allowed to complete the statutory
requirements for such election.

Such conclusion, contrary to the finding of the Court of Appeals, is in line with our
decisions in In Re:Florencio Mallare,[47] Co v. Electoral Tribunal of the House of
Representatives,[48] and Re:Application for Admission to the Philippine Bar, Vicente D.
Ching.[49]

In Mallare, Estebans exercise of the right of suffrage when he came of age was deemed
to be a positive act of election of Philippine citizenship. [50] The Court of Appeals, however,
said that the case cannot support herein petitioners cause, pointing out that, unlike petitioner,
Esteban is a natural child of a Filipina, hence, no other act would be necessary to confer on
him the rights and privileges of a Filipino citizen, [51] and that Esteban was born in
1929[52] prior to the adoption of the 1935 Constitution and the enactment of Commonwealth
Act No. 625.[53]

In the Co case, Jose Ong, Jr. did more than exercise his right of suffrage, as he
established his life here in the Philippines.[54] Again, such circumstance, while similar to that
of herein petitioners, was not appreciated because it was ruled that any election of Philippine
citizenship on the part of Ong would have resulted in absurdity, because the law itself had
already elected Philippine citizenship for him[55] as, apparently, while he was still a minor, a
certificate of naturalization was issued to his father.[56]

In Ching, it may be recalled that we denied his application for admission to the
Philippine Bar because, in his case, all the requirements, to wit: (1) a statement of election
under oath; (2) an oath of allegiance to the Constitution and Government of the Philippines;
and (3) registration of the statement of election and of the oath with the nearest civil registry
were complied with only fourteen (14) years after he reached the age of majority. Ching
offered no reason for the late election of Philippine citizenship. [57]

In all, the Court of Appeals found the petitioners argument of good faith and informal
election unacceptable and held:

Their reliance in the ruling contained in Re:Application for Admission to


the Philippine Bar, Vicente D. Ching, [which was decided on 1 October 1999],
is obviously flawed. It bears emphasis that the Supreme Court, in said case,
did not adopt the doctrine laid down in In Re: Florencio Mallare. On the
contrary, the Supreme Court was emphatic in pronouncing that the special
circumstances invoked by Ching, i.e., his continuous and uninterrupted stay in
the Philippines and his being a certified public accountant, a registered voter
and a former elected public official, cannot vest in him Philippine citizenship as
the law specifically lays down the requirements for acquisition of Philippine
citizenship by election.[58]

We are not prepared to state that the mere exercise of suffrage, being elected public
official, continuous and uninterrupted stay in the Philippines, and other similar acts showing
exercise of Philippine citizenship can take the place of election of citizenship. What we now
say is that where, as in petitioners case, the election of citizenship has in fact been done and
documented within the constitutional and statutory timeframe, the registration of the
documents of election beyond the frame should be allowed if in the meanwhile positive acts
of citizenship have publicly, consistently, and continuously been done. The actual exercise of
Philippine citizenship, for over half a century by the herein petitioners, is actual notice to the
Philippine public which is equivalent to formal registration of the election of Philippine
citizenship.

For what purpose is registration?


In Pascua v. Court of Appeals,[59] we elucidated the principles of civil law on
registration:

To register is to record or annotate. American and Spanish authorities


are unanimous on the meaning of the term to register as to enter in a register;
to record formally and distinctly; to enroll; to enter in a list.[60] In general,
registration refers to any entry made in the books of the registry, including
both registration in its ordinary and strict sense, and cancellation, annotation,
and even the marginal notes. In strict acceptation, it pertains to the entry made
in the registry which records solemnly and permanently the right of ownership
and other real rights.[61] Simply stated, registration is made for the purpose
of notification.[62]

Actual knowledge may even have the effect of registration as to the person who has
knowledge thereof. Thus, [i]ts purpose is to give notice thereof to all persons (and it) operates
as a notice of the deed, contract, or instrument to others. [63] As pertinent is the holding that
registration neither adds to its validity nor converts an invalid instrument into a valid one
between the parties.[64] It lays emphasis on the validity of an unregistered document.

Comparable jurisprudence may be consulted.

In a contract of partnership, we said that the purpose of registration is to give notice


to third parties; that failure to register the contract does not affect the liability of the
partnership and of the partners to third persons; and that neither does such failure affect the
partnerships juridical personality.[65] An unregistered contract of partnership is valid as among
the partners, so long as it has the essential requisites, because the main purpose of
registration is to give notice to third parties, and it can be assumed that the members
themselves knew of the contents of their contract.[66] The non-registration of a deed of
donation does not also affect its validity. Registration is not a requirement for the validity of
the contract as between the parties, for the effect of registration serves chiefly to bind third
persons.[67]

Likewise relevant is the pronouncement that registration is not a mode of acquiring a


right. In an analogous case involving an unrecorded deed of sale, we reiterated the settled
rule that registration is not a mode of acquiring ownership.
Registration does not confer ownership. It is not a mode of acquiring
dominion, but only a means of confirming the fact of its existence with notice
to the world at large.[68]

Registration, then, is the confirmation of the existence of a fact. In the instant case,
registration is the confirmation of election as such election. It is not the registration of the act
of election, although a valid requirement under Commonwealth Act No. 625, that will confer
Philippine citizenship on the petitioners. It is only a means of confirming the fact that
citizenship has been claimed.

Indeed, we even allow the late registration of the fact of birth and of
marriage.[69] Thus, has it been admitted through existing rules that the late registration of
the fact of birth of a child does not erase the fact of birth. Also, the fact of marriage cannot
be declared void solely because of the failure to have the marriage certificate registered with
the designated government agency.
Notably, the petitioners timely took their oath of allegiance to the Philippines. This was
a serious undertaking. It was commitment and fidelity to the state coupled with a pledge to
renounce absolutely and forever all allegiance to any other state. This was unqualified
acceptance of their identity as a Filipino and the complete disavowal of any other nationality.

Petitioners have passed decades of their lives in the Philippines as Filipinos. Their
present status having been formed by their past, petitioners can no longer have any national
identity except that which they chose upon reaching the age of reason.

Corollary to this fact, we cannot agree with the view of the Court of Appeals that since
the ACR presented by the petitioners are no longer valid on account of the new requirement
to present an E-series ACR, they are deemed not properly documented.[70] On the contrary,
petitioners should not be expected to secure E-series ACR because it would be inconsistent
with the election of citizenship and its constructive registration through their acts made public,
among others, their exercise of suffrage, election as public official, and continued and
uninterrupted stay in the Philippines since birth. The failure to register as aliens is, obviously,
consistent with petitioners election of Philippine citizenship.

The leanings towards recognition of the citizenship of children of Filipino mothers have
been indicated not alone by the jurisprudence that liberalized the requirement on time of
election, and recognized positive acts of Philippine citizenship.

The favor that is given to such children is likewise evident in the evolution of the
constitutional provision on Philippine citizenship.

Thus, while the 1935 Constitution requires that children of Filipino mothers elect
Philippine citizenship upon reaching their age of majority, [71] upon the effectivity of the 1973
Constitution, they automatically become Filipinos[72] and need not elect Philippine citizenship
upon reaching the age of majority. The 1973 provision reads:

Section 1. The following are citizens of the Philippines:

(1) xxx.
(2) Those whose fathers and mothers are citizens of
the Philippines.[73]
Better than the relaxation of the requirement, the 1987 Constitution now classifies
them as natural-born citizens upon election of Philippine citizenship. Thus, Sec. 2, Article IV
thereof provides:

Section 2. Natural-born citizens are those who are citizens of


the Philippines from birth without having to perform any act to acquire or
perfect their Philippine citizenship. Those who elect Philippine citizenship
in accordance with paragraph (3), Section 1 hereof[74] shall be deemed
natural-born citizens. (Emphasis supplied.)

The constitutional bias is reflected in the deliberations of the 1986 Constitutional


Commission.

MR. CONCEPCION. x x x.

xxxx

x x x x As regards those born of Filipino mothers, the 1935 Constitution merely


gave them the option to choose Philippine citizenship upon reaching the age of
majority, even, apparently, if the father were an alien or unknown. Upon the
other hand, under the 1973 Constitution, children of mixed marriages involving
an alien father and a Filipino mother are Filipino citizens, thus liberalizing the
counterpart provision in the 1935 Constitution by dispensing with the need to
make a declaration of intention upon reaching the age of majority. I understand
that the committee would further liberalize this provision of the 1935
Constitution. The Committee seemingly proposes to further liberalize the policy
of the 1935 Constitution by making those who became citizens of
the Philippines through a declaration of intention to choose their mothers
citizenship upon reaching the majority age by declaring that such children are
natural-born citizens of the Philippines.[75]

xxxx

xxx Why does the draft resolution adopt the provision of the 1973 Constitution
and not that of the 1935? [76]

xxxx
FR. BERNAS. x x x Precisely, the reason behind the modification of the 1935
rule on citizenship was a recognition of the fact that it reflected a certain male
chauvinism, and it was for the purpose of remedying that this proposed
provision was put in. The idea was that we should not penalize the mother of a
child simply because she fell in love with a foreigner. Now, the question on what
citizenship the child would prefer arises. We really have no way of guessing the
preference of the infant. But if we recognize the right of the child to choose,
then let him choose when he reaches the age of majority. I think dual
citizenship is just a reality imposed on us because we have no control of the
laws on citizenship of other countries. We recognize a child of a Filipino
mother. But whether or not she is considered a citizen of another country is
something completely beyond our control. But certainly it is within the
jurisdiction of the Philippine government to require that [at] a certain point, a
child be made to choose. But I do not think we should penalize the child before
he is even able to choose. I would, therefore, support the retention of the
modification made in 1973 of the male chauvinistic rule of the 1935
Constitution.[77]

xxxx

MR. REGALADO. With respect to a child who became a Filipino citizen by


election, which the Committee is now planning to consider a natural-born
citizen, he will be so the moment he opts for Philippine citizenship. Did the
Committee take into account the fact that at the time of birth, all he had was
just an inchoate right to choose Philippine citizenship, and yet, by subsequently
choosing Philippine citizenship, it would appear that his choice retroacted to the
date of his birth so much so that under the Gentlemans proposed amendment,
he would be a natural-born citizen?[78]

FR. BERNAS. But the difference between him and the natural-born who lost his
status is that the natural-born who lost his status, lost it voluntarily; whereas,
this individual in the situation contemplated in Section 1, paragraph 3 never
had the chance to choose.[79]

xxxx

[on the period within which to elect Philippine citizenship]

MR. RODRIGO. [T]his provision becomes very, very important because his
election of Philippine citizenship makes him not only a Filipino citizen but a natural-
born Filipino citizen, entitling him to run for Congress, to be a Justice of the
Supreme Court x x x.[80]
We are guided by this evolvement from election of Philippine citizenship upon reaching
the age of majority under the 1935 Philippine Constitution to dispensing with the election
requirement under the 1973 Philippine Constitution to express classification of these children
as natural-born citizens under the 1987 Constitution towards the conclusion that the omission
of the 1941 statutory requirement of registration of the documents of election should not
result in the obliteration of the right to Philippine citizenship.

Having a Filipino mother is permanent. It is the basis of the right of the petitioners to
elect Philippine citizenship. Petitioners elected Philippine citizenship in form and
substance. The failure to register the election in the civil registry should not defeat the election
and resultingly negate the permanent fact that they have a Filipino mother. The lacking
requirements may still be complied with subject to the imposition of appropriate
administrative penalties, if any. The documents they submitted supporting their allegations
that they have already registered with the civil registry, although belatedly, should be
examined for validation purposes by the appropriate agency, in this case, the Bureau of
Immigration. Other requirements embodied in the administrative orders and other issuances
of the Bureau of Immigration and the Department of Justice shall be complied with within a
reasonable time.

WHEREFORE, the Decision dated 29 August 2007, and the Resolution dated 29 May
2008 of the Court of Appeals in CA-G.R. SP No. 89532 affirming the Judgment dated 2
February 2005, and the Resolution dated 8 April 2005 of the Bureau of Immigration in BSI-
D.C. No. AFF-04-574 OC-STF-04-09/23-1416 are hereby SET ASIDE with respect to
petitioners Balgamelo Cabiling Ma, Felix Cabiling Ma, Jr., and Valeriano Cabiling
Ma. Petitioners are given ninety (90) days from notice within which to COMPLY with the
requirements of the Bureau of Immigration embodied in its Judgment of 2 February 2005. The
Bureau of Immigration shall ENSURE that all requirements, including the payment of their
financial obligations to the state, if any, have been complied with subject to the imposition of
appropriate administrative fines; REVIEW the documents submitted by the petitioners;
and ACT thereon in accordance with the decision of this Court.

SO ORDERED.
EN BANC

G.R. No. L-83882 January 24, 1989

IN RE PETITION FOR HABEAS CORPUS OF WILLIE YU, petitioner,


vs.
MIRIAM DEFENSOR-SANTIAGO, BIENVENIDO P. ALANO, JR., MAJOR PABALAN,
DELEO HERNANDEZ, BLODDY HERNANDEZ, BENNY REYES and JUN ESPIRITU
SANTO, respondent.

Pelaez, Adriano and Gregorio and Bonifacio A. Alentajan for petitioner.

Chavez, Hechanova & Lim Law Offices collaborating counsel for petitioner.

Augusto Jose y. Arreza for respondents.

PADILLA, J.:

The present controversy originated with a petition for habeas corpus filed with the Court on
4 July 1988 seeking the release from detention of herein petitioner. 1 After manifestation and
motion of the Solicitor General of his decision to refrain from filing a return of the writ on
behalf of the CID, respondent Commissioner thru counsel filed the return. 2Counsel for the
parties were heard in oral argument on 20 July 1988. The parties were allowed to submit
marked exhibits, and to file memoranda. 3 An internal resolution of 7 November 1988 referred
the case to the Court en banc. In its 10 November 1988 resolution, denying the petition
for habeas corpus, the Court disposed of the pending issues of (1) jurisdiction of the CID over
a naturalized Filipino citizen and (2) validity of warrantless arrest and detention of the same
person.

Petitioner filed a motion for reconsideration with prayer for restraining order dated 24
November 1988. 4 On 29 November 1988, the Court resolved to deny with finality the
aforesaid motion for reconsideration, and further resolved to deny the urgent motion for
issuance of a restraining order dated 28 November 1988. 5

Undaunted, petitioner filed a motion for clarification with prayer for restraining order on 5
December 1988.

Acting on said motion, a temporary restraining order was issued by the Court on 7 December
1988. 6 Respondent Commissioner filed a motion to lift TRO on 13 December 1988, the basis
of which is a summary judgment of deportation against Yu issued by the CID Board of
Commissioners on 2 December 1988. 7 Petitioner also filed a motion to set case for oral
argument on 8 December 1988.

In the meantime, an urgent motion for release from arbitrary detention 8 was filed by
petitioner on 13 December 1988. A memorandum in furtherance of said motion for release
dated 14 December 1988 was filed on 15 December 1988 together with a vigorous opposition
to the lifting of the TRO.
The lifting of the Temporary Restraining Order issued by the Court on 7 December 1988 is
urgently sought by respondent Commissioner who was ordered to cease and desist from
immediately deporting petitioner Yu pending the conclusion of hearings before the Board of
Special Inquiry, CID. To finally dispose of the case, the Court will likewise rule on petitioner's
motion for clarification with prayer for restraining order dated 5 December 1988, 9 urgent
motion for release from arbitrary detention dated 13 December 1988, 10 the memorandum in
furtherance of said motion for release dated 14 December 1988, 11 motion to set case for oral
argument dated 8 December 1988. 12

Acting on the motion to lift the temporary restraining order (issued on 7 December 1988)
dated 9 December 1988, 13and the vigorous opposition to lift restraining order dated 15
December 1988, 14 the Court resolved to give petitioner Yu a non-extendible period of three
(3) days from notice within which to explain and prove why he should still be considered a
citizen of the Philippines despite his acquisition and use of a Portuguese passport. 15

Petitioner filed his compliance with the resolution of 15 December 1988 on 20 December
1988 16 followed by an earnest request for temporary release on 22 December 1988.
Respondent filed on 2 January 1989 her comment reiterating her previous motion to lift
temporary restraining order. Petitioner filed a reply thereto on 6 January 1989.

Petitioner's own compliance reveals that he was originally issued a Portuguese passport in
1971, 17 valid for five (5) years and renewed for the same period upon presentment before
the proper Portuguese consular officer. Despite his naturalization as a Philippine citizen on 10
February 1978, on 21 July 1981, petitioner applied for and was issued Portuguese Passport
No. 35/81 serial N. 1517410 by the Consular Section of the Portuguese Embassy in Tokyo.
Said Consular Office certifies that his Portuguese passport expired on 20 July 1986. 18 While
still a citizen of the Philippines who had renounced, upon his naturalization, "absolutely and
forever all allegiance and fidelity to any foreign prince, potentate, state or sovereignty" and
pledged to "maintain true faith and allegiance to the Republic of the Philippines," 19 he
declared his nationality as Portuguese in commercial documents he signed, specifically, the
Companies registry of Tai Shun Estate Ltd. 20 filed in Hongkong sometime in April 1980.

To the mind of the Court, the foregoing acts considered together constitute an express
renunciation of petitioner's Philippine citizenship acquired through naturalization. In Board of
Immigration Commissioners us, Go Gallano, 21express renunciation was held to mean a
renunciation that is made known distinctly and explicitly and not left to inference or
implication. Petitioner, with full knowledge, and legal capacity, after having renounced
Portuguese citizenship upon naturalization as a Philippine citizen 22 resumed or reacquired his
prior status as a Portuguese citizen, applied for a renewal of his Portuguese passport 23 and
represented himself as such in official documents even after he had become a naturalized
Philippine citizen. Such resumption or reacquisition of Portuguese citizenship is grossly
inconsistent with his maintenance of Philippine citizenship.

This Court issued the aforementioned TRO pending hearings with the Board of Special Inquiry,
CID. However, pleadings submitted before this Court after the issuance of said TRO have
unequivocally shown that petitioner has expressly renounced his Philippine citizenship. The
material facts are not only established by the pleadings — they are not disputed by petitioner.
A rehearing on this point with the CID would be unnecessary and superfluous. Denial, if any,
of due process was obviated when petitioner was given by the Court the opportunity to show
proof of continued Philippine citizenship, but he has failed.
While normally the question of whether or not a person has renounced his Philippine
citizenship should be heard before a trial court of law in adversary proceedings, this has
become unnecessary as this Court, no less, upon the insistence of petitioner, had to look into
the facts and satisfy itself on whether or not petitioner's claim to continued Philippine
citizenship is meritorious.

Philippine citizenship, it must be stressed, is not a commodity or were to be displayed when


required and suppressed when convenient. This then resolves adverse to the petitioner his
motion for clarification and other motions mentioned in the second paragraph, page 3 of this
Decision.

WHEREFORE, premises considered, petitioner's motion for release from detention is DENIED.
Respondent's motion to lift the temporary restraining order is GRANTED. This Decision is
immediately executory.

SO ORDERED.
EN BANC

G.R. No. 195649 April 16, 2013

CASAN MACODE MAQUILING, Petitioner,


vs.
COMMISSION ON ELECTIONS, ROMMEL ARNADO y CAGOCO, LINOG G.
BALUA, Respondents.

DECISION

SERENO, CJ.:

THE CASE

This is a Petition for Certiorari ender Rule 64 in conjunction with Rule 65 of the Rules of Court
to review the Resolutions of the Commission on Elections (COMELEC). The Resolution 1 in SPA
No. 10-1 09(DC) of the COMELEC First Division dated 5 October 201 0 is being assailed for
applying Section 44 of the Local Government Code while the Resolution 2 of the COMELEC En
Banc dated 2 February 2011 is being questioned for finding that respondent Rommel Arnado
y Cagoco (respondent Arnado/Arnado) is solely a Filipino citizen qualified to run for public
office despite his continued use of a U.S. passport.

FACTS

Respondent Arnado is a natural born Filipino citizen. 3 However, as a consequence of his


subsequent naturalization as a citizen of the United States of America, he lost his Filipino
citizenship. Arnado applied for repatriation under Republic Act (R.A.) No. 9225 before the
Consulate General of the Philippines in San Franciso, USA and took the Oath of Allegiance to
the Republic of the Philippines on 10 July 2008.4 On the same day an Order of Approval of his
Citizenship Retention and Re-acquisition was issued in his favor.5

The aforementioned Oath of Allegiance states:

I, Rommel Cagoco Arnado, solemnly swear that I will support and defend the Constitution of
the Republic of the Philippines and obey the laws and legal orders promulgated by the duly
constituted authorities of the Philippines and I hereby declare that I recognize and accept the
supreme authority of the Philippines and will maintain true faith and allegiance thereto; and
that I impose this obligation upon myself voluntarily without mental reservation or purpose
of evasion.6

On 3 April 2009 Arnado again took his Oath of Allegiance to the Republic and executed an
Affidavit of Renunciation of his foreign citizenship, which states:

I, Rommel Cagoco Arnado, do solemnly swear that I absolutely and perpetually renounce all
allegiance and fidelity to the UNITED STATES OF AMERICA of which I am a citizen, and I
divest myself of full employment of all civil and political rights and privileges of the United
States of America.

I solemnly swear that all the foregoing statement is true and correct to the best of my
knowledge and belief.7
On 30 November 2009, Arnado filed his Certificate of Candidacy for Mayor of Kauswagan,
Lanao del Norte, which contains, among others, the following statements:

I am a natural born Filipino citizen / naturalized Filipino citizen.

I am not a permanent resident of, or immigrant to, a foreign country.

I am eligible for the office I seek to be elected to.

I will support and defend the Constitution of the Republic of the Philippines and will maintain
true faith and allegiance thereto. I will obey the laws, legal orders and decrees promulgated
by the duly constituted authorities.

I impose this obligation upon myself voluntarily without mental reservation or purpose of
evasion.8

On 28 April 2010, respondent Linog C. Balua (Balua), another mayoralty candidate, filed a
petition to disqualify Arnado and/or to cancel his certificate of candidacy for municipal mayor
of Kauswagan, Lanao del Norte in connection with the 10 May 2010 local and national
elections.9

Respondent Balua contended that Arnado is not a resident of Kauswagan, Lanao del Norte
and that he is a foreigner, attaching thereto a certification issued by the Bureau of
Immigration dated 23 April 2010 indicating the nationality of Arnado as "USA-American."10To
further bolster his claim of Arnado’s US citizenship, Balua presented in his Memorandum a
computer-generated travel record11 dated 03 December 2009 indicating that Arnado has been
using his US Passport No. 057782700 in entering and departing the Philippines. The said
record shows that Arnado left the country on 14 April 2009 and returned on 25 June 2009,
and again departed on 29 July 2009, arriving back in the Philippines on 24 November 2009.

Balua likewise presented a certification from the Bureau of Immigration dated 23 April 2010,
certifying that the name "Arnado, Rommel Cagoco" appears in the available Computer
Database/Passenger manifest/IBM listing on file as of 21 April 2010, with the following
pertinent travel records:

DATE OF Arrival : 01/12/2010

NATIONALITY : USA-AMERICAN

PASSPORT : 057782700

DATE OF Arrival : 03/23/2010

NATIONALITY : USA-AMERICAN

PASSPORT : 05778270012

On 30 April 2010, the COMELEC (First Division) issued an Order13 requiring the respondent to
personally file his answer and memorandum within three (3) days from receipt thereof.
After Arnado failed to answer the petition, Balua moved to declare him in default and to
present evidence ex-parte.

Neither motion was acted upon, having been overtaken by the 2010 elections where Arnado
garnered the highest number of votes and was subsequently proclaimed as the winning
candidate for Mayor of Kauswagan, Lanao del Norte.

It was only after his proclamation that Arnado filed his verified answer, submitting the
following documents as evidence:14

1. Affidavit of Renunciation and Oath of Allegiance to the Republic of the Philippines


dated 03 April 2009;

2. Joint-Affidavit dated 31 May 2010 of Engr. Virgil Seno, Virginia Branzuela, Leoncio
Daligdig, and Jessy Corpin, all neighbors of Arnado, attesting that Arnado is a long-
time resident of Kauswagan and that he has been conspicuously and continuously
residing in his family’s ancestral house in Kauswagan;

3. Certification from the Punong Barangay of Poblacion, Kauswagan, Lanao del Norte
dated 03 June 2010 stating that Arnado is a bona fide resident of his barangay and
that Arnado went to the United States in 1985 to work and returned to the Philippines
in 2009;

4. Certification dated 31 May 2010 from the Municipal Local Government Operations
Office of Kauswagan stating that Dr. Maximo P. Arnado, Sr. served as Mayor of
Kauswagan, from January 1964 to June 1974 and from 15 February 1979 to 15 April
1986; and

5. Voter Certification issued by the Election Officer of Kauswagan certifying that Arnado
has been a registered voter of Kauswagan since 03 April 2009.

THE RULING OF THE COMELEC FIRST DIVISION

Instead of treating the Petition as an action for the cancellation of a certificate of candidacy
based on misrepresentation,15 the COMELEC First Division considered it as one for
disqualification. Balua’s contention that Arnado is a resident of the United States was
dismissed upon the finding that "Balua failed to present any evidence to support his
contention,"16 whereas the First Division still could "not conclude that Arnado failed to meet
the one-year residency requirement under the Local Government Code."17

In the matter of the issue of citizenship, however, the First Division disagreed with Arnado’s
claim that he is a Filipino citizen.18

We find that although Arnado appears to have substantially complied with the requirements
of R.A. No. 9225, Arnado’s act of consistently using his US passport after renouncing his US
citizenship on 03 April 2009 effectively negated his Affidavit of Renunciation.

xxxx

Arnado’s continued use of his US passport is a strong indication that Arnado had no real
intention to renounce his US citizenship and that he only executed an Affidavit of Renunciation
to enable him to run for office. We cannot turn a blind eye to the glaring inconsistency between
Arnado’s unexplained use of a US passport six times and his claim that he re-acquired his
Philippine citizenship and renounced his US citizenship. As noted by the Supreme Court in the
Yu case, "a passport is defined as an official document of identity and nationality issued to a
person intending to travel or sojourn in foreign countries." Surely, one who truly divested
himself of US citizenship would not continue to avail of privileges reserved solely for US
nationals.19

The dispositive portion of the Resolution rendered by the COMELEC

First Division reads:

WHEREFORE, in view of the foregoing, the petition for disqualification and/or to cancel the
certificate of candidacy of Rommel C. Arnado is hereby GRANTED. Rommel C. Arnado’s
proclamation as the winning candidate for Municipal Mayor of Kauswagan, Lanao del Nore is
hereby ANNULLED. Let the order of succession under Section 44 of the Local Government
Code of 1991 take effect.20

The Motion for Reconsideration and


the Motion for Intervention

Arnado sought reconsideration of the resolution before the COMELEC En Banc on the ground
that "the evidence is insufficient to justify the Resolution and that the said Resolution is
contrary to law."21 He raised the following contentions:22

1. The finding that he is not a Filipino citizen is not supported by the evidence
consisting of his Oath of Allegiance and the Affidavit of Renunciation, which show that
he has substantially complied with the requirements of R.A. No. 9225;

2. The use of his US passport subsequent to his renunciation of his American citizenship
is not tantamount to a repudiation of his Filipino citizenship, as he did not perform any
act to swear allegiance to a country other than the Philippines;

3. He used his US passport only because he was not informed of the issuance of his
Philippine passport, and that he used his Philippine passport after he obtained it;

4. Balua’s petition to cancel the certificate of candidacy of Arnado was filed out of time,
and the First Division’s treatment of the petition as one for disqualification constitutes
grave abuse of discretion amounting to excess of jurisdiction; 23

5. He is undoubtedly the people’s choice as indicated by his winning the elections;

6. His proclamation as the winning candidate ousted the COMELEC from jurisdiction
over the case; and

7. The proper remedy to question his citizenship is through a petition for quo warranto,
which should have been filed within ten days from his proclamation.

Petitioner Casan Macode Maquiling (Maquiling), another candidate for mayor of Kauswagan,
and who garnered the second highest number of votes in the 2010 elections, intervened in
the case and filed before the COMELEC En Banc a Motion for Reconsideration together with
an Opposition to Arnado’s Amended Motion for Reconsideration. Maquiling argued that while
the First Division correctly disqualified Arnado, the order of succession under Section 44 of
the Local Government Code is not applicable in this case. Consequently, he claimed that the
cancellation of Arnado’s candidacy and the nullification of his proclamation, Maquiling, as the
legitimate candidate who obtained the highest number of lawful votes, should be proclaimed
as the winner.

Maquiling simultaneously filed his Memorandum with his Motion for Intervention and his
Motion for Reconsideration. Arnado opposed all motions filed by Maquiling, claiming that
intervention is prohibited after a decision has already been rendered, and that as a second-
placer, Maquiling undoubtedly lost the elections and thus does not stand to be prejudiced or
benefitted by the final adjudication of the case.

RULING OF THE COMELEC EN BANC

In its Resolution of 02 February 2011, the COMELEC En Banc held that under Section 6 of
Republic Act No. 6646, the Commission "shall continue with the trial and hearing of the action,
inquiry or protest even after the proclamation of the candidate whose qualifications for office
is questioned."

As to Maquiling’s intervention, the COMELEC En Banc also cited Section 6 of R.A. No. 6646
which allows intervention in proceedings for disqualification even after elections if no final
judgment has been rendered, but went on further to say that Maquiling, as the second placer,
would not be prejudiced by the outcome of the case as it agrees with the dispositive portion
of the Resolution of the First Division allowing the order of succession under Section 44 of the
Local Government Code to take effect.

The COMELEC En Banc agreed with the treatment by the First Division of the petition as one
for disqualification, and ruled that the petition was filed well within the period prescribed by
law,24 having been filed on 28 April 2010, which is not later than 11 May 2010, the date of
proclamation.

However, the COMELEC En Banc reversed and set aside the ruling of the First Division and
granted Arnado’s Motion for Reconsideration, on the following premises:

First:

By renouncing his US citizenship as imposed by R.A. No. 9225, the respondent embraced his
Philippine citizenship as though he never became a citizen of another country. It was at that
time, April 3, 2009, that the respondent became a pure Philippine Citizen again.

xxxx

The use of a US passport … does not operate to revert back his status as a dual citizen prior
to his renunciation as there is no law saying such. More succinctly, the use of a US passport
does not operate to "un-renounce" what he has earlier on renounced. The First Division’s
reliance in the case of In Re: Petition for Habeas Corpus of Willy Yu v. Defensor-Santiago, et
al. is misplaced. The petitioner in the said case is a naturalized citizen who, after taking his
oath as a naturalized Filipino, applied for the renewal of his Portuguese passport. Strict policy
is maintained in the conduct of citizens who are not natural born, who acquire their citizenship
by choice, thus discarding their original citizenship. The Philippine State expects strict conduct
of allegiance to those who choose to be its citizens. In the present case, respondent is not a
naturalized citizen but a natural born citizen who chose greener pastures by working abroad
and then decided to repatriate to supposedly help in the progress of Kauswagan. He did not
apply for a US passport after his renunciation. Thus the mentioned case is not on all fours
with the case at bar.

xxxx

The respondent presented a plausible explanation as to the use of his US passport. Although
he applied for a Philippine passport, the passport was only issued on June 18, 2009. However,
he was not notified of the issuance of his Philippine passport so that he was actually able to
get it about three (3) months later. Yet as soon as he was in possession of his Philippine
passport, the respondent already used the same in his subsequent travels abroad. This fact
is proven by the respondent’s submission of a certified true copy of his passport showing that
he used the same for his travels on the following dates: January 31, 2010, April 16, 2010,
May 20, 2010, January 12, 2010, March 31, 2010 and June 4, 2010. This then shows that the
use of the US passport was because to his knowledge, his Philippine passport was not yet
issued to him for his use. As probably pressing needs might be undertaken, the respondent
used whatever is within his control during that time.25

In his Separate Concurring Opinion, COMELEC Chairman Sixto Brillantes cited that the use of
foreign passport is not one of the grounds provided for under Section 1 of Commonwealth Act
No. 63 through which Philippine citizenship may be lost.

"The application of the more assimilative principle of continuity of citizenship is more


appropriate in this case. Under said principle, once a person becomes a citizen, either by birth
or naturalization, it is assumed that he desires to continue to be a citizen, and this assumption
stands until he voluntarily denationalizes or expatriates himself. Thus, in the instant case
respondent after reacquiring his Philippine citizenship should be presumed to have remained
a Filipino despite his use of his American passport in the absence of clear, unequivocal and
competent proof of expatriation. Accordingly, all doubts should be resolved in favor of
retention of citizenship."26

On the other hand, Commissioner Rene V. Sarmiento dissented, thus:

Respondent evidently failed to prove that he truly and wholeheartedly abandoned his
allegiance to the United States. The latter’s continued use of his US passport and enjoyment
of all the privileges of a US citizen despite his previous renunciation of the afore-mentioned
citizenship runs contrary to his declaration that he chose to retain only his Philippine
citizenship. Respondent’s submission with the twin requirements was obviously only for the
purpose of complying with the requirements for running for the mayoralty post in connection
with the May 10, 2010 Automated National and Local Elections.

Qualifications for elective office, such as citizenship, are continuing requirements; once any
of them is lost during his incumbency, title to the office itself is deemed forfeited. If a
candidate is not a citizen at the time he ran for office or if he lost his citizenship after his
election to office, he is disqualified to serve as such. Neither does the fact that respondent
obtained the plurality of votes for the mayoralty post cure the latter’s failure to comply with
the qualification requirements regarding his citizenship.

Since a disqualified candidate is no candidate at all in the eyes of the law, his having received
the highest number of votes does not validate his election. It has been held that where a
petition for disqualification was filed before election against a candidate but was adversely
resolved against him after election, his having obtained the highest number of votes did not
make his election valid. His ouster from office does not violate the principle of vox populi
suprema est lex because the application of the constitutional and statutory provisions on
disqualification is not a matter of popularity. To apply it is to breath[e] life to the sovereign
will of the people who expressed it when they ratified the Constitution and when they elected
their representatives who enacted the law.27

THE PETITION BEFORE THE COURT

Maquiling filed the instant petition questioning the propriety of declaring Arnado qualified to
run for public office despite his continued use of a US passport, and praying that Maquiling
be proclaimed as the winner in the 2010 mayoralty race in Kauswagan, Lanao del Norte.

Ascribing both grave abuse of discretion and reversible error on the part of the COMELEC En
Banc for ruling that Arnado is a Filipino citizen despite his continued use of a US passport,
Maquiling now seeks to reverse the finding of the COMELEC En Banc that Arnado is qualified
to run for public office.

Corollary to his plea to reverse the ruling of the COMELEC En Banc or to affirm the First
Division’s disqualification of Arnado, Maquiling also seeks the review of the applicability of
Section 44 of the Local Government Code, claiming that the COMELEC committed reversible
error in ruling that "the succession of the vice mayor in case the respondent is disqualified is
in order."

There are three questions posed by the parties before this Court which will be addressed
seriatim as the subsequent questions hinge on the result of the first.

The first question is whether or not intervention is allowed in a disqualification case.

The second question is whether or not the use of a foreign passport after renouncing foreign
citizenship amounts to undoing a renunciation earlier made.

A better framing of the question though should be whether or not the use of a foreign passport
after renouncing foreign citizenship affects one’s qualifications to run for public office.

The third question is whether or not the rule on succession in the Local Government Code is
applicable to this case.

OUR RULING

Intervention of a rival candidate in a


disqualification case is proper when
there has not yet been any
proclamation of the winner.

Petitioner Casan Macode Maquiling intervened at the stage when respondent Arnado filed a
Motion for Reconsideration of the First Division Resolution before the COMELEC En Banc. As
the candidate who garnered the second highest number of votes, Maquiling contends that he
has an interest in the disqualification case filed against Arnado, considering that in the event
the latter is disqualified, the votes cast for him should be considered stray and the second-
placer should be proclaimed as the winner in the elections.

It must be emphasized that while the original petition before the COMELEC is one for
cancellation of the certificate of candidacy and / or disqualification, the COMELEC First Division
and the COMELEC En Banc correctly treated the petition as one for disqualification.

The effect of a disqualification case is enunciated in Section 6 of R.A. No. 6646:

Sec. 6. Effect of Disqualification Case. - Any candidate who has been declared by final
judgment to be disqualified shall not be voted for, and the votes cast for him shall not be
counted. If for any reason a candidate is not declared by final judgment before an election to
be disqualified and he is voted for and receives the winning number of votes in such election,
the Court or Commission shall continue with the trial and hearing of the action, inquiry, or
protest and, upon motion of the complainant or any intervenor, may during the pendency
thereof order the suspension of the proclamation of such candidate whenever the evidence of
his guilt is strong.

Mercado v. Manzano28

clarified the right of intervention in a disqualification case. In that case, the Court said:

That petitioner had a right to intervene at that stage of the proceedings for the disqualification
against private respondent is clear from Section 6 of R.A. No. 6646, otherwise known as the
Electoral Reforms Law of 1987, which provides: Any candidate who has been declared by final
judgment to be disqualified shall not be voted for, and the votes cast for him shall not be
counted. If for any reason a candidate is not declared by final judgment before an election to
be disqualified and he is voted for and receives the winning number of votes in such election,
the Court or Commission shall continue with the trial and hearing of the action, inquiry, or
protest and, upon motion of the complainant or any intervenor, may during the pendency
thereof order the suspension of the proclamation of such candidate whenever the evidence of
guilt is strong. Under this provision, intervention may be allowed in proceedings for
disqualification even after election if there has yet been no final judgment rendered. 29

Clearly then, Maquiling has the right to intervene in the case. The fact that the COMELEC En
Banc has already ruled that Maquiling has not shown that the requisites for the exemption to
the second-placer rule set forth in Sinsuat v. COMELEC 30 are present and therefore would not
be prejudiced by the outcome of the case, does not deprive Maquiling of the right to elevate
the matter before this Court.

Arnado’s claim that the main case has attained finality as the original petitioner and
respondents therein have not appealed the decision of the COMELEC En Banc, cannot be
sustained. The elevation of the case by the intervenor prevents it from attaining finality. It is
only after this Court has ruled upon the issues raised in this instant petition that the
disqualification case originally filed by Balua against Arnado will attain finality.

The use of foreign passport after renouncing one’s foreign citizenship is a positive
and voluntary act of representation as to one’s nationality and citizenship; it does
not divest Filipino citizenship regained by repatriation but it recants the Oath of
Renunciation required to qualify one to run for an elective position.
Section 5(2) of The Citizenship Retention and Re-acquisition Act of 2003 provides:

Those who retain or re-acquire Philippine citizenship under this Act shall enjoy full civil and
political rights and be subject to all attendant liabilities and responsibilities under existing
laws of the Philippines and the following conditions:

xxxx

(2)Those seeking elective public in the Philippines shall meet the qualification for holding such
public office as required by the Constitution and existing laws and, at the time of the filing of
the certificate of candidacy, make a personal and sworn renunciation of any and all foreign
before any public officer authorized to administer an oath.

x x x31

Rommel Arnado took all the necessary steps to qualify to run for a public office. He took the
Oath of Allegiance and renounced his foreign citizenship. There is no question that after
performing these twin requirements required under Section 5(2) of R.A. No. 9225 or the
Citizenship Retention and Re-acquisition Act of 2003, he became eligible to run for public
office.

Indeed, Arnado took the Oath of Allegiance not just only once but twice: first, on 10 July 2008
when he applied for repatriation before the Consulate General of the Philippines in San
Francisco, USA, and again on 03 April 2009 simultaneous with the execution of his Affidavit
of Renunciation. By taking the Oath of Allegiance to the Republic, Arnado re-acquired his
Philippine citizenship. At the time, however, he likewise possessed American citizenship.
Arnado had therefore become a dual citizen.

After reacquiring his Philippine citizenship, Arnado renounced his American citizenship by
executing an Affidavit of Renunciation, thus completing the requirements for eligibility to run
for public office.

By renouncing his foreign citizenship, he was deemed to be solely a Filipino citizen, regardless
of the effect of such renunciation under the laws of the foreign country. 32

However, this legal presumption does not operate permanently and is open to attack when,
after renouncing the foreign citizenship, the citizen performs positive acts showing his
continued possession of a foreign citizenship.33

Arnado himself subjected the issue of his citizenship to attack when, after renouncing his
foreign citizenship, he continued to use his US passport to travel in and out of the country
before filing his certificate of candidacy on 30 November 2009. The pivotal question to
determine is whether he was solely and exclusively a Filipino citizen at the time he filed his
certificate of candidacy, thereby rendering him eligible to run for public office.

Between 03 April 2009, the date he renounced his foreign citizenship, and 30 November 2009,
the date he filed his COC, he used his US passport four times, actions that run counter to the
affidavit of renunciation he had earlier executed. By using his foreign passport, Arnado
positively and voluntarily represented himself as an American, in effect declaring before
immigration authorities of both countries that he is an American citizen, with all attendant
rights and privileges granted by the United States of America.
The renunciation of foreign citizenship is not a hollow oath that can simply be professed at
any time, only to be violated the next day. It requires an absolute and perpetual renunciation
of the foreign citizenship and a full divestment of all civil and political rights granted by the
foreign country which granted the citizenship.

Mercado v. Manzano34 already hinted at this situation when the Court declared:

His declarations will be taken upon the faith that he will fulfill his undertaking made under
oath. Should he betray that trust, there are enough sanctions for declaring the loss of his
Philippine citizenship through expatriation in appropriate proceedings. In Yu v. Defensor-
Santiago, we sustained the denial of entry into the country of petitioner on the ground that,
after taking his oath as a naturalized citizen, he applied for the renewal of his Portuguese
passport and declared in commercial documents executed abroad that he was a Portuguese
national. A similar sanction can be taken against anyone who, in electing Philippine citizenship,
renounces his foreign nationality, but subsequently does some act constituting renunciation
of his Philippine citizenship.

While the act of using a foreign passport is not one of the acts enumerated in Commonwealth
Act No. 63 constituting renunciation and loss of Philippine citizenship, 35 it is nevertheless an
act which repudiates the very oath of renunciation required for a former Filipino citizen who
is also a citizen of another country to be qualified to run for a local elective position.

When Arnado used his US passport on 14 April 2009, or just eleven days after he renounced
his American citizenship, he recanted his Oath of Renunciation 36 that he "absolutely and
perpetually renounce(s) all allegiance and fidelity to the UNITED STATES OF AMERICA"37 and
that he "divest(s) himself of full employment of all civil and political rights and privileges of
the United States of America."38

We agree with the COMELEC En Banc that such act of using a foreign passport does not divest
Arnado of his Filipino citizenship, which he acquired by repatriation. However, by representing
himself as an American citizen, Arnado voluntarily and effectively reverted to his earlier status
as a dual citizen. Such reversion was not retroactive; it took place the instant Arnado
represented himself as an American citizen by using his US passport.

This act of using a foreign passport after renouncing one’s foreign citizenship is fatal to
Arnado’s bid for public office, as it effectively imposed on him a disqualification to run for an
elective local position.

Arnado’s category of dual citizenship is that by which foreign citizenship is acquired through
a positive act of applying for naturalization. This is distinct from those considered dual citizens
by virtue of birth, who are not required by law to take the oath of renunciation as the mere
filing of the certificate of candidacy already carries with it an implied renunciation of foreign
citizenship.39 Dual citizens by naturalization, on the other hand, are required to take not only
the Oath of Allegiance to the Republic of the Philippines but also to personally renounce foreign
citizenship in order to qualify as a candidate for public office.

By the time he filed his certificate of candidacy on 30 November 2009, Arnado was a dual
citizen enjoying the rights and privileges of Filipino and American citizenship. He was qualified
to vote, but by the express disqualification under Section 40(d) of the Local Government
Code,40 he was not qualified to run for a local elective position.
In effect, Arnado was solely and exclusively a Filipino citizen only for a period of eleven days,
or from 3 April 2009 until 14 April 2009, on which date he first used his American passport
after renouncing his American citizenship.

This Court has previously ruled that:

Qualifications for public office are continuing requirements and must be possessed not only
at the time of appointment or election or assumption of office but during the officer's entire
tenure. Once any of the required qualifications is lost, his title may be seasonably challenged.
x x x.41

The citizenship requirement for elective public office is a continuing one. It must be possessed
not just at the time of the renunciation of the foreign citizenship but continuously. Any act
which violates the oath of renunciation opens the citizenship issue to attack.

We agree with the pronouncement of the COMELEC First Division that "Arnado’s act of
consistently using his US passport effectively negated his "Affidavit of Renunciation." 42 This
does not mean, that he failed to comply with the twin requirements under R.A. No. 9225, for
he in fact did.

It was after complying with the requirements that he performed positive acts which effectively
disqualified him from running for an elective public office pursuant to Section 40(d) of the
Local Government Code of 1991.

The purpose of the Local Government Code in disqualifying dual citizens from running for any
elective public office would be thwarted if we were to allow a person who has earlier renounced
his foreign citizenship, but who subsequently represents himself as a foreign citizen, to hold
any public office.

Arnado justifies the continued use of his US passport with the explanation that he was not
notified of the issuance of his Philippine passport on 18 June 2009, as a result of which he
was only able to obtain his Philippine passport three (3) months later. 43

The COMELEC En Banc differentiated Arnado from Willy Yu, the Portuguese national who
sought naturalization as a Filipino citizen and later applied for the renewal of his Portuguese
passport. That Arnado did not apply for a US passport after his renunciation does not make
his use of a US passport less of an act that violated the Oath of Renunciation he took. It was
still a positive act of representation as a US citizen before the immigration officials of this
country.

The COMELEC, in ruling favorably for Arnado, stated "Yet, as soon as he was in possession of
his Philippine passport, the respondent already used the same in his subsequent travels
abroad."44 We cannot agree with the COMELEC. Three months from June is September. If
indeed, Arnado used his Philippine passport as soon as he was in possession of it, he would
not have used his US passport on 24 November 2009.

Besides, Arnado’s subsequent use of his Philippine passport does not correct the fact that
after he renounced his foreign citizenship and prior to filing his certificate of candidacy, he
used his US passport. In the same way that the use of his foreign passport does not undo his
Oath of Renunciation, his subsequent use of his Philippine passport does not undo his earlier
use of his US passport.
Citizenship is not a matter of convenience. It is a badge of identity that comes with attendant
civil and political rights accorded by the state to its citizens. It likewise demands the
concomitant duty to maintain allegiance to one’s flag and country. While those who acquire
dual citizenship by choice are afforded the right of suffrage, those who seek election or
appointment to public office are required to renounce their foreign citizenship to be deserving
of the public trust. Holding public office demands full and undivided allegiance to the Republic
and to no other.

We therefore hold that Arnado, by using his US passport after renouncing his American
citizenship, has recanted the same Oath of Renunciation he took. Section 40(d) of the Local
Government Code applies to his situation. He is disqualified not only from holding the public
office but even from becoming a candidate in the May 2010 elections.

We now resolve the next issue.

Resolving the third issue necessitates revisiting Topacio v. Paredes 45 which is the
jurisprudential spring of the principle that a second-placer cannot be proclaimed as the winner
in an election contest. This doctrine must be re-examined and its soundness once again put
to the test to address the ever-recurring issue that a second-placer who loses to an ineligible
candidate cannot be proclaimed as the winner in the elections.

The Facts of the case are as follows:

On June 4, 1912, a general election was held in the town of Imus, Province of Cavite, to fill
the office of municipal president. The petitioner, Felipe Topacio, and the respondent, Maximo
Abad, were opposing candidates for that office. Topacio received 430 votes, and Abad 281.
Abad contested the election upon the sole ground that Topacio was ineligible in that he was
reelected the second time to the office of the municipal president on June 4, 1912, without
the four years required by Act No. 2045 having intervened. 46

Abad thus questioned the eligibility of To p a c i o on the basis of a statutory prohibition for
seeking a second re-election absent the four year interruption.

The often-quoted phrase in Topacio v. Paredes is that "the wreath of victory cannot be
transferred from an ineligible candidate to any other candidate when the sole question is the
eligibility of the one receiving a plurality of the legally cast ballots." 47

This phrase is not even the ratio decidendi; it is a mere obiter dictum. The Court was
comparing "the effect of a decision that a candidate is not entitled to the office because of
fraud or irregularities in the elections x x x with that produced by declaring a person ineligible
to hold such an office."

The complete sentence where the phrase is found is part of a comparison and contrast
between the two situations, thus:

Again, the effect of a decision that a candidate is not entitled to the office because of fraud
or irregularities in the elections is quite different from that produced by declaring a person
ineligible to hold such an office. In the former case the court, after an examination of the
ballots may find that some other person than the candidate declared to have received a
plurality by the board of canvassers actually received the greater number of votes, in which
case the court issues its mandamus to the board of canvassers to correct the returns
accordingly; or it may find that the manner of holding the election and the returns are so
tainted with fraud or illegality that it cannot be determined who received a plurality of the
legally cast ballots. In the latter case, no question as to the correctness of the returns or the
manner of casting and counting the ballots is before the deciding power, and generally the
only result can be that the election fails entirely. In the former, we have a contest in the strict
sense of the word, because of the opposing parties are striving for supremacy. If it be found
that the successful candidate (according to the board of canvassers) obtained a plurality in
an illegal manner, and that another candidate was the real victor, the former must retire in
favor of the latter. In the other case, there is not, strictly speaking, a contest, as the wreath
of victory cannot be transferred from an ineligible candidate to any other candidate when the
sole question is the eligibility of the one receiving a plurality of the legally cast ballots. In the
one case the question is as to who received a plurality of the legally cast ballots; in the other,
the question is confined to the personal character and circumstances of a single
individual.48 (Emphasis supplied)

Note that the sentence where the phrase is found starts with "In the other case, there is not,
strictly speaking, a contest" in contrast to the earlier statement, "In the former, we have a
contest in the strict sense of the word, because of the opposing parties are striving for
supremacy."

The Court in Topacio v. Paredes cannot be said to have held that "the wreath of victory cannot
be transferred from an ineligible candidate to any other candidate when the sole question is
the eligibility of the one receiving a plurality of the legally cast ballots."

A proper reading of the case reveals that the ruling therein is that since the Court of First
Instance is without jurisdiction to try a disqualification case based on the eligibility of the
person who obtained the highest number of votes in the election, its jurisdiction being
confined "to determine which of the contestants has been duly elected" the judge exceeded
his jurisdiction when he "declared that no one had been legally elected president of the
municipality of Imus at the general election held in that town on 4 June 1912" where "the
only question raised was whether or not Topacio was eligible to be elected and to hold the
office of municipal president."

The Court did not rule that Topacio was disqualified and that Abad as the second placer cannot
be proclaimed in his stead. The Court therein ruled:

For the foregoing reasons, we are of the opinion and so hold that the respondent judge
exceeded his jurisdiction in declaring in those proceedings that no one was elected municipal
president of the municipality of Imus at the last general election; and that said order and all
subsequent proceedings based thereon are null and void and of no effect; and, although this
decision is rendered on respondents' answer to the order to show cause, unless respondents
raised some new and additional issues, let judgment be entered accordingly in 5 days, without
costs. So ordered.49

On closer scrutiny, the phrase relied upon by a host of decisions does not even have a legal
basis to stand on. It was a mere pronouncement of the Court comparing one process with
another and explaining the effects thereof. As an independent statement, it is even illogical.

Let us examine the statement:


"x x x the wreath of victory cannot be transferred from an ineligible candidate to any other
candidate when the sole question is the eligibility of the one receiving a plurality of the legally
cast ballots."

What prevents the transfer of the wreath of victory from the ineligible candidate to another
candidate?

When the issue being decided upon by the Court is the eligibility of the one receiving a plurality
of the legally cast ballots and ineligibility is thereafter established, what stops the Court from
adjudging another eligible candidate who received the next highest number of votes as the
winner and bestowing upon him that "wreath?"

An ineligible candidate who receives the highest number of votes is a wrongful winner. By
express legal mandate, he could not even have been a candidate in the first place, but by
virtue of the lack of material time or any other intervening circumstances, his ineligibility
might not have been passed upon prior to election date. Consequently, he may have had the
opportunity to hold himself out to the electorate as a legitimate and duly qualified candidate.
However, notwithstanding the outcome of the elections, his ineligibility as a candidate remains
unchanged. Ineligibility does not only pertain to his qualifications as a candidate but
necessarily affects his right to hold public office. The number of ballots cast in his favor cannot
cure the defect of failure to qualify with the substantive legal requirements of eligibility to run
for public office.

The popular vote does not cure the


ineligibility of a candidate.

The ballot cannot override the constitutional and statutory requirements for qualifications and
disqualifications of candidates. When the law requires certain qualifications to be possessed
or that certain disqualifications be not possessed by persons desiring to serve as elective
public officials, those qualifications must be met before one even becomes a candidate. When
a person who is not qualified is voted for and eventually garners the highest number of votes,
even the will of the electorate expressed through the ballot cannot cure the defect in the
qualifications of the candidate. To rule otherwise is to trample upon and rent asunder the very
law that sets forth the qualifications and disqualifications of candidates. We might as well
write off our election laws if the voice of the electorate is the sole determinant of who should
be proclaimed worthy to occupy elective positions in our republic.

This has been, in fact, already laid down by the Court in Frivaldo v. COMELEC 50 when we
pronounced:

x x x. The fact that he was elected by the people of Sorsogon does not excuse this patent
violation of the salutary rule limiting public office and employment only to the citizens of this
country. The qualifications prescribed for elective office cannot be erased by the electorate
alone.

The will of the people as expressed through the ballot cannot cure the vice of ineligibility,
especially if they mistakenly believed, as in this case, that the candidate was qualified.
Obviously, this rule requires strict application when the deficiency is lack of citizenship. If a
person seeks to serve in the Republic of the Philippines, he must owe his total loyalty to this
country only, abjuring and renouncing all fealty and fidelity to any other state.51 (Emphasis
supplied)
This issue has also been jurisprudentially clarified in Velasco v. COMELEC 52 where the Court
ruled that the ruling in Quizon and Saya-ang cannot be interpreted without qualifications lest
"Election victory x x x becomes a magic formula to bypass election eligibility requirements." 53

We have ruled in the past that a candidate’s victory in the election may be considered a
sufficient basis to rule in favor of the candidate sought to be disqualified if the main issue
involves defects in the candidate’s certificate of candidacy. We said that while provisions
relating to certificates of candidacy are mandatory in terms, it is an established rule of
interpretation as regards election laws, that mandatory provisions requiring certain steps
before elections will be construed as directory after the elections, to give effect to the will of
the people. We so ruled in Quizon v. COMELEC and Saya-ang v. COMELEC:

The present case perhaps presents the proper time and opportunity to fine-tune our above
ruling. We say this with the realization that a blanket and unqualified reading and application
of this ruling can be fraught with dangerous significance for the rule of law and the integrity
of our elections. For one, such blanket/unqualified reading may provide a way around the law
that effectively negates election requirements aimed at providing the electorate with the basic
information to make an informed choice about a candidate’s eligibility and fitness for office.

The first requirement that may fall when an unqualified reading is made is Section 39 of the
LGC which specifies the basic qualifications of local government officials. Equally susceptive
of being rendered toothless is Section 74 of the OEC that sets out what should be stated in a
COC. Section 78 may likewise be emasculated as mere delay in the resolution of the petition
to cancel or deny due course to a COC can render a Section 78 petition useless if a candidate
with false COC data wins. To state the obvious, candidates may risk falsifying their COC
qualifications if they know that an election victory will cure any defect that their COCs may
have. Election victory then becomes a magic formula to bypass election eligibility
requirements. (Citations omitted)

What will stop an otherwise disqualified individual from filing a seemingly valid COC,
concealing any disqualification, and employing every strategy to delay any disqualification
case filed against him so he can submit himself to the electorate and win, if winning the
election will guarantee a disregard of constitutional and statutory provisions on qualifications
and disqualifications of candidates?

It is imperative to safeguard the expression of the sovereign voice through the ballot by
ensuring that its exercise respects the rule of law. To allow the sovereign voice spoken through
the ballot to trump constitutional and statutory provisions on qualifications and
disqualifications of candidates is not democracy or republicanism. It is electoral anarchy.
When set rules are disregarded and only the electorate’s voice spoken through the ballot is
made to matter in the end, it precisely serves as an open invitation for electoral anarchy to
set in.1âwphi1

Maquiling is not a second-placer as


he obtained the highest number of
votes from among the qualified
candidates.

With Arnado’s disqualification, Maquiling then becomes the winner in the election as he
obtained the highest number of votes from among the qualified candidates.
We have ruled in the recent cases of Aratea v. COMELEC54 and Jalosjos v. COMELEC55 that a
void COC cannot produce any legal effect.

Thus, the votes cast in favor of the ineligible candidate are not considered at all in determining
the winner of an election.

Even when the votes for the ineligible candidate are disregarded, the will of the electorate is
still respected, and even more so. The votes cast in favor of an ineligible candidate do not
constitute the sole and total expression of the sovereign voice. The votes cast in favor of
eligible and legitimate candidates form part of that voice and must also be respected.

As in any contest, elections are governed by rules that determine the qualifications and
disqualifications of those who are allowed to participate as players. When there are
participants who turn out to be ineligible, their victory is voided and the laurel is awarded to
the next in rank who does not possess any of the disqualifications nor lacks any of the
qualifications set in the rules to be eligible as candidates.

There is no need to apply the rule cited in Labo v. COMELEC 56 that when the voters are well
aware within the realm of notoriety of a candidate’s disqualification and still cast their votes
in favor said candidate, then the eligible candidate obtaining the next higher number of votes
may be deemed elected. That rule is also a mere obiter that further complicated the rules
affecting qualified candidates who placed second to ineligible ones.

The electorate’s awareness of the candidate’s disqualification is not a prerequisite for the
disqualification to attach to the candidate. The very existence of a disqualifying circumstance
makes the candidate ineligible. Knowledge by the electorate of a candidate’s disqualification
is not necessary before a qualified candidate who placed second to a disqualified one can be
proclaimed as the winner. The second-placer in the vote count is actually the first-placer
among the qualified candidates.

That the disqualified candidate has already been proclaimed and has assumed office is of no
moment. The subsequent disqualification based on a substantive ground that existed prior to
the filing of the certificate of candidacy voids not only the COC but also the proclamation.

Section 6 of R.A. No. 6646 provides:

Section 6. Effect of Disqualification Case. - Any candidate who has been declared by final
judgment to be disqualified shall not be voted for, and the votes cast for him shall not be
counted. If for any reason a candidate is not declared by final judgment before an election to
be disqualified and he is voted for and receives the winning number of votes in such election,
the Court or Commission shall continue with the trial and hearing of the action, inquiry, or
protest and, upon motion of the complainant or any intervenor, may during the pendency
thereof order the suspension of the proclamation of such candidate whenever the evidence of
his guilt is strong.

There was no chance for Arnado’s proclamation to be suspended under this rule because
Arnado failed to file his answer to the petition seeking his disqualification. Arnado only filed
his Answer on 15 June 2010, long after the elections and after he was already proclaimed as
the winner.
The disqualifying circumstance surrounding Arnado’s candidacy involves his citizenship. It
does not involve the commission of election offenses as provided for in the first sentence of
Section 68 of the Omnibus Election Code, the effect of which is to disqualify the individual
from continuing as a candidate, or if he has already been elected, from holding the office.

The disqualifying circumstance affecting Arnado is his citizenship. As earlier discussed, Arnado
was both a Filipino and an American citizen when he filed his certificate of candidacy. He was
a dual citizen disqualified to run for public office based on Section 40(d) of the Local
Government Code.

Section 40 starts with the statement "The following persons are disqualified from running for
any elective local position." The prohibition serves as a bar against the individuals who fall
under any of the enumeration from participating as candidates in the election.

With Arnado being barred from even becoming a candidate, his certificate of candidacy is thus
rendered void from the beginning. It could not have produced any other legal effect except
that Arnado rendered it impossible to effect his disqualification prior to the elections because
he filed his answer to the petition when the elections were conducted already and he was
already proclaimed the winner.

To hold that such proclamation is valid is to negate the prohibitory character of the
disqualification which Arnado possessed even prior to the filing of the certificate of candidacy.
The affirmation of Arnado's disqualification, although made long after the elections, reaches
back to the filing of the certificate of candidacy. Arnado is declared to be not a candidate at
all in the May 201 0 elections.

Arnado being a non-candidate, the votes cast in his favor should not have been counted. This
leaves Maquiling as the qualified candidate who obtained the highest number of votes.
Therefore, the rule on succession under the Local Government Code will not apply.

WHEREFORE, premises considered, the Petition is GRANTED. The Resolution of the COMELEC
En Bane dated 2 February 2011 is hereby ANNULLED and SET ASIDE. Respondent ROMMEL
ARNADO y CAGOCO is disqualified from running for any local elective position. CASAN
MACODE MAQUILING is hereby DECLARED the duly elected Mayor of Kauswagan, Lanao del
Norte in the 10 May 2010 elections.

This Decision is immediately executory.

Let a copy of this Decision be served personally upon the parties and the Commission on
Elections.

No pronouncement as to costs.

SO ORDERED.

MARIA LOURDES P. A. SERENO


Chief Justice

WE CONCUR:
[G.R. No. 120295. June 28, 1996]

JUAN G. FRIVALDO, petitioner, vs. COMMISSION ON ELECTIONS, and RAUL R.


LEE, respondents.

[G.R. No. 123755. June 28, 1996]

RAUL R. LEE, petitioner, vs. COMMISSION ON ELECTIONS and JUAN G.


FRIVALDO, respondents.

DECISION
PANGANIBAN, J.:

The ultimate question posed before this Court in these twin cases is: Who should be
declared the rightful governor of Sorsogon

(i) Juan G. Frivaldo, who unquestionably obtained the highest number of votes in three
successive elections but who was twice declared by this Court to be disqualified to hold such
office due to his alien citizenship, and who now claims to have re-assumed his lost Philippine
citizenship thru repatriation;

(ii) Raul R. Lee, who was the second placer in the canvass, but who claims that the votes cast
in favor of Frivaldo should be considered void; that the electorate should be deemed to have
intentionally thrown away their ballots; and that legally, he secured the most number of valid
votes; or

(iii) The incumbent Vice-Governor, Oscar G. Deri, who obviously was not voted directly to the
position of governor, but who according to prevailing jurisprudence should take over the said
post inasmuch as, by the ineligibility of Frivaldo, a "permanent vacancy in the contested office
has occurred"?

In ruling for Frivaldo, the Court lays down new doctrines on repatriation,
clarifies/reiterates/amplifies existing jurisprudence on citizenship and elections, and upholds
the superiority of substantial justice over pure legalisms.
G.R. No. 123755.
This is a special civil action under Rules 65 and 58 of the Rules of Court for certiorari and
preliminary injunction to review and annul a Resolution of the respondent Commission on
Elections (Comelec), First Division,1 promulgated on December 19,19952 and another
Resolution of the Comelec en bane promulgated February 23, 19963 denying petitioner's
motion for reconsideration.

The Facts
On March 20, 1995, private respondent Juan G. Frivaldo filed his Certificate of Candidacy
for the office of Governor of Sorsogon in the May 8, 1995 elections. On March 23, 1995,
petitioner Raul R. Lee, another candidate, filed a petition 4 with the Comelec docketed as SPA
No. 95-028 praying that Frivaldo "be disqualified from seeking or holding any public office or
position by reason of not yet being a citizen of the Philippines," and that his Certificate of
Candidacy be cancelled. On May 1, 1995, the Second Division of the Comelec promulgated a
Resolution5 granting the petition with the following disposition:6

"WHEREFORE, this Division resolves to GRANT the petition and declares that respondent is
DISQUALIFIED to run for the Office of Governor of Sorsogon on the ground that he is NOT a
citizen of the Philippines. Accordingly, respondent's certificate of candidacy is cancelled."

The Motion for Reconsideration filed by Frivaldo remained unacted upon until after
the May 8, 1995 elections. So, his candidacy continued and he was voted for during the
elections held on said date. On May 11, 1995, the Comelec en banc7 affirmed the
aforementioned Resolution of the Second Division.
The Provincial Board of Canvassers completed the canvass of the election returns and a
Certificate of Votes8.dated May 27, 1995 was issued showing the following votes obtained by
the candidates for the position of Governor of Sorsogon:

Antonio H. Escudero, Jr. 51,060

Juan G. Frivaldo 73,440

RaulR.Lee 53,304

Isagani P. Ocampo 1,925

On June 9, 1995, Lee filed in said SPA No. 95-028, a (supplemental) petition9 praying for
his proclamation as the duly-elected Governor of Sorsogon.
In an order10 dated June 21, 1995, but promulgated according to the petition "only on
June 29, 1995," the Comelec en bane directed "the Provincial Board of Canvassers of
Sorsogon to reconvene for the purpose of proclaiming candidate Raul Lee as the winning
gubernatorial candidate in the province of Sorsogon on June 29,1995 x x x." Accordingly,
at 8:30 in the evening of June 30,1995, Lee was proclaimed governor of Sorsogon.
On July 6, 1995, Frivaldo filed with the Comelec a new petition, 11 docketed as SPC No.
95-317, praying for the annulment of the June 30, 1995 proclamation of Lee and for his own
proclamation. He alleged that on June 30, 1995, at 2:00 in the afternoon, he took his oath of
allegiance as a citizen of the Philippines after "his petition for repatriation under P.D. 725
which he filed with the Special Committee on Naturalization in September 1994 had been
granted." As such, when "the said order (dated June 21, 1995) (of the Comelec) x x x was
released and received by Frivaldo on June 30, 1995 at 5:30 o'clock in the evening, there was
no more legal impediment to the proclamation (of Frivaldo) as governor x x x." In the
alternative, he averred that pursuant to the two cases of Labo vs. Comelec,12 the Vice-
Governor not Lee should occupy said position of governor.
On December 19, 1995, the Comelec First Division promulgated the herein assailed
Resolution13 holding that Lee, "not having garnered the highest number of votes," was not
legally entitled to be proclaimed as duly-elected governor; and that Frivaldo, "having garnered
the highest number of votes, and xxx having reacquired his Filipino citizenship by repatriation
on June 30, 1995 under the provisions of Presidential Decree No. 725 xxx (is) qualified to
hold the office of governor of Sorsogon"; thus:

"PREMISES CONSIDERED, the Commission (First Division), therefore RESOLVES to GRANT


the Petition.

Consistent with the decisions of the Supreme Court, the proclamation of Raul R. Lee as
Governor of Sorsogon is hereby ordered annulled, being contrary to law, he not having
garnered the highest number of votes to warrant his proclamation.

Upon the finality of the annulment of the proclamation of Raul R. Lee, the Provincial Board of
Canvassers is directed to immediately reconvene and, on the basis of the completed canvass,
proclaim petitioner Juan G. Frivaldo as the duly elected Governor of Sorsogon having garnered
the highest number of votes, and he having reacquired his Filipino citizenship by repatriation
on June 30,1995 under the provisions of Presidential Decree No. 725 and, thus, qualified to
hold the office of Governor of Sorsogon.

Conformably with Section 260 of the Omnibus Election Code (B.P. Blg. 881), the Clerk of the
Commission is directed to notify His Excellency the President of the Philippines, and the
Secretary of the Sangguniang Panlalawigan of the Province of Sorsogon of this resolution
immediately upon the due implementation thereof."

On December 26,1995, Lee filed a motion for reconsideration which was denied by the
Comelec en banc in its Resolution14 promulgated on February 23, 1996. On February 26,
1996, the present petition was filed. Acting on the prayer for a temporary restraining order,
this Court issued on February 27, 1996 a Resolution which inter alia directed the parties "to
maintain the status quo prevailing prior to the filing of this petition."

The Issues in G.R. No. 123755

Petitioner Lee's "position on the matter at hand briefly be capsulized in the following
propositions":15

"First - The initiatory petition below was so far insufficient in form and substance to warrant
the exercise by the COMELEC of its jurisdiction with the result that, in effect, the COMELEC
acted without jurisdiction in taking cognizance of and deciding said petition;

Second- The judicially declared disqualification of respondent was a continuing condition and
rendered him ineligible to run for, to be elected to and to hold the Office of Governor;

Third - The alleged repatriation of respondent was neither valid nor is the effect thereof
retroactive as to cure his ineligibility and qualify him to hold the Office of Governor; and

Fourth - Correctly read and applied, the Labo Doctrine fully supports the validity of petitioner's
proclamation as duly elected Governor of Sorsogon."

G.R. No. 120295


This is a petition to annul three Resolutions of the respondent Comelec, the first two of
which are also at issue in G.R. No. 123755, as follows:

1. Resolution16 of the Second Division, promulgated on May 1, 1995, disqualifying Frivaldo


from running for governor of Sorsogon in the May 8, 1995 elections "on the ground that he is
not a citizen of the Philippines";

2. Resolution17 of the Comelec en bane, promulgated on May 11, 1995; and

3. Resolution18 of the Comelec en bane, promulgated also on May 11, 1995 suspending the
proclamation of, among others, Frivaldo.

The Facts and the Issue

The facts of this case are essentially the same as those in G.R. No. 123755. However,
Frivaldo assails the above-mentioned resolutions on a different ground: that under Section
78 of the Omnibus Election Code, which is reproduced hereinunder:

"Section 78. Petition to deny due course or to cancel a certificate of candidacy. A verified
petition seeking to deny due course or to cancel a certificate of candidacy may be filed by any
person exclusively on the ground that any material representation contained therein as
required under Section 74 hereof is false. The petition may be filed at any time not later than
twenty-five days from the time of the filing of the certificate of candidacy and shall be
decided, after notice and hearing, not later than fifteen days before the election." (Italics
supplied.)

the Comelec had no jurisdiction to issue said Resolutions because they were not rendered
"within the period allowed by law," i.e., "not later than fifteen days before the election."
Otherwise stated, Frivaldo contends that the failure of the Comelec to act on the petition
for disqualification within the period of fifteen days prior to the election as provided by law is
a jurisdictional defect which renders the said Resolutions null and void.
By Resolution on March 12, 1996, the Court consolidated G.R. Nos. 120295 and 123755
since they are intimately related in their factual environment and are identical in the ultimate
question raised, viz., who should occupy the position of governor of the province of Sorsogon.
On March 19, 1995, the Court heard oral argument from the parties and required them
thereafter to file simultaneously their respective memoranda.

The Consolidated Issues

From the foregoing submissions, the consolidated issues may be restated as follows:

1. Was the repatriation of Frivaldo valid and legal? If so, did it seasonably cure his lack of
citizenship as to qualify him to be proclaimed and to hold the Office of Governor? If not, may
it be given retroactive effect? If so, from when?
2. Is Frivaldo's "judicially declared" disqualification for lack of Filipino citizenship a continuing
bar to his eligibility to run for, be elected to or hold the governorship of Sorsogon?

3. Did the respondent Comelec have jurisdiction over the initiatory petition in SPC No. 95-317
considering that : said petition is not "a pre-proclamation case, an election protest or a quo
warranto case"?

4. Was the proclamation of Lee, a runner-up in the election, valid and legal in light of existing
jurisprudence?

5. Did the respondent Commission on Elections exceed its jurisdiction in promulgating the
assailed Resolutions, all of which prevented Frivaldo from assuming the governorship of
Sorsogon, considering that they were not rendered within ( the period referred to in Section
78 of the Omnibus Election Code, viz., "not later than fifteen days before the elections"?

The First Issue: Frivaldo's Repatriation

The validity and effectivity of Frivaldo's repatriation is the lis mota, the threshold legal
issue in this case. All the other matters raised are secondary to this.
The Local Government Code of 199119 expressly requires Philippine citizenship as a
qualification for elective local officials, including that of provincial governor, thus:

"Sec. 39. Qualifications. (a) An elective local official must be a citizen of the Philippines; a
registered voter in the barangay, municipality, city, or province or, in the case of a member
of the sangguniang panlalawigan, sangguniang panlungsod, or sangguniang bayan, the
district where he intends to be elected; a resident therein for at least one (1) year immediately
preceding the day of the election; and able to read and write Filipino or any other local
language or dialect.

(b) Candidates for the position of governor, vice governor or member of the sangguniang
panlalawigan, or mayor, vice mayor or member of the sangguniang panlungsod of highly
urbanized cities must be at least twenty-three (23) years of age on election day.

xxx xxx xxx


Inasmuch as Frivaldo had been declared by this Court 20 as a non-citizen, it is therefore
incumbent upon him to show that he has reacquired citizenship; in fine, that he possesses
the qualifications prescribed under the said statute (R. A. 7160).
Under Philippine law,21 citizenship may be reacquired by direct act of Congress, by
naturalization or by repatriation. Frivaldo told this Court in G.R. No. 104654 22 and during the
oral argument in this case that he tried to resume his citizenship by direct act of Congress, but
that the bill allowing him to do so "failed to materialize, notwithstanding the endorsement of
several members of the House of Representatives" due, according to him, to the "maneuvers
of his political rivals." In the same case, his attempt at naturalization was rejected by this
Court because of jurisdictional, substantial and procedural defects.
Despite his lack of Philippine citizenship, Frivaldo was overwhelmingly elected governor
by the electorate of Sorsogon, with a margin of 27,000 votes in the 1988 elections, 57,000
in 1992, and 20,000 in 1995 over the same opponent Raul Lee. Twice, he was judicially
declared a non-Filipino and thus twice disqualified from holding and discharging his popular
mandate. Now, he comes to us a third time, with a fresh vote from the people of Sorsogon
and a favorable decision from the Commission on Elections to boot. Moreover, he now boasts
of having successfully passed through the third and last mode of reacquiring citizenship: by
repatriation under P.D. No. 725, with no less than the Solicitor General himself, who was the
prime opposing counsel in the previous cases he lost, this time, as counsel for co-respondent
Comelec, arguing the validity of his cause (in addition to his able private counsel Sixto S.
Brillantes, Jr.). That he took his oath of allegiance under the provisions of said Decree at 2:00
p.m. on June 30, 1995 is not disputed. Hence, he insists that henot Leeshould have been
proclaimed as the duly-elected governor of Sorsogon when the Provincial Board of Canvassers
met at 8:30 p.m. on the said date since, clearly and unquestionably, he garnered the highest
number of votes in the elections and since at that time, he already reacquired his citizenship.
En contrario, Lee argues that Frivaldo's repatriation is tainted ; with serious defects,
which we shall now discuss in seriatim.
First, Lee tells us that P.D. No. 725 had "been effectively repealed," asserting that "then
President Corazon Aquino exercising legislative powers under the Transitory Provisions of the
1987 Constitution, forbade the grant of citizenship by Presidential Decree or Executive
Issuances as the same poses a serious and contentious issue of policy which the present
government, in the exercise of prudence and sound discretion, should best leave to the
judgment of the first Congress under the 1987 Constitution," adding that in her memorandum
dated March 27,1987 to the members of the Special Committee on Naturalization constituted
for purposes of Presidential Decree No. 725, President Aquino directed them "to cease and
desist from undertaking any and all proceedings within your functional area of responsibility
as defined under Letter of Instructions (LOI) No. 270 dated April 11, 1975, as amended."23
This memorandum dated March 27, 198724 cannot by any stretch of legal hermeneutics
be construed as a law sanctioning or authorizing a repeal of P.D. No. 725. Laws are repealed
only by subsequent ones25 and a repeal may be express or implied. It is obvious that no
express repeal was made because then President Aquino in her memorandum based on the
copy furnished us by Lee did not categorically and/or impliedly state that P.D. 725 was being
repealed or was being rendered without any legal effect. In fact, she did not even mention it
specifically by its number or text. On the other hand, it is a basic rule of statutory construction
that repeals by implication are not favored. An implied repeal will not be allowed "unless it is
convincingly and unambiguously demonstrated that the two laws are clearly repugnant and
patently inconsistent that they cannot co-exist."26
The memorandum of then President Aquino cannot even be regarded as a legislative
enactment, for not every pronouncement of the Chief Executive even under the Transitory
Provisions of the 1987 Constitution can nor should be regarded as an exercise of her law-
making powers. At best, it could be treated as an executive policy addressed to the Special
Committee to halt the acceptance and processing of applications for repatriation pending
whatever "judgment the first Congress under the 1987 Constitution" might make. In other
words, the former President did not repeal P.D. 725 but left it to the first Congress once
createdto deal with the matter. If she had intended to repeal such law, she should have
unequivocally said so instead of referring the matter to Congress. The fact is she carefully
couched her presidential issuance in terms that clearly indicated the intention of "the present
government, in the exercise of prudence and sound discretion" to leave the matter of repeal
to the new Congress. Any other interpretation of the said Presidential Memorandum, such as
is now being proffered to the Court by Lee, would visit unmitigated violence not only upon
statutory construction but on common sense as well.
Second. Lee also argues that "serious congenital irregularities flawed the repatriation
proceedings," asserting that Frivaldo's application therefor was "filed on June 29, 1995 x x x
(and) was approved in just one day or on June 30, 1995 x x x," which "prevented a judicious
review and evaluation of the merits thereof." Frivaldo counters that he filed his application for
repatriation with the Office of the President in Malacanang Palace on August 17, 1994. This is
confirmed by the Solicitor General. However, the Special Committee was reactivated only
on June 8, 1995, when presumably the said Committee started processing his application.
On June 29, 1995, he filled up and re-submitted the FORM that the Committee required.
Under these circumstances, it could not be said that there was "indecent haste" in the
processing of his application.
Anent Lee's charge that the "sudden reconstitution of the Special Committee on
Naturalization was intended solely for the personal interest of respondent," 27 the Solicitor
General explained during the oral argument on March 19, 1996 that such allegation is simply
baseless as there were many others who applied and were considered for repatriation, a list
of whom was submitted by him to this Court, through a Manifestation 28 filed on April 3, 1996.
On the basis of the parties' submissions, we are convinced that the presumption of
regularity in the performance of official duty and the presumption of legality in the repatriation
of Frivaldo have not been successfully rebutted by Lee. The mere fact that the proceedings
were speeded up is by itself not a ground to conclude that such proceedings were necessarily
tainted. After all, the requirements of repatriation under P.D. No. 725 are not difficult to
comply with, nor are they tedious and cumbersome. In fact, P.D. 725 29 itself requires very
little of an applicant, and even the rules and regulations to implement the said decree were
left to the Special Committee to promulgate. This is not unusual since, unlike in naturalization
where an alien covets a first-time entry into Philippine political life, in repatriation the
applicant is a former natural-born Filipino who is merely seeking to reacquire his previous
citizenship. In the case of Frivaldo, he was undoubtedly a natural-born citizen who openly and
faithfully served his country and his province prior to his naturalization in the United States a
naturalization he insists was made necessary only to escape the iron clutches of a dictatorship
he abhorred and could not in conscience embrace and who, after the fall of the dictator and
the re-establishment of democratic space, wasted no time in returning to his country of birth
to offer once more his talent and services to his people.
So too, the fact that ten other persons, as certified to by the Solicitor General, were
granted repatriation argues convincingly and conclusively against the existence of favoritism
vehemently posited by Raul Lee. At any rate, any contest on the legality of Frivaldo's
repatriation should have been pursued before the Committee itself, and, failing there, in the
Office of the President, pursuant to the doctrine of exhaustion of administrative remedies.
Third. Lee further contends that assuming the assailed repatriation to be valid,
nevertheless it could only be effective as at 2:00 p.m. of June 30, 1995 whereas the
citizenship qualification prescribed by the Local Government Code "must exist on the date of
his election, if not when the certificate of candidacy is filed," citing our decision in G.R.
10465430 which held that "both the Local Government Code and the Constitution require that
only Philippine citizens can run and be elected to Public office" Obviously, however, this was
a mere obiter as the only issue in said case was whether Frivaldo's naturalization was valid
or not and NOT the effective date thereof. Since the Court held his naturalization to be invalid,
then the issue of when an aspirant for public office should be a citizen was NOT resolved at
all by the Court. Which question we shall now directly rule on.
Under Sec. 39 of the Local Government Code, "(a)n elective local official must be:
* a citizen of the Philippines;
* a registered voter in the barangay, municipality, city, or province x x x where he
intends to be elected;
* a resident therein for at least one (1) year immediately preceding the day of the
election;
* able to read and write Filipino or any other local language or dialect."
* In addition, "candidates for the position of governor x x x must be at least twenty-
three (23) years of age on election day."
From the above, it will be noted that the law does not specify any particular date or time
when the candidate must possess citizenship, unlike that for residence (which must consist
of at least one year's residency immediately preceding the day of election) and age (at least
twenty three years of age on election day).
Philippine citizenship is an indispensable requirement for holding an elective public
office,31 and the purpose of the citizenship qualification is none other than to ensure that no
alien, i.e., no person owing allegiance to another nation, shall govern our people and our
country or a unit of territory thereof. Now, an official begins to govern or to discharge his
functions only upon his proclamation and on the day the law mandates his term of office to
begin. Since Frivaldo re-assumed his citizenship on June 30, 1995the very day 32 the term of
office of governor (and other elective officials) beganhe was therefore already qualified to be
proclaimed, to hold such office and to discharge the functions and responsibilities thereof as
of said date. In short, at that time, he was already qualified to govern his native Sorsogon.
This is the liberal interpretation that should give spirit, life and meaning to our law on
qualifications consistent with the purpose for which such law was enacted. So too, even from
a literal (as distinguished from liberal) construction, it should be noted that Section 39 of the
Local Government Code speaks of "Qualifications" of "ELECTIVE OFFICIALS," not of
candidates. Why then should such qualification be required at the time of election or at the
time of the filing of the certificates of candidacies, as Lee insists? Literally, such qualifications
unless otherwise expressly conditioned, as in the case of age and residence should thus be
possessed when the "elective [or elected] official" begins to govern, i.e., at the time he is
proclaimed and at the start of his term in this case, on June 30, 1995. Paraphrasing this
Court's ruling in Vasquez vs. Giapand Li Seng Giap & Sons,33 if the purpose of the citizenship
requirement is to ensure that our people and country do not end up being governed by aliens,
i.e., persons owing allegiance to another nation, that aim or purpose would not be thwarted
but instead achieved by construing the citizenship qualification as applying to the time of
proclamation of the elected official and at the start of his term.
But perhaps the more difficult objection was the one raised during the oral argument 34 to
the effect that the citizenship qualification should be possessed at the time the candidate (or
for that matter the elected official) registered as a voter. After all, Section 39, apart from
requiring the official to be a citizen, also specifies as another item of qualification, that he be
a "registered voter." And, under the law35 a "voter" must be a citizen of the Philippines. So
therefore, Frivaldo could not have been a voter-much less a validly registered one if he was
not a citizen at the time of such registration.
The answer to this problem again lies in discerning the purpose of the requirement. If the
law intended the citizenship qualification to be possessed prior to election consistent with the
requirement of being a registered voter, then it would not have made citizenship a SEPARATE
qualification. The law abhors a redundancy. It therefore stands to reason that the law intended
CITIZENSHIP to be a qualification distinct from being a VOTER, even if being a voter presumes
being a citizen first. It also stands to reason that the voter requirement was included as
another qualification (aside from "citizenship"), not to reiterate the need for nationality but to
require that the official be registered as a voter IN THE AREA OR TERRITORY he seeks to
govern, i.e., the law states: "a registered voter in the barangay, municipality, city, or province
x x x where he intends to be elected." It should be emphasized that the Local Government
Code requires an elective official to be a registered voter. It does not require him to
vote actually. Hence, registrationnot the actual votingis the core of this "qualification." In
other words, the law's purpose in this second requirement is to ensure that the prospective
official is actually registered in the area he seeks to govern and not anywhere else.
Before this Court, Frivaldo has repeatedly emphasizedand Lee has not disputed that he
"was and is a registered voter of Sorsogon, and his registration as a voter has been sustained
as valid by judicial declaration x x x In fact, he cast his vote in his precinct on May 8, 1995."36
So too, during the oral argument, his counsel stead-fastly maintained that "Mr. Frivaldo
has always been a registered voter of Sorsogon. He has voted in 1987,1988,1992, then he
voted again in 1995. In fact, his eligibility as a voter was questioned, but the court dismissed
(sic) his eligibility as a voter and he was allowed to vote as in fact, he voted in all the previous
elections including on May 8,1995.37
It is thus clear that Frivaldo is a registered voter in the province where he intended to be
elected.
There is yet another reason why the prime issue of citizenship should be reckoned from
the date of proclamation, not necessarily the date of election or date of filing of the certificate
of candidacy. Section 253 of the Omnibus Election Code38 gives any voter, presumably
including the defeated candidate, the opportunity to question the ELIGIBILITY (or the
disloyalty) of a candidate. This is the only provision of the Code that authorizes a remedy on
how to contest before the Comelec an incumbent's ineligibility arising from failure to meet the
qualifications enumerated under Sec. 39 of the Local Government Code. Such remedy of Quo
Warranto can be availed of "within ten days after proclamation" of the winning candidate.
Hence, it is only at such time that the issue of ineligibility may be taken cognizance of by the
Commission. And since, at the very moment of Lee's proclamation (8:30 p.m., June 30,
1995), Juan G. Frivaldo was already and indubitably a citizen, having taken his oath of
allegiance earlier in the afternoon of the same day, then he should have been the candidate
proclaimed as he unquestionably garnered the highest number of votes in the immediately
preceding elections and such oath had already cured his previous "judicially-declared"
alienage. Hence, at such time, he was no longer ineligible.
But to remove all doubts on this important issue, we also hold that the repatriation of
Frivaldo RETRO ACTED to the date of the filing of his application on August 17,1994.
It is true that under the Civil Code of the Philippines,39 "(l)aws shall have no retroactive
effect, unless the contrary is provided." But there are settled exceptions40 to this general rule,
such as when the statute is CURATIVE or REMEDIAL in nature or when it CREATES NEW
RIGHTS.
According to Tolentino,41 curative statutes are those which undertake to cure errors and
irregularities, thereby validating judicial or administrative proceedings, acts of public officers,
or private deeds and contracts which otherwise would not produce their intended
consequences by reason of some statutory disability or failure to comply with some technical
requirement. They operate on conditions already existing, and are necessarily retroactive in
operation. Agpalo,42 on the other hand, says that curative statutes are "healing acts x x x
curing defects and adding to the means of enforcing existing obligations x x x (and) are
intended to supply defects, abridge superfluities in existing laws, and curb certain evils x x x
By their very nature, curative statutes are retroactive xxx (and) reach back to past events to
correct errors or irregularities and to render valid and effective attempted acts which would
be otherwise ineffective for the purpose the parties intended."
On the other hand, remedial or procedural laws, i.e., those statutes relating to remedies
or modes of procedure, which do not create new or take away vested rights, but only operate
in furtherance of the remedy or confirmation of such rights, ordinarily do not come within the
legal meaning of a retrospective law, nor within the general rule against the retrospective
operation of statutes.43
A reading of P.D. 725 immediately shows that it creates a new right, and also provides
for a new remedy, thereby filling certain voids in our laws. Thus, in its preamble, P.D. 725
expressly recognizes the plight of "many Filipino women (who) had lost their Philippine
citizenship by marriage to aliens" and who could not, under the existing law (C. A. No. 63, as
amended) avail of repatriation until "after the death of their husbands or the termination of
their marital status" and who could neither be benefitted by the 1973 Constitution's new
provision allowing "a Filipino woman who marries an alien to retain her Philippine citizenship
xxx" because "such provision of the new Constitution does not apply to Filipino women who
had married aliens before said constitution took effect." Thus, P.D. 725 granted a new right
to these womenthe right to re-acquire Filipino citizenship even during their marital coverture,
which right did not exist prior to P.D. 725. On the other hand, said statute also provided a
new remedy and a new right in favor of other "natural born Filipinos who (had) lost their
Philippine citizenship but now desire to re-acquire Philippine citizenship," because prior to the
promulgation of P.D. 725 such former Filipinos would have had to undergo the tedious and
cumbersome process of naturalization, but with the advent of P.D. 725 they could now re-
acquire their Philippine citizenship under the simplified procedure of repatriation.
The Solicitor General44 argues:

"By their very nature, curative statutes are retroactive, (DBP vs. CA, 96 SCRA 342), since
they are intended to supply defects, abridge superfluities in existing laws (Del
Castillo vs. Securities and Exchange Commission, 96 Phil. 119) and curb certain evils
(Santos vs. Duata, 14 SCRA 1041).

In this case, P.D. No. 725 was enacted to cure the defect in the existing naturalization law,
specifically C. A. No. 63 wherein married Filipino women are allowed to repatriate only upon
the death of their husbands, and natural-born Filipinos who lost their citizenship by
naturalization and other causes faced the difficulty of undergoing the rigid procedures of C.A.
63 for reacquisition of Filipino citizenship by naturalization.

Presidential Decree No. 725 provided a remedy for the aforementioned legal aberrations and
thus its provisions are considered essentially remedial and curative."

In light of the foregoing, and prescinding from the wording of the preamble, it is
unarguable that the legislative intent was precisely to give the statute retroactive operation.
"(A) retrospective operation is given to a statute or amendment where the intent that it should
so operate clearly appears from a consideration of the act as a whole, or from the terms
thereof."45It is obvious to the Court that the statute was meant to "reach back" to those
persons, events and transactions not otherwise covered by prevailing law and jurisprudence.
And inasmuch as it has been held that citizenship is a political and civil right equally as
important as the freedom of speech, liberty of abode, the right against unreasonable searches
and seizures and other guarantees enshrined in the Bill of Rights, therefore the legislative
intent to give retrospective operation to P.D. 725 must be given the fullest effect possible.
"(I)t has been said that a remedial statute must be so construed as to make it effect the
evident purpose for -which it was enacted, so that if the reason of the statute extends to past
transactions, as well as to those in the future, then it will be so applied although the statute
does not in terms so direct, unless to do so would impair some vested right or violate some
constitutional guaranty."46 This is all the more true of P.D. 725, which did not specify any
restrictions on or delimit or qualify the right of repatriation granted therein.
At this point, a valid question may be raised: How can the retroactivity of P.D. 725 benefit
Frivaldo considering that said law was enacted on June 5,1975, while Frivaldo lost his Filipino
citizenship much later, on January 20, 1983, and applied for repatriation even later, on August
17, 1994?
While it is true that the law was already in effect at the time that Frivaldo became an
American citizen, nevertheless, it is not only the law itself (P.D. 725) which is tobe given
retroactive effect, but even the repatriation granted under said law to Frivaldo on June 30,
1995 is to be deemed to have retroacted to the date of his application therefor, August 17,
1994. The reason for this is simply that if, as in this case, it was the intent of the legislative
authority that the law should apply to past events i.e., situations and transactions existing
even before the law came into being in order to benefit the greatest number of former Filipinos
possible thereby enabling them to enjoy and exercise the constitutionally guaranteed right of
citizenship, and such legislative intention is to be given the fullest effect and expression,
then there is all the more reason to have the law apply in a retroactive or retrospective
manner to situations, events and transactions subsequent to the passage of such law. That
is, the repatriation granted to Frivaldo on June 30, 1995 can and should be made to take
effect as of date of his application. As earlier mentioned, there is nothing in the law that would
bar this or would show a contrary intention on the part of the legislative authority; and there
is no showing that damage or prejudice to anyone, or anything unjust or injurious would result
from giving retroactivity to his repatriation. Neither has Lee shown that there will result the
impairment of any contractual obligation, disturbance of any vested right or breach of some
constitutional guaranty.
Being a former Filipino who has served the people repeatedly, Frivaldo deserves a liberal
interpretation of Philippine laws and whatever defects there were in his nationality should now
be deemed mooted by his repatriation.
Another argument for retroactivity to the date of filing is that it would prevent prejudice
to applicants. If P.D. 725 were not to be given retroactive effect, and the Special Committee
decides not to act, i.e., to delay the processing of applications for any substantial length of
time, then the former Filipinos who may be stateless, as Frivaldohaving already renounced
his American citizenship was, may be prejudiced for causes outside their control. This should
not be. In case of doubt in the interpretation or application of laws, it is to be presumed that
the law-making body intended right and justice to prevail.47
And as experience will show, the Special Committee was able to process, act upon and grant
applications for repatriation within relatively short spans of time after the same were
filed.48The fact that such interregna were relatively insignificant minimizes the likelihood of
prejudice to the government as a result of giving retroactivity to repatriation. Besides, to the
mind of the Court, direct prejudice to the government is possible only where a person's
repatriation has the effect of wiping out a liability of his to the government arising in
connection with or as a result of his being an alien, and accruing only during the interregnum
between application and approval, a situation that is not present in the instant case.
And it is but right and just that the mandate of the people, already twice frustrated,
should now prevail. Under the circumstances, there is nothing unjust or iniquitous in treating
Frivaldo's repatriation as having become effective as of the date of his application, i.e.,
on August 17, 1994. This being so, all questions about his possession of the nationality
qualification whether at the date of proclamation (June 30, 1995) or the date of election (May
8, 1995) or date of filing his certificate of candidacy (March 20, 1995) would become moot.

Based on the foregoing, any question regarding Frivaldo's status as a registered voter would
also be deemed settled. Inasmuch as he is considered as having been repatriatedi.e., his
Filipino citizenship restored as of August 17, 1994, his previous registration as a voter is
likewise deemed validated as of said date.

It is not disputed that on January 20, 1983 Frivaldo became an American. Would the
retroactivity of his repatriation not effectively give him dual citizenship, which under Sec. 40
of the Local Government Code would disqualify him "from running for any elective local
position?"49 We answer this question in the negative, as there is cogent reason to hold that
Frivaldo was really STATELESS at the time he took said oath of allegiance and even before
that, when he ran for governor in 1988. In his Comment, Frivaldo wrote that he "had long
renounced and had long abandoned his American citizenshiplong before May 8, 1995. At best,
Frivaldo was stateless in the interim when he abandoned and renounced his US citizenship
but before he was repatriated to his Filipino citizenship."50

On this point, we quote from the assailed Resolution dated December 19, 1995:51

"By the laws of the United States, petitioner Frivaldo lost his American citizenship when he
took his oath of allegiance to the Philippine Government when he ran for Governor in 1988,
in 1992, and in 1995. Every certificate of candidacy contains an oath of allegiance to the
Philippine Government."

These factual findings that Frivaldo has lost his foreign nationality long before the
elections of 1995 have not been effectively rebutted by Lee. Furthermore, it is basic that such
findings of the Commission are conclusive upon this Court, absent any showing of
capriciousness or arbitrariness or abuse.52

The Second Issue: Is Lack of Citizenship a Continuing Disqualification?

Lee contends that the May 1,1995 Resolution53 of the Comelec Second Division in SPA
No. 95-028 as affirmed in toto by Comelec En Banc in its Resolution of May 11, 1995 "became
final and executory after five (5) days or on May 17,1995, no restraining order having been
issued by this Honorable Court."54 Hence, before Lee "was proclaimed as the elected governor
on June 30, 1995, there was already a final and executory judgment disqualifying" Frivaldo.
Lee adds that this Court's two rulings (which Frivaldo now concedes were legally "correct")
declaring Frivaldo an alien have also become final and executory way before the 1995
elections, and these "judicial pronouncements of his political status as an American citizen
absolutely and for all time disqualified (him) from running for, and holding any public office
in the Philippines."
We do not agree.
It should be noted that our first ruling in G.R. No. 87193 disqualifying Frivaldo was
rendered in connection with the 1988 elections while that in G.R. No. 104654 was in
connection with the 1992 elections. That he was disqualified for such elections is final and can
no longer be changed. In the words of the respondent Commission (Second Division) in its
assailed Resolution:55

"The records show that the Honorable Supreme Court had decided that Frivaldo was not a
Filipino citizen and thus disqualified for the purpose of the 1988 and 1992 elections. However,
there is no record of any 'final judgment' of the disqualification of Frivaldo as a candidate for
the May 8, 1995 elections. What the Commission said in its Order of June 21, 1995
(implemented on June 30, 1995), directing the proclamation of Raul R. Lee, was that Frivaldo
was not a Filipino citizen 'having been declared by the Supreme Court in its Order dated March
25, 1995, not a citizen of the Philippines.' This declaration of the Supreme Court, however,
was in connection with the 1992 elections."

Indeed, decisions declaring the acquisition or denial of citizenship cannot govern a


person's future status with finality. This is because a person may subsequently reacquire, or
for that matter lose, his citizenship under any of the modes recognized by law for the purpose.
Hence, in Lee vs. Commissioner of Immigration, 56 we held:

"Everytime the citizenship of a person is material or indispensable in a judicial or


administrative case, whatever the corresponding court or administrative authority decides
therein as to such citizenship is generally not considered res judicata, hence it has to be
threshed out again and again, as the occasion demands."

The Third Issue: Comelec's Jurisdiction


Over The Petition in SPC No. 95-317
Lee also avers that respondent Comelec had no jurisdiction to entertain the petition in
SPC No. 95-317 because the only "possible types of proceedings that may be entertained by
the Comelec are a pre-proclamation case, an election protest or a quo warranto case." Again,
Lee reminds us that he was proclaimed on June 30, 1995 but that Frivaldo filed SPC No. 95-
317 questioning his (Lee's) proclamation only on July 6, 1995 "beyond the 5-day reglementary
period." Hence, according to him, Frivaldo's "recourse was to file either an election protest or
a quo warranto action."
This argument is not meritorious. The Constitution 57 has given the Comelec ample power
to "exercise exclusive original jurisdiction over all contests relating to the elections, returns
and qualifications of all elective x x x provincial x x x officials." Instead of dwelling at length
on the various petitions that Comelec, in the exercise of its constitutional prerogatives, may
entertain, suffice
it to say that this Court has invariably recognized the Commission's authority to hear and
decide petitions for annulment of proclamations of which SPC No. 95-317 obviously is
one.58Thus, in Mentang vs. COMELEC,59 we ruled:

"The petitioner argues that after proclamation and assumption of office, a pre-proclamation
controversy is no longer viable. Indeed, we are aware of cases holding that pre-proclamation
controversies may no longer be entertained by the COMELEC after the winning candidate has
been proclaimed, (citing Gallardo vs. Rimando, 187 SCRA 463; Salvacion vs. COMELEC, 170
SCRA 513; Casimiro vs. COMELEC, 171 SCRA 468.) This rule, however, is premised on an
assumption that the proclamation is no proclamation at all and the proclaimed candidate's
assumption of office cannot deprive the COMELEC of the power to make such declaration of
nullity. (citing Aguam vs. COMELEC, 23 SCRA 883; Agbayani vs. COMELEC, 186 SCRA 484.)"

The Court however cautioned that such power to annul a proclamation must "be done
within ten (10) days following the proclamation." Inasmuch as Frivaldo's petition was filed
only six (6) days after Lee's proclamation, there is no question that the Comelec correctly
acquired jurisdiction over the same.

The Fourth Issue: Was Lee's Proclamation Valid


Frivaldo assails the validity of the Lee proclamation. We uphold him for the following
reasons:

First. To paraphrase this Court in Labo vs. COMELEC,60 "the fact remains that he (Lee) was
not the choice of the sovereign will," and in Aquino vs. COMELEC,61 Lee is "a second placer,
xxx just that, a second placer."

In spite of this, Lee anchors his claim to the governorship on the pronouncement of this
Court in the aforesaid Labo62 case, as follows:

"The rule would have been different if the electorate fully aware in fact and in law of a
candidate's disqualification so as to bring such awareness within the realm of notoriety, would
nonetheless cast their votes in favor of the ineligible candidate. In such case, the electorate
may be said to have waived the validity and efficacy of their votes by notoriously misapplying
their franchise or throwing away their votes, in which case, the eligible candidate obtaining
the next higher number of votes may be deemed elected."

But such holding is qualified by the next paragraph, thus:

"But this is not the situation obtaining in the instant dispute. It has not been shown, and none
was alleged, that petitioner Labo was notoriously known as an ineligible candidate, much less
the electorate as having known of such fact. On the contrary, petitioner Labo was even
allowed by no less than the Comelec itself in its resolution dated May 10, 1992 to be voted
for the office of the city mayor as its resolution dated May 9,1992 denying due course to
petitioner Labo's certificate of candidacy had not yet become final and subject to the final
outcome of this case."

The last-quoted paragraph in Labo, unfortunately for Lee, is the ruling appropriate in this
case because Frivaldo was in 1995 in an identical situation as Labo was in 1992 when the
Comelec's cancellation of his certificate of candidacy was not yet final on election day as there
was in both cases a pending motion for reconsideration, for which reason Comelec issued an
(omnibus) resolution declaring that Frivaldo (like Labo in 1992) and several others can still
be voted for in the May 8, 1995 election, as in fact, he was.
Furthermore, there has been no sufficient evidence presented to show that the electorate
of Sorsogon was "fully aware in fact and in law" of Frivaldo's alleged disqualification as to
"bring such awareness within the realm of notoriety", in other words, that the voters
intentionally wasted their ballots knowing that, in spite of their voting for him, he was
ineligible. If Labo has any relevance at all, it is that the vice-governor and not Leeshould be
proclaimed, since in losing the election, Lee was, to paraphrase Labo again, "obviously not
the choice of the people" of Sorsogon. This is the emphatic teaching of Labo:

"The rule, therefore, is: the ineligibility of a candidate receiving majority votes does not entitle
the eligible candidate receiving the next highest number of votes to be declared elected. A
minority or defeated candidate cannot be deemed elected to the office."

Second. As we have earlier declared Frivaldo to have seasonably re-acquired his


citizenship and inasmuch as he obtained the highest number of votes in the 1995 elections,
henot Lee should be proclaimed. Hence, Lee's proclamation was patently erroneous and
should now be corrected.
The Fifth Issue: Is Section 78 of the Election Code Mandatory?

In G.R. No. 120295, Frivaldo claims that the assailed Resolution of the Comelec (Second
Division) dated May 1, 1995 and the confirmatory en banc Resolution of May 11, 1995
disqualifying him for want of citizenship should be annulled because they were rendered
beyond the fifteen (15) day period prescribed by Section 78 of the Omnibus Election Code
which reads as follows:

"Section 78. Petition to deny due course or to cancel a certificate of candidacy. A verified
petition seeking to deny due course or to cancel a certificate of candidacy may be filed by any
person exclusively on the ground that any material representation contained therein as
required under Section 74 hereof is false. The petition may be filed at any time not later than
twenty-five days from the time of the filing of the certificate of candidacy and shall be decided
after notice and hearing, not later than fifteen days before the election" (italics supplied.)

This claim is now moot and academic inasmuch as these resolutions are deemed
superseded by the subsequent ones issued by the Commission (First Division) on December
19, 1995, affirmed en banc63 on February 23, 1996, which both upheld his election. At any
rate, it is obvious that Section 78 is merely directory as Section 6 of R.A. No. 6646 authorizes
the Commission to try and decide petitions for disqualifications even after the elections, thus:

"SEC. 6. Effect of Disqualification Case. Any candidate who has been declared by final
judgment to be disqualified shall not be voted for, and the votes cast for him shall not be
counted. If for any reason a candidate is not declared by final judgment before an election to
be disqualified and he is voted for and receives the -winning number of votes in such election,
the Court or Commission shall continue with the trial and hearing of the action, inquiry or
protest and, upon motion of the complainant or any intervenor, may during the pendency
thereof order the suspension of the proclamation of such candidate whenever the evidence of
his guilt is strong." (Italics supplied)

Refutation of Mr. Justice Davide's Dissent

In his dissenting opinion, the esteemed Mr. Justice Hilario G. Davide, Jr. argues that
President Aquino's memorandum dated March 27, 1987 should be viewed as a suspension
(not a repeal, as urged by Lee) of P.D. 725. But whether it decrees a suspension or a repeal
is a purely academic distinction because the said issuance is not a statute that can amend or
abrogate an existing law. The existence and subsistence of P.D. 725 were recognized in the
first Frivaldo case;64 viz, "(u)nder CA No. 63 as amended by CA No. 473 and P.D. No. 725,
Philippine citizenship maybe reacquired by xxx repatriation" He also contends that by allowing
Frivaldo to register and to remain as a registered voter, the Comelec and in effect this Court
abetted a "mockery" of our two previous judgments declaring him a non-citizen. We do not
see such abetting or mockery. The retroactivity of his repatriation, as discussed earlier, legally
cured whatever defects there may have been in his registration as a voter for the purpose of
the 1995 elections. Such retroactivity did not change his disqualifications in 1988 and 1992,
which were the subjects of such previous rulings.
Mr. Justice Davide also believes that Quo Warranto is not the sole remedy to question the
ineligibility of a candidate, citing the Comelec's authority under Section 78 of the Omnibus
Election Code allowing the denial of a certificate of candidacy on the ground of a false material
representation therein as required by Section 74. Citing Loong, he then states his
disagreement with our holding that Section 78 is merely directory. We really have no quarrel.
Our point is that Frivaldo was in error in his claim in G.R. No. 120295 that the Comelec
Resolutions promulgated on May 1, 1995 and May 11, 1995 were invalid because they were
issued "not later than fifteen days before the election" as prescribed by Section 78. In
dismissing the petition in G.R. No. 120295, we hold that the Comelec did not commit grave
abuse of discretion because "Section 6 of R. A. 6646 authorizes the Comelec to try and decide
disqualifications even after the elections." In spite of his disagreement with us on this
point, i.e., that Section 78 "is merely directory," we note that just like us, Mr. Justice Davide
nonetheless votes to "DISMISS G.R. No. 120295." One other point. Loong, as quoted in the
dissent, teaches that a petition to deny due course under Section 78 must be filed within
the 25-day period prescribed therein. The present case however deals with the period during
which the Comelec may decide such petition. And we hold that it may be decided even after
the fifteen day period mentioned in Section 78. Here, we rule that a decision promulgated
by the Comelec even after the elections is valid but Loong held that a petition filed beyond the
25-day period is out of time. There is no inconsistency nor conflict.
Mr. Justice Davide also disagrees with the Court's holding that, given the unique factual
circumstances of Frivaldo, repatriation may be given retroactive effect. He argues that such
retroactivity "dilutes" our holding in the first Frivaldo case. But the first (and even the second
Frivaldo) decision did not directly involve repatriation as a mode of acquiring citizenship. If
we may repeat, there is no question that Frivaldo was not a Filipino for purposes of
determining his qualifications in the 1988 and 1992 elections. That is settled. But his
supervening repatriation has changed his political status not in 1988 or 1992, but only in the
1995 elections.
Our learned colleague also disputes our holding that Frivaldo was stateless prior to his
repatriation, saying that "informal renunciation or abandonment is not a ground to lose
American citizenship." Since our courts are charged only with the duty of the determining who
are Philippine nationals, we cannot rule on the legal question of who are or who are not
Americans. It is basic in international law that a State determines ONLY those who are its own
citizens not who are the citizens of other countries. 65 The issue here is: the Comelec made a
finding of fact that Frivaldo was stateless and such finding has not been shown by Lee to be
arbitrary or whimsical. Thus, following settled case law, such finding is binding and final.
The dissenting opinion also submits that Lee who lost by chasmic margins to Frivaldo in
all three previous elections, should be declared winner because "Frivaldo's ineligibility for
being an American was publicly known." First, there is absolutely no empirical evidence for
such "public" knowledge. Second, even if there is, such knowledge can be true post facto only
of the last two previous elections. Third, even the Comelec and now this Court were/are still
deliberating on his nationality before, during and after the 1995 elections. How then can there
be such "public" knowledge?
Mr. Justice Davide submits that Section 39 of the Local Government Code refers to the
qualifications of elective local officials, i.e., candidates, and not elected officials, and that the
citizenship qualification [under par. (a) of that section] must be possessed by candidates, not
merely at the commencement of the term, but by election day at the latest. We see it
differently. Section 39, par. (a) thereof speaks of "elective local official" while par. (b) to (f)
refer to "candidates." If the qualifications under par. (a) were intended to apply to
"candidates" and not elected officials, the legislature would have said so, instead of
differentiating par. (a) from the rest of the paragraphs. Secondly, if Congress had meant that
the citizenship qualification should be possessed at election day or prior thereto, it would have
specifically stated such detail, the same way it did in pars. (b) to (f) for other qualifications
of candidates for governor, mayor, etc.
Mr. Justice Davide also questions the giving of retroactive effect to Frivaldo's repatriation
on the ground, among others, that the law specifically provides that it is only after taking the
oath of allegiance that applicants shall be deemed to have reacquired Philippine citizenship.
We do not question what the provision states. We hold however that the provision should be
understood thus: that after taking the oath of allegiance the applicant is deemed to have
reacquired Philippine citizenship, which reacquisition (or repatriation) is deemed for all
purposes and intents to have retroacted to the date of his application therefor.
In any event, our "so too" argument regarding the literal meaning of the word "elective"
in reference to Section 39 of the Local Government Code, as well as regarding Mr. Justice
Davide's thesis that the very wordings of P.D. 725 suggest non-retroactivity, were already
taken up rather extensively earlier in this Decision.
Mr. Justice Davide caps his paper with a clarion call: "This Court must be the first to
uphold the Rule of Law." We agree we must all follow the rule of law. But that is NOT the
issue here. The issue is how should the law be interpreted and applied in this case so it can
be followed, so it can rule!
At balance, the question really boils down to a choice of philosophy and perception of how
to interpret and apply laws relating to elections: literal or liberal; the letter or the spirit; the
naked provision or its ultimate purpose; legal syllogism or substantial justice; in isolation or
in the context of social conditions; harshly against or gently in favor of the voters' obvious
choice. In applying election laws, it would be far better to err in favor of popular sovereignty
than to be right in complex but little understood legalisms. Indeed, to inflict a thrice rejected
candidate upon the electorate of Sorsogon would constitute unmitigated judicial tyranny and
an unacceptable assault upon this Court's conscience.

EPILOGUE

In sum, we rule that the citizenship requirement in the Local Government Code is to be
possessed by an elective official at the latest as of the time he is proclaimed and at the start
of the term of office to which he has been elected. We further hold P.D. No. 725 to be in full
force and effect up to the present, not having been suspended or repealed expressly nor
impliedly at any time, and Frivaldo's repatriation by virtue thereof to have been properly
granted and thus valid and effective. Moreover, by reason of the remedial or curative nature
of the law granting him a new right to resume his political status and the legislative intent
behind it, as well as his unique situation of having been forced to give up his citizenship and
political aspiration as his means of escaping a regime he abhorred, his repatriation is to be
given retroactive effect as of the date of his application therefor, during the pendency of which
he was stateless, he having given ' up his U. S. nationality. Thus, in contemplation of law, he
possessed the vital requirement of Filipino citizenship as of the start of the term of office of
governor, and should have been proclaimed instead of Lee. Furthermore, since his
reacquisition of citizenship retroacted to August 17, 1994, his registration as a voter of
Sorsogon is deemed to have been validated as of said date as well. The foregoing, of course,
are precisely consistent with our holding that lack of the citizenship requirement is not a
continuing disability or disqualification to run for and hold public office. And once again, we
emphasize herein our previous rulings recognizing the Comelec's authority and jurisdiction to
hear and decide petitions for annulment of proclamations.
This Court has time and again liberally and equitably construed the electoral laws of our
country to give fullest effect to the manifest will of our people,66 for in case of doubt, political
laws must be
interpreted to give life and spirit to the popular mandate freely expressed through the
ballot. Otherwise stated, legal niceties and technicalities cannot stand in the way of the
sovereign will. Consistently, we have held:

"x x x (L)aws governing election contests must be liberally construed to the end that the will
of the people in the choice of public officials may not be defeated by mere technical objections
(citations omitted)."67

The law and the courts must accord Frivaldo every possible protection, defense and
refuge, in deference to the popular will. Indeed, this Court has repeatedly stressed the
importance of giving effect to the sovereign will in order to ensure the survival of our
democracy. In any action involving the possibility of a reversal of the popular electoral choice,
this Court must exert utmost effort to resolve the issues in a manner that would give effect
to the will of the majority, for it is merely sound public policy to cause elective offices to be
filled by those who are the choice of the majority. To successfully challenge a winning
candidate's qualifications, the petitioner must clearly demonstrate that the ineligibility is so
patently antagonistic68 to constitutional and legal principles that overriding such ineligibility
and thereby giving effect to the apparent will of the people, would ultimately create greater
prejudice to the very democratic institutions and juristic traditions that our Constitution and
laws so zealously protect and promote. In this undertaking, Lee has miserably failed.
In Frivaldo's case, it would have been technically easy to find fault with his cause. The
Court could have refused to grant retroactivity to the effects of his repatriation and hold him
still ineligible due to his failure to show his citizenship at the time he registered as a voter
before the 1995 elections. Or, it could have disputed the factual findings of the Comelec that
he was stateless at the time of repatriation and thus hold his consequent dual citizenship as
a disqualification "from running for any elective local position." But the real essence of justice
does not emanate from quibblings over patchwork legal technicality. It proceeds from the
spirit's gut consciousness of the dynamic role of law as a brick in the ultimate development
of the social edifice. Thus, the Court struggled against and eschewed the easy, legalistic,
technical and sometimes harsh anachronisms of the law in order to evoke substantial justice
in the larger social context consistent with Frivaldo's unique situation approximating
venerability in Philippine political life. Concededly, he sought American citizenship only to
escape the clutches of the dictatorship. At this stage, we cannot seriously entertain any doubt
about his loyalty and dedication to this country. At the first opportunity, he returned to this
land, and sought to serve his people once more. The people of Sorsogon overwhelmingly
voted for him three times. He took an oath of allegiance to this Republic every time he filed
his certificate of candidacy and during his failed naturalization bid. And let it not be
overlooked, his demonstrated tenacity and sheer determination to re-assume his nationality
of birth despite several legal set-backs speak more loudly, in spirit, in fact and in truth than
any legal technicality, of his consuming intention and burning desire to re-embrace his native
Philippines even now at the ripe old age of 81 years. Such loyalty to and love of country as
well as nobility of purpose cannot be lost on this Court of justice and equity. Mortals of lesser
mettle would have given up. After all, Frivaldo was assured of a life of ease and plenty as a
citizen of the most powerful country in the world. But he opted, nay, single-mindedly insisted
on returning to and serving once more his struggling but beloved land of birth. He therefore
deserves every liberal interpretation of the law which can be applied in his favor. And in the
final analysis, over and above Frivaldo himself, the indomitable people of Sorsogon most
certainly deserve to be governed by a leader of their overwhelming choice.
WHEREFORE, in consideration of the foregoing:
(1) The petition in G.R. No. 123755 is hereby DISMISSED. The assailed Resolutions of
the respondent Commission are AFFIRMED.
(2) The petition in G.R. No. 120295 is also DISMISSED for being moot and academic. In
any event, it has no merit.
No costs.
SO ORDERED.
G.R. No. 104654 June 6, 1994

REPUBLIC OF THE PHILIPPINES, petitioner,


vs.
HON. ROSALIO G. DE LA ROSA, PRESIDING JUDGE OF THE REGIONAL TRIAL COURT,
BRANCH 28, MANILA and JUAN G. FRIVALDO, respondents.

G.R. No. 105715 June 6, 1994

RAUL R. LEE, petitioner,


vs.
COMMISSION ON ELECTIONS and JUAN G. FRIVALDO, respondents.

G.R. No. 105735 June 6, 1994

RAUL R. LEE, petitioner,


vs.
COMMISSION ON ELECTIONS and JUAN G. FRIVALDO, respondents.

The Solicitor General for petitioner in G.R. No. 104654.

Yolando F. Lim counsel for private respondent.

QUIASON, J.:

In Frivaldo v. Commission on Elections, 174 SCRA 245 (1989), this Court declared private
respondent, Juan G. Frivaldo, an alien and therefore disqualified from serving as Governor of
the Province of Sorsogon.

Once more, the citizenship of private respondent is put in issue in


these petitions docketed as G.R. No.104654 and G.R. No. 105715 and G.R. No. 105735. The
petitions were consolidated since they principally involve the same issues and parties.

G.R. No. 104654

This is a petition for certiorari under Rule 45 of the Revised Rules of Court in relation to R.A.
No. 5440 and Section 25 of the Interim Rules, filed by the Republic of the Philippines: (1) to
annul the Decision dated February 27, 1992 of the Regional Trial Court, Branch 28, Manila, in
SP Proc. No. 91-58645, which re-admitted private respondent as a Filipino citizen under the
Revised Naturalization Law (C.A. No. 63 as amended by C.A. No. 473); and (2) to nullify the
oath of allegiance taken by private respondent on February 27, 1992.

On September 20, 1991, petitioner filed a petition for naturalization captioned: "In the Matter
of Petition of Juan G. Frivaldo to be Re-admitted as a Citizen of the Philippines under
Commonwealth Act No. 63" (Rollo, pp. 17-23).
In an Order dated October 7, 1991 respondent Judge set the petition for hearing on March
16, 1992, and directed the publication of the said order and petition in the Official Gazette
and a newspaper of general circulation, for three consecutive weeks, the last publication of
which should be at least six months before the said date of hearing. The order further required
the posting of a copy thereof and the petition in a conspicuous place in the Office of the Clerk
of Court of the Regional Trial Court, Manila (Rollo, pp. 24-26).

On January 14, 1992, private respondent filed a "Motion to Set Hearing Ahead of Schedule,"
where he manifested his intention to run for public office in the May 1992 elections. He alleged
that the deadline for filing the certificate of candidacy was March 15, one day before the
scheduled hearing. He asked that the hearing set on March 16 be cancelled and be moved to
January 24 (Rollo, pp. 27-28).

The motion was granted in an Order dated January 24, 1992, wherein the hearing of the
petition was moved to February 21, 1992. The said order was not published nor a copy thereof
posted.

On February 21, the hearing proceeded with private respondent as the sole witness. He
submitted the following documentary evidence: (1) Affidavit of Publication of the Order dated
October 7, 1991 issued by the publisher of The Philippine Star (Exh. "A"); (2) Certificate of
Publication of the order issued
by the National Printing Office (Exh. "B"); (3) Notice of Hearing of Petition (Exh. "B-1"); (4)
Photocopy of a Citation issued by the National Press Club with private respondent’s picture
(Exhs. "C" and "C-2"); (5) Certificate of Appreciation issued by the Rotary Club of Davao (Exh.
"D"); (6) Photocopy
of a Plaque of Appreciation issued by the Republican College, Quezon City (Exh. "E"); (7)
Photocopy of a Plaque of Appreciation issued by the Davao-Bicol Association (Exh. "F"); (8)
Certification issued by the Records Management and Archives Office that the record of birth
of private respondent was not on file (Exh. "G"); and (8) Certificate of Naturalization issued
by the United States District Court (Exh. "H").

Six days later, on February 27, respondent Judge rendered the assailed Decision, disposing
as follows:

WHEREFORE, the petition is GRANTED. Petitioner JUAN G. FRIVALDO, is re-


admitted as a citizen of the Republic of the Philippines by naturalization,
thereby vesting upon him, all the rights and privileges of a natural born Filipino
citizen (Rollo, p. 33).

On the same day, private respondent was allowed to take his oath of allegiance before
respondent Judge (Rollo, p. 34).

On March 16, a "Motion for Leave of Court to Intervene and to Admit Motion for
Reconsideration" was filed by Quiterio H. Hermo. He alleged that the proceedings were tainted
with jurisdictional defects, and prayed for a new trial to conform with the requirements of the
Naturalization Law.

After receiving a copy of the Decision on March 18, 1992, the Solicitor General interposed a
timely appeal directly with the Supreme Court.

G.R. No. 105715


This is a petition for certiorari, mandamus with injunction under Rule 65 of the Revised Rules
of Court in relation to Section 5(2) of Article VIII of the Constitution with prayer for temporary
restraining order filed by Raul R. Lee against the Commission on Elections (COMELEC) and
private respondent, to annul the en banc Resolution of the COMELEC, which dismissed his
petition docketed as SPC Case No. 92-273. The said petition sought to annul the proclamation
of private respondent as Governor-elect of the Province of Sorsogon.

Petitioner was the official candidate of the Laban ng Demokratikong Pilipino (LDP) for the
position of governor of the Province of Sorsogon in the May 1992 elections. Private respondent
was the official candidate of the Lakas-National Union of Christian Democrats (Lakas-NUCD)
for the same position.

Private respondent was proclaimed winner on May 22, 1992.

On June 1, petitioner filed a petition with the COMELEC to annul the proclamation of private
respondent as Governor-elect of the Province of Sorsogon on the grounds: (1) that the
proceedings and composition of the Provincial Board of Canvassers were not in accordance
with law; (2) that private respondent is an alien, whose grant of Philippine citizenship is being
questioned by the State in G.R. No. 104654; and (3) that private respondent is not a duly
registered voter. Petitioner further prayed that the votes case in favor of private respondent
be considered as stray votes, and that he, on the basis of the remaining valid votes cast, be
proclaimed winner.

On June 10, the COMELEC issued the questioned en banc resolution which dismissed the
petition for having been filed out of time, citing Section 19 of R.A. No. 7166. Said section
provides that the period to appeal a ruling of the board of canvassers on questions affecting
its composition or proceedings was three days.

In this petition, petitioner argues that the COMELEC acted with grave abuse of discretion when
it ignored the fundamental issue of private respondent’s disqualification in the guise of
technicality.

Petitioner claims that the inclusion of private respondent’s name in the list of registered voters
in Sta. Magdalena, Sorsogon was invalid because at the time he registered as a voter in 1987,
he was as American citizen.

Petitioner further claims that the grant of Filipino citizenship to private respondent is not yet
conclusive because the case is still on appeal before us.

Petitioner prays for: (1) the annulment of private respondent’s proclamation as Governor of
the Province of Sorsogon; (2) the deletion of private respondent’s name from the list of
candidates for the position of governor; (3) the proclamation of the governor-elect based on
the remaining votes, after the exclusion of the votes for private respondent; (4) the issuance
of a temporary restraining order to enjoin private respondent from taking his oath and
assuming office; and (5) the issuance of a writ of mandamus to compel the COMELEC to
resolve the pending disqualification case docketed as SPA Case No. 92-016, against private
respondent.

G.R. No. 105735


This is a petition for mandamus under Rule 65 of the Revised Rules of Court in relation to
Section 5(2) of Article VIII of the Constitution, with prayer for temporary restraining order.
The parties herein are identical with the parties in G.R. No. 105715.

In substance, petitioner prays for the COMELEC’s immediate resolution of SPA Case No. 92-
016, which is a petition for the cancellation of private respondent’s certificate of candidacy
filed on March 23, 1992 by Quiterio H. Hermo, the intervenor in G.R. No. 104654 (Rollo, p.
18).

The petition for cancellation alleged: (1) that private respondent is an American citizen, and
therefore ineligible to run as candidate for the position of governor of the Province of
Sorsogon; (2) that the trial court’s decision
re-admitting private respondent as a Filipino citizen was fraught with legal infirmities
rendering it null and void; (3) that assuming the decision to be valid, private respondent’s
oath of allegiance, which was taken on the same day the questioned decision was
promulgated, violated Republic Act No. 530, which provides for a two-year waiting period
before the oath of allegiance can be taken by the applicant; and (4) that the hearing of the
petition on February 27, 1992, was held less than four months from the date of the last
publication of the order and petition. The petition prayed for the cancellation of private
respondent’s certificate of candidacy and the deletion of his name from the list of registered
voters in Sta. Magdalena, Sorsogon.

In his answer to the petition for cancellation, private respondent denied the allegations therein
and averred: (1) that Quiterio H. Hermo, not being a candidate for the same office for which
private respondent was aspiring, had no standing to file the petition; (2) that the decision re-
admitting him to Philippine citizenship was presumed to be valid; and (3) that no case had
been filed to exclude his name as a registered voter.

Raul R. Lee intervened in the petition for cancellation of private respondent’s certificate of
candidacy (Rollo, p. 37.).

On May 13, 1992, said intervenor urged the COMELEC to decide the petition for cancellation,
citing Section 78 of the Omnibus Election Code, which provides that all petitions on matters
involving the cancellation of a certificate of candidacy must be decided "not later than fifteen
days before election," and the case of Alonto v. Commission on Election, 22 SCRA 878 (1968),
which ruled that all pre-proclamation controversies should be summarily decided (Rollo,
p. 50).

The COMELEC concedes that private respondent has not yet reacquired his Filipino citizenship
because the decision granting him the same is not yet final and executory (Rollo, p. 63).
However, it submits that the issue of disqualification of a candidate is not among the grounds
allowed in a
pre-proclamation controversy, like SPC Case No. 92-273. Moreover, the said petition was filed
out of time.

The COMELEC contends that the preparation for the elections occupied much of its time, thus
its failure to immediately resolve SPA Case No. 92-016. It argues that under Section 5 of Rule
25 of the COMELEC Rules of Procedure, it is excused from deciding a disqualification case
within the period provided by law for reasons beyond its control. It also assumed that the
same action was subsequently abandoned by petitioner when he filed before it a petition
for quo warranto docketed as EPC No. 92-35. The quo warranto proceedings sought private
respondent’s disqualification because of his American citizenship.
II

G.R. No. 104654

We shall first resolve the issue concerning private respondent’s citizenship.

In his comment to the State’s appeal of the decision granting him Philippine citizenship in
G.R. No. 104654, private respondent alleges that the precarious political atmosphere in the
country during Martial Law compelled him to seek political asylum in the United States, and
eventually to renounce his Philippine citizenship.

He claims that his petition for naturalization was his only available remedy for his reacquisition
of Philippine citizenship. He tried to reacquire his Philippine citizenship through repatriation
and direct act of Congress. However, he was later informed that repatriation proceedings were
limited to army deserters or Filipino women who had lost their citizenship by reason of their
marriage to foreigners (Rollo, pp. 49-50). His request to Congress for sponsorship of a bill
allowing him to reacquire his Philippine citizenship failed to materialize, notwithstanding the
endorsement of several members of the House of Representatives in his favor (Rollo, p. 51).
He attributed this to the maneuvers of his political rivals.

He also claims that the re-scheduling of the hearing of the petition to an earlier date, without
publication, was made without objection from the Office of the Solicitor General. He makes
mention that on the date of the hearing, the court was jam-packed.

It is private respondent’s posture that there was substantial compliance with the law and that
the public was well-informed of his petition for naturalization due to the publicity given by the
media.

Anent the issue of the mandatory two-year waiting period prior to the taking of the oath of
allegiance, private respondent theorizes that the rationale of the law imposing the waiting
period is to grant the public an opportunity to investigate the background of the applicant and
to oppose the grant of Philippine citizenship if there is basis to do so. In his case, private
respondent alleges that such requirement may be dispensed with, claiming that his life, both
private and public, was well-known. Private respondent cites his achievement as a freedom
fighter and a former Governor of the Province of Sorsogon for six terms.

The appeal of the Solicitor General in behalf of the Republic of the Philippines is meritorious.
The naturalization proceedings in SP Proc. No. 91-58645 was full of procedural flaws,
rendering the decision an anomaly.

Private respondent, having opted to reacquire Philippine citizenship thru naturalization under
the Revised Naturalization Law, is duty bound to follow the procedure prescribed by the said
law. It is not for an applicant to decide for himself and to select the requirements which he
believes, even sincerely, are applicable to his case and discard those which be believes are
inconvenient or merely of nuisance value. The law does not distinguish between an applicant
who was formerly a Filipino citizen and one who was never such a citizen. It does not provide
a special procedure for the reacquisition of Philippine citizenship by former Filipino citizens
akin to the repatriation of a woman who had lost her Philippine citizenship by reason of her
marriage to an alien.
The trial court never acquired jurisdiction to hear the petition for naturalization of private
respondent. The proceedings conducted, the decision rendered and the oath of allegiance
taken therein, are null and void for failure to comply with the publication and posting
requirements under the Revised Naturalization Law.

Under Section 9 of the said law, both the petition for naturalization and the order setting it
for hearing must be published once a week for three consecutive weeks in the Official Gazette
and a newspaper of general circulation respondent cites his achievements as a freedom fighter
and a former Governor of the Province of Sorsogon for six terms.

The appeal of the Solicitor General in behalf of the Republic of


the Philippines is meritorious. The naturalization proceedings in SP Proc.
No. 91-58645 was full of procedural flaws, rendering the decision an anomaly.

Private respondent, having opted to reacquire Philippine citizenship thru naturalization under
the Revised Naturalization Law, is duty bound to follow the procedure prescribed by the said
law. It is not for an applicant to decide for himself and to select the requirements which he
believes, even sincerely, are applicable to his case and discard those which he believes are
inconvenient or merely of nuisance value. The law does not distinguish between an applicant
who was formerly a Filipino citizen and one who was never such a citizen. It does not provide
a special procedure for the reacquisition of Philippine citizenship by former Filipino citizens
akin to the repatriation of a woman who had lost her Philippine citizenship by reason of her
marriage to an alien.

The trial court never acquired jurisdiction to hear the petition for naturalization of private
respondent. The proceedings conducted, the decision rendered and the oath of allegiance
taken therein, are null and void for failure to comply with the publication and posting
requirements under the Revised Naturalization Law.

Under Section 9 of the said law, both the petition for naturalization and the order setting it
for hearing must be published once a week for three consecutive weeks in the Official Gazette
and a newspaper of general circulation. Compliance therewith is jurisdictional (Po Yi Bo v.
Republic, 205 SCRA 400 [1992]). Moreover, the publication and posting of the petition and
the order must be in its full test for the court to acquire jurisdiction (Sy v. Republic, 55 SCRA
724 [1974]).

The petition for naturalization lacks several allegations required by Sections 2 and 6 of the
Revised Naturalization Law, particularly: (1) that the petitioner is of good moral character;
(2) that he resided continuously in the Philippines for at least ten years; (3) that he is able to
speak and write English and any one of the principal dialects; (4) that he will reside
continuously in the Philippines from the date of the filing of the petition until his admission to
Philippine citizenship; and (5) that he has filed a declaration of intention or if he is excused
from said filing, the justification therefor.

The absence of such allegations is fatal to the petition (Po Yi Bi v. Republic, 205 SCRA 400
[1992]).

Likewise, the petition is not supported by the affidavit of at least two credible persons who
vouched for the good moral character of private respondent as required by Section 7 of the
Revised Naturalization Law. Private respondent also failed to attach a copy of his certificate
of arrival to the petition as required by Section 7 of the said law.
The proceedings of the trial court was marred by the following irregularities: (1) the hearing
of the petition was set ahead of the scheduled date of hearing, without a publication of the
order advancing the date of hearing, and the petition itself; (2) the petition was heard within
six months from the last publication of the petition; (3) petitioner was allowed to take his
oath of allegiance before the finality of the judgment; and (4) petitioner took his oath of
allegiance without observing the two-year waiting period.

A decision in a petition for naturalization becomes final only after 30 days from its
promulgation and, insofar as the Solicitor General is concerned, that period is counted from
the date of his receipt of the copy of the decision (Republic v. Court of First Instance of Albay,
60 SCRA 195 [1974]).

Section 1 of R.A. No. 530 provides that no decision granting citizenship in naturalization
proceedings shall be executory until after two years from its promulgation in order to be able
to observe if: (1) the applicant has left the country; (2) the applicant has dedicated himself
continuously to a lawful calling or profession; (3) the applicant has not been convicted of any
offense or violation of government promulgated rules; and (4) the applicant has committed
any act prejudicial to the interest of the country or contrary to government announced
policies.

Even discounting the provisions of R.A. No. 530, the courts cannot implement any decision
granting the petition for naturalization before its finality.

G.R. No. 105715

In view of the finding in G.R. No. 104654 that private respondent is not yet a Filipino citizen,
we have to grant the petition in G.R. No. 105715 after treating it as a petition
for certiorari instead of a petition for mandamus. Said petition assails the en banc resolution
of the COMELEC, dismissing SPC Case No. 92-273, which in turn is a petition to annul private
respondent’s proclamation on three grounds: 1) that the proceedings and composition of the
Provincial Board of Canvassers were not in accordance with law; 2) that private respondent
is an alien, whose grant of Filipino citizenship is being questioned by the State in G.R. No.
104654; and 3) that private respondent is not a duly registered voter. The COMELEC
dismissed the petition on the grounds that it was filed outside the three-day period for
questioning the proceedings
and composition of the Provincial Board of Canvassers under Section 19 of R.A. No. 7166.

The COMELEC failed to resolve the more serious issue — the disqualification of private
respondent to be proclaimed Governor on grounds of lack of Filipino citizenship. In this aspect,
the petition is one for quo warranto. In Frivaldo v. Commission on Elections, 174 SCRA 245
(1989), we held that a petition for quo warranto, questioning the respondent’s title and
seeking to prevent him from holding office as Governor for alienage, is not covered by the
ten-day period for appeal prescribed in Section 253 of the Omnibus Election Code.
Furthermore, we explained that "qualifications for public office are continuing requirements
and must be possessed not only at the time of appointment or election or assumption of office
but during the officer’s entire tenure; once any of the required qualification is lost, his title
may be seasonably challenged."

Petitioner’s argument, that to unseat him will frustrate the will of the electorate, is untenable.
Both the Local Government Code and the Constitution require that only Filipino citizens can
run and be elected to public office. We can only surmise that the electorate, at the time they
voted for private respondent, was of the mistaken belief that he had legally reacquired Filipino
citizenship.

Petitioner in G.R. No. 105715, prays that the votes cast in favor of private respondent be
considered stray and that he, being the candidate obtaining the second highest number of
votes, be declared winner. In Labo, Jr. v. COMELEC, 176 SCRA 1 (1989), we ruled that where
the candidate who obtained the highest number of votes is later declared to be disqualified to
hold the office to which he was elected, the candidate who garnered the second highest
number of votes is not entitled to be declared winner (See also Geronimo v. Ramos, 136 SCRA
435 [1985]; Topacio v. Paredes, 23 Phil. 238 [1912]).

G.R. No. 105735

In view of the discussions of G.R. No. 104654 and G.R. No. 105715, we find the petition in
G.R. No. 105735 moot and academic.

WHEREFORE, the petitions in G.R. No. 104654 and G.R. No. 105715 are both GRANTED while
the petition in G.R. No. 105735 is DISMISSED. Private respondent is declared NOT a citizen
of the Philippines and therefore DISQUALIFIED from continuing to serve as GOVERNOR of the
Province of Sorsogon. He is ordered to VACATE his office and to SURRENDER the same to the
Vice-Governor of the Province of Sorsogon once this decision becomes final and executory.
No pronouncement as to costs.

SO ORDERED.

G.R. No. 86564 August 1, 1989

RAMON L. LABO, JR., petitioner,


vs.
THE COMMISSION ON ELECTIONS (COMELEC) EN BANC AND LUIS L.
LARDIZABAL, respondents

Estelito P. Mendoza for petitioner.

Rillera and Quintana for private respondent.

CRUZ, J.:

The petitioner asks this Court to restrain the Commission on Elections from looking into the
question of his citizenship as a qualification for his office as Mayor of Baguio City. The
allegation that he is a foreigner, he says, is not the issue. The issue is whether or not the
public respondent has jurisdiction to conduct any inquiry into this matter, considering that
the petition for quo warranto against him was not filed on time.

It is noteworthy that this argument is based on the alleged tardiness not of the petition itself
but of the payment of the filing fee, which the petitioner contends was an indispensable
requirement. The fee is, curiously enough, all of P300.00 only. This brings to mind the popular
verse that for want of a horse the kingdom was lost. Still, if it is shown that the petition was
indeed filed beyond the reglementary period, there is no question that this petition must be
granted and the challenge abated.

The petitioner's position is simple. He was proclaimed mayor-elect of Baguio City, on January
20, 1988. The petition for quo warranto was filed by the private respondent on January 26,
1988, but no filing fee was paid on that date. This fee was finally paid on February 10, 1988,
or twenty-one days after his proclamation. As the petition by itself alone was ineffectual
without the filing fee, it should be deemed filed only when the fee was paid. This was done
beyond the reglementary period provided for under Section 253 of the Omnibus Election Code
reading as follows:

SEC. 253. Petition for quo warranto. — Any voter contesting the election of a
Member of the Batasang Pambansa, regional, provincial, or city officer on the
ground of ineligibility or of disloyalty to the Republic of the Philippines shall file
a sworn petition for quo warranto with the Commission within ten days after
the proclamation of the result of the election.

The petitioner adds that the payment of the filing fee is required under Rule 36, Section 5, of
the Procedural Rules of the COMELEC providing that —

Sec. 5. No petition for quo warranto shall be given due course without the
payment of a filing fee in the amount of Three Hundred Pesos (P300.00) and
the legal research fee as required by law.

and stresses that there is abundant jurisprudence holding that the payment of the filing fee
is essential to the timeliness of the filling of the petition itself. He cites many rulings of the
Court to this effect, specifically Manchester v. Court of Appeals. 1

For his part, the private respondent denies that the filing fee was paid out of time. In fact he
says, it was flied ahead of time. His point is that when he filed his "Petition for Quo Warranto
with Prayer for Immediate Annulment of Proclamation and Restraining Order or Injunction"
on January 26, 1988, the COMELEC treated it as a pre-proclamation controversy and docketed
it as SPC Case No. 88-288. No docket fee was collected although it was offered. It was only
on February 8, 1988, that the COMELEC decided to treat his petition as solely for quo
warranto and re-docketed it as EPC Case No. 88-19, serving him notice on February 10, 1988.
He immediately paid the filing fee on that date.

The private respondent argues further that during the period when the COMELEC regarded
his petition as a pre-proclamation controversy, the time for filing an election protest or quo
warranto proceeding was deemed suspended under Section 248 of the Omnibus Election
Code. 2 At any rate, he says, Rule 36, Section 5, of the COMELEC Rules of Procedure cited by
the petitioner, became effective only on November 15, 1988, seven days after publication of
the said Rules in the Official Gazette pursuant to Section 4, Rule 44 thereof. 3 These rules
could not retroact to January 26,1988, when he filed his petition with the COMELEC.

In his Reply, the petitioner argues that even if the Omnibus Election Code did not require it,
the payment of filing fees was still necessary under Res. No. 1996 and, before that, Res. No.
1450 of the respondent COMELEC, promulgated on January 12, 1988, and February 26, 1980,
respectively. To this, the private respondent counters that the latter resolution was intended
for the local elections held on January 30, 1980, and did not apply to the 1988 local elections,
which were supposed to be governed by the first-mentioned resolution. However, Res. No.
1996 took effect only on March 3, 1988, following the lapse of seven days after its publication
as required by RA No. 6646, otherwise known as the Electoral Reform Law of 1987, which
became effective on January 5, 1988. Its Section 30 provides in part:

Sec. 30. Effectivity of Regulations and Orders of the Commission. — The rules
and regulations promulgated by the Commission shall take effect on the
seventh day after their publication in the Official Gazette or in at least (2) daily
newspapers of general circulation in the Philippines.

The Court has considered the arguments of the parties and holds that the petition for quo
warranto was filed on time. We agree with the respondents that the fee was paid during the
ten-day period as extended by the pendency of the petition when it was treated by the
COMELEC as a pre-proclamation proceeding which did not require the payment of a filing fee.
At that, we reach this conclusion only on the assumption that the requirement for the payment
of the fees in quo warranto proceedings was already effective. There is no record that Res.
No. 1450 was even published; and as for Res. No. 1996, this took effect only on March 3,
1988, seven days after its publication in the February 25, 1988 issues of the Manila Chronicle
and the Philippine Daily Inquirer, or after the petition was filed.

The petitioner forgets Tañ;ada v. Tuvera 4 when he argues that the resolutions became
effective "immediately upon approval" simply because it was so provided therein. We held in
that case that publication was still necessary under the due process clause despite such
effectivity clause.

In any event, what is important is that the filing fee was paid, and whatever delay there may
have been is not imputable to the private respondent's fault or neglect. It is true that in
the Manchester Case, we required the timely payment of the filing fee as a precondition for
the timeliness of the filing of the case itself. In Sun Insurance Office, Ltd. v.
Asuncion, 5 however this Court, taking into account the special circumstances of that case,
declared:

This Court reiterates the rule that the trial court acquires jurisdiction over a
case only upon the payment of the prescribed filing fee. However, the court
may allow the payment of the said fee within a reasonable time. In the event
of non-compliance therewith, the case shall be dismissed.

The same idea is expressed in Rule 42, Section 18, of the COMELEC Rules of Procedure
adopted on June 20, 1988, thus:

Sec. 18. Non-payment of prescribed fees. — If the fees above prescribed are
not paid, the Commission may refuse to take action thereon until they are paid
and may dismiss the action or the proceeding. (Emphasis supplied.)

The Court notes that while arguing the technical point that the petition for quo
warranto should be dismissed for failure to pay the filing fee on time, the petitioner would at
the same time minimize his alleged lack of citizenship as "a futile technicality," It is
regrettable, to say the least, that the requirement of citizenship as a qualification for public
office can be so demeaned. What is worse is that it is regarded as an even less important
consideration than the reglementary period the petitioner insists upon.

This matter should normally end here as the sole issue originally raised by the petitioner is
the timeliness of the quo warranto proceedings against him. However, as his citizenship is the
subject of that proceeding, and considering the necessity for an early resolution of that more
important question clearly and urgently affecting the public interest, we shall directly address
it now in this same action.

The Court has similarly acted in a notable number of cases, thus:

From the foregoing brief statement of the nature of the instant case, it would
appear that our sole function in this proceeding should be to resolve the single
issue of whether or not the Court of Appeals erred in ruling that the motion for
new trial of the GSIS in question should indeed be deemed pro forma.But going
over the extended pleadings of both parties, the Court is immediately
impressed that substantial justice may not be timely achieved, if we should
decide this case upon such a technical ground alone. We have carefully read all
the allegations and arguments of the parties, very ably and comprehensively
expounded by evidently knowledgeable and unusually competent counsel, and
we feel we can better serve the interests of justice by broadening the scope of
our inquiry, for as the record before us stands, we see that there is enough
basis for us to end the basic controversy between the parties here and now,
dispensing, however, with procedural steps which would not anyway affect
substantially the merits of their respective claims. 6

xxx

While it is the fault of the petitioner for appealing to the wrong court and
thereby allowing the period for appeal to lapse, the more correct procedure was
for the respondent court to forward the case to the proper court which was the
Court of Appeals for appropriate action. Considering, however, the length of
time that this case has been pending, we apply the rule in the case of Del
Castillo v. Jaymalin, (112 SCRA 629) and follow the principle enunciated in
Alger Electric, Inc. v. Court of Appeals, (135 SCRA 37) which states:

... it is a cherished rule of procedure for this Court to always


strive to settle the entire controversy in a single proceeding
leaving no root or branch to bear the seeds of future litigation.
No useful purpose will be served if this case is remanded to the
trial court only to have its decision raised again to the
Intermediate Appellate Court and from there to this Court. (p.
43)

Only recently in the case of Beautifont, Inc., et al. v. Court of Appeals, et


al. (G.R. No. 50141, January 29, 1988), we stated that:

... But all those relevant facts are now before this Court. And those facts dictate
the rendition of a verdict in the petitioner's favor. There is therefore no point in
referring the case back to the Court of Appeals. The facts and the legal
propositions involved will not change, nor should the ultimate judgment.
Considerable time has already elapsed and, to serve the ends of justice, it is
time that the controversy is finally laid to rest. (See Sotto v. Samson, 5 SCRA
733; Republic v. Paredes, 108 Phil. 57; Lianga Lumber Co. v. Lianga Timber
Co., Inc., 76 SCRA 197; Erico v. Heirs of Chigas, 98 SCRA 575; Francisco v.
City of Davao, 12 SCRA 628; Valencia v. Mabilangan, 105 Phil.
162).lâwphî1.ñèt Sound practice seeks to accommodate the theory which
avoids waste of time, effort and expense, both to the parties and the
government, not to speak of delay in the disposal of the case (cf. Fernandez v.
Garcia, 92 Phil. 592, 597). A marked characteristic of our judicial set-up is that
where the dictates of justice so demand ... the Supreme Court should act, and
act with finality.' (Li Siu Liat v. Republic, 21 SCRA 1039, 1046, citing Samal v.
CA, 99 Phil. 230 and U.S. v. Gimenez, 34 Phil. 74). In this case, the dictates of
justice do demand that this Court act, and act with finality. 7

xxx

Remand of the case to the lower court for further reception of evidence is not
necessary where the court is in a position to resolve the dispute based on the
records before it. On many occasions, the Court, in the public interest and the
expeditious administration of justice, has resolved actions on the merits instead
of remanding them to the trial court for further proceedings, such as where the
ends of justice would not be subserved by the remand of the case or when
public interest demands an early disposition of the case or where the trial court
had already received all the evidence of the parties. 8

This course of action becomes all the more justified in the present case where, to repeat for
stress, it is claimed that a foreigner is holding a public office.

We also note in his Reply, the petitioner says:

In adopting private respondent's comment, respondent COMELEC implicitly


adopted as "its own" private respondent's repeated assertion that petitioner is
no longer a Filipino citizen. In so doing, has not respondent COMELEC
effectively disqualified itself, by reason of prejudgment, from resolving the
petition for quo warranto filed by private respondent still pending before it? 9

This is still another reason why the Court has seen fit to rule directly on the merits of this
case.

Going over the record, we find that there are two administrative decisions on the question of
the petitioner's citizenship. The first was rendered by the Commission on Elections on May
12, 1982, and found the petitioner to be a citizen of the Philippines. 10 The second was
rendered by the Commission on Immigration and Deportation on September 13, 1988, and
held that the petitioner was not a citizen of the Philippines. 11

The first decision was penned by then COMELEC Chigas, Vicente Santiago, Jr., with
Commissioners Pabalate Savellano and Opinion concurring in full and Commissioner Bacungan
concurring in the dismissal of the petition "without prejudice to the issue of the respondent's
citizenship being raised anew in a proper case." Commissioner Sagadraca reserved his vote,
while Commissioner Felipe was for deferring decision until representations shall have been
made with the Australian Embassy for official verification of the petitioner's alleged
naturalization as an Australian.

The second decision was unanimously rendered by Chairman Miriam Defensor-Santiago and
Commissioners Alano and Geraldez of the Commission on Immigration and Deportation. It is
important to observe that in the proceeding before the COMELEC, there was no direct proof
that the herein petitioner had been formally naturalized as a citizen of Australia. This
conjecture, which was eventually rejected, was merely inferred from the fact that he had
married an Australian citizen, obtained an Australian passport, and registered as an alien with
the CID upon his return to this country in 1980.

On the other hand, the decision of the CID took into account the official statement of the
Australian Government dated August 12, 1984, through its Consul in the Philippines, that the
petitioner was still an Australian citizen as of that date by reason of his naturalization in 1976.
That statement 12 is reproduced in full as follows:

I, GRAHAM COLIN WEST, Consul of Australia in the Philippines, by virtue of a certificate of


appointment signed and sealed by the Australian Minister of State for Foreign Affairs on 19
October 1983, and recognized as such by Letter of Patent signed and sealed by the Philippines
Acting Minister of Foreign Affairs on 23 November 1983, do hereby provide the following
statement in response to the subpoena Testificandum dated 9 April 1984 in regard to the
Petition for disqualification against RAMON LABO, JR. Y LOZANO (SPC No. 84-73), and do
hereby certify that the statement is true and correct.

STATEMENT

A) RAMON LABO, JR. Y LOZANO, date of birth 23 December 1934, was married
in the Philippines to an Australian citizen. As the spouse of an Australian citizen,
he was not required to meet normal requirements for the grant of citizenship
and was granted Australian citizenship by Sydney on 28 July 1976.

B) Any person over the age of 16 years who is granted Australian


citizenship must take an oath of allegiance or make an affirmation of allegiance.
The wording of the oath of affirmation is: "I ..., renouncing all other allegiance
..." etc. This need not necessarily have any effect on his former nationality as
this would depend on the citizenship laws of his former country.

C) The marriage was declared void in the Australian Federal Court in Sydney
on 27 June 1980 on the ground that the marriage had been bigamous.

D) According to our records LABO is still an Australian citizen.

E) Should he return to Australia, LABO may face court action in respect of


Section 50 of Australian Citizenship Act 1948 which relates to the giving of false
or misleading information of a material nature in respect of an application for
Australian citizenship. If such a prosecution was successful, he could be
deprived of Australian citizenship under Section 21 of the Act.

F) There are two further ways in which LABO could divest himself of Australian
citizenship:

(i) He could make a declaration of Renunciation of Australian citizenship under


Section 18 of the Australian Citizenship Act, or

(ii) If he acquired another nationality, (for example, Filipino) by a formal and


voluntary act other than marriage, then he would automatically lose as
Australian citizenship under Section 17 of the Act.
IN WITNESS WHEREOF, I HAVE HEREUNTO SET MAY HAND AND SEAL OF THE
AUSTRALIAN EMBASSY, MANILA, THIS 12th DAY OF APRIL 1984. DONE AT
MANILA IN THE PHILIPPINES.

(Signed) GRAHAM C. WEST Consul

This was affirmed later by the letter of February 1, 1988, addressed to the
private respondent by the Department of Foreign Affairs reading as follows: 13

Sir:

With reference to your letter dated 1 February 1988, I wish to inform you that
inquiry made with the Australian Government through the Embassy of the
Philippines in Canberra has elicited the following information:

1) That Mr. Ramon L. Labo, Jr. acquired Australian citizenship on 28 July 1976.

2) That prior to 17 July 1986, a candidate for Australian citizenship had to either
swear an oath of allegiance or make an affirmation of allegiance which carries
a renunciation of "all other allegiance.

Very truly yours, For the Secretary of Foreign Affairs: (SGD) RODOLFO
SEVERINO, JR. Assistant Secretary

The decision also noted the oath of allegiance taken by every naturalized Australian reading
as follows:

OATH OF ALLEGIANCE

I, A.B., renouncing all other allegiance, swear by Almighty God that I will be
faithful and bear true allegiance to Her Majesty Elizabeth the Second, Queen of
Australia, Her heirs and successors according to law, and that I will faithfully
observe the laws of Australia and fulfill my duties as an Australian citizen. 14

and the Affirmation of Allegiance, which declares:

AFFIRMATION OF ALLEGIANCE

I, A.B., renouncing all other allegiance, solemnly and sincerely promise and
declare that I will be faithful and bear true allegiance to Her Majesty Elizabeth
the Second, Queen of Australia, Her heirs and successors according to law, and
that I will faithfully observe the Laws of Australia and fulfill my duties as an
Australian citizen. 15

The petitioner does not question the authenticity of the above evidence. Neither does he deny
that he obtained Australian Passport No. 754705, which he used in coming back to the
Philippines in 1980, when he declared before the immigration authorities that he was an alien
and registered as such under Alien Certificate of Registration No. B-323985. 16 He later asked
for the change of his status from immigrant to a returning former Philippine citizen and was
granted Immigrant Certificate of Residence No. 223809. 17 He also categorically declared that
he was a citizen of Australia in a number of sworn statements voluntarily made by him and.
even sought to avoid the jurisdiction of the barangay court on the ground that he was a
foreigner. 18

The decision of the COMELEC in 1982 quaintly dismisses all these acts as "mistakes" that did
not divest the petitioner of his citizenship, although, as earlier noted, not all the members
joined in this finding. We reject this ruling as totally baseless. The petitioner is not an
unlettered person who was not aware of the consequences of his acts, let alone the fact that
he was assisted by counsel when he performed these acts.

The private respondent questions the motives of the COMELEC at that time and stresses
Labo's political affiliation with the party in power then, but we need not go into that now.

There is also the claim that the decision can no longer be reversed because of the doctrine
of res judicata, but this too must be dismissed. This doctrine does not apply to questions of
citizenship, as the Court has ruled in several cases. 19 Moreover, it does not appear that it
was properly and seasonably pleaded, in a motion to dismiss or in the answer, having been
invoked only when the petitioner filed his reply 20 to the private respondent's comment.
Besides, one of the requisites of res judicata, to wit, identity of parties, is not present in this
case.

The petitioner's contention that his marriage to an Australian national in 1976 did not
automatically divest him of Philippine citizenship is irrelevant. There is no claim or finding that
he automatically ceased to be a Filipino because of that marriage. He became a citizen of
Australia because he was naturalized as such through a formal and positive process, simplified
in his case because he was married to an Australian citizen. As a condition for such
naturalization, he formally took the Oath of Allegiance and/or made the Affirmation of
Allegiance, both quoted above. Renouncing all other allegiance, he swore "to be faithful and
bear true allegiance to Her Majesty Elizabeth the Second, Queen of Australia ..." and to fulfill
his duties "as an Australian citizen."

The petitioner now claims that his naturalization in Australia made him at worst only a dual
national and did not divest him of his Philippine citizenship. Such a specious argument cannot
stand against the clear provisions of CA No. 63, which enumerates the modes by which
Philippine citizenship may be lost. Among these are: (1) naturalization in a foreign country;
(2) express renunciation of citizenship; and (3) subscribing to an oath of allegiance to support
the Constitution or laws of a foreign country, all of which are applicable to the petitioner. It is
also worth mentioning in this connection that under Article IV, Section 5, of the present
Constitution, "Dual allegiance of citizens is inimical to the national interest and shall be dealt
with by law."

Even if it be assumed that, as the petitioner asserts, his naturalization in Australia was
annulled after it was found that his marriage to the Australian citizen was bigamous, that
circumstance alone did not automatically restore his Philippine citizenship. His divestiture of
Australian citizenship does not concern us here. That is a matter between him and his adopted
country. What we must consider is the fact that he voluntarily and freely rejected Philippine
citizenship and willingly and knowingly embraced the citizenship of a foreign country. The
possibility that he may have been subsequently rejected by Australia, as he claims, does not
mean that he has been automatically reinstated as a citizen of the Philippines.

Under CA No. 63 as amended by PD No. 725, Philippine citizenship may be reacquired by


direct act of Congress, by naturalization, or by repatriation. It does not appear in the record,
nor does the petitioner claim, that he has reacquired Philippine citizenship by any of these
methods. He does not point to any judicial decree of naturalization as to any statute directly
conferring Philippine citizenship upon him. Neither has he shown that he has complied with
PD No. 725, providing that:

... (2) natural-born Filipinos who have lost their Philippine citizenship may
reacquire Philippine citizenship through repatriation by applying with the
Special Committee on Naturalization created by Letter of Instruction No. 270,
and, if their applications are approved, taking the necessary oath of allegiance
to the Republic of the Philippines, after which they shall be deemed to have
reacquired Philippine citizenship. The Commission on Immigration and
Deportation shall thereupon cancel their certificate of registration. (Emphasis
supplied.)

That is why the Commission on Immigration and Deportation rejected his application for the
cancellation of his alien certificate of registration. And that is also the reason we must deny
his present claim for recognition as a citizen of the Philippines.

The petitioner is not now, nor was he on the day of the local elections on January 18, 1988,
a citizen of the Philippines. In fact, he was not even a qualified voter under the Constitution
itself because of his alienage. 21 He was therefore ineligible as a candidate for mayor of Baguio
City, under Section 42 of the Local Government Code providing in material part as follows:

Sec. 42. Qualifications. — An elective local official must be a citizen of the


Philippines, at least twenty-three years of age on election day, a qualified voter
registered as such in the barangay, municipality, city or province where he
proposes to be elected, a resident therein for at least one year at the time of
the filing of his certificate of candidacy, and able to read and write English,
Filipino, or any other local language or dialect.

The petitioner argues that his alleged lack of citizenship is a "futile technicality" that should
not frustrate the will of the electorate of Baguio City, who elected him by a "resonant and
thunderous majority." To be accurate, it was not as loud as all that, for his lead over the
second-placer was only about 2,100 votes. In any event, the people of that locality could not
have, even unanimously, changed the requirements of the Local Government Code and the
Constitution. The electorate had no power to permit a foreigner owing his total allegiance to
the Queen of Australia, or at least a stateless individual owing no allegiance to the Republic
of the Philippines, to preside over them as mayor of their city. Only citizens of the Philippines
have that privilege over their countrymen.

The probability that many of those who voted for the petitioner may have done so in the belief
that he was qualified only strengthens the conclusion that the results of the election cannot
nullify the qualifications for the office now held by him. These qualifications are continuing
requirements; once any of them is lost during incumbency, title to the office itself is deemed
forfeited. In the case at bar, the citizenship and voting requirements were not subsequently
lost but were not possessed at all in the first place on the day of the election. The petitioner
was disqualified from running as mayor and, although elected, is not now qualified to serve
as such.

Finally, there is the question of whether or not the private respondent, who filed the quo
warranto petition, can replace the petitioner as mayor. He cannot. The simple reason is that
as he obtained only the second highest number of votes in the election, he was obviously not
the choice of the people of Baguio city.
The latest ruling of the Court on this issue is Santos v. Commission on Elections 22 decided in
1985. In that case, the candidate who placed second was proclaimed elected after the votes
for his winning rival, who was disqualified as a turncoat and considered a non-candidate, were
all disregarded as stray. In effect, the second placer won by default. That decision was
supported by eight members of the Court then 23 with three dissenting 24 and another two
reserving their vote. 25 One was on official leave. 26

Re-examining that decision, the Court finds, and so holds, that it should be reversed in favor
of the earlier case of Geronimo v. Ramos, 27 Which represents the more logical and democratic
rule. That case, which reiterated the doctrine first announced in 1912 in Topacio vs.
Paredes 28 was supported by ten members of the Court 29 without any dissent, although one
reserved his vote, 30 another took no part 31 and two others were on leave. 32 There the Court
held:

... it would be extremely repugnant to the basic concept of the constitutionally


guaranteed right to suffrage if a candidate who has not acquired the majority
or plurality of votes is proclaimed a winner and imposed as the representative
of a constituency, the majority of which have positively declared through their
ballots that they do not choose him.

Sound policy dictates that public elective offices are filled by those who have
received the highest number of votes cast in the election for that office, and it
is a fundamental Idea in all republican forms of government that no one can be
declared elected and no measure can be declared carried unless he or it
receives a majority or plurality of the legal votes cast in the election. (20 Corpus
Juris 2nd, S 243, p. 676.)

The fact that the candidate who obtained the highest number of votes is later
declared to be disqualified or not eligible for the office to which he was elected
does not necessarily entitle the candidate who obtained the second highest
number of votes to be declared the winner of the elective office. The votes cast
for a dead, disqualified, or non-eligible person may not be valid to vote the
winner into office or maintain him there. However, in the absence of a statute
which clearly asserts a contrary political and legislative policy on the matter, if
the votes were cast in the sincere belief that the candidate was alive, qualified,
or eligible, they should not be treated as stray, void or meaningless.

It remains to stress that the citizen of the Philippines must take pride in his status as such
and cherish this priceless gift that, out of more than a hundred other nationalities, God has
seen fit to grant him. Having been so endowed, he must not lightly yield this precious
advantage, rejecting it for another land that may offer him material and other attractions that
he may not find in his own country. To be sure, he has the right to renounce the Philippines
if he sees fit and transfer his allegiance to a state with more allurements for him. 33 But having
done so, he cannot expect to be welcomed back with open arms once his taste for his adopted
country turns sour or he is himself disowned by it as an undesirable alien.

Philippine citizenship is not a cheap commodity that can be easily recovered after its
renunciation. It may be restored only after the returning renegade makes a formal act of re-
dedication to the country he has abjured and he solemnly affirms once again his total and
exclusive loyalty to the Republic of the Philippines. This may not be accomplished by election
to public office.
WHEREFORE, petitioner Ramon J. Labo, Jr. is hereby declared NOT a citizen of the Philippines
and therefore DISQUALIFIED from continuing to serve as Mayor of Baguio City. He is ordered
to VACATE his office and surrender the same to the Vice-Mayor of Baguio City, once this
decision becomes final and executory. The temporary restraining order dated January 31,
1989, is LIFTED.

Fernan, (C.J.), Narvasa, Melencio-Herrera, Paras, Feliciano, Gancayco, Padilla, Bidin,


Sarmiento, Cortes, Griñ;o-Aquino Medialdea and Regalado, JJ., concur.

Separate Opinions

GUTTIERREZ, JR., J.,concurring:

As in the case of Frivaldo v. Commission on Elections (G. R. No. 87193, June 23, 1989) and
inspire of what would otherwise be insuperable procedural obstacles, I am constrained to
concur in the Court's decision so forcefully and felicitously written by Mr. Justice Isagani A.
Cruz. I do so because I cannot see how the Court can countenance a citizen of a foreign
country or one who has renounced Filipino citizenship sitting as the mayor of one of the most
important cities in the Philippines.

What was raised to the Court was only the issue of the COMELEC's jurisdiction to inquire into
the citizenship of the petitioner. Ordinarily, we would have limited ourselves to sustaining the
jurisdiction of the COMELEC and remanding the case for further proceedings and the rendition
of a decision. Under Section 7, Article IXA of the Constitution, a decision, order, or ruling of
the COMELEC may be brought to the Supreme Court on certiorari by the aggrieved party
within thirty day from receipt of a copy thereof. No decision on the petitioner's citizenship has
been rendered and no decision can, as yet, be elevated to us for review. I, therefore, reiterate
my statement in Frivaldo that my concurrence is limited only to cases involving citizenship
and disloyalty but not to any of the many other grounds for disqualification cited in my
concurring opinion.

Our decision to disqualify the petitioner is particularly distressing to me because I am


impressed by the singular achievements in the beautification of Baguio City, in the peace and
order situation, and in the resurgence of civic pride so visible to anyone who has gone up to
Baguio since Mr. Labo assumed the mayorship. However, I see no other way this case can be
resolved except by adopting a pragmatic approach. It is beyond dispute that a non-citizen
cannot be the mayor of Baguio City. I join the rest of the Court.

Fernan, C.J., Narvasa, Melencio-Herrera, Paras, Feliciano, Gancayco, Padilla, Bidin,


Sarmiento, Cortes, Griñ;o-Aquino Medialdea and Regalado, JJ., concur.
G.R. No. 83820 May 25, 1990

JOSE B. AZNAR (as Provincial Chairman of PDP Laban in Cebu), petitioner,


vs.
COMMISSION ON ELECTIONS and EMILIO MARIO RENNER OSMEÑA, respondents.

Rufino B. Requina for petitioner.

Angara, Abello, Concepcion, Regala & Cruz for private respondent.

PARAS, J.:

Before Us is a petition for certiorari assailing the Resolution of the Commission on Elections
(COMELEC) dated June 11, 1988, which dismissed the petition for the disqualification of
private respondent Emilio "Lito" Osmeña as candidate for Provincial Governor of Cebu
Province.

The facts of the case are briefly as follows:

On November 19, 1987, private respondent Emilio "Lito" Osmeña filed his certificate of
candidacy with the COMELEC for the position of Provincial Governor of Cebu Province in the
January 18, 1988 local elections.

On January 22, 1988, the Cebu PDP-Laban Provincial Council (Cebu-PDP Laban, for short), as
represented by petitioner Jose B. Aznar in his capacity as its incumbent Provincial Chairman,
filed with the COMELEC a petition for the disqualification of private respondent on the ground
that he is allegedly not a Filipino citizen, being a citizen of the United States of America.

On January 27, 1988, petitioner filed a Formal Manifestation submitting a Certificate issued
by the then Immigration and Deportation Commissioner Miriam Defensor Santiago certifying
that private respondent is an American and is a holder of Alien Certificate of Registration
(ACR) No. B-21448 and Immigrant Certificate of Residence (ICR) No. 133911, issued at Manila
on March 27 and 28, 1958, respectively. (Annex "B-1").

The petitioner also filed a Supplemental Urgent Ex-Parte Motion for the Issuance of a
Temporary Restraining Order to temporarily enjoin the Cebu Provincial Board of Canvassers
from tabulating/canvassing the votes cast in favor of private respondent and proclaiming him
until the final resolution of the main petition.

Thus, on January 28, 1988, the COMELEC en banc resolved to order the Board to continue
canvassing but to suspend the proclamation.

At the hearing before the COMELEC (First Division), the petitioner presented the following
exhibits tending to show that private respondent is an American citizen: Application for Alien
Registration Form No. 1 of the Bureau of Immigration signed by private respondent dated
November 21, 1979 (Exh. "B"); Alien Certificate of Registration No. 015356 in the name of
private respondent dated November 21, 1979 (Exh. "C"); Permit to Re-enter the Philippines
dated November 21, 1979 (Exh. "D"); Immigration Certificate of Clearance dated January 3,
1980 (Exh. "E"). (pp. 117-118, Rollo)
Private respondent, on the other hand, maintained that he is a Filipino citizen, alleging: that
he is the legitimate child of Dr. Emilio D. Osmeña, a Filipino and son of the late President
Sergio Osmeña, Sr.; that he is a holder of a valid and subsisting Philippine Passport No.
0855103 issued on March 25, 1987; that he has been continuously residing in the Philippines
since birth and has not gone out of the country for more than six months; and that he has
been a registered voter in the Philippines since 1965. (pp. 107-108, Rollo)

On March 3, 1988, COMELEC (First Division) directed the Board of Canvassers to proclaim the
winning candidates. Having obtained the highest number of votes, private respondent was
proclaimed the Provincial Governor of Cebu.

Thereafter, on June 11, 1988, COMELEC (First Division) dismissed the petition for
disqualification for not having been timely filed and for lack of sufficient proof that private
respondent is not a Filipino citizen.

Hence, the present petition.

The petition is not meritorious.

There are two instances where a petition questioning the qualifications of a registered
candidate to run for the office for which his certificate of candidacy was filed can be raised
under the Omnibus Election Code (B.P. Blg. 881), to wit:

(1) Before election, pursuant to Section 78 thereof which provides that:

'Section 78. Petition to deny due course or to cancel a certificate of candidacy.


— A verified petition seeking to deny due course or to cancel a certificate of
candidacy may be filed by any person exclusively on the ground that any
material representation contained therein as required under Section 74 hereof
is false. The petition may be filed at any time not later than twenty-five days
from the time of the filing of the certificate of candidacy and shall be decided,
after the notice and hearing, not later than fifteen days before the election.

and

(2) After election, pursuant to Section 253 thereof, viz:

'Sec. 253. Petition for quo warranto. — Any voter contesting the election of any
Member of the Batasang Pambansa, regional, provincial, or city officer on the
ground of ineligibility or of disloyalty to the Republic of the Philippines shall file
a sworn petition for quo warranto with the Commission within ten days after
the proclamation of the results of the election.

The records show that private respondent filed his certificate of candidacy on November 19,
1987 and that the petitioner filed its petition for disqualification of said private respondent on
January 22, 1988. Since the petition for disqualification was filed beyond the twenty five-day
period required in Section 78 of the Omnibus Election Code, it is clear that said petition was
filed out of time.

The petition for the disqualification of private respondent cannot also be treated as a petition
for quo warranto under Section 253 of the same Code as it is unquestionably premature,
considering that private respondent was proclaimed Provincial Governor of Cebu only on
March 3, 1988.

However, We deem it is a matter of public interest to ascertain the respondent's citizenship


and qualification to hold the public office to which he has been proclaimed elected. There is
enough basis for us to rule directly on the merits of the case, as the COMELEC did below.

Petitioner's contention that private respondent is not a Filipino citizen and, therefore,
disqualified from running for and being elected to the office of Provincial Governor of Cebu, is
not supported by substantial and convincing evidence.

In the proceedings before the COMELEC, the petitioner failed to present direct proof that
private respondent had lost his Filipino citizenship by any of the modes provided for under
C.A. No. 63. Among others, these are: (1) by naturalization in a foreign country; (2) by
express renunciation of citizenship; and (3) by subscribing to an oath of allegiance to support
the Constitution or laws of a foreign country. From the evidence, it is clear that private
respondent Osmeña did not lose his Philippine citizenship by any of the three mentioned
hereinabove or by any other mode of losing Philippine citizenship.

In concluding that private respondent had been naturalized as a citizen of the United States
of America, the petitioner merely relied on the fact that private respondent was issued alien
certificate of registration and was given clearance and permit to re-enter the Philippines by
the Commission on Immigration and Deportation. Petitioner assumed that because of the
foregoing, the respondent is an American and "being an American", private respondent "must
have taken and sworn to the Oath of Allegiance required by the U.S. Naturalization Laws." (p.
81, Rollo)

Philippine courts are only allowed to determine who are Filipino citizens and who are not.
Whether or not a person is considered an American under the laws of the United States does
not concern Us here.

By virtue of his being the son of a Filipino father, the presumption that private respondent is
a Filipino remains. It was incumbent upon the petitioner to prove that private respondent had
lost his Philippine citizenship. As earlier stated, however, the petitioner failed to positively
establish this fact.

The cases of Juan Gallanosa Frivaldo v. COMELEC et al, (G.R. No. 87193, June 21, 1989)
and Ramon L. Labo v. COMELEC et al (G.R. No. 86564, August 1, 1989) are not applicable to
the case at bar.

In the Frivaldo case, evidence shows that he was naturalized as a citizen of the United States
in 1983 per certification from the United States District Court, Northern District of California,
as duly authenticated by Vice Consul Amado P. Cortez of the Philippine Consulate General in
San Francisco, California, U.S.A.

Frivaldo expressly admitted in his answer that he was naturalized in the United States but
claimed that he was forced to embrace American citizenship to protect himself from the
persecution of the Marcos government. The Court, however, found this suggestion of
involuntariness unacceptable, pointing out that there were many other Filipinos in the United
States similarly situated as Frivaldo who did not find it necessary to abandon their status as
Filipinos.
Likewise, in the case of Labo, records show that Labo was married to an Australian citizen
and that he was naturalized as an Australian citizen in 1976, per certification from the
Australian Government through its Consul in the Philippines. This was later affirmed by the
Department of Foreign Affairs.

The authenticity of the above evidence was not disputed by Labo. In fact, in a number of
sworn statements, Labo categorically declared that he was a citizen of Australia.

In declaring both Frivaldo and Labo not citizens of the Philippines, therefore, disqualified from
serving as Governor of the Province of Sorsogon and Mayor of Baguio City, respectively, the
Court considered the fact that by their own admissions, they are indubitably aliens, no longer
owing any allegiance to the Republic of the Philippines since they have sworn their total
allegiance to a foreign state.

In the instant case, private respondent vehemently denies having taken the oath of allegiance
of the United States (p. 81, Rollo). He is a holder of a valid and subsisting Philippine passport
and has continuously participated in the electoral process in this country since 1963 up to the
present, both as a voter and as a candidate (pp. 107-108, Rollo). Thus, private respondent
remains a Filipino and the loss of his Philippine citizenship cannot be presumed.

In the learned dissent of Mr. Justice Teodoro Padilla, he stresses the fact that because Osmeña
obtained Certificates of Alien Registration as an American citizen, the first in 1958 when he
was 24 years old and the second in 1979, he, Osmeña should be regarded as having expressly
renounced Philippine citizenship. To Our mind, this is a case of non sequitur (It does not
follow). Considering the fact that admittedly Osmeña was both a Filipino and an American,
the mere fact that he has a Certificate stating he is an American does not mean that he is not
still a Filipino. Thus, by way of analogy, if a person who has two brothers named Jose and
Mario states or certifies that he has a brother named Jose, this does not mean that he does
not have a brother named Mario; or if a person is enrolled as student simultaneously in two
universities, namely University X and University Y, presents a Certification that he is a student
of University X, this does not necessarily mean that he is not still a student of University Y.
In the case of Osmeña, the Certification that he is an American does not mean that he is not
still a Filipino, possessed as he is, of both nationalities or citizenships. Indeed, there is no
express renunciation here of Philippine citizenship; truth to tell, there is even no implied
renunciation of said citizenship. When We consider that the renunciation needed to lose
Philippine citizenship must be "express", it stands to reason that there can be no such loss of
Philippine 'citizenship when there is no renunciation either "'express" or "implied".

Parenthetically, the statement in the 1987 Constitution that "dual allegiance of citizens is
inimical to the national interest and shall be dealt with by law"(Art. IV, Sec. 5) has no
retroactive effect. And while it is true that even before the 1987 Constitution, Our country
had already frowned upon the concept of dual citizenship or allegiance, the fact is it actually
existed. Be it noted further that under the aforecited proviso, the effect of such dual
citizenship or allegiance shall be dealt with by a future law. Said law has not yet been enacted.

WHEREFORE, the petition for certiorari is hereby DISMISSED and the Resolution of the
COMELEC is hereby AFFIRMED.

SO ORDERED.
[G.R. No. 135083. May 26, 1999]

ERNESTO S. MERCADO, petitioner, vs. EDUARDO BARRIOS MANZANO and the


COMMISSION ON ELECTIONS, respondents.

DECISION
MENDOZA, J.:

Petitioner Ernesto S. Mercado and private respondent Eduardo B. Manzano were


candidates for vice mayor of the City of Makati in the May 11, 1998 elections. The other one
was Gabriel V. Daza III.The results of the election were as follows:

Eduardo B. Manzano 103,853


Ernesto S. Mercado 100,894
Gabriel V. Daza III 54,275[1]

The proclamation of private respondent was suspended in view of a pending petition for
disqualification filed by a certain Ernesto Mamaril who alleged that private respondent was
not a citizen of the Philippines but of the United States.
In its resolution, dated May 7, 1998,[2] the Second Division of the COMELEC granted the
petition of Mamaril and ordered the cancellation of the certificate of candidacy of private
respondent on the ground that he is a dual citizen and, under 40(d) of the Local Government
Code, persons with dual citizenship are disqualified from running for any elective position. The
COMELECs Second Division said:

What is presented before the Commission is a petition for disqualification of Eduardo Barrios
Manzano as candidate for the office of Vice-Mayor of Makati City in the May 11, 1998
elections. The petition is based on the ground that the respondent is an American citizen
based on the record of the Bureau of Immigration and misrepresented himself as a natural-
born Filipino citizen.

In his answer to the petition filed on April 27, 1998, the respondent admitted that he is
registered as a foreigner with the Bureau of Immigration under Alien Certificate of Registration
No. B-31632 and alleged that he is a Filipino citizen because he was born in 1955 of a Filipino
father and a Filipino mother. He was born in the United States, San Francisco, California, on
September 14, 1955, and is considered an American citizen under US Laws. But
notwithstanding his registration as an American citizen, he did not lose his Filipino citizenship.

Judging from the foregoing facts, it would appear that respondent Manzano is both a Filipino
and a US citizen. In other words, he holds dual citizenship.

The question presented is whether under our laws, he is disqualified from the position for
which he filed his certificate of candidacy. Is he eligible for the office he seeks to be elected?

Under Section 40(d) of the Local Government Code, those holding dual citizenship are
disqualified from running for any elective local position.
WHEREFORE, the Commission hereby declares the respondent Eduardo Barrios Manzano
DISQUALIFIED as candidate for Vice-Mayor of Makati City.

On May 8, 1998, private respondent filed a motion for reconsideration.[3] The motion
remained pending even until after the election held on May 11, 1998.
Accordingly, pursuant to Omnibus Resolution No. 3044, dated May 10, 1998, of the
COMELEC, the board of canvassers tabulated the votes cast for vice mayor of Makati City but
suspended the proclamation of the winner.
On May 19, 1998, petitioner sought to intervene in the case for
disqualification.[4] Petitioners motion was opposed by private respondent.
The motion was not resolved. Instead, on August 31, 1998, the COMELEC en banc
rendered its resolution. Voting 4 to 1, with one commissioner abstaining, the COMELEC en
banc reversed the ruling of its Second Division and declared private respondent qualified to
run for vice mayor of the City of Makati in the May 11, 1998 elections. [5] The pertinent portions
of the resolution of the COMELEC en banc read:

As aforesaid, respondent Eduardo Barrios Manzano was born in San Francisco, California,
U.S.A. He acquired US citizenship by operation of the United States Constitution and laws
under the principle of jus soli.

He was also a natural born Filipino citizen by operation of the 1935 Philippine Constitution, as
his father and mother were Filipinos at the time of his birth. At the age of six (6), his parents
brought him to the Philippines using an American passport as travel document. His parents
also registered him as an alien with the Philippine Bureau of Immigration. He was issued an
alien certificate of registration. This, however, did not result in the loss of his Philippine
citizenship, as he did not renounce Philippine citizenship and did not take an oath of allegiance
to the United States.

It is an undisputed fact that when respondent attained the age of majority, he registered
himself as a voter, and voted in the elections of 1992, 1995 and 1998, which effectively
renounced his US citizenship under American law. Under Philippine law, he no longer had U.S.
citizenship.

At the time of the May 11, 1998 elections, the resolution of the Second Division, adopted on
May 7, 1998, was not yet final. Respondent Manzano obtained the highest number of votes
among the candidates for vice-mayor of Makati City, garnering one hundred three thousand
eight hundred fifty three (103,853) votes over his closest rival, Ernesto S. Mercado, who
obtained one hundred thousand eight hundred ninety four (100,894) votes, or a margin of
two thousand nine hundred fifty nine (2,959) votes. Gabriel Daza III obtained third place with
fifty four thousand two hundred seventy five (54,275) votes. In applying election laws, it
would be far better to err in favor of the popular choice than be embroiled in complex legal
issues involving private international law which may well be settled before the highest court
(Cf. Frivaldo vs. Commission on Elections, 257 SCRA 727).

WHEREFORE, the Commission en banc hereby REVERSES the resolution of the Second
Division, adopted on May 7, 1998, ordering the cancellation of the respondents certificate of
candidacy.

We declare respondent Eduardo Luis Barrios Manzano to be QUALIFIED as a candidate for the
position of vice-mayor of Makati City in the May 11, 1998, elections.
ACCORDINGLY, the Commission directs the Makati City Board of Canvassers, upon proper
notice to the parties, to reconvene and proclaim the respondent Eduardo Luis Barrios Manzano
as the winning candidate for vice-mayor of Makati City.

Pursuant to the resolution of the COMELEC en banc, the board of canvassers, on the
evening of August 31, 1998, proclaimed private respondent as vice mayor of the City of
Makati.
This is a petition for certiorari seeking to set aside the aforesaid resolution of the
COMELEC en banc and to declare private respondent disqualified to hold the office of vice
mayor of Makati City.Petitioner contends that

[T]he COMELEC en banc ERRED in holding that:

A. Under Philippine law, Manzano was no longer a U.S. citizen when he:

1. He renounced his U.S. citizenship when he attained the age of majority when he was
already 37 years old; and,

2. He renounced his U.S. citizenship when he (merely) registered himself as a voter and voted
in the elections of 1992, 1995 and 1998.

B. Manzano is qualified to run for and or hold the elective office of Vice-Mayor of the City of
Makati;

C. At the time of the May 11, 1998 elections, the resolution of the Second Division adopted
on 7 May 1998 was not yet final so that, effectively, petitioner may not be declared the winner
even assuming that Manzano is disqualified to run for and hold the elective office of Vice-
Mayor of the City of Makati.

We first consider the threshold procedural issue raised by private respondent


Manzano whether petitioner Mercado has personality to bring this suit considering that he was
not an original party in the case for disqualification filed by Ernesto Mamaril nor was
petitioners motion for leave to intervene granted.

I. PETITIONER'S RIGHT TO BRING THIS SUIT

Private respondent cites the following provisions of Rule 8 of the Rules of Procedure of
the COMELEC in support of his claim that petitioner has no right to intervene and, therefore,
cannot bring this suit to set aside the ruling denying his motion for intervention:

Section 1. When proper and when may be permitted to intervene. Any person allowed to
initiate an action or proceeding may, before or during the trial of an action or proceeding, be
permitted by the Commission, in its discretion to intervene in such action or proceeding, if he
has legal interest in the matter in litigation, or in the success of either of the parties, or an
interest against both, or when he is so situated as to be adversely affected by such action or
proceeding.

....
Section 3. Discretion of Commission. In allowing or disallowing a motion for intervention, the
Commission or the Division, in the exercise of its discretion, shall consider whether or not the
intervention will unduly delay or prejudice the adjudication of the rights of the original parties
and whether or not the intervenors rights may be fully protected in a separate action or
proceeding.

Private respondent argues that petitioner has neither legal interest in the matter in litigation
nor an interest to protect because he is a defeated candidate for the vice-mayoralty post of
Makati City [who] cannot be proclaimed as the Vice-Mayor of Makati City even if the private
respondent be ultimately disqualified by final and executory judgment.
The flaw in this argument is it assumes that, at the time petitioner sought to intervene in
the proceedings before the COMELEC, there had already been a proclamation of the results
of the election for the vice mayoralty contest for Makati City, on the basis of which petitioner
came out only second to private respondent. The fact, however, is that there had been no
proclamation at that time. Certainly, petitioner had, and still has, an interest in ousting private
respondent from the race at the time he sought to intervene. The rule in Labo v.
COMELEC,[6] reiterated in several cases,[7] only applies to cases in which the election of the
respondent is contested, and the question is whether one who placed second to the
disqualified candidate may be declared the winner. In the present case, at the time petitioner
filed a Motion for Leave to File Intervention on May 20, 1998, there had been no proclamation
of the winner, and petitioners purpose was precisely to have private respondent disqualified
from running for [an] elective local position under 40(d) of R.A. No. 7160. If Ernesto Mamaril
(who originally instituted the disqualification proceedings), a registered voter of Makati City,
was competent to bring the action, so was petitioner since the latter was a rival candidate for
vice mayor of Makati City.
Nor is petitioners interest in the matter in litigation any less because he filed a motion for
intervention only on May 20, 1998, after private respondent had been shown to have garnered
the highest number of votes among the candidates for vice mayor. That petitioner had a right
to intervene at that stage of the proceedings for the disqualification against private
respondent is clear from 6 of R.A. No. 6646, otherwise known as the Electoral Reforms Law
of 1987, which provides:

Any candidate who has been declared by final judgment to be disqualified shall not be voted
for, and the votes cast for him shall not be counted. If for any reason a candidate is not
declared by final judgment before an election to be disqualified and he is voted for and
receives the winning number of votes in such election, the Court or Commission shall continue
with the trial and hearing of the action, inquiry, or protest and, upon motion of the
complainant or any intervenor, may during the pendency thereof order the suspension of the
proclamation of such candidate whenever the evidence of guilt is strong.

Under this provision, intervention may be allowed in proceedings for disqualification even
after election if there has yet been no final judgment rendered.
The failure of the COMELEC en banc to resolve petitioners motion for intervention was
tantamount to a denial of the motion, justifying petitioner in filing the instant petition for
certiorari. As the COMELEC en banc instead decided the merits of the case, the present
petition properly deals not only with the denial of petitioners motion for intervention but also
with the substantive issues respecting private respondents alleged disqualification on the
ground of dual citizenship.
This brings us to the next question, namely, whether private respondent Manzano
possesses dual citizenship and, if so, whether he is disqualified from being a candidate for
vice mayor of Makati City.

II. DUAL CITIZENSHIP AS A GROUND FOR DISQUALIFICATION

The disqualification of private respondent Manzano is being sought under 40 of the Local
Government Code of 1991 (R.A. No. 7160), which declares as disqualified from running for
any elective local position: . . . (d) Those with dual citizenship. This provision is incorporated
in the Charter of the City of Makati.[8]
Invoking the maxim dura lex sed lex, petitioner, as well as the Solicitor General, who
sides with him in this case, contends that through 40(d) of the Local Government Code,
Congress has command[ed] in explicit terms the ineligibility of persons possessing dual
allegiance to hold local elective office.
To begin with, dual citizenship is different from dual allegiance. The former arises when,
as a result of the concurrent application of the different laws of two or more states, a person
is simultaneously considered a national by the said states.[9] For instance, such a situation
may arise when a person whose parents are citizens of a state which adheres to the principle
of jus sanguinis is born in a state which follows the doctrine of jus soli. Such a person, ipso
facto and without any voluntary act on his part, is concurrently considered a citizen of both
states. Considering the citizenship clause (Art. IV) of our Constitution, it is possible for the
following classes of citizens of the Philippines to possess dual citizenship:
(1) Those born of Filipino fathers and/or mothers in foreign countries which follow the
principle of jus soli;
(2) Those born in the Philippines of Filipino mothers and alien fathers if by the laws of
their fathers country such children are citizens of that country;
(3) Those who marry aliens if by the laws of the latters country the former are considered
citizens, unless by their act or omission they are deemed to have renounced Philippine
citizenship.
There may be other situations in which a citizen of the Philippines may, without
performing any act, be also a citizen of another state; but the above cases are clearly possible
given the constitutional provisions on citizenship.
Dual allegiance, on the other hand, refers to the situation in which a person
simultaneously owes, by some positive act, loyalty to two or more states. While dual
citizenship is involuntary, dual allegiance is the result of an individuals volition.
With respect to dual allegiance, Article IV, 5 of the Constitution provides: Dual allegiance
of citizens is inimical to the national interest and shall be dealt with by law. This provision was
included in the 1987 Constitution at the instance of Commissioner Blas F. Ople who explained
its necessity as follows:[10]

. . . I want to draw attention to the fact that dual allegiance is not dual citizenship. I have
circulated a memorandum to the Bernas Committee according to which a dual allegiance - and
I reiterate a dual allegiance - is larger and more threatening than that of mere double
citizenship which is seldom intentional and, perhaps, never insidious. That is often a function
of the accident of mixed marriages or of birth on foreign soil. And so, I do not question double
citizenship at all.
What we would like the Committee to consider is to take constitutional cognizance of the
problem of dual allegiance. For example, we all know what happens in the triennial elections
of the Federation of Filipino-Chinese Chambers of Commerce which consists of about 600
chapters all over the country. There is a Peking ticket, as well as a Taipei ticket. Not widely
known is the fact that the Filipino-Chinese community is represented in the Legislative Yuan
of the Republic of China in Taiwan. And until recently, the sponsor might recall, in Mainland
China in the Peoples Republic of China, they have the Associated Legislative Council for
overseas Chinese wherein all of Southeast Asia including some European and Latin countries
were represented, which was dissolved after several years because of diplomatic friction. At
that time, the Filipino-Chinese were also represented in that Overseas Council.

When I speak of double allegiance, therefore, I speak of this unsettled kind of allegiance of
Filipinos, of citizens who are already Filipinos but who, by their acts, may be said to be bound
by a second allegiance, either to Peking or Taiwan. I also took close note of the concern
expressed by some Commissioners yesterday, including Commissioner Villacorta, who were
concerned about the lack of guarantees of thorough assimilation, and especially Commissioner
Concepcion who has always been worried about minority claims on our natural resources.

Dual allegiance can actually siphon scarce national capital to Taiwan, Singapore, China or
Malaysia, and this is already happening. Some of the great commercial places in downtown
Taipei are Filipino-owned, owned by Filipino-Chinese it is of common knowledge in Manila. It
can mean a tragic capital outflow when we have to endure a capital famine which also means
economic stagnation, worsening unemployment and social unrest.

And so, this is exactly what we ask that the Committee kindly consider incorporating a new
section, probably Section 5, in the article on Citizenship which will read as follows: DUAL
ALLEGIANCE IS INIMICAL TO CITIZENSHIP AND SHALL BE DEALT WITH ACCORDING TO LAW.

In another session of the Commission, Ople spoke on the problem of these citizens with
dual allegiance, thus:[11]

. . . A significant number of Commissioners expressed their concern about dual citizenship in


the sense that it implies a double allegiance under a double sovereignty which some of us
who spoke then in a freewheeling debate thought would be repugnant to the sovereignty
which pervades the Constitution and to citizenship itself which implies a uniqueness and which
elsewhere in the Constitution is defined in terms of rights and obligations exclusive to that
citizenship including, of course, the obligation to rise to the defense of the State when it is
threatened, and back of this, Commissioner Bernas, is, of course, the concern for national
security. In the course of those debates, I think some noted the fact that as a result of the
wave of naturalizations since the decision to establish diplomatic relations with the Peoples
Republic of China was made in 1975, a good number of these naturalized Filipinos still
routinely go to Taipei every October 10; and it is asserted that some of them do renew their
oath of allegiance to a foreign government maybe just to enter into the spirit of the occasion
when the anniversary of the Sun Yat-Sen Republic is commemorated. And so, I have detected
a genuine and deep concern about double citizenship, with its attendant risk of double
allegiance which is repugnant to our sovereignty and national security. I appreciate what the
Committee said that this could be left to the determination of a future legislature. But
considering the scale of the problem, the real impact on the security of this country, arising
from, let us say, potentially great numbers of double citizens professing double allegiance,
will the Committee entertain a proposed amendment at the proper time that will prohibit, in
effect, or regulate double citizenship?
Clearly, in including 5 in Article IV on citizenship, the concern of the Constitutional
Commission was not with dual citizens per se but with naturalized citizens who maintain their
allegiance to their countries of origin even after their naturalization. Hence, the phrase dual
citizenship in R.A. No. 7160, 40(d) and in R.A. No. 7854, 20 must be understood as referring
to dual allegiance. Consequently, persons with mere dual citizenship do not fall under this
disqualification. Unlike those with dual allegiance, who must, therefore, be subject to strict
process with respect to the termination of their status, for candidates with dual citizenship, it
should suffice if, upon the filing of their certificates of candidacy, they elect Philippine
citizenship to terminate their status as persons with dual citizenship considering that their
condition is the unavoidable consequence of conflicting laws of different states. As Joaquin G.
Bernas, one of the most perceptive members of the Constitutional Commission, pointed out:
[D]ual citizenship is just a reality imposed on us because we have no control of the laws on
citizenship of other countries. We recognize a child of a Filipino mother. But whether or not
she is considered a citizen of another country is something completely beyond our control. [12]
By electing Philippine citizenship, such candidates at the same time forswear allegiance
to the other country of which they are also citizens and thereby terminate their status as dual
citizens. It may be that, from the point of view of the foreign state and of its laws, such an
individual has not effectively renounced his foreign citizenship. That is of no moment as the
following discussion on 40(d) between Senators Enrile and Pimentel clearly shows: [13]
SENATOR ENRILE. Mr. President, I would like to ask clarification of line 41, page 17: Any
person with dual citizenship is disqualified to run for any elective local position. Under
the present Constitution, Mr. President, someone whose mother is a citizen of the
Philippines but his father is a foreigner is a natural-born citizen of the Republic. There
is no requirement that such a natural born citizen, upon reaching the age of majority,
must elect or give up Philippine citizenship.
On the assumption that this person would carry two passports, one belonging to the
country of his or her father and one belonging to the Republic of the Philippines, may
such a situation disqualify the person to run for a local government position?
SENATOR PIMENTEL. To my mind, Mr. President, it only means that at the moment when
he would want to run for public office, he has to repudiate one of his citizenships.
SENATOR ENRILE. Suppose he carries only a Philippine passport but the country of origin
or the country of the father claims that person, nevertheless, as a citizen? No one can
renounce. There are such countries in the world.
SENATOR PIMENTEL. Well, the very fact that he is running for public office would, in effect,
be an election for him of his desire to be considered as a Filipino citizen.
SENATOR ENRILE. But, precisely, Mr. President, the Constitution does not require an
election. Under the Constitution, a person whose mother is a citizen of the Philippines
is, at birth, a citizen without any overt act to claim the citizenship.
SENATOR PIMENTEL. Yes. What we are saying, Mr. President, is: Under the Gentlemans
example, if he does not renounce his other citizenship, then he is opening himself to
question. So, if he is really interested to run, the first thing he should do is to say in
the Certificate of Candidacy that: I am a Filipino citizen, and I have only one
citizenship.
SENATOR ENRILE. But we are talking from the viewpoint of Philippine law, Mr.
President. He will always have one citizenship, and that is the citizenship invested upon
him or her in the Constitution of the Republic.
SENATOR PIMENTEL. That is true, Mr. President. But if he exercises acts that will prove
that he also acknowledges other citizenships, then he will probably fall under this
disqualification.
This is similar to the requirement that an applicant for naturalization must renounce all
allegiance and fidelity to any foreign prince, potentate, state, or sovereignty [14] of which at
the time he is a subject or citizen before he can be issued a certificate of naturalization as a
citizen of the Philippines. In Parado v. Republic,[15] it was held:

[W]hen a person applying for citizenship by naturalization takes an oath that he renounces
his loyalty to any other country or government and solemnly declares that he owes his
allegiance to the Republic of the Philippines, the condition imposed by law is satisfied and
complied with. The determination whether such renunciation is valid or fully complies with the
provisions of our Naturalization Law lies within the province and is an exclusive prerogative
of our courts. The latter should apply the law duly enacted by the legislative department of
the Republic. No foreign law may or should interfere with its operation and application. If the
requirement of the Chinese Law of Nationality were to be read into our Naturalization Law,
we would be applying not what our legislative department has deemed it wise to require, but
what a foreign government has thought or intended to exact. That, of course, is absurd. It
must be resisted by all means and at all cost. It would be a brazen encroachment upon the
sovereign will and power of the people of this Republic.

III. PETITIONER'S ELECTION OF PHILIPPINE CITIZENSHIP

The record shows that private respondent was born in San Francisco, California on
September 4, 1955, of Filipino parents. Since the Philippines adheres to the principle of jus
sanguinis, while the United States follows the doctrine of jus soli, the parties agree that, at
birth at least, he was a national both of the Philippines and of the United States. However,
the COMELEC en banc held that, by participating in Philippine elections in 1992, 1995, and
1998, private respondent effectively renounced his U.S. citizenship under American law, so
that now he is solely a Philippine national.
Petitioner challenges this ruling. He argues that merely taking part in Philippine elections
is not sufficient evidence of renunciation and that, in any event, as the alleged renunciation
was made when private respondent was already 37 years old, it was ineffective as it should
have been made when he reached the age of majority.
In holding that by voting in Philippine elections private respondent renounced his
American citizenship, the COMELEC must have in mind 349 of the Immigration and Nationality
Act of the United States, which provided that A person who is a national of the United States,
whether by birth or naturalization, shall lose his nationality by: . . . (e) Voting in a political
election in a foreign state or participating in an election or plebiscite to determine the
sovereignty over foreign territory. To be sure this provision was declared unconstitutional by
the U.S. Supreme Court in Afroyim v. Rusk[16] as beyond the power given to the U.S. Congress
to regulate foreign relations. However, by filing a certificate of candidacy when he ran for his
present post, private respondent elected Philippine citizenship and in effect renounced his
American citizenship. Private respondents certificate of candidacy, filed on March 27, 1998,
contained the following statements made under oath:
6. I AM A FILIPINO CITIZEN (STATE IF NATURAL-BORN OR
NATURALIZED) NATURAL-BORN
....
10. I AM A REGISTERED VOTER OF PRECINCT NO. 747-A, BARANGAY SAN LORENZO,
CITY/MUNICIPALITY OF MAKATI, PROVINCE OF NCR .
11. I AM NOT A PERMANENT RESIDENT OF, OR IMMIGRANT TO, A FOREIGN
COUNTRY.
12. I AM ELIGIBLE FOR THE OFFICE I SEEK TO BE ELECTED. I WILL SUPPORT AND
DEFEND THE CONSTITUTION OF THE PHILIPPINES AND WILL MAINTAIN TRUE
FAITH AND ALLEGIANCE THERETO; THAT I WILL OBEY THE LAWS, LEGAL ORDERS
AND DECREES PROMULGATED BY THE DULY CONSTITUTED AUTHORITIES OF THE
REPUBLIC OF THE PHILIPPINES; AND THAT I IMPOSE THIS OBLIGATION UPON
MYSELF VOLUNTARILY, WITHOUT MENTAL RESERVATION OR PURPOSE OF
EVASION. I HEREBY CERTIFY THAT THE FACTS STATED HEREIN ARE TRUE AND
CORRECT OF MY OWN PERSONAL KNOWLEDGE.
The filing of such certificate of candidacy sufficed to renounce his American citizenship,
effectively removing any disqualification he might have as a dual citizen. Thus, in Frivaldo v.
COMELEC it was held:[17]

It is not disputed that on January 20, 1983 Frivaldo became an American. Would the
retroactivity of his repatriation not effectively give him dual citizenship, which under Sec. 40
of the Local Government Code would disqualify him from running for any elective local
position? We answer this question in the negative, as there is cogent reason to hold that
Frivaldo was really STATELESS at the time he took said oath of allegiance and even before
that, when he ran for governor in 1988. In his Comment, Frivaldo wrote that he had long
renounced and had long abandoned his American citizenship-long before May 8, 1995. At
best, Frivaldo was stateless in the interim-when he abandoned and renounced his US
citizenship but before he was repatriated to his Filipino citizenship.

On this point, we quote from the assailed Resolution dated December 19, 1995:

By the laws of the United States, petitioner Frivaldo lost his American citizenship when he
took his oath of allegiance to the Philippine Government when he ran for Governor in 1988,
in 1992, and in 1995.Every certificate of candidacy contains an oath of allegiance to the
Philippine Government.

These factual findings that Frivaldo has lost his foreign nationality long before the elections
of 1995 have not been effectively rebutted by Lee. Furthermore, it is basic that such findings
of the Commission are conclusive upon this Court, absent any showing of capriciousness or
arbitrariness or abuse.

There is, therefore, no merit in petitioners contention that the oath of allegiance contained
in private respondents certificate of candidacy is insufficient to constitute renunciation of his
American citizenship. Equally without merit is petitioners contention that, to be effective, such
renunciation should have been made upon private respondent reaching the age of majority
since no law requires the election of Philippine citizenship to be made upon majority age.
Finally, much is made of the fact that private respondent admitted that he is registered
as an American citizen in the Bureau of Immigration and Deportation and that he holds an
American passport which he used in his last travel to the United States on April 22,
1997. There is no merit in this. Until the filing of his certificate of candidacy on March 21,
1998, he had dual citizenship. The acts attributed to him can be considered simply as the
assertion of his American nationality before the termination of his American citizenship. What
this Court said in Aznar v. COMELEC[18] applies mutatis mutandis to private respondent in the
case at bar:

. . . Considering the fact that admittedly Osmea was both a Filipino and an American, the
mere fact that he has a Certificate stating he is an American does not mean that he is
not still a Filipino. . . . [T]he Certification that he is an American does not mean that he is not
still a Filipino, possessed as he is, of both nationalities or citizenships. Indeed, there is no
express renunciation here of Philippine citizenship; truth to tell, there is even no implied
renunciation of said citizenship. When We consider that the renunciation needed to lose
Philippine citizenship must be express, it stands to reason that there can be no such loss of
Philippine citizenship when there is no renunciation, either express or implied.

To recapitulate, by declaring in his certificate of candidacy that he is a Filipino citizen;


that he is not a permanent resident or immigrant of another country; that he will defend and
support the Constitution of the Philippines and bear true faith and allegiance thereto and that
he does so without mental reservation, private respondent has, as far as the laws of this
country are concerned, effectively repudiated his American citizenship and anything which he
may have said before as a dual citizen.
On the other hand, private respondents oath of allegiance to the Philippines, when
considered with the fact that he has spent his youth and adulthood, received his education,
practiced his profession as an artist, and taken part in past elections in this country, leaves
no doubt of his election of Philippine citizenship.
His declarations will be taken upon the faith that he will fulfill his undertaking made under
oath. Should he betray that trust, there are enough sanctions for declaring the loss of his
Philippine citizenship through expatriation in appropriate proceedings. In Yu v. Defensor-
Santiago,[19] we sustained the denial of entry into the country of petitioner on the ground
that, after taking his oath as a naturalized citizen, he applied for the renewal of his Portuguese
passport and declared in commercial documents executed abroad that he was a Portuguese
national. A similar sanction can be taken against any one who, in electing Philippine
citizenship, renounces his foreign nationality, but subsequently does some act constituting
renunciation of his Philippine citizenship.
WHEREFORE, the petition for certiorari is DISMISSED for lack of merit.
SO ORDERED.
[G.R. No. 142840. May 7, 2001]

ANTONIO BENGSON III, petitioner, vs. HOUSE OF REPRESENTATIVES ELECTORAL


TRIBUNAL and TEODORO C. CRUZ, respondents.

DECISION
KAPUNAN, J.:

The citizenship of respondent Teodoro C. Cruz is at issue in this case, in view of the
constitutional requirement that "no person shall be a Member of the House of Representatives
unless he is a natural-born citizen."[1]
Respondent Cruz was a natural-born citizen of the Philippines. He was born in San
Clemente, Tarlac, on April 27, 1960, of Filipino parents. The fundamental law then applicable
was the 1935 Constitution.[2]
On November 5, 1985, however, respondent Cruz enlisted in the United States Marine
Corps and, without the consent of the Republic of the Philippines, took an oath of allegiance
to the United States. As a consequence, he lost his Filipino citizenship for under
Commonwealth Act No. 63, Section 1(4), a Filipino citizen may lose his citizenship by, among
others, "rendering service to or accepting commission in the armed forces of a foreign
country." Said provision of law reads:

Section 1. How citizenship may be lost. -- A Filipino citizen may lose his citizenship in any of
the following ways and/or events:

xxx

(4) By rendering services to, or accepting commission in, the armed forces of a foreign
country: Provided, That the rendering of service to, or the acceptance of such commission in,
the armed forces of a foreign country, and the taking of an oath of allegiance incident thereto,
with the consent of the Republic of the Philippines, shall not divest a Filipino of his Philippine
citizenship if either of the following circumstances is present:

(a) The Republic of the Philippines has a defensive and/or offensive pact of alliance with said
foreign country; or

(b) The said foreign country maintains armed forces on Philippine territory with the consent
of the Republic of the Philippines: Provided, That the Filipino citizen concerned, at the time of
rendering said service, or acceptance of said commission, and taking the oath of allegiance
incident thereto, states that he does so only in connection with his service to said foreign
country; And provided, finally, That any Filipino citizen who is rendering service to, or is
commissioned in, the armed forces of a foreign country under any of the circumstances
mentioned in paragraph (a) or (b), shall not be permitted to participate nor vote in any
election of the Republic of the Philippines during the period of his service to, or commission
in, the armed forces of said country. Upon his discharge from the service of the said foreign
country, he shall be automatically entitled to the full enjoyment of his civil and political rights
as a Filipino citizen x x x.
Whatever doubt that remained regarding his loss of Philippine citizenship was erased by
his naturalization as a U.S. citizen on June 5, 1990, in connection with his service in the U.S.
Marine Corps.
On March 17, 1994, respondent Cruz reacquired his Philippine citizenship through
repatriation under Republic Act No. 2630.[3] He ran for and was elected as the Representative
of the Second District of Pangasinan in the May 11, 1998 elections. He won by a convincing
margin of 26,671 votes over petitioner Antonio Bengson III, who was then running for
reelection.
Subsequently, petitioner filed a case for Quo Warranto Ad Cautelam with respondent
House of Representatives Electoral Tribunal (HRET) claiming that respondent Cruz was not
qualified to become a member of the House of Representatives since he is not a natural-born
citizen as required under Article VI, Section 6 of the Constitution.[4]
On March 2, 2000, the HRET rendered its decision[5] dismissing the petition for quo
warranto and declaring respondent Cruz the duly elected Representative of the Second
District of Pangasinan in the May 1998 elections. The HRET likewise denied petitioner's motion
for reconsideration of the decision in its resolution dated April 27, 2000. [6]
Petitioner thus filed the present petition for certiorari assailing the HRET's decision on the
following grounds:

1. The HRET committed serious errors and grave abuse of discretion, amounting to excess of
jurisdiction, when it ruled that private respondent is a natural-born citizen of the
Philippines despite the fact that he had ceased being such in view of the loss and renunciation
of such citizenship on his part.

2. The HRET committed serious errors and grave abuse of discretion, amounting to excess of
jurisdiction, when it considered private respondent as a citizen of the Philippines despite the
fact that he did not validly acquire his Philippine citizenship.

3. Assuming that private respondent's acquisition of Philippine citizenship was invalid, the
HRET committed serious errors and grave abuse of discretion, amounting to excess of
jurisdiction, when it dismissed the petition despite the fact that such reacquisition could not
legally and constitutionally restore his natural-born status.[7]

The issue now before us is whether respondent Cruz, a natural-born Filipino who became
an American citizen, can still be considered a natural-born Filipino upon his reacquisition of
Philippine citizenship.
Petitioner asserts that respondent Cruz may no longer be considered a natural-born
Filipino since he lost his Philippine citizenship when he swore allegiance to the United States
in 1995, and had to reacquire the same by repatriation. He insists that Article IV, Section 2
of the Constitution expressly states that natural-born citizens are those who are citizens from
birth without having to perform any act to acquire or perfect such citizenship.
Respondent on the other hand contends that he reacquired his status as a natural-born
citizen when he was repatriated since the phrase "from birth" in Article IV, Section 2 refers to
the innate, inherent and inborn characteristic of being a natural-born citizen.
The petition is without merit.
The 1987 Constitution enumerates who are Filipino citizens as follows:
(1) Those who are citizens of the Philippines at the time of the adoption of this
Constitution;
(2) Those whose fathers or mothers are citizens of the Philippines;
(3) Those born before January 17, 1973 of Filipino mothers, who elect Philippine
citizenship upon reaching the age of majority, and
(4) Those who are naturalized in accordance with law.[8]
There are two ways of acquiring citizenship: (1) by birth, and (2) by naturalization. These
ways of acquiring citizenship correspond to the two kinds of citizens: the natural-born citizen,
and the naturalized citizen. A person who at the time of his birth is a citizen of a particular
country, is a natural-born citizen thereof.[9]
As defined in the same Constitution, natural-born citizens "are those citizens of the
Philippines from birth without having to perform any act to acquire or perfect his Philippine
citizenship."[10]
On the other hand, naturalized citizens are those who have become Filipino citizens
through naturalization, generally under Commonwealth Act No. 473, otherwise known as the
Revised Naturalization Law, which repealed the former Naturalization Law (Act No. 2927),
and by Republic Act No. 530.[11] To be naturalized, an applicant has to prove that he possesses
all the qualifications[12] and none of the disqualifications[13] provided by law to become a
Filipino citizen. The decision granting Philippine citizenship becomes executory only after two
(2) years from its promulgation when the court is satisfied that during the intervening period,
the applicant has (1) not left the Philippines; (2) has dedicated himself to a lawful calling or
profession; (3) has not been convicted of any offense or violation of Government promulgated
rules; or (4) committed any act prejudicial to the interest of the nation or contrary to any
Government announced policies.[14]
Filipino citizens who have lost their citizenship may however reacquire the same in the
manner provided by law. Commonwealth Act. No. 63 (C.A. No. 63), enumerates the three
modes by which Philippine citizenship may be reacquired by a former citizen: (1) by
naturalization, (2) by repatriation, and (3) by direct act of Congress.[15]
Naturalization is a mode for both acquisition and reacquisition of Philippine citizenship. As
a mode of initially acquiring Philippine citizenship, naturalization is governed by
Commonwealth Act No. 473, as amended. On the other hand, naturalization as a mode for
reacquiring Philippine citizenship is governed by Commonwealth Act No. 63.[16] Under this
law, a former Filipino citizen who wishes to reacquire Philippine citizenship must possess
certain qualifications[17] and none of the disqualifications mentioned in Section 4 of C.A.
473.[18]
Repatriation, on the other hand, may be had under various statutes by those who lost
their citizenship due to: (1) desertion of the armed forces; [19] (2) service in the armed forces
of the allied forces in World War II;[20] (3) service in the Armed Forces of the United States
at any other time;[21] (4) marriage of a Filipino woman to an alien; [22] and (5) political and
economic necessity.[23]
As distinguished from the lengthy process of naturalization, repatriation simply consists
of the taking of an oath of allegiance to the Republic of the Philippines and registering said
oath in the Local Civil Registry of the place where the person concerned resides or last resided.
In Angat v. Republic,[24] we held:
xxx. Parenthetically, under these statutes [referring to RA Nos. 965 and 2630], the person
desiring to reacquire Philippine citizenship would not even be required to file a petition in
court, and all that he had to do was to take an oath of allegiance to the Republic of the
Philippines and to register that fact with the civil registry in the place of his residence or where
he had last resided in the Philippines. [Italics in the original.] [25]

Moreover, repatriation results in the recovery of the original nationality.[26] This


means that a naturalized Filipino who lost his citizenship will be restored to his prior status as
a naturalized Filipino citizen. On the other hand, if he was originally a natural-born citizen
before he lost his Philippine citizenship, he will be restored to his former status as a natural-
born Filipino.
In respondent Cruz's case, he lost his Filipino citizenship when he rendered service in the
Armed Forces of the United States. However, he subsequently reacquired Philippine
citizenship under R.A. No. 2630, which provides:

Section 1. Any person who had lost his Philippine citizenship by rendering service to, or
accepting commission in, the Armed Forces of the United States, or after separation from the
Armed Forces of the United States, acquired United States citizenship, may reacquire
Philippine citizenship by taking an oath of allegiance to the Republic of the Philippines and
registering the same with Local Civil Registry in the place where he resides or last resided in
the Philippines. The said oath of allegiance shall contain a renunciation of any other
citizenship.

Having thus taken the required oath of allegiance to the Republic and having registered
the same in the Civil Registry of Magantarem, Pangasinan in accordance with the aforecited
provision, respondent Cruz is deemed to have recovered his original status as a natural-born
citizen, a status which he acquired at birth as the son of a Filipino father.[27] It bears stressing
that the act of repatriation allows him to recover, or return to, his original status before
he lost his Philippine citizenship.
Petitioner's contention that respondent Cruz is no longer a natural-born citizen since he
had to perform an act to regain his citizenship is untenable. As correctly explained by the
HRET in its decision, the term "natural-born citizen" was first defined in Article III, Section 4
of the 1973 Constitution as follows:

Sec. 4. A natural-born citizen is one who is a citizen of the Philippines from birth without
having to perform any act to acquire or perfect his Philippine citizenship.

Two requisites must concur for a person to be considered as such: (1) a person must be
a Filipino citizen from birth and (2) he does not have to perform any act to obtain or perfect
his Philippine citizenship.
Under the 1973 Constitution definition, there were two categories of Filipino citizens which
were not considered natural-born: (1) those who were naturalized and (2) those born before
January 17, 1973,[28] of Filipino mothers who, upon reaching the age of majority, elected
Philippine citizenship. Those "naturalized citizens" were not considered natural-born obviously
because they were not Filipinos at birth and had to perform an act to acquire Philippine
citizenship. Those born of Filipino mothers before the effectivity of the 1973 Constitution were
likewise not considered natural-born because they also had to perform an act to perfect their
Philippine citizenship.
The present Constitution, however, now considers those born of Filipino mothers before
the effectivity of the 1973 Constitution and who elected Philippine citizenship upon reaching
the majority age as natural-born. After defining who are natural-born citizens, Section 2 of
Article IV adds a sentence: "Those who elect Philippine citizenship in accordance with
paragraph (3), Section 1 hereof shall be deemed natural-born citizens." Consequently, only
naturalized Filipinos are considered not natural-born citizens. It is apparent from the
enumeration of who are citizens under the present Constitution that there are only two classes
of citizens: (1) those who are natural-born and (2) those who are naturalized in accordance
with law. A citizen who is not a naturalized Filipino, i.e., did not have to undergo the process
of naturalization to obtain Philippine citizenship, necessarily is a natural-born
Filipino. Noteworthy is the absence in said enumeration of a separate category for persons
who, after losing Philippine citizenship, subsequently reacquire it. The reason therefor is clear:
as to such persons, they would either be natural-born or naturalized depending on the reasons
for the loss of their citizenship and the mode prescribed by the applicable law for the
reacquisition thereof. As respondent Cruz was not required by law to go through naturalization
proceedings in order to reacquire his citizenship, he is perforce a natural-born Filipino. As
such, he possessed all the necessary qualifications to be elected as member of the House of
Representatives.
A final point. The HRET has been empowered by the Constitution to be the "sole judge"
of all contests relating to the election, returns, and qualifications of the members of the
House.[29] The Court's jurisdiction over the HRET is merely to check "whether or not there has
been a grave abuse of discretion amounting to lack or excess of jurisdiction" on the part of
the latter.[30] In the absence thereof, there is no occasion for the Court to exercise its
corrective power and annul the decision of the HRET nor to substitute the Court's judgment
for that of the latter for the simple reason that it is not the office of a petition for certiorari to
inquire into the correctness of the assailed decision. [31] There is no such showing of grave
abuse of discretion in this case.
WHEREFORE, the petition is hereby DISMISSED.
Davide, Jr., Bellosillo, Puno, and Ynares-Santiago, JJ, concur.
Melo, and Vitug JJ., no part, Chairman and member, respectively, of HRET which
rendered the appealed judgement.
Mendoza, J., no part, being ponente of decision under review.
Panganiban, J., has separate concurring opinion.
Quisumbing, Buena, and de Leon, JJ., on leave.
Pardo, and Gonzaga-Reyes, JJ., joins the concurring opinion of J. Panganiban.
Sandoval-Gutierrez, J., dissents.
G.R. No. L-21289 October 4, 1971

MOY YA LIM YAO alias EDILBERTO AGUINALDO LIM and LAU YUEN
YEUNG, petitioners-appellants,
vs.
THE COMMISSIONER OF IMMIGRATION, respondent-appellee.

Aruego, Mamaril & Associates for petitioners-appellants.

Office of the Solicitor General Arturo A. Alafriz, Assistant Solicitor General Frine' C. Zaballero
and Solicitor Sumilang V. Bernardo for respondent-appellee.

BARREDO, J.:

Appeal from the following decision of the Court of First Instance of Manila in its Civil Case No.
49705 entitled Moy Ya Lim Yao, etc., et al. vs. The Commissioner of Immigration which, brief
as it is, sufficiently depicts the factual setting of and the fundamental issues involved in this
case thus:

In the instant case, petitioners seek the issuance of a writ of injunction against
the Commissioner of Immigration, "restraining the latter and/or his authorized
representative from ordering plaintiff Lau Yuen Yeung to leave the Philippines
and causing her arrest and deportation and the confiscation of her bond, upon
her failure to do so."

The prayer for preliminary injunction embodied in the complaint, having been
denied, the case was heard on the merits and the parties submitted their
respective evidence.

The facts of the case, as substantially and correctly stated by the Solicitor
General are these:

On February 8, 1961, Lau Yuen Yeung applied for a passport visa


to enter the Philippines as a non-immigrant. In the interrogation
made in connection with her application for a temporary visitor's
visa to enter the Philippines, she stated that she was a Chinese
residing at Kowloon, Hongkong, and that she desired to take a
pleasure trip to the Philippines to visit her great (grand) uncle
Lau Ching Ping for a period of one month (Exhibits "l," "1-a," and
"2"). She was permitted to come into the Philippines on March
13, 1961, and was permitted to stay for a period of one month
which would expire on April 13, 1961. On the date of her arrival,
Asher Y, Cheng filed a bond in the amount of P1,000.00 to
undertake, among others that said Lau Yuen Yeung would
actually depart from the Philippines on or before the expiration
of her authorized period of stay in this country or within the
period as in his discretion the Commissioner of Immigration or
his authorized representative might properly allow. After
repeated extensions, petitioner Lau Yuen Yeung was allowed to
stay in the Philippines up to February 13, 1962 (Exhibit "4"). On
January 25, 1962, she contracted marriage with Moy Ya Lim Yao
alias Edilberto Aguinaldo Lim an alleged Filipino citizen. Because
of the contemplated action of respondent to confiscate her bond
and order her arrest and immediate deportation, after the
expiration of her authorized stay, she brought this action for
injunction with preliminary injunction. At the hearing which took
place one and a half years after her arrival, it was admitted that
petitioner Lau Yuen Yeung could not write either English or
Tagalog. Except for a few words, she could not speak either
English or Tagalog. She could not name any Filipino neighbor,
with a Filipino name except one, Rosa. She did not know the
names of her brothers-in-law, or sisters-in-law.

Under the facts unfolded above, the Court is of the considered opinion, and so
holds, that the instant petition for injunction cannot be sustained for the same
reason as set forth in the Order of this Court, dated March 19, 1962, the
pertinent portions of which read:

First, Section 15 of the Revised Naturalization Law provides:

Effect of the naturalization on wife and children. — Any woman


who is now or may hereafter be married to a citizen of the
Philippines, and who might herself be lawfully naturalized shall
be deemed a citizen of the Philippines.

The above-quoted provision is clear and its import unequivocal and hence it
should be held to mean what it plainly and explicitly expresses in unmistakable
terms. The clause "who might herself be lawfully naturalized" incontestably
implies that an alien woman may be deemed a citizen of the Philippines by
virtue of her marriage to a Filipino citizen only if she possesses all the
qualifications and none of the disqualifications specified in the law, because
these are the explicit requisites provided by law for an alien to be naturalized.
(Lee Suan Ay, Alberto Tan and Lee Chiao vs. Emilio Galang, etc., G. R. No. L-
11855). However, from the allegation of paragraph 3 of the complaint, to wit:

3. That plaintiff Lau Yuen Yeung, Chinese by birth, who might


herself be lawfully naturalized as a Filipino citizen (not being
disqualified to become such by naturalization), is a Filipino citizen
by virtue of her marriage on January 25, 1962 to plaintiff MOY
YA LIM YAO alias EDILBERTO AGUINALDO LIM, under the
Naturalization Laws of the Philippines.

it can be deduced beyond debate that petitioner Lau Yuen Yeung while claiming
not to be disqualified, does not and cannot allege that she possesses all the
qualifications to be naturalized, naturally because, having been admitted as a
temporary visitor only on March 13, 1961, it is obvious at once that she lacks
at least, the requisite length of residence in the Philippines (Revised
Naturalization Law, Sec. 2, Case No. 2, Sec. 3, Case No. 3).

Were if the intention of the law that the alien woman, to be


deemed a citizen of the Philippines by virtue of marriage to a
Filipino citizen, need only be not disqualified under the
Naturalization Law, it would have been worded "and who herself
is not disqualified to become a citizen of the Philippines."

Second, Lau Yuen Yeung, a temporary Chinese woman visitor, whose


authorized stay in the Philippines, after repeated extensions thereof, was to
expire last February 28, 1962, having married her co-plaintiff only on January
25, 1962, or just a little over one month before the expiry date of her stay, it
is evident that said marriage was effected merely for convenience to defeat or
avoid her then impending compulsory departure, not to say deportation. This
cannot be permitted.

Third, as the Solicitor General has well stated:

5. That petitioner Lau Yuen Yeung, having been admitted as a


temporary alien visitor on the strength of a deliberate and
voluntary representation that she will enter and stay only for a
period of one month and thereby secured a visa, cannot go back
on her representation to stay permanently without first departing
from the Philippines as she had promised. (Chung Tiao Bing, et
al. vs. Commissioner of Immigration, G. R. No. L-9966,
September 29, 1956; Ong Se Lun vs. Board of Commissioners,
G. R. No. L-6017, September 16, 1954; Sec. 9, last par., Phil.
Immigration Law).

The aforequoted argument of the Solicitor General is well buttressed not only
by the decided cases of the Supreme Court on the point mentioned above, but
also on the very provisions of Section 9, sub-paragraph (g) of the Philippine
Immigration Act of 1940 which reads:

An alien who is admitted as a non-immigrant cannot remain in


the Philippines permanently. To obtain permanent admission, a
non-immigrant alien must depart voluntarily to some foreign
country and procure from the appropriate Philippine Consul the
proper visa and thereafter undergo examination by the Officers
of the Bureau of Immigration at a Philippine port of entry for
determination of his admissibility in accordance with the
requirements of this Act. (This paragraph is added by Republic
Act 503). (Sec. 9, subparagraph (g) of the Philippine Immigration
Act of 1940).

And fourth, respondent Commissioner of Immigration is charged with the


administration of all laws relating to immigration (Sec. 3, Com. Act No. 613)
and in the performance of his duties in relation to alien immigrants, the law
gives the Commissioner of Immigration a wide discretion, a quasi-judicial
function in determining cases presented to him (Pedro Uy So vs. Commissioner
of Immigration CA-G. R. No. 23336-R, Dec. 15, 1960), so that his decision
thereon may not be disturbed unless he acted with abuse of discretion or in
excess of his jurisdiction.
It may also be not amiss to state that wife Lau Yuen Yeung, while she barely
and insufficiently talk in broken Tagalog and English, she admitted that she
cannot write either language.

The only matter of fact not clearly passed upon by His Honor which could have some bearing
in the resolution of this appeal is the allegation in the brief of petitioners-appellants, not
denied in the governments brief, that "in the hearing ..., it was shown thru the testimony of
the plaintiff Lau Yuen Yeung that she does not possess any of the disqualifications for
naturalization." Of course, as an additional somehow relevant factual matter, it is also
emphasized by said appellants that during the hearing in the lower court, held almost ten
months after the alleged marriage of petitioners, "Lau Yuen Yeung was already carrying in
her womb for seven months a child by her husband."

Appellants have assigned six errors allegedly committed by the court a quo, thus:

THE LOWER COURT ERRED IN HOLDING THAT THE CLAUSE "WHO MIGHT
HERSELF BE LAWFULLY NATURALIZED" (OF SECTION 15, REVISED
NATURALIZATION LAW) INCONTESTABLY IMPLIES THAT AN ALIEN WOMAN
MAY BE DEEMED A CITIZEN OF THE PHILIPPINES BY VIRTUE OF HER MARRIAGE
TO A FILIPINO CITIZEN, ONLY IF SHE POSSESSES ALL THE QUALIFICATIONS
AND NONE OF THE DISQUALIFICATIONS SPECIFIED IN THE LAW.

II

THE LOWER COURT ERRED IN HOLDING THAT A WOMAN FOREIGNER WHO


DOES NOT POSSESS ANY OF THE DISQUALIFICATIONS FOR CITIZENSHIP AND
WHO MARRIED A FILIPINO CITIZEN IS STILL CONSIDERED AN ALIEN EVEN
AFTER SUCH MARRIAGE AS TO FALL WITHIN THE REQUIREMENT OF SECTION
9, SUB-PARAGRAPH (9) OF THE PHILIPPINE IMMIGRATION ACT OF 1940.

III

THE COURT ERRED IN CONCLUDING THAT LAU YUEN YEUNG'S MARRIAGE TO


A FILIPINO CITIZEN WAS ONLY FOR CONVENIENCE, MERELY BECAUSE THE
SAME WAS CELEBRATED JUST OVER A MONTH BEFORE THE EXPIRY DATE OF
HER AUTHORIZED STAY.

IV

THE LOWER COURT ERRED IN FAILING TO FIND THAT THE COMMISSIONER OF


IMMIGRATION ACTED WITH ABUSE OF DISCRETION OR IN EXCESS OF HIS
JURISDICTION WHEN SAID OFFICER THREATENED TO SEND OUT OF THE
COUNTRY PLAINTIFF LAU YUEN YEUNG WITH WARNING THAT HER FAILURE TO
DO SO WOULD MEAN CONFISCATION OF HER BOND, ARREST AND IMMEDIATE
DEPORTATION, IN SPITE OF THE FACT THAT LAU YUEN YEUNG IS NOW A
FILIPINO CITIZEN.

V
THE LOWER COURT ERRED IN DISMISSING PLAINTIFFS-APPELLANTS'
COMPLAINT AND IN REFUSING TO PERMANENTLY ENJOIN THE COMMISSIONER
FROM ORDERING PLAINTIFF LAU YUEN YEUNG TO LEAVE THE PHILIPPINES AS
A TEMPORARY VISITOR WHICH SHE IS NOT.

VI

THE LOWER COURT ERRED IN REFUSING TO GRANT PLAINTIFFS-APPELLANTS'


MOTION FOR PRELIMINARY INJUNCTION EMBODIED IN THEIR COMPLAINT, IN
AN ORDER DATED MARCH 19, 1962. (PAGES 36-41, RECORD ON APPEAL) .

We need not discuss these assigned errors separately. In effect, the above decision upheld
the two main grounds of objection of the Solicitor General to the petition in the court
below, viz:

That petitioner Lau Yuen Yeung, having been admitted as a temporary alien
visitor on the strength of a deliberate and voluntary representation that she will
enter and stay only for a period of one month and thereby secured a visa,
cannot go back on her representation to stay permanently without first
departing from the Philippines as she had promised. (Chung Tiao Bing, et al.
vs. Commissioner of Immigration, G.R. No. L-9966, September 29, 1956; Ong
Se Lun vs. Board of Commissioners, G.R. No. L-6017, Sept. 16, 1954, Sec. 9,
last par. Phil. Immigration Law);

That the mere marriage of a Filipino citizen to an alien does not automatically
confer on the latter Philippine citizenship. The alien wife must possess all the
qualifications required by law to become a Filipino citizen by naturalization and
none of the disqualifications. (Lee Suan Ay, Alberto Tan and Lee Chiao vs.
Galang, etc., G. R. No. L-11855, Dec. 25, 1959)

It is obvious from the nature of these objection that their proper resolution would necessarily
cover all the points raised in appellants' assignments of error, hence, We will base our
discussions, more or less, on said objections.

The first objection of the Solicitor General which covers the matters dealt with in appellants'
second and fourth assignments of error does not require any lengthy discussion. As a matter
of fact, it seem evident that the Solicitor General's pose that an alien who has been admitted
into the Philippines as a non-immigrant cannot remain here permanently unless he voluntarily
leaves the country first and goes to a foreign country to secure thereat from the appropriate
Philippine consul the proper visa and thereafter undergo examination by officers of the Bureau
of Immigration at a Philippine port of entry for determination of his admissibility in accordance
with the requirements of the Philippine Immigration Act of 1940, as amended by Republic Act
503, is premised on the assumption that petitioner Lau Yuen Yeung is not a Filipino citizen.
We note the same line of reasoning in the appealed decision of the court a quo. Accordingly,
it is but safe to assume that were the Solicitor General and His Honor of the view that said
petitioner had become ipso facto a Filipina by virtue of her marriage to her Filipino husband,
they would have held her as entitled to assume the status of a permanent resident without
having to depart as required of aliens by Section 9 (g) of the law.
In any event, to set this point at rest, We hereby hold that portion of Section 9 (g) of the
Immigration Act providing:

An alien who is admitted as a non-immigrant cannot remain in the Philippines


permanently. To obtain permanent admission, a non-immigrant alien must
depart voluntarily to some foreign country and procure from the appropriate
Philippine consul the proper visa and thereafter undergo examination by the
officers of the Bureau of Immigration at a Philippine port of entry for
determination of his admissibility in accordance with the requirements of this
Act.

does not apply to aliens who after coming into the Philippines as temporary visitors,
legitimately become Filipino citizens or acquire Filipino citizenship. Such change of nationality
naturally bestows upon their the right to stay in the Philippines permanently or not, as they
may choose, and if they elect to reside here, the immigration authorities may neither deport
them nor confiscate their bonds. True it is that this Court has vehemently expressed
disapproval of convenient ruses employed by alien to convert their status from temporary
visitors to permanent residents in circumvention of the procedure prescribed by the legal
provision already mentioned, such as in Chiong Tiao Bing vs. Commissioner of Immigration,
99 Phil. 1020, wherein, thru Mr. Justice J.B.L. Reyes, the Court, reiterating the ruling in Ong
Se Lun vs. Board of Immigration Commissioners, 95 PMI. 785, said:

... It is clear that if an alien gains admission to the Islands on the strength of a
deliberate and voluntary representation that he will enter only for a limited
time, and thereby secures the benefit of a temporary visa, the law will not allow
him subsequently to go back on his representation and stay permanently,
without first departing from the Philippines as he had promised. No officer can
relieve him of the departure requirements of section 9 of the Immigration Act,
under the guise of "change" or "correction", for the law makes no distinctions,
and no officer is above the law. Any other ruling would, as stated in our previous
decision, encourage aliens to enter the Islands on false pretences; every alien
so permitted to enter for a limited time, might then claim a right to permanent
admission, however flimsy such claim should be, and thereby compel our
government to spend time, money and effort to examining and verifying
whether or not every such alien really has a right to take up permanent
residence here. In the meanwhile, the alien would be able to prolong his stay
and evade his return to the port whence he came, contrary to what he promised
to do when he entered. The damages inherent in such ruling are self-evident.

On the other hand, however, We cannot see any reason why an alien who has been here as
a temporary visitor but who has in the meanwhile become a Filipino should be required to still
leave the Philippines for a foreign country, only to apply thereat for a re-entry here and
undergo the process of showing that he is entitled to come back, when after all, such right
has become incontestible as a necessary concomitant of his assumption of our nationality by
whatever legal means this has been conferred upon him. Consider for example, precisely the
case of the minor children of an alien who is naturalized. It is indubitable that they
become ipso facto citizens of the Philippines. Could it be the law that before they can be
allowed permanent residence, they still have to be taken abroad so that they may be
processed to determine whether or not they have a right to have permanent residence here?
The difficulties and hardships which such a requirement entails and its seeming
unreasonableness argue against such a rather absurd construction. Indeed, as early as 1957,
in Ly Giok Ha vs. Galang, 101 Phil. 459, Mr. Justice Concepcion, our present Chief Justice,
already ruled thus:

... (P)etitioners allege that, upon her marriage to a Filipino, Ly Giok Ha became
also a citizen of the Philippines. Indeed, if this conclusion were correct, it would
follow that, in consequence of her marriage, she had been naturalized as such
citizen, and, hence the decision appealed from would have to be affirmed, for
section 40(c) of Commonwealth Act 613 provides that "in the event of the
naturalization as a Philippine citizen ... of the alien on whose behalf the bond
deposit is given, the bond shall be cancelled or the sum deposited shall be
returned to the depositor or his legal representative." (At. pp. 462-463)

In other words, the applicable statute itself more than implies that the naturalization of an
alien visitor as a Philippine citizen logically produces the effect of conferring upon him ipso
facto all the rights of citizenship including that of being entitled to permanently stay in the
Philippines outside the orbit of authority of the Commissioner of Immigration vis-a-vis aliens,
if only because by its very nature and express provisions, the Immigration Law is a law only
for aliens and is inapplicable to citizens of the Philippines. In the sense thus discussed
therefore, appellants' second and fourth assignments of error are well taken.

II

Precisely, the second objection, of the Solicitor General sustained by the trial judge is that
appellant Lau Yuen Yeung's marriage to appellant Moya Lim Yao alias Edilberto Aguinaldo
whose Filipino citizenship is not denied did not have the effect of making her a Filipino, since
it has not been shown that she "might herself be lawfully naturalized," it appearing clearly in
the record that she does not possess all the qualifications required of applicants for
naturalization by the Revised Naturalization Law, Commonwealth Act 473, even if she has
proven that she does not suffer from any of the disqualifications thereunder. In other words,
the Solicitor General implicitly concedes that had it been established in the proceedings below
that appellant Lau Yuen Yeung possesses all the qualifications required by the law of
applicants for naturalization, she would have been recognized by the respondent as a Filipino
citizen in the instant case, without requiring her to submit to the usual proceedings for
naturalization.

To be sure, this position of the Solicitor General is in accord with what used to be the view of
this Court since Lee Suan Ay, et al. v. Emilio Galang, etc., et al., G.R. No. L-11855,
promulgated December 23, 1959, 106 Phil., 706,713, 1 for it was only in Zita Ngo Burca vs.
Republic, G.R. NO. L-24252 which was promulgated on January 30, 1967 (19 SCRA 186), that
over the pen of Mr. Justice Conrado Sanchez, this Court held that for an alien woman who
marries a Filipino to be deemed a Filipina, she has to apply for naturalization in accordance
with the procedure prescribed by the Revised Naturalization Law and prove in said
naturalization proceeding not only that she has all the qualifications and none of the
disqualifications provided in the law but also that she has complied with all the formalities
required thereby like any other applicant for naturalization, 2 albeit said decision is not yet
part of our jurisprudence inasmuch as the motion for its reconsideration is still pending
resolution. Appellants are in effect urging Us, however, in their first and second assignments
of error, not only to reconsider Burca but to even reexamine Lee Suan Ay which, as a matter
of fact, is the prevailing rule, having been reiterated in all subsequent decisions up to Go Im
Ty.3
Actually, the first case in which Section 15 of the Naturalization Law, Commonwealth Act 473,
underwent judicial construction was in the first Ly Giok Ha case, 4 one almost identical to the
one at bar. Ly Giok Ha, a woman of Chinese nationality, was a temporary visitor here whose
authority to stay was to expire on March 14, 1956. She filed a bond to guaranty her timely
departure. On March 8, 1956, eight days before the expiration of her authority to stay, she
married a Filipino by the name of Restituto Lacasta. On March 9, 1956, her husband notified
the Commissioner of Immigration of said marriage and, contending that his wife had become
a Filipina by reason of said marriage, demanded for the cancellation of her bond, but instead
of acceding to such request, the Commissioner required her to leave, and upon her failure to
do so, on March 16, 1956, the Commissioner confiscated her bond; a suit was filed for the
recovery of the bond; the lower court sustained her contention that she had no obligation to
leave, because she had become Filipina by marriage, hence her bond should be returned. The
Commissioner appealed to this Court. In the said appeal, Mr. Justice Roberto Concepcion, our
present Chief Justice, spoke for the Court, thus:

The next and most important question for determination is whether her
marriage to a Filipino justified or, at least, excused the aforesaid failure of Ly
Giok Ha to depart from the Philippines on or before March 14, 1956. In
maintaining the affirmative view, petitioners alleged that, upon her marriage
to a Filipino, Ly Giok Ha became, also, a citizen of the Philippines. Indeed, if
this conclusion were correct, it would follow that, in consequence of her
marriage, she had been naturalized as such citizen, and, hence, the decision
appealed from would have to be affirmed, for section 40(c) of Commonwealth
Act No. 613 provides that "in the event of the naturalization as a Philippine
citizen ... of the alien on whose behalf the bond deposit is given, the bond shall
be cancelled or the sum deposited shall be returned to the depositor or his legal
representative." Thus the issue boils down to whether an alien female who
marries a male citizen of the Philippines follows ipso facto his political status.

The pertinent part of section 15 of Commonwealth Act No. 473, upon which
petitioners rely, reads:

Any woman who is now or may hereafter be married to a citizen


of the Philippines, and who might herself be lawfully naturalized
shall be deemed a citizen of the Philippines.

Pursuant thereto, marriage to a male Filipino does not vest Philippine


citizenship to his foreign wife, unless she "herself may be lawfully naturalized."
As correctly held in an opinion of the Secretary of Justice (Op. No. 52, series of
1950),* this limitation of section 15 excludes, from the benefits of
naturalization by marriage, those disqualified from being naturalized as citizens
of the Philippines under section 4 of said Commonwealth Act No. 473, namely:

(a) Persons opposed to organized government or affiliated with


any association or group of persons who uphold and teach
doctrines opposing all organized governments;

(b) Persons defending or teaching the necessity or propriety of


violence, personal assault, or assassination for the success and
predominance of their ideas;

(c) Polygamists or believers in the practice of polygamy;


(d) Persons convicted of crimes involving moral turpitude;

(e) Persons suffering from mental alienation or incurable


contagious diseases;

(f) Persons who, during the period of their residence in the


Philippines, have not mingled socially with the Filipinos, or who
have not evinced a sincere desire to learn and embrace the
customs, traditions, and ideals of the Filipinos;

(g) Citizens or subjects of nations with whom the ... Philippines


are at war, during the period of such war;

(h) Citizens or subjects of a foreign country other than the United


States, whose laws does not grant Filipinos the right to become
naturalized citizens or subjects thereof.

In the case at bar, there is neither proof nor allegation in the pleadings that Ly
Giok Ha does not fall under any of the classes disqualified by law. Moreover, as
the parties who claim that, despite her failure to depart from the Philippines
within the period specified in the bond in question, there has been no breach
thereof, petitioners have the burden of proving her alleged change of political
status, from alien to citizen. Strictly speaking, petitioners have not made out,
therefore a case against the respondents-appellants.

Considering, however, that neither in the administrative proceedings, nor in the


lower court, had the parties seemingly felt that there was an issue on whether
Ly Giok Ha may "be lawfully naturalized," and this being a case of first
impression in our courts, we are of the opinion that, in the interest of equity
and justice, the parties herein should be given an opportunity to introduce
evidence, if they have any, on said issue. (At pp. 462-464.) .

As may be seen, although not specifically in so many words, no doubt was left in the above
decision as regards the following propositions: .

1. That under Section 15 of Commonwealth Act 473, the Revised Naturalization Law, the
marriage of an alien woman to a Filipino makes her a Filipina, if she "herself might be lawfully
naturalized";

2. That this Court declared as correct the opinion of the Secretary of Justice that the limitation
of Section 15 of the Naturalization Law excludes from the benefits of naturalization by
marriage, only those disqualified from being naturalized under Section 4 of the law qouted in
the decision;

3. That evidence to the effect that she is not disqualified may be presented in the action to
recover her bond confiscated by the Commissioner of Immigration;

4. That upon proof of such fact, she may be recognized as Filipina; and

5. That in referring to the disqualification enumerated in the law, the Court somehow left the
impression that no inquiry need be made as to qualifications, 5 specially considering that the
decision cited and footnotes several opinions of the Secretary of Justice, the immediate
superior of the Commissioner of Immigration, the most important of which are the following:

Paragraph (a), section 13 of Act No. 2927, as amended, (now section 15,
Commonwealth Act No. 473), provided that "any woman who is now or may
hereafter be married to a citizen of the Philippines, and who might herself be
lawfully naturalized shall be deemed a citizen of the Philippines." A similar
provision in the naturalization law of the United States has been construed as
not requiring the woman to have the qualifications of residence, good character,
etc., as in the case of naturalization by judicial proceedings, but merely that
she is of the race of persons who may be naturalized. (Kelly v. Owen [Dist. Col.
1868] 7 Wall 496, 5F, 11, 12; ex parte Tryason [D. C. Wash. 1914] 215 F. 449,
27 Op. Atty. Gen. 507). (Op. No. 168, s. 1940 of Justice Sec. Jose Abad
Santos.)

In a previous opinion rendered for your Office, I stated that the clause "who
might herself be lawfully naturalized", should be construed as not requiring the
woman to have the qualifications of residence, good character, etc., as in cases
of naturalization by judicial proceedings, but merely that she is of the race of
persons who may be naturalized. (Op. No. 79, s. 1940)

Inasmuch as the race qualification has been removed by the Revised


Naturalization Law, it results that any woman who married a citizen of the
Philippines prior to or after June 17, 1939, and the marriage not having been
dissolved, and on the assumption that she possesses none of the
disqualifications mentioned in Section 4 of Commonwealth Act No. 473, follows
the citizenship of her husband. (Op. No. 176, s. 1940 of Justice Sec. Jose Abad
Santos.)

From the foregoing narration of facts, it would seem that the only material point
of inquiry is as to the citizenship of Arce Machura. If he shall be found to be a
citizen of the Philippines, his wife, Mrs. Lily James Machura, shall likewise be
deemed a citizen of the Philippines pursuant to the provision of Section 15,
Commonwealth Act No. 473, which reads in part as follows:

Any woman who is now or may hereafter be married to a citizen


of the Philippines, and who might herself be lawfully naturalized
shall be deemed a citizen of the Philippines.

The phrase "who might herself be lawfully naturalized", as contained in the


above provision, means that the woman who is married to a Filipino citizen
must not belong to any of the disqualified classes enumerated in Section 4 of
the Naturalization Law (Ops., Sec. of Jus., No. 28, s. 1950; No. 43, s. 1948,
No. 95, s. 1941; Nos. 79 and 168, s. 1940). Under the facts stated in the within
papers, Mrs. Machura does not appear to be among the disqualified classes
mentioned in the law.

It having been shown that Arce Machura or Arsenio Guevara was born as an
illegitimate of a Filipino mother, he should be considered as a citizen of the
Philippines in consonance with the well-settled rule that an illegitimate child
follows the citizenship of his only legally recognized parent, the mother (Op.,
Sec. of Jus., Nos. 58, 98 & 281, s. 1948; No. 96, s. 1949). Her husband being
a Filipino, Mrs. Machura must necessarily be deemed as a citizen of the
Philippines by marriage (Sec. 15, Com. Act No. 473.) (Op. No. 52, s. 1950 of
Justice Sec. Ricardo Nepomuceno.)

The logic and authority of these opinions, compelling as they are, must have so appealed to
this Court that five days later, on May 22, 1957, in Ricardo Cua v. The Board of
Commissioners, 101 Phil. 521, Mr. Justice J.B.L. Reyes, reiterated the same ruling on the
basis of the following facts:

Tjioe Wu Suan, an Indonesian, arrived in Manila on November 1, 1952, but it turned out that
her passport was forged. On December 10, 1953, a warrant was issued for her arrest for
purpose of deportation. Later, on December 20, 1953, she married Ricardo Cua, a Filipino,
and because of said marriage, the Board of Special Inquiry considered her a Filipina. Upon a
review of the case, however, the Board of Immigration Commissioners insisted on continuing
with the deportation proceedings and so, the husband filed prohibition and mandamus
proceedings. The lower court denied the petition. Although this Court affirmed said decision,
it held, on the other hand, that:

Granting the validity of marriage, this Court has ruled in the recent case of Ly
Giok Ha v. Galang, supra, p. 459, that the bare fact of a valid marriage to a
citizen does not suffice to confer his citizenship upon the wife. Section 15 of the
Naturalization Law requires that the alien woman who marries a Filipino must
show, in addition, that she "might herself be lawfully naturalized" as a Filipino
citizen. As construed in the decision cited, this last condition requires proof that
the woman who married a Filipino is herself not disqualified under section 4 of
the Naturalization Law.

No such evidence appearing on record, the claim of assumption of Filipino


citizenship by Tjioe Wu Suan, upon her marriage to petitioner, is untenable.
The lower court, therefore, committed no error in refusing to interfere with the
deportation proceedings, where she can anyway establish the requisites
indispensable for her acquisition of Filipino citizenship, as well as the alleged
validity of her Indonesian passport. (Ricardo Cua v. The Board of Immigration
Commissioners, G. R. No. L-9997, May 22, 1957, 101 Phil. 521, 523.)
[Emphasis supplied] .

For emphasis, it is reiterated that in the above two cases, this Court expressly gave the parties
concerned opportunity to prove the fact that they were not suffering from any of the
disqualifications of the law without the need of undergoing any judicial naturalization
proceeding. It may be stated, therefore, that according to the above decisions, the law in this
country, on the matter of the effect of marriage of an alien woman to a Filipino is that she
thereby becomes a Filipina, if it can be proven that at the time of such marriage, she does
not possess any of the disqualifications enumerated in Section 4 of the Naturalization Law,
without the need of submitting to any naturalization proceedings under said law.

It is to be admitted that both of the above decisions made no reference to qualifications, that
is, as to whether or not they need also to be proved, but, in any event, it is a fact that the
Secretary of Justice understood them to mean that such qualifications need not be possessed
nor proven. Then Secretary of Justice Jesus Barrera, who later became a distinguished
member of this Court,6 so ruled in opinions rendered by him subsequent to Ly Giok Ha, the
most illustrative of which held: .
At the outset it is important to note that an alien woman married to a Filipino
citizen needs only to show that she "might herself be lawfully naturalized" in
order to acquire Philippine citizenship. Compliance with other conditions of the
statute, such as those relating to the qualifications of an applicant for
naturalization through judicial proceedings, is not necessary. (See: Leonard v.
Grant, 5 Fed. 11; 27 Ops. Atty. Gen [U.S.] 507; Ops. Sec. of Justice, No. 776,
s. 1940, and No. 111, s. 1953.

This view finds support in the case of Ly Giok Ha et al. v. Galang et al., G.R.
No. L-10760, promulgated May 17, 1957, where the Supreme Court, construing
the abovequoted section of the Naturalization Law, held that "marriage to a
male Filipino does not vest Philippine citizenship to his foreign wife," unless she
"herself may be lawfully naturalized," and that "this limitation of Section 15
excludes, from the benefits of naturalization by marriage, those disqualified
from being naturalized as citizens of the Philippines under Section 4 of said
Commonwealth Act No. 473." In other words, disqualification for any of the
causes enumerated in Section 4 of the Act is the decisive factor that defeats
the right of the foreign wife of a Philippine citizen to acquire Philippine
citizenship.

xxx xxx xxx

Does petitioner, Lim King Bian, belong to any of these groups The
Commissioner of Immigration does not say so but merely predicates his
negative action on the ground that a warrant of deportation for "overstaying"
is pending against the petitioner.

We do not believe the position is well taken. Since the grounds for
disqualification for naturalization are expressly enumerated in the law, a
warrant of deportation not based on a finding of unfitness to become
naturalized for any of those specified causes may not be invoked to negate
acquisition of Philippine citizenship by a foreign wife of a Philippine citizen under
Section 15 of the Naturalization Law. (Inclusio unius est exclusio alterius) (Op.
No. 12, s. 1958 of Justice Undersec. Jesus G. Barrera.)

Regarding the steps that should be taken by an alien woman married to a


Filipino citizen in order to acquire Philippine citizenship, the procedure followed
in the Bureau of Immigration is as follows: The alien woman must file a petition
for the cancellation of her alien certificate of registration alleging, among other
things, that she is married to a Filipino citizen and that she is not disqualified
from acquiring her husband's citizenship pursuant to section 4 of
Commonwealth Act No. 473, as amended. Upon the filing of said petition, which
should be accompanied or supported by the joint affidavit of the petitioner and
her Filipino husband to the effect that the petitioner does not belong to any of
the groups disqualified by the cited section from becoming naturalized Filipino
citizen (please see attached CEB Form 1), the Bureau of Immigration conducts
an investigation and thereafter promulgates its order or decision granting or
denying the petition. (Op. No. 38, s. 19058 of Justice Sec. Jesus G. Barrera.)

This view finds support in the case of Ly Giok Ha et al., v. Galang et al. (G.R.
No. L-10760, promulgated May 17, 1957), where the Supreme Court,
construing the above-quoted section in the Revised Naturalization Law, held
that "marriage to a male Filipino does not vest Philippine citizenship to his
foreign wife, unless she herself may be lawfully naturalized," and that "this
limitation of Section 15 excludes, from the benefits of naturalization by
marriage, those disqualified from being naturalized as citizens of the Philippines
under Section 4 of said Commonwealth Act No. 473." In other words,
disqualification for any of the causes enumerated in section 4 of the Act is the
decisive factor that defeats the right of an alien woman married to a Filipino
citizen to acquire Philippine citizenship. (Op. 57, s. 1958 of Justice Sec. Jesus
G. Barrera.)

The contention is untenable. The doctrine enunciated in the Ly Giok Ha case is


not a new one. In that case, the Supreme Court held that under paragraph I of
Section 15 Of Commonwealth Act No. 473, 'marriage to a male Filipino does
not vest Philippine citizenship to his foreign wife unless she "herself may be
lawfully naturalized"', and, quoting several earlier opinions of the Secretary of
Justice, namely: No. 52, s. 1950; No. 168, s. 1940; No. 95, s. 1941; No. 63,
s. 1948; No. 28. s. 1950, "this limitation of section 15 excludes from the
benefits of naturalization by marriage, those disqualified from being naturalized
as citizens of the Philippines under section 4 of said Commonwealth Act No.
473." (Op. 134, s. 1962 of Justice Undersec. Magno S. Gatmaitan.)

It was not until more than two years later that, in one respect, the above construction of the
law was importantly modified by this Court in Lee Suan Ay, supra, in which the facts were as
follows:

Upon expiration of the appellant Lee Suan Ay's authorized period of temporary
stay in the Philippines (25 March 1955), on 26 March 1955 the Commissioner
of Immigration asked the bondsman to present her to the Bureau of
Immigration within 24 hours from receipt of notice, otherwise the bond will be
confiscated(Annex 1). For failure of the bondsman to comply with the foregoing
order, on 1 April 1955. the Commissioner of Immigration ordered the cash bond
confiscated (Annex E). Therefore, there was an order issued by the
Commissioner of Immigration confiscating or forfeiting the cash bond. Unlike
in forfeiture of bail bonds in criminal proceedings, where the Court must enter
an order forfeiting the bail bond and the bondsman must be given an
opportunity to present his principal or give a satisfactory reason for his inability
to do so, before final judgment may be entered against the bondsman,(section
15, Rule 110; U.S. v. Bonoan, 22 Phil. 1.) in forfeiture of bonds posted for the
temporary stay of an alien in the Philippines, no court proceeding is necessary.
Once a breach of the terms and conditions of the undertaking in the bond is
committed, the Commissioner of Immigration may, under the terms and
conditions thereof, declare it forfeited in favor of the Government. (In the
meanwhile, on April 1, 1955, Lee Suan Ay and Alberto Tan, a Filipino, were
joined in marriage by the Justice of the Peace of Las Piñas, Rizal.)

Mr. Justice Sabino Padilla speaking for a unanimous court which included Justices Concepcion
and Reyes who had penned Ly Giok Ha, and Ricardo Cua, ruled thus:

The fact that Lee Suan Ay (a Chinese) was married to a Filipino citizen does not
relieve the bondsman from his liability on the bond. The marriage took place
on 1 April 1955, and the violation of the terms and conditions of the undertaking
in the bond — failure to depart from the Philippines upon expiration of her
authorized period of temporary stay in the Philippines (25 March 1955) and
failure to report to the Commissioner of Immigration within 24 hours from
receipt of notice — were committed before the marriage. Moreover, the
marriage of a Filipino citizen to an alien does not automatically confer Philippine
citizenship upon the latter. She must possess the qualifications required by law
to become a Filipino citizen by naturalization.* There is no showing that the
appellant Lee Suan Ay possesses all the qualifications and none of the
disqualifications provided for by law to become a Filipino citizen by
naturalization.

Pertinently to be noted at once in this ruling, which, to be sure, is the one relied upon in the
appealed decision now before Us, is the fact that the footnote of the statement therein that
the alien wife "must possess the qualifications required by law to become a Filipino citizen by
naturalization" makes reference to Section 15, Commonwealth Act 473 and precisely, also
to Ly Giok Ha v. Galang, supra. As will be recalled, on the other hand, in the opinions of the
Secretary of Justice explicitly adopted by the Court in Ly Giok Ha, among them, Opinion No.
176, Series of 1940, above-quoted, it was clearly held that "(I)n a previous opinion rendered
for your Office, I stated that the clause "who might herself be lawfully naturalized", should be
construed as not requiring the woman to have the qualifications of residence, good character,
etc., as in cases of naturalization by judicial proceedings but merely that she is of the race by
persons who may be naturalized. (Op. No. 79, s. 1940)

Since Justice Padilla gave no reason at all for the obviously significant modification of the
construction of the law, it could be said that there was need for clarification of the seemingly
new posture of the Court. The occasion for such clarification should have been in Kua Suy,
etc., et al. vs. The Commissioner of Immigration, G.R. No. L-13790, October 31, 1963,
penned by Mr. Justice J.B.L. Reyes, who had rendered the opinion in Ricardo Cua, supra,
which followed that in Ly Giok Ha, supra, but apparently seeing no immediate relevancy in
the case on hand then of the particular point in issue now, since it was not squarely raised
therein similarly as in Lee Suan Ay, hence, anything said on the said matter would at best be
no more than obiter dictum, Justice Reyes limited himself to holding that "Under Section 15
of the Naturalization Act, the wife is deemed a citizen of the Philippines only if she "might
herself be lawfully naturalized," so that the fact of marriage to a citizen, by itself alone, does
not suffice to confer citizenship, as this Court has previously ruled in Ly Giok Ha v. Galang,
54 O.G. 356, and in Cua v. Board of Immigration Commissioners, 53 O.G. 8567; and there is
here no evidence of record as to the qualifications or absence of disqualifications of appellee
Kua Suy", without explaining the apparent departure already pointed out from Ly Giok Ha and
Ricardo Cua. Even Justice Makalintal, who wrote a separate concurring and dissenting opinion
merely lumped together Ly Giok Ha, Ricardo Cua and Lee Suan Ay and opined that both
qualifications and non-disqualifications have to be shown without elucidating on what seemed
to be departure from the said first two decisions.

It was only on November 30, 1963 that to Mr. Justice Roberto Regala fell the task of
rationalizing the Court's position. In Lo San Tuang v. Galang, G.R. No. L-18775, November
30, 1963, 9 SCRA 638, the facts were simply these: Lo San Tuang, a Chinese woman, arrived
in the Philippines on July 1, 1960 as a temporary visitor with authority to stay up to June 30,
1961. She married a Filipino on January 7, 1961, almost six months before the expiry date of
her permit, and when she was requested to leave after her authority to stay had expired, she
refused to do so, claiming she had become a Filipina by marriage, and to bolster her position,
she submitted an affidavit stating explicitly that she does not possess any of the
disqualifications enumerated in the Naturalization Law, Commonwealth Act 473. When the
case reached the court, the trial judge held for the government that in addition to not having
any of the disqualifications referred to, there was need that Lo San Tuang should have also
possessed all the qualifications of residence, moral character, knowledge of a native principal
dialect, etc., provided by the law. Recognizing that the issue squarely to be passed upon was
whether or not the possession of all the qualifications were indeed needed to be shown apart
from non-disqualification, Justice Regala held affirmatively for the Court, reasoning out thus:
.

It is to be noted that the petitioner has anchored her claim for citizenship on
the basis of the decision laid down in the case of Leonard v. Grant, 5 Swy. 603,
5 F 11, where the Circuit Court of Oregon held that it was only necessary that
the woman "should be a person of the class or race permitted to be naturalized
by existing laws, and that in respect of the qualifications arising out of her
conduct or opinions, being the wife of a citizen, she is to be regarded as
qualified for citizenship, and therefore considered a citizen." (In explanation of
its conclusion, the Court said: "If, whenever during the life of the woman or
afterwards, the question of her citizenship arises in a legal proceeding, the
party asserting her citizenship by reason of her marriage with a citizen must
not only prove such marriage, but also that the woman then possessed all the
further qualifications necessary to her becoming naturalized under existing
laws, the statute will be practically nugatory, if not a delusion and a share. The
proof of the facts may have existed at the time of the marriage, but years after,
when a controversy arises upon the subject, it may be lost or difficult to find.")

In other words, all that she was required to prove was that she was a free white
woman or a woman of African descent or nativity, in order to be deemed an
American citizen, because, with respect to the rest of the qualifications on
residence, moral character, etc., she was presumed to be qualified.

Like the law in the United States, our former Naturalization Law (Act No. 2927,
as amended by Act No. 3448) specified the classes of persons who alone might
become citizens of the Philippines, even as it provided who were disqualified.
Thus, the pertinent provisions of that law provided:

Section 1. Who may become Philippine citizens — Philippine


citizenship may be acquired by (a) natives of the Philippines who
are not citizens thereof under the Jones Law; (b) natives of the
Insular possessions of the United States; (c) citizens of the
United States, or foreigners who under the laws of the United
States may become citizens of said country if residing therein.

Section 2. Who are disqualified. — The following cannot be


naturalized as Philippine citizens: (a) Persons opposed to
organized government or affiliated with any association or group
of persons who uphold and teach doctrines opposing all
organized government; (b) persons defending or teaching the
necessity or propriety of violence, personal assault or
assassination for the success and predominance of their ideas;
(c) polygamists or believers in the practice of polygamy; (d)
persons convicted of crimes involving moral turpitude; (e)
persons suffering from mental alienation or incurable contagious
diseases; (f) citizens or subjects of nations with whom the United
States and the Philippines are at war, during the period of such
war.

Section 3. Qualifications. — The persons comprised in subsection


(a) of section one of this Act, in order to be able to acquire
Philippine citizenship, must be not less than twenty-one years of
age on the day of the hearing of their petition.

The persons comprised in subsections (b) and (c) of said section


one shall, in addition to being not less than twenty-one years of
age on the day of the hearing of the petition, have all and each
of the following qualifications:

First. Residence in the Philippine Islands for a continuous period


of not less than five years, except as provided in the next
following section;

Second. To have conducted themselves in a proper and


irreproachable manner during the entire period of their residence
in the Philippine Islands, in their relation with the constituted
government as well as with the community in which they are
living;

Third. To hold in the Philippine Islands real estate worth not less
than one thousand pesos, Philippine currency, or have some
known trade or profession; and

Fourth. To speak and write English, Spanish, or some native


tongue.

In case the petitioner is a foreign subject, he shall, besides,


declare in writing and under oath his intention of renouncing
absolutely and perpetually all faith and allegiance to the foreign
authority, state or sovereignty of which he was a native, citizen
or subject.

Applying the interpretation given by Leonard v. Grant supra, to our law as it


then stood, alien women married to citizens of the Philippines must, in order to
be deemed citizens of the Philippines, be either (1) natives of the Philippines
who were not citizens thereof under the Jones Law, or (2) natives of other
Insular possessions of the United States, or (3) citizens of the United States or
foreigners who under the laws of the United States might become citizens of
that country if residing therein. With respect to the qualifications set forth in
Section 3 of the former law, they were deemed to have the same for all intents
and purposes.

But, with the approval of the Revised Naturalization Law (Commonwealth Act
No. 473) on June 17, 1939, Congress has since discarded class or racial
consideration from the qualifications of applicants for naturalization (according
to its proponent, the purpose in eliminating this consideration was, first, to
remove the features of the existing naturalization act which discriminated in
favor of the Caucasians and against Asiatics who are our neighbors, and are
related to us by racial affinity and, second, to foster amity with all nations
[Sinco, Phil. Political Law 502 — 11 ed.]), even as it retained in Section 15 the
phrase in question. The result is that the phrase "who might herself be lawfully
naturalized" must be understood in the context in which it is now found, in a
setting so different from that in which it was found by the Court in Leonard v.
Grant.

The only logical deduction from the elimination of class or racial consideration
is that, as the Solicitor General points out, the phrase "who might herself be
lawfully naturalized" must now be understood as referring to those who under
Section 2 of the law are qualified to become citizens of the Philippines.

There is simply no support for the view that the phrase "who might herself be
lawfully naturalized" must now be understood as requiring merely that the alien
woman must not belong to the class of disqualified persons under Section 4 of
the Revised Naturalization Law. Such a proposition misreads the ruling laid
down in Leonard v. Grant. A person who is not disqualified is not necessarily
qualified to become a citizen of the Philippines, because the law treats
"qualifications" and "disqualifications" in separate sections. And then it must
not be lost sight of that even under the interpretation given to the former law,
it was to be understood that the alien woman was not disqualified under Section
2 of that law. Leonard v. Grant did not rule that it was enough if the alien
woman does not belong to the class of disqualified persons in order that she
may be deemed to follow the citizenship of her husband: What that case held
was that the phrase "who might herself be lawfully naturalized, merely means
that she belongs to the class or race of persons qualified to become citizens by
naturalization — the assumption being always that she is not otherwise
disqualified.

We therefore hold that under the first paragraph of Section 15 of the


Naturalization Law, an alien woman, who is married to a citizen of the
Philippines, acquires the citizenship of her husband only if she has all the
qualifications and none of the disqualifications provided by law. Since there is
no proof in this case that petitioner has all the qualifications and is not in any
way disqualified, her marriage to a Filipino citizen does not automatically make
her a Filipino citizen. Her affidavit to the effect that she is not in any way
disqualified to become a citizen of this country was correctly disregarded by the
trial court, the same being self-serving.

Naturally, almost a month later in Sun Peck Yong v. Commissioner of Immigration, G.R. No.
L-20784, December 27, 1963, 9 SCRA 875, wherein the Secretary of Foreign Affairs reversed
a previous resolution of the preceding administration to allow Sun Peck Yong and her minor
son to await the taking of the oath of Filipino citizenship of her husband two years after the
decision granting him nationalization and required her to leave and this order was contested
in court, Justice Barrera held:

In the case of Lo San Tuang v. Commissioner of Immigration (G.R. No. L-


18775, promulgated November 30, 1963; Kua Suy vs. Commissioner of
Immigration, L-13790, promulgated October 31, 1963), we held that the fact
that the husband became a naturalized citizen does not automatically make the
wife a citizen of the Philippines. It must also be shown that she herself
possesses all the qualifications, and none of the disqualifications, to become a
citizen. In this case, there is no allegation, much less showing, that petitioner-
wife is qualified to become a Filipino citizen herself. Furthermore, the fact that
a decision was favorably made on the naturalization petition of her husband is
no assurance that he (the husband) would become a citizen, as to make a basis
for the extension of her temporary stay.

On the same day, in Tong Siok Sy v. Vivo, G.R. No. L-21136, December 27, 1963, 9 SCRA
876, Justice Barrera reiterated the same ruling and citing particularly Lo San Tuang and Kua
Suy, held that the marriage of Tong Siok Sy to a Filipino on November 12, 1960 at Taichung,
Taiwan and her taking oath of Filipino citizenship before the Philippine Vice-Consul at Taipeh,
Taiwan on January 6, 1961 did not make her a Filipino citizen, since she came here only in
1961 and obviously, she had not had the necessary ten-year residence in the Philippines
required by the law.

Such then was the status of the jurisprudential law on the matter under discussion when
Justice Makalintal sought a reexamination thereof in Choy King Tee v. Galang, G.R. No. L-
18351, March 26, 1965, 13 SCRA 402. Choy King Tee's husband was granted Philippine
citizenship on January 13, 1959 and took the oath on January 31 of the same year. Choy King
Tee first came to the Philippines in 1955 and kept commuting between Manila and Hongkong
since then, her last visa before the case being due to expire on February 14, 1961. On January
27, 1961, her husband asked the Commissioner of Immigration to cancel her alien certificate
of registration, as well as their child's, for the reason that they were Filipinos, and when the
request was denied as to the wife, a mandamus was sought, which the trial court granted.
Discussing anew the issue of the need for qualifications, Justice Makalintal not only reiterated
the arguments of Justice Regala in Lo San Tuang but added further that the ruling is believed
to be in line with the national policy of selective admission to Philippine citizenship. 7

No wonder, upon this authority, in Austria v. Conchu, G.R. No. L-20716, June 22, 1965, 14
SCRA 336, Justice J.P. Bengzon readily reversed the decision of the lower court granting the
writs of mandamus and prohibition against the Commissioner of Immigration, considering
that Austria's wife, while admitting she did not possess all the qualifications for naturalization,
had submitted only an affidavit that she had none of the disqualifications therefor. So also did
Justice Dizon similarly hold eight days later in Brito v. Commissioner, G.R. No. L-16829, June
30, 1965, 14 SCRA 539.

Then came the second Ly Giok Ha case8 wherein Justice J. B. L. Reyes took occasion to expand
on the reasoning of Choy King Tee by illustrating with examples "the danger of relying
exclusively on the absence of disqualifications, without taking into account the other
affirmative requirements of the law."9

Lastly, in Go Im Ty v. Republic, G.R. No. L-17919, decided on July 30, 1966, 10 Justice
Zaldivar held for the Court that an alien woman who is widowed during the dependency of
the naturalization proceedings of her husband, in order that she may be allowed to take the
oath as Filipino, must, aside from proving compliance with the requirements of Republic Act
530, show that she possesses all the qualifications and does not suffer from any of the
disqualifications under the Naturalization Law, citing in the process the decision to such effect
discussed above, 11even as he impliedly reversed pro tanto the ruling in Tan Lin v. Republic,
G.R. No. L-13786, May 31, 1961, 2 SCRA 383.

Accordingly, in Burca, Justice Sanchez premised his opinion on the assumption that the point
now under discussion is settled law.
In the case now at bar, the Court is again called upon to rule on the same issue. Under Section
15 of the Naturalization Law, Commonwealth Act 473, providing that:

SEC. 15. Effect of the naturalization on wife and children. — Any woman, who
is now or may hereafter be married to a citizen of the Philippines, and who
might herself be lawfully naturalized shall be deemed a citizen of the
Philippines.

Minor children of persons naturalized under this law who have been born in the
Philippines shall be considered citizens thereof.

A foreign-born minor child, if dwelling in the Philippines at the time of the


naturalization of the parent, shall automatically become a Philippine citizen,
and a foreign-born child, who is not in the Philippines at the time the parent is
naturalized, shall be deemed a Philippine citizen only during his minority, unless
he begins to reside permanently in the Philippines when still a minor, in which
case, he will continue to be a Philippine citizen even after becoming of age.

A child born outside of the Philippines after the naturalization of his parent,
shall be considered a Philippine citizen unless within one year after reaching
the age of majority he fails to register himself as a Philippine citizen at the
American Consulate of the country where he resides, and to take the necessary
oath of allegiance.

is it necessary, in order that an alien woman who marries a Filipino or who is married to a
man who subsequently becomes a Filipino, may become a Filipino citizen herself, that, aside
from not suffering from any of the disqualifications enumerated in the law, she must also
possess all the qualifications required by said law? if nothing but the unbroken line from Lee
Suan Ay to Go Im Ty, as recounted above, were to be considered, it is obvious that an
affirmative answer to the question would be inevitable, specially, if it is noted that the present
case was actually submitted for decision on January 21, 1964 yet, shortly after Lo San Tuang,
Tong Siok Sy and Sun Peck Yong, all supra, and even before Choy King Tee, supra, were
decided. There are other circumstances, however, which make it desirable, if not necessary,
that the Court take up the matter anew. There has been a substantial change in the
membership of the Court since Go Im Ty, and of those who were in the Court already when
Burca was decided, two members, Justice Makalintal and Castro concurred only in the result,
precisely, according to them, because (they wanted to leave the point now under discussion
open in so far as they are concerned. 12 Truth to tell, the views and arguments discussed at
length with copious relevant authorities, in the motion for reconsideration as well as in the
memorandum of the amici curae 13 in the Burca case cannot just be taken lightly and
summarily ignored, since they project in the most forceful manner, not only the legal and
logical angles of the issue, but also the imperative practical aspects thereof in the light of the
actual situation of the thousands of alien wives of Filipinos who have so long, even decades,
considered themselves as Filipinas and have always lived and acted as such, officially or
otherwise, relying on the long standing continuous recognition of their status as such by the
administrative authorities in charge of the matter, as well as by the courts. Under these
circumstances, and if only to afford the Court an opportunity to consider the views of the five
justices who took no part in Go Im Ty (including the writer of this opinion), the Court decided
to further reexamine the matter. After all, the ruling first laid in Lee Suan Ay, and later in Lo
San Tuang, Choy King Tee stand the second (1966) Ly Giok Ha, did not categorically repudiate
the opinions of the Secretary of Justice relied upon by the first (1959) Ly Giok Ha. Besides,
some points brought to light during the deliberations in this case would seem to indicate that
the premises of the later cases can still bear further consideration.

Whether We like it or not, it is undeniably factual that the legal provision We are construing,
Section 15, aforequoted, of the Naturalization Law has been taken directly, copied and
adopted from its American counterpart. To be more accurate, said provision is nothing less
than a reenactment of the American provision. A brief review of its history proves this beyond
per adventure of doubt.

The first Naturalization Law of the Philippines approved by the Philippine Legislature under
American sovereignty was that of March 26, 1920, Act No. 2927. Before then, as a
consequence of the Treaty of Paris, our citizenship laws were found only in the Organic Laws,
the Philippine Bill of 1902, the Act of the United States Congress of March 23, 1912 and later
the Jones Law of 1916. In fact, Act No. 2927 was enacted pursuant to express authority
granted by the Jones Law. For obvious reasons, the Philippines gained autonomy on the
subjects of citizenship and immigration only after the effectivity of the Philippine
Independence Act. This made it practically impossible for our laws on said subject to have
any perspective or orientation of our own; everything was American.

The Philippine Bill of 1902 provided pertinently: .

SECTION 4. That all inhabitants of the Philippine Islands continuing to reside


herein who were Spanish subjects on the eleventh day of April, eighteen-
hundred and ninety-nine, and then resided in said Islands, and their children
born subsequent thereto, shall be deemed and held to be citizens of the
Philippine Islands and as such entitled to the protection of the United States,
except such as shall have elected to preserve their allegiance to the Crown of
Spain in accordance with the provisions of the treaty of peace between the
United States and Spain signed at Paris December tenth, eighteen hundred and
ninety-eight.

This Section 4 of the Philippine Bill of 1902 was amended by Act of Congress of March 23,
1912, by adding a provision as follows:

Provided, That the Philippine Legislature is hereby authorized to provide by law


for the acquisition of Philippine citizenship by those natives of the Philippine
Islands who do not come within the foregoing provisions, the natives of other
insular possessions of the United States, and such other persons residing in the
Philippine Islands who would become citizens of the United States, under the
laws of the United States, if residing therein.

The Jones Law reenacted these provisions substantially: .

SECTION 2. That all inhabitants of the Philippine Islands who were Spanish
subjects on the eleventh day of April, eighteen hundred and ninety-nine, and
then resided in said islands, and their children born subsequent thereto, shall
be deemed and held to be citizens of the Philippine Islands, except such as shall
have elected to preserve their allegiance to the Crown of Spain in accordance
with the provisions of the treaty of peace between the United States and Spain,
signed at Paris December tenth, eighteen hundred and ninety-eight and except
such others as have since become citizens of some other country: Provided,
That the Philippine Legislature, herein provided for, is hereby authorized to
provide by law for the acquisition of Philippine citizenship by those natives of
the Philippine Islands who do not come within the foregoing provisions, the
natives of the insular possessions of the United States, and such other persons
residing in the Philippine Islands who are citizens of the United States under
the laws of the United States if residing therein.

For aught that appears, there was nothing in any of the said organic laws regarding the effect
of marriage to a Filipino upon the nationality of an alien woman, albeit under the Spanish Civil
Code provisions on citizenship, Articles 17 to 27, which were, however, abrogated upon the
change of sovereignty, it was unquestionable that the citizenship of the wife always followed
that of the husband. Not even Act 2927 contained any provision regarding the effect of
naturalization of an alien, upon the citizenship of his alien wife, nor of the marriage of such
alien woman with a native born Filipino or one who had become a Filipino before the marriage,
although Section 13 thereof provided thus: .

SEC. 13. Right of widow and children of petitioners who have died. — In case
a petitioner should die before the final decision has been rendered, his widow
and minor children may continue the proceedings. The decision rendered in the
case shall, so far as the widow and minor children are concerned, produce the
same legal effect as if it had been rendered during the life of the petitioner.

It was not until November 30, 1928, upon the approval of Act 3448, amending Act 2977, that
the following provisions were added to the above Section 13:

SECTION 1. The following new sections are hereby inserted between sections
thirteen and fourteen of Act Numbered Twenty-nine hundred and Twenty-
seven:

SEC. 13(a). Any woman who is now or may hereafter be married


to a citizen of the Philippine Islands and who might herself be
lawfully naturalized, shall be deemed a citizen of the Philippine
Islands.

SEC. 13(b). Children of persons who have been duly naturalized


under this law, being under the age of twenty-one years at the
time of the naturalization of their parents, shall, if dwelling in the
Philippine Islands, be considered citizens thereof.

SEC. 13(c). Children of persons naturalized under this law who


have been born in the Philippine Islands after the naturalization
of their parents shall be considered citizens thereof.

When Commonwealth Act 473, the current naturalization law, was enacted on June 17, 1939,
the above Section 13 became its Section 15 which has already been quoted earlier in this
decision. As can be seen, Section 13 (a) abovequoted was re-enacted practically word for
word in the first paragraph of this Section 15 except for the change of Philippine Islands to
Philippines. And it could not have been on any other basis than this legislative history of our
naturalization law that each and everyone of the decisions of this Court from the first Ly Giok
Ha to Go Im Ty, discussed above, were rendered.
As stated earlier, in the opinion of Chief Justice Concepcion in the first Ly Giok Ha, it was quite
clear that for an alien woman who marries a Filipino to become herself a Filipino citizen, there
is no need for any naturalization proceeding because she becomes a Filipina ipso facto from
the time of such marriage, provided she does not suffer any of the disqualifications
enumerated in Section 4 of Commonwealth Act 473, with no mention being made of whether
or not the qualifications enumerated in Section 2 thereof need be shown. It was only in Lee
Suan Ay in 1959 that the possession of qualifications were specifically required, but it was not
until 1963, in Lo San Tuang, that Justice Regala reasoned out why the possession of the
qualifications provided by the law should also be shown to be possessed by the alien wife of
a Filipino, for her to become a Filipina by marriage.

As may be recalled, the basic argument advanced by Justice Regala was briefly as follows:
That "like the law in the United States, our Naturalization Law specified the classes of persons
who alone might become citizens, even as it provided who were disqualified," and inasmuch
as Commonwealth Act 473, our Naturalization Law since 1939 did not reenact the section
providing who might become citizens, allegedly in order to remove racial discrimination in
favor of Caucasians and against Asiatics, "the only logical deduction ... is that the phrase "who
might herself be lawfully naturalized" must now be understood as referring to those who
under Section 2 of the law are qualified to become citizens of the Philippines" and "there is
simply no support for the view that the phrase "who might herself be lawfully naturalized"
must now be understood as requiring merely that the alien woman must not belong to the
class of disqualified persons under Section 4 of the Revised Naturalization Law." 14

A similar line of reasoning was followed in Choy King Tee, which for ready reference may be
qouted:

The question has been settled by the uniform ruling of this Court in a number
of cases. The alien wife of a Filipino citizen must first prove that she has all the
qualifications required by Section 2 and none of the disqualifications
enumerated in Section 4 of the Naturalization Law before she may be deemed
a Philippine citizen (Lao Chay v. Galang, L-190977, Oct. 30, 1964, citing Lo San
Tuang v. Galang, L-18775, Nov. 30, 1963; Sun Peck Yong v. Commissioner of
Immigration, L-20784, December 27, 1963; Tong Siok Sy v. Vivo, L-21136,
December 27, 1963). The writer of this opinion has submitted the question
anew to the court for a possible reexamination of the said ruling in the light of
the interpretation of a similar law in the United States after which Section 15
of our Naturalization Law was patterned. That law was section 2 of the Act of
February 10, 1855 (Section 1994 of the Revised Statutes of the U.S.). The local
law, Act No. 3448, was passed on November 30, 1928 as an amendment to the
former Philippine Naturalization Law, Act No. 2927, which was approved on
March 26, 1920. Under this Naturalization Law, acquisition of Philippine
citizenship was limited to three classes of persons, (a) Natives of the Philippines
who were not citizens thereof; (b) natives of the other insular possessions of
the United States; and (c) citizens of the United States, or foreigners who,
under the laws of the United States, may become citizens of the latter country
if residing therein. The reference in subdivision (c) to foreigners who may
become American Citizens is restrictive in character, for only persons of certain
specified races were qualified thereunder. In other words, in so far as racial
restrictions were concerned there was at the time a similarity between the
naturalization laws of the two countries and hence there was reason to accord
here persuasive force to the interpretation given in the United States to the
statutory provision concerning the citizenship of alien women marrying
American citizens.

This Court, however, believes that such reason has ceased to exist since the
enactment of the Revised Naturalization Law, (Commonwealth Act No. 473) on
June 17, 1939. The racial restrictions have been eliminated in this Act, but the
provision found in Act No. 3448 has been maintained. It is logical to presume
that when Congress chose to retain the said provision — that to be deemed a
Philippine citizen upon marriage the alien wife must be one "who might herself
be lawfully naturalized," the reference is no longer to the class or race to which
the woman belongs, for class or race has become immaterial, but to the
qualifications and disqualifications for naturalization as enumerated in Sections
2 and 4 of the statute. Otherwise the requirement that the woman "might
herself be lawfully naturalized" would be meaningless surplusage, contrary to
settled norms of statutory construction.

The rule laid down by this Court in this and in other cases heretofore decided
is believed to be in line with the national policy of selective admission to
Philippine citizenship, which after all is a privilege granted only to those who
are found worthy thereof, and not indiscriminately to anybody at all on the
basis alone of marriage to a man who is a citizen of the Philippines, irrespective
of moral character, ideological beliefs, and identification with Filipino ideals,
customs and traditions.

Appellee here having failed to prove that she has all the qualifications for
naturalization, even, indeed, that she has none of the disqualifications, she is
not entitled to recognition as a Philippine citizen.

In the second Ly Giok Ha, the Court further fortified the arguments in favor of the same
conclusion thus:

On cross-examination, she (Ly Giok Ha) failed to establish that: (1) she has
been residing in the Philippines for a continuous period of at least (10) years
(p. 27, t.s.n., id.); (2) she has a lucrative trade, profession, or lawful
occupation (p. 13, t.s.n., id.); and (3) she can speak and write English, or any
of the principal Philippine languages (pp. 12, 13, t.s.n., id.).

While the appellant Immigration Commissioner contends that the words


emphasized indicate that the present Naturalization Law requires that an alien
woman who marries a Filipino husband must possess the qualifications
prescribed by section 2 in addition to not being disqualified under any of the
eight ("a" to "h") subheadings of section 4 of Commonwealth Act No. 473, in
order to claim our citizenship by marriage, both the appellee and the court
below (in its second decision) sustain the view that all that the law demands is
that the woman be not disqualified under section 4.

At the time the present case was remanded to the court of origin (1960) the
question at issue could be regarded as not conclusively settled, there being
only the concise pronouncement in Lee Suan Ay, et al. v. Galang, G. R. No. L-
11855, Dec. 23, 1959, to the effect that:
The marriage of a Filipino citizen to an alien does not
automatically confer Philippine citizenship upon the latter. She
must possess the qualifications required by law to become a
Filipino citizen by naturalization.

Since that time, however, a long line of decisions of this Court has firmly
established the rule that the requirement of section 15 of Commonwealth Act
473 (the Naturalization Act), that an alien woman married to a citizen should
be one who "might herself be lawfully naturalized," means not only woman free
from the disqualifications enumerated in section 4 of the Act but also one who
possesses the qualifications prescribed by section 2 of Commonwealth Act 473
(San Tuan v. Galang, L-18775, Nov. 30, 1963; Sun Peck Yong v. Com. of
Immigration, L-20784, Dee. 27, 1963; Tong Siok Sy v. Vivo, L-21136, Dec. 27,
1963; Austria v. Conchu, L-20716, June 22, 1965; Choy King Tee v. Galang, L-
18351, March 26, 1965; Brito v. Com. of Immigration, L-16829, June 30,
1965).

Reflection will reveal why this must be so. The qualifications prescribed under
section 2 of the Naturalization Act, and the disqualifications enumerated in its
section 4 are not mutually exclusive; and if all that were to be required is that
the wife of a Filipino be not disqualified under section 4, the result might well
be that citizenship would be conferred upon persons in violation of the policy of
the statute. For example, section 4 disqualifies only —

(c) Polygamists or believers in the practice of polygamy; and

(d) Persons convicted of crimes involving moral turpitude,

so that a blackmailer, or a maintainer of gambling or bawdy houses, not


previously convicted by a competent court would not be thereby disqualified;
still, it is certain that the law did not intend such person to be admitted as a
citizen in view of the requirement of section 2 that an applicant for citizenship
"must be of good moral character."

Similarly, the citizen's wife might be a convinced believer in racial supremacy,


in government by certain selected classes, in the right to vote exclusively by
certain "herrenvolk", and thus disbelieve in the principles underlying the
Philippine Constitution; yet she would not be disqualified under section 4, as
long as she is not "opposed to organized government," nor affiliated to groups
"upholding or teaching doctrines opposing all organized governments", nor
"defending or teaching the necessity or propriety of violence, personal assault
or assassination for the success or predominance of their ideas." Et sic de
caeteris.

The foregoing instances should suffice to illustrate the danger of relying


exclusively on the absence of disqualifications, without taking into account the
other affirmative requirements of the law, which, in the case at bar, the
appellee Ly Giok Ha admittedly does not possess.

As to the argument that the phrase "might herself be lawfully naturalized" was
derived from the U.S. Revised Statutes (section 1994) and should be given the
same territorial and racial significance given to it by American courts, this Court
has rejected the same in Lon San Tuang v. Galang, L-18775, November 30,
1963; and in Choy King Tee v. Galang, L-18351, March 26, 1965.

It is difficult to minimize the persuasive force of the foregoing rationalizations, but a closer
study thereof cannot bat reveal certain relevant considerations which adversely affect the
premises on which they are predicated, thus rendering the conclusions arrived thereby not
entirely unassailable.

1. The main proposition, for instance, that in eliminating Section 1 of Act 2927 providing who
are eligible for Philippine citizenship, the purpose of Commonwealth Act 473, the Revised
Naturalization Law, was to remove the racial requirements for naturalization, thereby opening
the door of Filipino nationality to Asiatics instead of allowing the admission thereto of
Caucasians only, suffers from lack of exact accuracy. It is important to note, to start with,
that Commonwealth Act 473 did away with the whole Section 1 of Act 2927 which reads,
thus:

SECTION 1. Who may become Philippine citizens. — Philippine citizenship may


be acquired by: (a) natives of the Philippines who are not citizens thereof under
the Jones Law; (b) natives of the other Insular possessions of the United
States; (c) citizens of the United States, or foreigners who under the laws of
the United States may become citizens of said country if residing therein.

and not only subdivision (c) thereof. Nowhere in this whole provision was there any mention
of race or color of the persons who were then eligible for Philippine citizenship. What is more
evident from said provision is that it reflected the inevitable subordination of our legislation
during the pre-Commonwealth American regime to the understandable stations flowing from
our staffs as a territory of the United States by virtue of the Treaty of Paris. In fact, Section
1 of Act 2927 was precisely approved pursuant to express authority without which it could
not have been done, granted by an amendment to Section 4 of the Philippine Bill of 1902
introduced by the Act of the United States Congress of March 23, 1912 and which was
reenacted as part of the Jones Law of 1916, the pertinent provisions of which have already
been footed earlier. In truth, therefore, it was because of the establishment of the Philippine
Commonwealth and in the exercise of our legislative autonomy on citizenship matters under
the Philippine Independence Act that Section 1 of Act 2927 was eliminated, 15 and not
purposely to eliminate any racial discrimination contained in our Naturalization Law. The
Philippine Legislature naturally wished to free our Naturalization Law from the impositions of
American legislation. In other words, the fact that such discrimination was removed was one
of the effects rather than the intended purpose of the amendment.

2. Again, the statement in Choy King Tee to the effect that "the reference in subdivision (c)
(of Section 1 of Act 2927) to foreigners who may become American citizens is restrictive in
character, for only persons of certain specified races were qualified thereunder" fails to
consider the exact import of the said subdivision. Explicitly, the thrust of the said subdivision
was to confine the grant under it of Philippine citizenship only to the three classes of persons
therein mentioned, the third of which were citizens of the United States and, corollarily,
persons who could be American citizens under her laws. The words used in the provision do
not convey any idea of favoring aliens of any particular race or color and of excluding others,
but more accurately, they refer to all the disqualifications of foreigners for American
citizenship under the laws of the United States. The fact is that even as of 1906, or long
before 1920, when our Act 2927 became a law, the naturalization, laws of the United States
already provided for the following disqualifications in the Act of the Congress of June 29,
1906:
SEC. 7. That no person who disbelieves in or who is opposed to organized
government, or who is a member of or affiliated with any organization
entertaining and teaching such disbelief in or opposition to organized
government, or who advocates or teaches the duty, necessity, or propriety of
the unlawful assaulting or killing of any officer or officers, either of specific
individuals or of officers generally, of the Government of the United States, or
of any other organized government, because of his or their official character,
or who is a polygamist, shall be naturalized or be made a citizen of the United
States.

and all these disqualified persons were, therefore, ineligible for Philippine citizenship under
Section 1 of Act 2927 even if they happened to be Caucasians. More importantly, as a matter
of fact, said American law, which was the first "Act to Establish a Bureau of Immigration and
Naturalization and to provide for a Uniform Rule for Naturalization of Aliens throughout the
United States" contained no racial disqualification requirement, except as to Chinese, the Act
of May 6, 1882 not being among the expressly repealed by this law, hence it is clear that
when Act 2927 was enacted, subdivision (e) of its Section 1 could not have had any
connotation of racial exclusion necessarily, even if it were traced back to its origin in the Act
of the United States Congress of 1912 already mentioned above. 16 Thus, it would seem that
the rationalization in the qouted decisions predicated on the theory that the elimination of
Section 1 of Act 2927 by Commonwealth Act 473 was purposely for no other end than the
abolition of racial discrimination in our naturalization law has no clear factual basis. 17

3. In view of these considerations, there appears to be no cogent reason why the construction
adopted in the opinions of the Secretary of Justice referred to in the first Ly Giok Ha decision
of the Chief Justice should not prevail. It is beyond dispute that the first paragraph of Section
15 of Commonwealth Act 473 is a reenactment of Section 13(a) of Act 2927, as amended by
Act 3448, and that the latter is nothing but an exact copy, deliberately made, of Section 1994
of the Raised Statutes of the United States as it stood before its repeal in 1922. 18 Before such
repeal, the phrase "who might herself be lawfully naturalized" found in said Section 15 had a
definite unmistakable construction uniformly foIlowed in all courts of the United States that
had occasion to apply the same and which, therefore, must be considered, as if it were written
in the statute itself. It is almost trite to say that when our legislators enacted said section,
they knew of its unvarying construction in the United States and that, therefore, in adopting
verbatim the American statute, they have in effect incorporated into the provision, as thus
enacted, the construction given to it by the American courts as well as the Attorney General
of the United States and all administrative authorities, charged with the implementation of
the naturalization and immigration laws of that country. (Lo Cham v. Ocampo, 77 Phil., 635
[1946]; Laxamana v. Baltazar, 92 Phil., 32 [1952]; Hartley v. Commissioner, 295 U.S. 216,
79 L. ed. 1399, 55 S Ct. 756 [19353; Helvering v. Winmill, 305 U.S. 79, 83 L ed. 52, 59 S Ct.
45 [1938]; Helvering v. R. J. Reynolds Tobacco Co., 306 U.S. 110, 83 L ed. 536, 59 S Ct. 423
[1939]. [p. 32, Memo of Amicus Curiae]).

A fairly comprehensive summary of the said construction by the American courts and
administrative authorities is contained in United States of America ex rel. Dora Sejnensky v.
Robert E. Tod, Commissioner of Immigration, Appt., 295 Fed. 523, decided November 14,
1922, 26 A. L. R. 1316 as follows:

Section 1994 of the Revised Statutes (Comp. Stat. 3948, 2 Fed. Sta. Anno. 2d
ed. p. 117) provides as follows: "Any woman who is now or may hereafter be
married to a citizen of the United States, and who might herself be lawfully
naturalized, shall be deemed a citizen."
Section 1944 of the Revised Stat. is said to originate in the Act of Congress of
February 10, 1855 (10 Stat. at L. 604, chap. 71), which in its second section
provided "that any woman, who might lawfully be naturalized under the existing
laws, married, or who shall be married to a citizen of the United States, shall
be deemed and taken to be a citizen."

And the American Statute of 1855 is substantially a copy of the earlier British
Statute 7 & 8 Vict. chap. 66, s 16, 1844, which provided that "any woman
married, or who shall be married, to a natural-born subject or person
naturalized, shall be deemed and taken to be herself naturalized, and have all
the rights and privileges of a natural born subject."

The Act of Congress of September 22, 1922 (42 Stat. at L. 1021, chap. 411,
Comp. Stat. 4358b, Fed. Stat. Anno. Supp. 1922, p. 255), being "An Act
Relative to the Naturalization and Citizenship of Married Women," in 2, provides
"that any woman who marries a citizen of the United States after the passage
of this Act, ... shall not become a citizen of the United States by reason of such
marriage ..."

Section 6 of the act also provides "that 1994 of the Revised Statutes ... are
repealed."

Section 6 also provides that `such repeal shall not terminate citizenship
acquired or retained under either of such sections, ..." meaning 2 and 6. So
that this Act of September 22, 1922, has no application to the facts of the
present case, as the marriage of the relator took place prior to its passage. This
case, therefore, depends upon the meaning to be attached to 1994 of the
Revised Statutes.

In 1868 the Supreme Court, in Kelly v. Owen, 7 Wall. 496, 498, 19 L. ed. 283,
284, construed this provision as found in the Act of 1855 as follows: "The term,
"who might lawfully be naturalized under the existing laws," only limits the
application of the law to free white women. The previous Naturalization Act,
existing at the time, only required that the person applying for its benefits
should be "a free white person," and not an alien enemy."

This construction limited the effect of the statute to those aliens who belonged
to the class or race which might be lawfully naturalized, and did not refer to
any of the other provisions of the naturalization laws as to residence or moral
character, or to any of the provisions of the immigration laws relating to the
exclusion or deportation of aliens.

In 1880, in Leonard v. Grant (C. C.) 5 Fed. 11, District Judge Deady also
construed the Act of 1855, declaring that "any woman who is now or may
hereafter be married to a citizen of the United States, and might herself be
lawfully naturalized, shall be deemed a citizen." He held that "upon the
authorities, and the reason, if not the necessity, of the case," the statute must
be construed as in effect declaring that an alien woman, who is of the class or
race that may be lawfully naturalized under the existing laws, and who marries
a citizen of the United States, is such a citizen also, and it was not necessary
that it should appear affirmatively that she possessed the other qualifications
at the time of her marriage to entitle her to naturalization.
In 1882, the Act of 1855 came before Mr. Justice Harlan, sitting in the circuit
court, in United States v. Kellar, 13 Fed. 82. An alien woman, a subject of
Prussia came to the United States and married here a naturalized citizen. Mr.
Justice Harlan, with the concurrence of Judge Treat, held that upon her
marriage she became ipso facto a citizen of the United States as fully as if she
had complied with all of the provisions of the statutes upon the subject of
naturalization. He added: "There can be no doubt of this, in view of the decision
of the Supreme Court of the United, States in Kelly v. Owen, 7 Wall. 496, 19 L.
ed. 283." The alien "belonged to the class of persons" who might be lawfully
naturalized.

In 1904, in Hopkins v. Fachant, 65 C. C. A. 1, 130 Fed. 839, an alien woman


came to the United States from France and entered the country contrary to the
immigration laws. The immigration authorities took her into custody at the port
of New York, with the view of deporting her. She applied for her release under
a writ of habeas corpus, and pending the disposition of the matter she married
a naturalized American citizen. The circuit court of appeals for the ninth Circuit
held, affirming the court below, that she was entitled to be discharged from
custody. The court declared: "The rule is well settled that her marriage to a
naturalized citizen of the United States entitled her to be discharged. The status
of the wife follows that of her husband, ... and by virtue of her marriage her
husband's domicil became her domicil." .

In 1908, the circuit court for the district of Rhode Island in Re Rustigian, 165.
Fed. 980, had before it the application of a husband for his final decree of
naturalization. It appeared that at that time his wife was held by the
immigration authorities at New York on the ground that she was afflicted with
a dangerous and contagious disease. Counsel on both sides agreed that the
effect of the husband's naturalization would be to confer citizenship upon the
wife. In view of that contingency District Judge Brown declined to pass upon
the husband's application for naturalization, and thought it best to wait until it
was determined whether the wife's disease was curable. He placed his failure
to act on the express ground that the effect of naturalizing the husband might
naturalize her. At the same time he express his opinion that the husband's
naturalization would not effect her naturalization, as she was not one who could
become lawfully naturalized. "Her own capacity (to become naturalized)," the
court stated "is a prerequisite to her attaining citizenship. If herself lacking in
that capacity, the married status cannot confer it upon her." Nothing, however,
was actually decided in that case, and the views expressed therein are really
nothing more than mere dicta. But, if they can be regarded as something more
than that, we find ourselves, with all due respect for the learned judge, unable
to accept them.

In 1909, in United States ex rel. Nicola v. Williams, 173 Fed. 626, District Judge
Learned Hand held that an alien woman, a subject of the Turkish Empire, who
married an American citizen while visiting Turkey, and then came to the United
States, could not be excluded, although she had, at the time of her entry, a
disease which under the immigration laws would have been sufficient ground
for her exclusion, if she bad not had the status of a citizen. The case was
brought into this court on appeal, and in 1911 was affirmed, in 106 C. C. A.
464, 184 Fed. 322. In that case, however at the time the relators married, they
might have been lawfully naturalized, and we said: "Even if we assume the
contention of the district attorney to be correct that marriage will not make a
citizen of a woman who would be excluded under our immigration laws, it does
not affect these relators."

We held that, being citizens, they could not be excluded as aliens; and it was
also said to be inconsistent with the policy of our law that the husband should
be a citizen and the wife an alien. The distinction between that case and the
one now before the court is that, in the former case, the marriage took place
before any order of exclusion had been made, while in this the marriage was
celebrated after such an order was made. But such an order is a mere
administrative provision, and has not the force of a judgment of a court, and
works no estoppel. The administrative order is based on the circumstances that
existed at the time the order of exclusion was made. If the circumstances
change prior to the order being carried into effect, it cannot be executed. For
example, if an order of exclusion should be based on the ground that the alien
was at the time afflicted with a contagious disease, and it should be made
satisfactorily to appear, prior to actual deportation, that the alien had entirely
recovered from the disease, we think it plain that the order could not be carried
into effect. So, in this case, if, after the making of the order of exclusion and
while she is permitted temporarily to remain, she in good faith marries an
American citizen, we cannot doubt the validity of her marriage, and that she
thereby acquired, under international law and under 1994 of the Revised
Statutes, American citizenship, and ceased to be an alien. There upon, the
immigration authorities lost their jurisdiction over her, as that jurisdiction
applies only to aliens, and not to citizens.

In 1910, District Judge Dodge, in Ex parte Kaprielian, 188 Fed. 694, sustained
the right of the officials to deport a woman under the following circumstances:
She entered this country in July, 1910, being an alien and having been born in
Turkey. She was taken into custody by the immigration authorities in the
following September, and in October a warrant for her deportation was issued.
Pending hearings as to the validity of that order, she was paroled in the custody
of her counsel. The ground alleged for her deportation was that she was
afflicted with a dangerous and contagious disease at the time of her entry. One
of the reasons assigned to defeat deportation was that the woman had married
a citizen of the United States pending the proceedings for her deportation.
Judge Dodge declared himself unable to believe that a marriage under such
circumstances "is capable of having the effect claimed, in view of the facts
shown." He held that it was no part of the intended policy of 1994 to annul or
override the immigration laws, so as to authorize the admission into the country
of the wife of a naturalized alien not otherwise entitled to enter, and that an
alien woman, who is of a class of persons excluded by law from admission to
the United States does not come within the provisions of that section. The court
relied wholly upon the dicta contained in the Rustigian Case. No other
authorities were cited.

In 1914, District Judge Neterer, in Ex parte Grayson, 215 Fed. 449, construed
1994 and held that where, pending proceedings to deport an alien native of
France as an alien prostitute, she was married to a citizen of the United States,
she thereby became a citizen, and was not subject to deportation until her
citizenship was revoked by due process of law. It was his opinion that if, as was
contended, her marriage was conceived in fraud, and was entered into for the
purpose of evading the immigration laws and preventing her deportation, such
fact should be established in a court of competent jurisdiction in an action
commenced for the purpose. The case was appealed and the appeal was
dismissed. 134 C. C. A. 666, 219 Fed. 1022.

It is interesting also to observe the construction placed upon the language of


the statute by the Department of Justice. In 1874, Attorney General Williams,
14 Ops. Atty. Gen. 402, passing upon the Act of February 10, 1855, held that
residence within the United States for the period required by the naturalization
laws was riot necessary in order to constitute an alien woman a citizen, she
having married a citizen of the United States abroad, although she never
resided in the United States, she and her husband having continued to reside
abroad after the marriage.

In 1909, a similar construction was given to the Immigration Act of May 5,


1907, in an opinion rendered by Attorney General Wickersham. It appeared an
unmarried woman, twenty-eight years of age and a native of Belgium, arrived
in New York and went at once to a town in Nebraska, where she continued to
reside. About fifteen months after her arrival she was taken before a United
States commissioner by way of instituting proceedings under the Immigration
Act (34 Stat. at L. 898, chap. 1134, Comp. Stat. 4242, 3 Fed. Stat. Anno. 2d
ed. p. 637) for her deportation, on the ground that she had entered this country
for the purpose of prostitution, and had been found an inmate of a house of
prostitution and practicing the same within three years after landing. It
appeared, however, that after she was taken before the United States
commissioner, but prior to her arrest under a warrant by the Department of
Justice, she was lawfully married to a native-born citizen of the United States.
The woman professed at the time of her marriage an intention to abandon her
previous mode of life and to remove with her husband to his home in
Pennsylvania. He knew what her mode of life had been, but professed to believe
in her good intentions. The question was raised as to the right to deport her,
the claim being advance that by her marriage she bad become an American
citizen and therefore could not be deported. The Attorney General ruled against
the right to deport her as she had become an American citizen. He held that
the words, "who might herself be lawfully naturalized," refer to a class or race
who might be lawfully naturalized, and that compliance with the other
conditions of the naturalization laws was not required. 27 Ops. Atty. Gen. 507.

Before concluding this opinion, we may add that it has not escaped our
observation that Congress, in enacting the Immigration Act of 1917, so as to
provide, in 19, "that the marriage to an American citizen of a female of the
sexually immoral classes ... shall not invest such female with United States
citizenship if the marriage of such alien female shall be solemnized after her
arrest or after the commission of acts which make her liable to deportation
under this act."

Two conclusions seem irresistibly to follow from the above change in the law:

(1) Congress deemed legislation essential to prevent women of the immoral


class avoiding deportation through the device of marrying an American citizen.
(2) If Congress intended that the marriage of an American citizen with an alien
woman of any other of the excluded classes, either before or after her
detention, should not confer upon her American citizenship, thereby entitling
her to enter the country, its intention would have been expressed, and 19 would
not have been confined solely to women of the immoral class.

Indeed, We have examined all the leading American decisions on the subject and We have
found no warrant for the proposition that the phrase "who might herself be lawfully
naturalized" in Section 1994 of the Revised Statutes was meant solely as a racial bar, even if
loose statements in some decisions and other treaties and other writings on the subject would
seem to give such impression. The case of Kelley v. Owen, supra, which appears to be the
most cited among the first of the decisions 19 simply held:

As we construe this Act, it confers the privileges of citizenship upon women


married to citizens of the United States, if they are of the class of persons for
whose naturalization the previous Acts of Congress provide. The terms
"married" or "who shall be married," do not refer in our judgment, to the time
when the ceremony of marriage is celebrated, but to a state of marriage. They
mean that, whenever a woman, who under previous Acts might be naturalized,
is in a state of marriage to a citizen, whether his citizenship existed at the
passage of the Act or subsequently, or before or after the marriage, she
becomes, by that fact, a citizen also. His citizenship, whenever it exists,
confers, under the Act, citizenship upon her. The construction which would
restrict the Act to women whose husbands, at the time of marriage, are citizens,
would exclude far the greater number, for whose benefit, as we think, the Act
was intended. Its object, in our opinion, was to allow her citizenship to follow
that of her husband, without the necessity of any application for naturalization
on her part; and, if this was the object, there is no reason for the restriction
suggested.

The terms, "who might lawfully be naturalized under the existing laws," only
limit the application of the law to free white women. The previous Naturalization
Act, existing at the time only required that the person applying for its benefits
should be "a free white person," and not an alien enemy. Act of April 14th,
1802, 2 Stat. at L. 153.

A similar construction was given to the Act by the Court of Appeals of New York,
in Burton v. Burton, 40 N. Y. 373; and is the one which gives the widest
extension to its provisions.

Note that write the court did say that "the terms, "who might lawfully be naturalized under
existing laws" only limit the application to free white women" 20 it hastened to add that "the
previous Naturalization Act, existing at the time, ... required that the person applying for its
benefits should be (not only) a "free white person" (but also) ... not an alien enemy." This is
simply because under the Naturalization Law of the United States at the time the case was
decided, the disqualification of enemy aliens had already been removed by the Act of July 30,
1813, as may be seen in the corresponding footnote hereof anon. In other words, if in the
case of Kelly v. Owen only the race requirement was mentioned, the reason was that there
was no other non-racial requirement or no more alien enemy disqualification at the time; and
this is demonstrated by the fact that the court took care to make it clear that under the
previous naturalization law, there was also such requirement in addition to race. This is
impotent, since as stated in re Rustigian, 165 Fed. Rep. 980, "The expression used by Mr.
Justice Field, (in Kelly v. Owen) the terms "who might lawfully be naturalized under existing
laws" only limit the application of the law to free white women, must be interpreted in the
application to the special facts and to the incapacities under the then existing laws," (at p.
982) meaning that whether or not an alien wife marrying a citizen would be a citizen was
dependent, not only on her race and nothing more necessarily, but on whether or not there
were other disqualifications under the law in force at the time of her marriage or the
naturalization of her husband.

4. As already stated, in Lo San Tuang, Choy King Tee and the second Ly Giok Ha, the Court
drew the evidence that because Section 1 of Act 2927 was eliminated by Commonwealth Act
473, it follows that in place of the said eliminated section particularly its subdivision (c), being
the criterion of whether or not an alien wife "may be lawfully naturalized," what should be
required is not only that she must not be disqualified under Section 4 but that she must also
possess the qualifications enumerated in Section 2, such as those of age, residence, good
moral character, adherence to the underlying principles of the Philippine Constitution,
irreproachable conduct, lucrative employment or ownership of real estate, capacity to speak
and write English or Spanish and one of the principal local languages, education of children in
certain schools, etc., thereby implying that, in effect, sails Section 2 has been purposely
intended to take the place of Section 1 of Act 2927. Upon further consideration of the proper
premises, We have come, to the conclusion that such inference is not sufficiently justified.

To begin with, nothing extant in the legislative history, which We have already explained
above of the mentioned provisions has been shown or can be shown to indicate that such was
the clear intent of the legislature. Rather, what is definite is that Section 15 is, an exact copy
of Section 1994 of the Revised Statutes of the United States, which, at the time of the approval
of Commonwealth Act 473 had already a settled construction by American courts and
administrative authorities.

Secondly, as may be gleaned from the summary of pertinent American decisions quoted
above, there can be no doubt that in the construction of the identically worded provision in
the Revised Statutes of the United States, (Section 1994, which was taken, from the Act of
February 10, 1855) all authorities in the United States are unanimously agreed that the
qualifications of residence, good moral character, adherence to the Constitution, etc. are not
supposed to be considered, and that the only eligibility to be taken into account is that of the
race or class to which the subject belongs, the conceptual scope of which, We have just
discussed. 21 In the very case of Leonard v. Grant, supra, discussed by Justice Regala in Lo
San Tuang, the explanation for such posture of the American authorities was made thus:

The phrase, "shall be deemed a citizen" in section 1994 Rev. St., or as it was
in the Act of 1855, supra, "shall be deemed and taken to be a citizen" while it
may imply that the person to whom it relates has not actually become a citizen
by ordinary means or in the usual way, as by the judgment of a competent
court, upon a proper application and proof, yet it does not follow that such
person is on that account practically any the less a citizen. The word "deemed"
is the equivalent of "considered" or "judged"; and, therefore, whatever an act
of Congress requires to be "deemed" or "taken" as true of any person or thing,
must, in law, be considered as having been duly adjudged or established
concerning "such person or thing, and have force and effect accordingly. When,
therefore, Congress declares that an alien woman shall, under certain
circumstances, be "deemed' an American citizen, the effect when the
contingency occurs, is equivalent to her being naturalized directly by an act of
Congress, or in the usual mode thereby prescribed.
Unless We disregard now the long settled familiar rule of statutory construction that in a
situation like this wherein our legislature has copied an American statute word for word, it is
understood that the construction already given to such statute before its being copied
constitute part of our own law, there seems to be no reason how We can give a different
connotation or meaning to the provision in question. At least, We have already seen that the
views sustaining the contrary conclusion appear to be based on in accurate factual premises
related to the real legislative background of the framing of our naturalization law in its present
form.

Thirdly, the idea of equating the qualifications enumerated in Section 2 of Commonwealth Act
473 with the eligibility requirements of Section 1 of Act 2927 cannot bear close scrutiny from
any point of view. There is no question that Section 2 of Commonwealth Act 473 is more or
less substantially the same as Section 3 of Act 2927. In other words, Section 1 of Act 2927
co-existed already with practically the same provision as Section 2 of Commonwealth Act 473.
If it were true that the phrase "who may be lawfully naturalized" in Section 13 (a) of Act 2927,
as amended by Act 3448, referred to the so-called racial requirement in Section 1 of the same
Act, without regard to the provisions of Section 3 thereof, how could the elimination of Section
1 have the effect of shifting the reference to Section 3, when precisely, according to the
American jurisprudence, which was prevailing at the time Commonwealth Act 473 was
approved, such qualifications as were embodied in said Section 3, which had their counterpart
in the corresponding American statutes, are not supposed to be taken into account and that
what should be considered only are the requirements similar to those provided for in said
Section 1 together with the disqualifications enumerated in Section 4?

Fourthly, it is difficult to conceive that the phrase "who might be lawfully naturalized" in
Section 15 could have been intended to convey a meaning different than that given to it by
the American courts and administrative authorities. As already stated, Act 3448 which
contained said phrase and from which it was taken by Commonwealth Act 473, was enacted
in 1928. By that, time, Section 1994 of the Revised Statutes of the United States was no
longer in force because it had been repealed expressly the Act of September 22, 1922 which
did away with the automatic naturalization of alien wives of American citizens and required,
instead, that they submit to regular naturalization proceedings, albeit under more liberal
terms than those of other applicants. In other words, when our legislature adopted the phrase
in question, which, as already demonstrated, had a definite construction in American law, the
Americans had already abandoned said phraseology in favor of a categorical compulsion for
alien wives to be natural judicially. Simple logic would seem to dictate that, since our
lawmakers, at the time of the approval of Act 3448, had two choices, one to adopt the
phraseology of Section 1994 with its settled construction and the other to follow the new
posture of the Americans of requiring judicial naturalization and it appears that they have
opted for the first, We have no alternative but to conclude that our law still follows the old or
previous American Law On the subject. Indeed, when Commonwealth Act 473 was approved
in 1939, the Philippine Legislature, already autonomous then from the American Congress,
had a clearer chance to disregard the old American law and make one of our own, or, at least,
follow the trend of the Act of the U.S. Congress of 1922, but still, our legislators chose to
maintain the language of the old law. What then is significantly important is not that the
legislature maintained said phraseology after Section 1 of Act 2927 was eliminated, but that
it continued insisting on using it even after the Americans had amended their law in order to
provide for what is now contended to be the construction that should be given to the phrase
in question. Stated differently, had our legislature adopted a phrase from an American statute
before the American courts had given it a construction which was acquiesced to by those
given upon to apply the same, it would be possible for Us to adopt a construction here different
from that of the Americans, but as things stand, the fact is that our legislature borrowed the
phrase when there was already a settled construction thereof, and what is more, it appears
that our legislators even ignored the modification of the American law and persisted in
maintaining the old phraseology. Under these circumstances, it would be in defiance of reason
and the principles of Statutory construction to say that Section 15 has a nationalistic and
selective orientation and that it should be construed independently of the previous American
posture because of the difference of circumstances here and in the United States. It is always
safe to say that in the construction of a statute, We cannot fall on possible judicial fiat or
perspective when the demonstrated legislative point of view seems to indicate otherwise.

5. Viewing the matter from another angle, there is need to emphasize that in reality and in
effect, the so called racial requirements, whether under the American laws or the Philippine
laws, have hardly been considered as qualifications in the same sense as those enumerated
in Section 3 of Act 2927 and later in Section 2 of Commonwealth Act 473. More accurately,
they have always been considered as disqualifications, in the sense that those who did not
possess them were the ones who could not "be lawfully naturalized," just as if they were
suffering from any of the disqualifications under Section 2 of Act 2927 and later those under
Section 4 of Commonwealth Act 473, which, incidentally, are practically identical to those in
the former law, except those in paragraphs (f) and (h) of the latter. 22 Indeed, such is the
clear impression anyone will surely get after going over all the American decisions and
opinions quoted and/or cited in the latest USCA (1970), Title 8, section 1430, pp. 598-602,
and the first decisions of this Court on the matter, Ly Giok Ha (1959) and Ricardo Cua, citing
with approval the opinions of the secretary of Justice. 23 Such being the case, that is, that the
so-called racial requirements were always treated as disqualifications in the same light as the
other disqualifications under the law, why should their elimination not be viewed or
understood as a subtraction from or a lessening of the disqualifications? Why should such
elimination have instead the meaning that what were previously considered as irrelevant
qualifications have become disqualifications, as seems to be the import of the holding in Choy
King Tee to the effect that the retention in Section 15 of Commonwealth Act 473 of the same
language of what used to be Section 13 (a) of Act 2927 (as amended by Act 3448),
notwithstanding the elimination of Section 1 of the latter, necessarily indicates that the
legislature had in mind making the phrase in question "who may be lawfully naturalized" refer
no longer to any racial disqualification but to the qualification under Section 2 of
Commonwealth Act 473? Otherwise stated, under Act 2927, there were two groups of persons
that could not be naturalized, namely, those falling under Section 1 and those falling under
Section 2, and surely, the elimination of one group, i.e. those belonging to Section 1, could
not have had, by any process of reasoning, the effect of increasing, rather than decreasing,
the disqualifications that used to be before such elimination. We cannot see by what alchemy
of logic such elimination could have convicted qualifications into disqualifications specially in
the light of the fact that, after all, these are disqualifications clearly set out as such in the law
distinctly and separately from qualifications and, as already demonstrated, in American
jurisprudence, qualifications had never been considered to be of any relevance in determining
"who might be lawfully naturalized," as such phrase is used in the statute governing the status
of alien wives of American citizens, and our law on the matter was merely copied verbatim
from the American statutes.

6. In addition to these arguments based on the applicable legal provisions and judicial
opinions, whether here or in the United States, there are practical considerations that militate
towards the same conclusions. As aptly stated in the motion for reconsideration of counsel
for petitioner-appellee dated February 23, 1967, filed in the case of Zita Ngo Burca v.
Republic, supra:

Unreasonableness of requiring alien wife to prove "qualifications" —


There is one practical consideration that strongly militates against a
construction that Section 15 of the law requires that an alien wife of a Filipino
must affirmatively prove that she possesses the qualifications prescribed under
Section 2, before she may be deemed a citizen. Such condition, if imposed upon
an alien wife, becomes unreasonably onerous and compliance therewith
manifestly difficult. The unreasonableness of such requirement is shown by the
following:

1. One of the qualifications required of an Applicant for


naturalization under Section 2 of the law is that the applicant
"must have resided in the Philippines for a continuous period of
not less than ten years." If this requirement is applied to an alien
wife married to a Filipino citizen, this means that for a period of
ten years at least, she cannot hope to acquire the citizenship of
her husband. If the wife happens to be a citizen of a country
whose law declares that upon her marriage to a foreigner she
automatically loses her citizenship and acquires the citizenship
of her husband, this could mean that for a period of ten years at
least, she would be stateless. And even after having acquired
continuous residence in the Philippines for ten years, there is no
guarantee that her petition for naturalization will be granted, in
which case she would remain stateless for an indefinite period of
time.

2. Section 2 of the law likewise requires of the applicant for


naturalization that he "must own real estate in the Philippines
worth not less than five thousand pesos, Philippine currency, or
must have some known lucrative trade, profession, or lawful
occupation." Considering the constitutional prohibition against
acquisition by an alien of real estate except in cases of hereditary
succession (Art. XIII, Sec. 5, Constitution), an alien wife desiring
to acquire the citizenship of her husband must have to prove that
she has a lucrative income derived from a lawful trade,
profession or occupation. The income requirement has been
interpreted to mean that the petitioner herself must be the one
to possess the said income. (Uy v. Republic, L-19578, Oct. 27,
1964; Tanpa Ong vs. Republic, L-20605, June 30, 1965; Li Tong
Pek v. Republic, L-20912, November 29, 1965). In other words,
the wife must prove that she has a lucrative income derived from
sources other than her husband's trade, profession or calling. It
is of common knowledge, and judicial notice may be taken of the
fact that most wives in the Philippines do not have gainful
occupations of their own. Indeed, Philippine law, recognizing the
dependence of the wife upon the husband, imposes upon the
latter the duty of supporting the former. (Art. 291, Civil Code).
It should be borne in mind that universally, it is an accepted
concept that when a woman marries, her primary duty is to be a
wife, mother and housekeeper. If an alien wife is not to be remiss
in this duty, how can she hope to acquire a lucrative income of
her own to qualify her for citizenship?
3. Under Section 2 of the law, the applicant for naturalization
"must have enrolled his minor children of school age, in any of
the public schools or private schools recognized by the Office of
the Private Education of the Philippines, where Philippine history,
government and civics are taught or prescribed as part of the
school curriculum during the entire period of residence in the
Philippines required of him prior to the hearing of his petition for
naturalization as Philippine citizen." If an alien woman has minor
children by a previous marriage to another alien before she
marries a Filipino, and such minor children had not been enrolled
in Philippine schools during her period of residence in the
country, she cannot qualify for naturalization under the
interpretation of this Court. The reason behind the requirement
that children should be enrolled in recognized educational
institutions is that they follow the citizenship of their father.
(Chan Ho Lay v. Republic, L-5666, March 30, 1954; Tan Hi v.
Republic, 88 Phil. 117 [1951]; Hao Lian Chu v. Republic, 87 Phil.
668 [1950]; Yap Chin v. Republic, L-4177, May 29, 1953; Lim
Lian Hong v. Republic, L-3575, Dec. 26, 1950). Considering that
said minor children by her first husband generally follow the
citizenship of their alien father, the basis for such requirement
as applied to her does not exist. Cessante ratione legis cessat
ipsa lex.

4. Under Section 3 of the law, the 10-year continuous residence


prescribed by Section 2 "shall be understood as reduced to five
years for any petitioner (who is) married to a Filipino woman." It
is absurd that an alien male married to a Filipino wife should be
required to reside only for five years in the Philippines to qualify
for citizenship, whereas an alien woman married to a Filipino
husband must reside for ten years.

Thus under the interpretation given by this Court, it is more difficult for an alien
wife related by marriage to a Filipino citizen to become such citizen, than for a
foreigner who is not so related. And yet, it seems more than clear that the
general purpose of the first paragraph of Section 15 was obviously to accord to
an alien woman, by reason of her marriage to a Filipino, a privilege not similarly
granted to other aliens. It will be recalled that prior to the enactment of Act No.
3448 in 1928, amending Act No. 2927 (the old Naturalization Law), there was
no law granting any special privilege to alien wives of Filipinos. They were
treated as any other foreigner. It was precisely to remedy this situation that
the Philippine legislature enacted Act No. 3448. On this point, the observation
made by the Secretary of Justice in 1941 is enlightening:

It is true that under, Article 22 of the (Spanish) Civil Code, the


wife follows the nationality of the husband; but the Department
of State of the United States on October 31, 1921, ruled that the
alien wife of a Filipino citizen is not a Filipino citizen, pointing out
that our Supreme Court in the leading case of Roa v. Collector of
Customs (23 Phil. 315) held that Articles 17 to 27 of the Civil
Code being political have been abrogated upon the cession of the
Philippine Islands to the United States. Accordingly, the stated
taken by the Attorney-General prior to the envictment of Act No.
3448, was that marriage of alien women to Philippine citizens did
not make the former citizens of this counting. (Op. Atty. Gen.,
March 16, 1928) .

To remedy this anomalous condition, Act No. 3448 was enacted


in 1928 adding section 13(a) to Act No. 2927 which provides that
"any woman who is now or may hereafter be married to a citizen
of the Philippine Islands, and who might herself be lawfully
naturalized, shall be deemed a citizen of the Philippine Islands.
(Op. No. 22, s. 1941; emphasis ours).

If Section 15 of the, Revised Naturalization Law were to be interpreted, as this


Court did, in such a way as to require that the alien wife must prove the
qualifications prescribed in Section 2, the privilege granted to alien wives would
become illusory. It is submitted that such a construction, being contrary to the
manifested object of the statute must be rejected.

A statute is to be construed with reference to its manifest object,


and if the language is susceptible of two constructions, one which
will carry out and the other defeat such manifest object, it should
receive the former construction. (In re National Guard, 71 Vt.
493, 45 A. 1051; Singer v. United States, 323 U.S. 338, 89 L.
ed. 285. See also, U.S. v. Navarro, 19 Phil. 134 [1911]; U. S. v.
Toribio, 15 Phil. 85 [1910).

... A construction which will cause objectionable results should


be avoided and the court will, if possible, place on the statute a
construction which will not result in injustice, and in accordance
with the decisions construing statutes, a construction which will
result in oppression, hardship, or inconveniences will also be
avoided, as will a construction which will prejudice public
interest, or construction resulting in unreasonableness, as well
as a construction which will result in absurd consequences.

So a construction should, if possible, be avoided if the result


would be an apparent inconsistency in legislative intent, as has
been determined by the judicial decisions, or which would result
in futility, redundancy, or a conclusion not contemplated by the
legislature; and the court should adopt that construction which
will be the least likely to produce mischief. Unless plainly shown
to have been the intention of the legislature, an interpretation
which would render the requirements of the statute uncertain
and vague is to be avoided, and the court will not ascribe to the
legislature an intent to confer an illusory right. ... (82 C.J.S.,
Statutes, sec. 326, pp. 623-632).

7. In Choy King Tee and the second Ly Giok Ha, emphasis was laid on the need for aligning
the construction of Section 15 with "the national policy of selective admission to Philippine
citizenship." But the question may be asked, is it reasonable to suppose that in the pursuit of
such policy, the legislature contemplated to make it more difficult if not practically impossible
in some instances, for an alien woman marrying a Filipino to become a Filipina than any
ordinary applicant for naturalization, as has just been demonstrated above? It seems but
natural and logical to assume that Section 15 was intended to extend special treatment to
alien women who by marrying a Filipino irrevocably deliver themselves, their possessions,
their fate and fortunes and all that marriage implies to a citizen of this country, "for better or
for worse." Perhaps there can and will be cases wherein the personal conveniences and
benefits arising from Philippine citizenship may motivate such marriage, but must the
minority, as such cases are bound to be, serve as the criterion for the construction of law?
Moreover, it is not farfetched to believe that in joining a Filipino family the alien woman is
somehow disposed to assimilate the customs, beliefs and ideals of Filipinos among whom,
after all, she has to live and associate, but surely, no one should expect her to do so even
before marriage. Besides, it may be considered that in reality the extension of citizenship to
her is made by the law not so much for her sake as for the husband. Indeed, We find the
following observations anent the national policy rationalization in Choy King Tee and Ly Giok
Ha (the second) to be quite persuasive:

We respectfully suggest that this articulation of the national policy begs the
question. The avowed policy of "selectives admission" more particularly refers
to a case where citizenship is sought to be acquired in a judicial proceeding for
naturalization. In such a case, the courts should no doubt apply the national
policy of selecting only those who are worthy to become citizens. There is here
a choice between accepting or rejecting the application for citizenship. But this
policy finds no application in cases where citizenship is conferred by operation
of law. In such cases, the courts have no choice to accept or reject. If the
individual claiming citizenship by operation of law proves in legal proceedings
that he satisfies the statutory requirements, the courts cannot do otherwise
than to declare that he is a citizen of the Philippines. Thus, an individual who is
able to prove that his father is a Philippine citizen, is a citizen of the Philippines,
"irrespective of his moral character, ideological beliefs, and identification with
Filipino ideals, customs, and traditions." A minor child of a person naturalized
under the law, who is able to prove the fact of his birth in the Philippines, is
likewise a citizen, regardless of whether he has lucrative income, or he adheres
to the principles of the Constitution. So it is with an alien wife of a Philippine
citizen. She is required to prove only that she may herself be lawfully
naturalized, i.e., that she is not one of the disqualified persons enumerated in
Section 4 of the law, in order to establish her citizenship status as a fact.

A paramount policy consideration of graver import should not be overlooked in


this regard, for it explains and justifies the obviously deliberate choice of words.
It is universally accepted that a State, in extending the privilege of citizenship
to an alien wife of one of its citizens could have had no other objective than to
maintain a unity of allegiance among the members of the family. (Nelson v.
Nelson, 113 Neb. 453, 203 N. W. 640 [1925]; see also "Convention on the
Nationality of Married Women: Historical Background and Commentary."
UNITED NATIONS, Department of Economic and Social Affairs E/CN, 6/399, pp.
8 et seq.). Such objective can only be satisfactorily achieved by allowing the
wife to acquire citizenship derivatively through the husband. This is particularly
true in the Philippines where tradition and law has placed the husband as head
of the family, whose personal status and decisions govern the life of the family
group. Corollary to this, our laws look with favor on the unity and solidarity of
the family (Art. 220, Civil Code), in whose preservation of State as a vital and
enduring interest. (See Art. 216, Civil Code). Thus, it has been said that by
tradition in our country, there is a theoretic identity of person and interest
between husband and wife, and from the nature of the relation, the home of
one is that of the other. (See De la Viña v. Villareal, 41 Phil. 13). It should
likewise be said that because of the theoretic identity of husband and wife, and
the primacy of the husband, the nationality of husband should be the nationality
of the wife, and the laws upon one should be the law upon the other. For as
the court, in Hopkins v. Fachant (9th Cir., 1904) 65 C.C.A., 1, 130 Fed. 839,
held: "The status of the wife follows that of the husband, ... and by virtue of
her marriage her husband's domicile became her domicile." And the
presumption under Philippine law being that the property relations of husband
and wife are under the regime of conjugal partnership (Art. 119, Civil Code),
the income of one is also that of the other.

It is, therefore, not congruent with our cherished traditions of family unity and
identity that a husband should be a citizen and the wife an alien, and that the
national treatment of one should be different from that of the other. Thus, it
cannot be that the husband's interests in property and business activities
reserved by law to citizens should not form part of the conjugal partnership and
be denied to the wife, nor that she herself cannot, through her own efforts but
for the benefit of the partnership, acquire such interests. Only in rare instances
should the identity of husband and wife be refused recognition, and we submit
that in respect of our citizenship laws, it should only be in the instances where
the wife suffers from the disqualifications stated in Section 4 of the Revised
Naturalization Law. (Motion for Reconsideration, Burca vs. Republic, supra.)

With all these considerations in mind, We are persuaded that it is in the best interest of all
concerned that Section 15 of the Naturalization Law be given effect in the same way as it was
understood and construed when the phrase "who may be lawfully naturalized," found in the
American statute from which it was borrowed and copied verbatim, was applied by the
American courts and administrative authorities. There is merit, of course in the view that
Philippine statutes should be construed in the light of Philippine circumstances, and with
particular reference to our naturalization laws. We should realize the disparity in the
circumstances between the United States, as the so-called "melting pot" of peoples from all
over the world, and the Philippines as a developing country whose Constitution is nationalistic
almost in the come. Certainly, the writer of this opinion cannot be the last in rather
passionately insisting that our jurisprudence should speak our own concepts and resort to
American authorities, to be sure, entitled to admiration, and respect, should not be regarded
as source of pride and indisputable authority. Still, We cannot close our eyes to the undeniable
fact that the provision of law now under scrutiny has no local origin and orientation; it is
purely American, factually taken bodily from American law when the Philippines was under
the dominating influence of statutes of the United States Congress. It is indeed a sad
commentary on the work of our own legislature of the late 1920's and 1930's that given the
opportunity to break away from the old American pattern, it took no step in that direction.
Indeed, even after America made it patently clear in the Act of Congress of September 22,
1922 that alien women marrying Americans cannot be citizens of the United States without
undergoing naturalization proceedings, our legislators still chose to adopt the previous
American law of August 10, 1855 as embodied later in Section 1994 of the Revised Statutes
of 1874, Which, it is worth reiterating, was consistently and uniformly understood as
conferring American citizenship to alien women marrying Americans ipso facto, without
having to submit to any naturalization proceeding and without having to prove that they
possess the special qualifications of residence, moral character, adherence to American ideals
and American constitution, provided they show they did not suffer from any of the
disqualifications enumerated in the American Naturalization Law. Accordingly, We now hold,
all previous decisions of this Court indicating otherwise notwithstanding, that under Section
15 of Commonwealth Act 473, an alien woman marrying a Filipino, native born or naturalized,
becomes ipso facto a Filipina provided she is not disqualified to be a citizen of the Philippines
under Section 4 of the same law. Likewise, an alien woman married to an alien who is
subsequently naturalized here follows the Philippine citizenship of her husband the moment
he takes his oath as Filipino citizen, provided that she does not suffer from any of the
disqualifications under said Section 4.

As under any other law rich in benefits for those coming under it, doubtless there will be
instances where unscrupulous persons will attempt to take advantage of this provision of law
by entering into fake and fictitious marriages or mala fide matrimonies. We cannot as a matter
of law hold that just because of these possibilities, the construction of the provision should be
otherwise than as dictated inexorably by more ponderous relevant considerations, legal,
juridical and practical. There can always be means of discovering such undesirable practice
and every case can be dealt with accordingly as it arises.

III.

The third aspect of this case requires necessarily a re-examination of the ruling of this Court
in Burca, supra, regarding the need of judicial naturalization proceedings before the alien wife
of a Filipino may herself be considered or deemed a Filipino. If this case which, as already
noted, was submitted for decision in 1964 yet, had only been decided earlier, before Go Im
Ty, the foregoing discussions would have been sufficient to dispose of it. The Court could have
held that despite her apparent lack of qualifications, her marriage to her co-petitioner made
her a Filipina, without her undergoing any naturalization proceedings, provided she could
sustain, her claim that she is not disqualified under Section 4 of the law. But as things stand
now, with the Burca ruling, the question We have still to decide is, may she be deemed a
Filipina without submitting to a naturalization proceeding?

Naturally, if Burca is to be followed, it is clear that the answer to this question must necessarily
be in the affirmative. As already stated, however, the decision in Burca has not yet become
final because there is still pending with Us a motion for its reconsideration which vigorously
submits grounds worthy of serious consideration by this Court. On this account, and for the
reasons expounded earlier in this opinion, this case is as good an occasion as any other to re-
examine the issue.

In the said decision, Justice Sanchez held for the Court:

We accordingly rule that: (1) An alien woman married to a Filipino who desires
to be a citizen of this country must apply therefore by filing a petition for
citizenship reciting that she possesses all the qualifications set forth in Section
2 and none of the disqualifications under Section 4, both of the Revised
Naturalization Law; (2) Said petition must be filed in the Court of First Instance
where petitioner has resided at least one year immediately preceding the filing
of the petition; and (3) Any action by any other office, agency, board or official,
administrative or otherwise — other than the judgment of a competent court
of justice — certifying or declaring that an alien wife of the Filipino citizen is
also a Filipino citizen, is hereby declared null and void.

3. We treat the present petition as one for naturalization. Or, in the words of
law, a "petition for citizenship". This is as it should be. Because a reading of the
petition will reveal at once that efforts were made to set forth therein, and to
prove afterwards, compliance with Sections 2 and 4 of the Revised
Naturalization law. The trial court itself apparently considered the petition as
one for naturalization, and, in fact, declared petitioner "a citizen of the
Philippines."

In other words, under this holding, in order for an alien woman marrying a Filipino to be
vested with Filipino citizenship, it is not enough that she possesses the qualifications
prescribed by Section 2 of the law and none of the disqualifications enumerated in its Section
4. Over and above all these, she has to pass thru the whole process of judicial naturalization
apparently from declaration of intention to oathtaking, before she can become a Filipina. In
plain words, her marriage to a Filipino is absolutely of no consequence to her nationality vis-
a-vis that of her Filipino husband; she remains to be the national of the country to which she
owed allegiance before her marriage, and if she desires to be of one nationality with her
husband, she has to wait for the same time that any other applicant for naturalization needs
to complete, the required period of ten year residence, gain the knowledge of English or
Spanish and one of the principle local languages, make her children study in Filipino schools,
acquire real property or engage in some lawful occupation of her own independently of her
husband, file her declaration of intention and after one year her application for naturalization,
with the affidavits of two credible witnesses of her good moral character and other
qualifications, etc., etc., until a decision is ordered in her favor, after which, she has to
undergo the two years of probation, and only then, but not before she takes her oath as
citizen, will she begin to be considered and deemed to be a citizen of the Philippines. Briefly,
she can become a Filipino citizen only by judicial declaration.

Such being the import of the Court's ruling, and it being quite obvious, on the other hand,
upon a cursory reading of the provision, in question, that the law intends by it to spell out
what is the "effect of naturalization on (the) wife and children" of an alien, as plainly indicated
by its title, and inasmuch as the language of the provision itself clearly conveys the thought
that some effect beneficial to the wife is intended by it, rather than that she is not in any
manner to be benefited thereby, it behooves Us to take a second hard look at the ruling, if
only to see whether or not the Court might have overlooked any relevant consideration
warranting a conclusion different from that complained therein. It is undeniable that the issue
before Us is of grave importance, considering its consequences upon tens of thousands of
persons affected by the ruling therein made by the Court, and surely, it is for Us to avoid,
whenever possible, that Our decision in any case should produce any adverse effect upon
them not contemplated either by the law or by the national policy it seeks to endorse.

AMICI CURIAE in the Burca case, respectable and impressive by their number and standing
in the Bar and well known for their reputation for intellectual integrity, legal acumen and
incisive and comprehensive resourcefulness in research, truly evident in the quality of the
memorandum they have submitted in said case, invite Our attention to the impact of the
decision therein thus:

The doctrine announced by this Honorable Court for the first time in the present
case -- that an alien woman who marries a Philippine citizen not only does
not ipso facto herself become a citizen but can acquire such citizenship only
through ordinary naturalization proceedings under the Revised Naturalization
Law, and that all administrative actions "certifying or declaring such woman to
be a Philippine citizen are null and void" — has consequences that reach far
beyond the confines of the present case. Considerably more people are
affected, and affected deeply, than simply Mrs. Zita N. Burca. The newspapers
report that as many as 15 thousand women married to Philippine citizens are
affected by this decision of the Court. These are women of many and diverse
nationalities, including Chinese, Spanish, British, American, Columbian,
Finnish, Japanese, Chilean, and so on. These members of the community, some
of whom have been married to citizens for two or three decades, have all
exercised rights and privileges reserved by law to Philippine citizens. They will
have acquired, separately or in conjugal partnership with their citizen
husbands, real property, and they will have sold and transferred such property.
Many of these women may be in professions membership in which is limited to
citizens. Others are doubtless stockholders or officers or employees in
companies engaged in business activities for which a certain percentage of
Filipino equity content is prescribed by law. All these married women are now
faced with possible divestment of personal status and of rights acquired and
privileges exercised in reliance, in complete good faith, upon a reading of the
law that has been accepted as correct for more than two decades by the very
agencies of government charged with the administration of that law. We must
respectfully suggest that judicial doctrines which would visit such
comprehensive and far-reaching injury upon the wives and mothers of
Philippine citizens deserve intensive scrutiny and reexamination.

To be sure, this appeal can be no less than what this Court attended to in Gan Tsitung vs.
Republic, G.R. No. L-20819, Feb. 21, 1967, 19 SCRA 401 — when Chief Justice Concepcion
observed:

The Court realizes, however, that the rulings in the Barretto and Delgado cases
— although referring to situations the equities of which are not identical to
those obtaining in the case at bar — may have contributed materially to the
irregularities committed therein and in other analogous cases, and induced the
parties concerned to believe, although erroneously, that the procedure followed
was valid under the law.

Accordingly, and in view of the implications of the issue under consideration,


the Solicitor General was required, not only, to comment thereon, but, also, to
state "how many cases there are, like the one at bar, in which certificates of
naturalization have been issued after notice of the filing of the petition for
naturalization had been published in the Official Gazette only once, within the
periods (a) from January 28, 1950" (when the decision in Delgado v. Republic
was promulgated) "to May 29, 1957" (when the Ong Son Cui was decided) "and
(b) from May 29, 1957 to November 29, 1965" (when the decision in the
present case was rendered).

After mature deliberation, and in the light of the reasons adduced in appellant's
motion for reconsideration and in the reply thereto of the Government, as well
as of the data contained in the latter, the Court holds that the doctrine laid
down in the Ong Son Cui case shall apply and affect the validity of certificates
of naturalization issued after, not on or before May 29, 1957.

Here We are met again by the same problem. In Gan Tsitung, the Court had to expressly
enjoin the prospective application of its construction of the law made in a previous
decision, 24 which had already become final, to serve the ends of justice and equity. In the
case at bar, We do not have to go that far. As already observed, the decision in Burca still
under reconsideration, while the ruling in Lee Suan Ay, Lo San Tuang, Choy King Tee and
others that followed them have at the most become the law of the case only for the parties
thereto. If there are good grounds therefor, all We have to do now is to reexamine the said
rulings and clarify or modify them.
For ready reference, We requote Section 15:

Sec. 15. Effect of the naturalization on wife and children. — Any woman who is
now or may hereafter be married to a citizen of the Philippines, and who might
herself be lawfully naturalized shall be deemed a citizen of the Philippines.

Minor children of persons naturalized under this law who have been born in the
Philippines shall be considered citizens thereof.

A foreign-born minor child, if dwelling in the Philippines at the time of


naturalization of the parents, shall automatically become a Philippine citizen,
and a foreign-born minor child, who is not in the Philippines at the time the
parent is naturalized, shall be deemed a Philippine citizen only during his
minority, unless he begins to reside permanently in the Philippines when still a
minor, in which case, he will continue to be a Philippine citizen even after
becoming of age.

A child born outside of the Philippines after the naturalization of his parent,
shall be considered a Philippine citizen, unless within one year after reaching
the age of majority, he fails to register himself as a Philippine citizen at the
American Consulate of the country where he resides, and to take the necessary
oath of allegiance.

It is obvious that the main subject-matter and purpose of the statute, the Revised
Naturalization Law or Commonwealth Act 473, as a whole, is to establish a complete
procedure for the judicial conferment of the status of citizenship upon qualified aliens. After
laying out such a procedure, remarkable for its elaborate and careful inclusion of all
safeguards against the possibility of any undesirable persons becoming a part of our citizenry,
it carefully but categorically states the consequence of the naturalization of an alien
undergoing such procedure it prescribes upon the members of his immediate family, his wife
and children, 25 and, to that end, in no uncertain terms it ordains that: (a) all his minor
children who have been born in the Philippines shall be "considered citizens" also; (b) all such
minor children, if born outside the Philippines but dwelling here at the time of such
naturalization "shall automatically become" Filipinos also, but those not born in the Philippines
and not in the Philippines at the time of such naturalization, are also redeemed citizens of this
country provided that they shall lose said status if they transfer their permanent residence to
a foreign country before becoming of age; (c) all such minor children, if born outside of the
Philippines after such naturalization, shall also be "considered" Filipino citizens, unless they
expatriate themselves by failing to register as Filipinos at the Philippine (American) Consulate
of the country where they reside and take the necessary oath of allegiance; and (d) as to the
wife, she "shall be deemed a citizen of the Philippines" if she is one "who might herself be
lawfully naturalized". 26

No doubt whatever is entertained, so Burca holds very correctly, as to the point that the minor
children, falling within the conditions of place and time of birth and residence prescribed in
the provision, are vested with Philippine citizenship directly by legislative fiat or by force of
the law itself and without the need for any judicial proceeding or declaration. (At p. 192, 19
SCRA). Indeed, the language of the provision, is not susceptible of any other interpretation.
But it is claimed that the same expression "shall be deemed a citizen of the Philippines" in
reference to the wife, does not necessarily connote the vesting of citizenship status upon her
by legislative fiat because the antecedent phrase requiring that she must be one "who might
herself be lawfully naturalized" implies that such status is intended to attach only after she
has undergone the whole process of judicial naturalization required of any person desiring to
become a Filipino. Stated otherwise, the ruling in Burca is that while Section 15 envisages
and intends legislative naturalization as to the minor children, the same section deliberately
treats the wife differently and leaves her out for the ordinary judicial naturalization.

Of course, it goes without saying that it is perfectly within the constitutional authority of the
Congress of the Philippines to confer or vest citizenship status by legislative fiat. (U.S. v.
Wong Kim Ark, 169 U.S. 649, 42 L ed. 890 [1898]; See, 1 Tañada & Carreon, Political Law of
the Philippines 152 [1961 ed.]) In fact, it has done so for particular individuals, like two
foreign religious prelates, 27 hence there is no reason it cannot do it for classes or groups of
persons under general conditions applicable to all of the members of such class or group, like
women who marry Filipinos, whether native-born or naturalized. The issue before Us in this
case is whether or not the legislature hag done so in the disputed provisions of Section 15 of
the Naturalization Law. And Dr. Vicente G. Sinco, one of the most respect authorities on
political law in the Philippines 28 observes in this connection thus: "A special form of
naturalization is often observed by some states with respect to women. Thus in the Philippines
a foreign woman married to a Filipino citizen becomes ipso facto naturalized, if she belongs
to any of the classes who may apply for naturalization under the Philippine Laws." (Sinco,
Phil. Political Law 498-499 [10th ed. 1954]; emphasis ours; this comment is substantially
reiterated in the 1962 edition, citing Ly Giok Ha and Ricardo Cua, supra.)

More importantly, it may be stated, at this juncture, that in construing the provision of the
United States statutes from which our law has been copied, 28a the American courts have held
that the alien wife does not acquire American citizenship by choice but by operation of law.
"In the Revised Statutes the words "and taken" are omitted. The effect of this statute is that
every alien woman who marries a citizen of the United States becomes perforce a citizen
herself, without the formality of naturalization, and regardless of her wish in that respect."
(USCA 8, p. 601 [1970 ed.], citing Mackenzie v. Hare, 1913, 134 P. 713, 165 Cal. 766,
affirmed 36 S. Ct. 106, 239 U.S. 299, 60 L ed. 297.) .

We need not recount here again how this provision in question was first enacted as paragraph
(a) of Section 13, by way of an insertion into Act 2927 by Act 3448 of November 30, 1928,
and that, in turn, and paragraph was copied verbatim from Section 1994 of the Revised
Statutes of the United States, which by that time already had a long accepted construction
among the courts and administrative authorities in that country holding that under such
provision an alien woman who married a citizen became, upon such marriage, likewise a
citizen by force of law and as a consequence of the marriage itself without having to undergo
any naturalization proceedings, provided that, it could be shown that at the time of such
marriage, she was not disqualified to be naturalized under the laws then in force. To repeat
the discussion We already made of these undeniable facts would unnecessarily make this
decision doubly extensive. The only point which might be reiterated for emphasis at this
juncture is that whereas in the United States, the American Congress, recognizing the
construction, of Section 1994 of the Revised Statutes to be as stated above, and finding it
desirable to avoid the effects of such construction, approved the Act of September 22, 1922
Explicitly requiring all such alien wives to submit to judicial naturalization albeit under more
liberal terms than those for other applicants for citizenship, on the other hand, the Philippine
Legislature, instead of following suit and adopting such a requirement, enacted Act 3448 on
November 30, 1928 which copied verbatim the aforementioned Section 1994 of the Revised
Statutes, thereby indicating its preference to adopt the latter law and its settled construction
rather than the reform introduced by the Act of 1922.
Obviously, these considerations leave Us no choice. Much as this Court may feel that as the
United States herself has evidently found it to be an improvement of her national policy vis-
a-vis the alien wives of her citizens to discontinue their automatic incorporation into the body
of her citizenry without passing through the judicial scrutiny of a naturalization proceeding,
as it used to be before 1922, it seems but proper, without evidencing any bit of colonial
mentality, that as a developing country, the Philippines adopt a similar policy, unfortunately,
the manner in which our own legislature has enacted our laws on the subject, as recounted
above, provides no basis for Us to construe said law along the line of the 1922 modification
of the American Law. For Us to do so would be to indulge in judicial legislation which it is not
institutionally permissible for this Court to do. Worse, this court would be going precisely
against the grain of the implicit Legislative intent.

There is at least one decision of this Court before Burca wherein it seems it is quite clearly
implied that this Court is of the view that under Section 16 of the Naturalization Law, the
widow and children of an applicant for naturalization who dies during the proceedings do not
have to submit themselves to another naturalization proceeding in order to avail of the
benefits of the proceedings involving the husband. Section 16 provides: .

SEC. 16. Right of widow and children of petitioners who have died. — In case
a petitioner should die before the final decision has been rendered, his widow
and minor children may continue the proceedings. The decision rendered in the
case shall, so far as the widow and minor children are concerned, produce the
same legal effect as if it had been rendered during the life of the petitioner.

In Tan Lin v. Republic, G.R. No. L-13706, May 31, 1961, 2 SCRA 383, this Court held:

Invoking the above provisions in their favor, petitioners-appellants argue (1)


that under said Sec. 16, the widow and minor children are allowed to continue
the same proceedings and are not substituted for the original petitioner; (2)
that the qualifications of the original petitioner remain to be in issue and not
those of the widow and minor children, and (3) that said Section 16 applies
whether the petitioner dies before or after final decision is rendered, but before
the judgment becomes executory.

There is force in the first and second arguments. Even the second sentence of
said Section 16 contemplate the fact that the qualifications of the original
petitioner remains the subject of inquiry, for the simple reason that it states
that "The decision rendered in the case shall, so far as the widow and minor
children are concerned, produce the same legal effect as if it had been rendered
during the life of the petitioner." This phraseology emphasizes the intent of the
law to continue the proceedings with the deceased as the theoretical petitioner,
for if it were otherwise, it would have been unnecessary to consider the decision
rendered, as far as it affected the widow and the minor children.

xxx xxx xxx

The Chua Chian case (supra), cited by the appellee, declared that a dead person
can not be bound to do things stipulated in the oath of allegiance, because an
oath is a personal matter. Therein, the widow prayed that she be allowed to
take the oath of allegiance for the deceased. In the case at bar, petitioner Tan
Lin merely asked that she be allowed to take the oath of allegiance and the
proper certificate of naturalization, once the naturalization proceedings of her
deceased husband, shall have been completed, not on behalf of the deceased
but on her own behalf and of her children, as recipients of the benefits of his
naturalization. In other words, the herein petitioner proposed to take the oath
of allegiance, as a citizen of the Philippines, by virtue of the legal provision that
"any woman who is now or may hereafter be married to a citizen of the
Philippines and who might herself be lawfully naturalized shall be deemed a
citizen of the Philippines. Minor children of persons naturalized under this law
who have been born in the Philippines shall be considered citizens thereof."
(Section 15, Commonwealth Act No. 473). The decision granting citizenship to
Lee Pa and the record of the case at bar, do not show that the petitioning widow
could not have been lawfully naturalized, at the time Lee Pa filed his petition,
apart from the fact that his 9 minor children were all born in the Philippines.
(Decision, In the Matter of the Petition of Lee Pa to be admitted a citizen of the
Philippines, Civil Case No. 16287, CFI, Manila, Annex A; Record on Appeal, pp.
8-11). The reference to Chua Chian case is, therefore, premature.

Section 16, as may be seen, is a parallel provision to Section 15. If the widow of an applicant
for naturalization as Filipino, who dies during the proceedings, is not required to go through
a naturalization preceeding, in order to be considered as a Filipino citizen hereof, it should
follow that the wife of a living Filipino cannot be denied the same privilege. This is plain
common sense and there is absolutely no evidence that the Legislature intended to treat them
differently.

Additionally, We have carefully considered the arguments advanced in the motion for
reconsideration in Burca, and We see no reason to disagree with the following views of
counsel: .

It is obvious that the provision itself is a legislative declaration of who may be


considered citizens of the Philippines. It is a proposition too plain to be disputed
that Congress has the power not only to prescribe the mode or manner under
which foreigners may acquire citizenship, but also the very power of conferring
citizenship by legislative fiat. (U. S. v. Wong Kim Ark, 169 U. S. 649, 42 L. Ed.
890 [1898] ; see 1 Tañada and Carreon, Political Law of the Philippines 152
[1961 ed.]) The Constitution itself recognizes as Philippine citizens "Those who
are naturalized in accordance with law" (Section 1[5], Article IV, Philippine
Constitution). Citizens by naturalization, under this provision, include not only
those who are naturalized in accordance with legal proceedings for the
acquisition of citizenship, but also those who acquire citizenship by "derivative
naturalization" or by operation of law, as, for example, the "naturalization" of
an alien wife through the naturalization of her husband, or by marriage of an
alien woman to a citizen. (See Tañada & Carreon, op. cit. supra, at 152, 172;
Velayo, Philippine Citizenship and Naturalization 2 [1965 ed.]; 1 Paras, Civil
Code 186 [1967 ed.]; see also 3 Hackworth, Digest of International Law 3).

The phrase "shall be deemed a citizen of the Philippines" found in Section 14 of


the Revised Naturalization Law clearly manifests an intent to confer citizenship.
Construing a similar phrase found in the old U.S. naturalization law (Revised
Statutes, 1994), American courts have uniformly taken it to mean that upon
her marriage, the alien woman becomes by operation of law a citizen of the
United States as fully as if she had complied with all the provisions of the
statutes upon the subject of naturalization. (U.S. v. Keller, 13 F. 82; U.S.
Opinions of the US Attorney General dated June 4, 1874 [14 Op. 4021, July 20,
1909 [27 Op. 507], December 1, 1910 [28 Op. 508], Jan. 15, 1920 [32 Op.
2091 and Jan. 12, 1923 [23 398]).

The phrase "shall be deemed a citizen," in Section 1994 Revised


Statute (U.S. Comp. Stat. 1091, 1268) or as it was in the Act of
1855 (10 Stat. at L. 604, Chapt. 71, Sec. 2), "shall be deemed
and taken to be a citizens" while it may imply that the person to
whom it relates has not actually become a citizen by the ordinary
means or in the usual way, as by the judgment of a competent
court, upon a proper application and proof, yet it does not follow
that such person is on that account practically any the less a
citizen. The word "deemed" is the equivalent of "considered" or
"judged," and therefore, whatever an Act of Congress requires
to be "deemed" or "taken" as true of any person or thing must,
in law, be considered as having been duly adjudged or
established concerning such person or thing, and have force and
effect accordingly. When, therefore, Congress declares that an
alien woman shall, under certain circumstances, be "deemed" an
American citizen, the effect when the contingency occurs, is
equivalent to her being naturalized directly by an Act of Congress
or in the usual mode thereby prescribed. (Van Dyne, Citizenship
of the United States 239, cited in Velayo, Philippine Citizenship
and Naturalization 146-147 [1965 ed.]; emphasis ours).

That this was likewise the intent of the Philippine legislature when it enacted
the first paragraph of Section 15 of the Revised Naturalization Law is shown by
a textual analysis of the entire statutory provision. In its entirety, Section 15
reads:

(See supra).

The phrases "shall be deemed" "shall be considered," and "shall automatically


become" as used in the above provision, are undoubtedly synonymous. The
leading idea or purpose of the provision was to confer Philippine citizenship by
operation of law upon certain classes of aliens as a legal consequence of their
relationship, by blood or by affinity, to persons who are already citizens of the
Philippines. Whenever the fact of relationship of the persons enumerated in the
provision concurs with the fact of citizenship of the person to whom they are
related, the effect is for said persons to become ipso factocitizens of the
Philippines. "Ipso facto" as here used does not mean that all alien wives and all
minor children of Philippine citizens, from the mere fact of relationship,
necessarily become such citizens also. Those who do not meet the statutory
requirements do not ipso facto become citizens; they must apply for
naturalization in order to acquire such status. What it does mean, however, is
that in respect of those persons enumerated in Section 15, the relationship to
a citizen of the Philippines is the operative fact which establishes the acquisition
of Philippine citizenship by them. Necessarily, it also determines the point of
time at which such citizenship commences. Thus, under the second paragraph
of Section 15, a minor child of a Filipino naturalized under the law, who was
born in the Philippines, becomes ipso facto a citizen of the Philippines from the
time the fact of relationship concurs with the fact of citizenship of his parent,
and the time when the child became a citizen does not depend upon the time
that he is able to prove that he was born in the Philippines. The child may prove
some 25 years after the naturalization of his father that he was born in the
Philippines and should, therefore, be "considered" a citizen thereof. It does not
mean that he became a Philippine citizen only at that later time. Similarly, an
alien woman who married a Philippine citizen may be able to prove only some
25 years after her marriage (perhaps, because it was only 25 years after the
marriage that her citizenship status became in question), that she is one who
might herself be lawfully naturalized." It is not reasonable to conclude that she
acquired Philippine citizenship only after she had proven that she "might herself
be lawfully naturalized." It is not reasonable to conclude that she acquired
Philippine citizenship only after she had proven that she "might herself be
lawfully naturalized."

The point that bears emphasis in this regard is that in adopting the very
phraseology of the law, the legislature could not have intended that an alien
wife should not be deemed a Philippine citizen unless and until she proves that
she might herself be lawfully naturalized. Far from it, the law states in plain
terms that she shall be deemed a citizen of the Philippines if she is one "who
might herself be lawfully naturalized." The proviso that she must be one "who
might herself be lawfully naturalized" is not a condition precedent to the vesting
or acquisition of citizenship; it is only a condition or a state of fact necessary to
establish her citizenship as a factum probandum, i.e., as a fact established and
proved in evidence. The word "might," as used in that phrase, precisely replies
that at the time of her marriage to a Philippine citizen, the alien woman "had
(the) power" to become such a citizen herself under the laws then in force.
(Owen v. Kelly, 6 DC 191 [1867], aff'd Kelly v. Owen, 76 US 496, 19 L ed 283
[1869). That she establishes such power long after her marriage does not alter
the fact that at her marriage, she became a citizen.

(This Court has held) that "an alien wife of a Filipino citizen may not acquire the
status of a citizen of the Philippines unless there is proof that she herself may
be lawfully naturalized" (Decision, pp. 3-4). Under this view, the "acquisition"
of citizenship by the alien wife depends on her having proven her qualifications
for citizenship, that is, she is not a citizen unless and until she proves that she
may herself be lawfully naturalized. It is clear from the words of the law that
the proviso does not mean that she must first prove that she "might herself be
lawfully naturalized" before she shall be deemed (by Congress, not by the
courts) a citizen. Even the "uniform" decisions cited by this Court (at fn. 2) to
support its holding did not rule that the alien wife becomes a citizen
only after she has proven her qualifications for citizenship. What those
decisions ruled was that the alien wives in those cases failed to prove their
qualifications and therefore they failed to establish their claim to citizenship.
Thus in Ly Giok Ha v. Galang, 101 Phil. 459 [l957], the case was remanded to
the lower court for determination of whether petitioner, whose claim to
citizenship by marriage to a Filipino was disputed by the Government, "might
herself be lawfully naturalized," for the purpose of " proving her alleged change
of political status from alien to citizen" (at 464). In Cua v. Board, 101 Phil. 521
[1957], the alien wife who was being deported, claimed she was a Philippine
citizen by marriage to a Filipino. This Court finding that there was no proof that
she was not disqualified under Section 4 of the Revised Naturalization Law,
ruled that: "No such evidence appearing on record, the claim of assumption of
Philippine citizenship by Tijoe Wu Suan, upon her marriage to petitioner, is
untenable." (at 523) It will be observed that in these decisions cited by this
Court, the lack of proof that the alien wives "might (themselves) be lawfully
naturalized" did not necessarily imply that they did not become, in truth and in
fact, citizens upon their marriage to Filipinos. What the decisions merely held
was that these wives failed to establish their claim to that status as a proven
fact.

In all instances where citizenship is conferred by operation of law, the time


when citizenship is conferred should not be confused with the time when
citizenship status is established as a proven fact. Thus, even a natural-born
citizen of the Philippines, whose citizenship status is put in issue in any
proceeding would be required to prove, for instance, that his father is a citizen
of the Philippines in order to factually establish his claim to citizenship.* His
citizenship status commences from the time of birth, although his claim thereto
is established as a fact only at a subsequent time. Likewise, an alien woman
who might herself be lawfully naturalized becomes a Philippine citizen at the
time of her marriage to a Filipino husband, not at the time she is able to
establish that status as a proven fact by showing that she might herself be
lawfully naturalized. Indeed, there is no difference between a statutory
declaration that a person is deemed a citizen of the Philippines provided his
father is such citizen from a declaration that an alien woman married to a
Filipino citizen of the Philippines provided she might herself be lawfully
naturalized. Both become citizens by operation of law; the former becomes a
citizen ipso facto upon birth; the later ipso facto upon marriage.

It is true that unless and until the alien wife proves that she might herself be
lawfully naturalized, it cannot be said that she has established her status as a
proven fact. But neither can it be said that on that account, she did not become
a citizen of the Philippines. If her citizenship status is not questioned in any
legal proceeding, she obviously has no obligation to establish her status as a
fact. In such a case, the presumption of law should be that she is what she
claims to be. (U.S. v. Roxas, 5 Phil. 375 [1905]; Hilado v. Assad, 51 O.G. 4527
[1955]). There is a presumption that a representation shown to have been
made is true. (Aetna Indemnity Co. v. George A. Fuller, Co., 73 A. 738, 74 A.
369, 111 ME. 321).

The question that keeps bouncing back as a consequence of the foregoing views is, what
substitute is them for naturalization proceedings to enable the alien wife of a Philippine citizen
to have the matter of her own citizenship settled and established so that she may not have
to be called upon to prove it everytime she has to perform an act or enter in to a transaction
or business or exercise a right reserved only to Filipinos? The ready answer to such question
is that as the laws of our country, both substantive and procedural, stand today, there is no
such procedure, but such paucity is no proof that the citizenship under discussion is not vested
as of the date of marriage or the husband's acquisition of citizenship, as the case may be, for
the truth is that the same situation objections even as to native-born Filipinos. Everytime the
citizenship of a person is material or indispensable in a judicial or administrative case,
whatever the corresponding court or administrative authority decides therein as to such
citizenship is generally not considered as res adjudicata, hence it has to be threshed out again
and again as the occasion may demand. This, as We view it, is the sense in which Justice
Dizon referred to "appropriate proceeding" in Brito v. Commissioner, supra. Indeed, only the
good sense and judgment of those subsequently inquiring into the matter may make the effort
easier or simpler for the persons concerned by relying somehow on the antecedent official
findings, even if these are not really binding.
It may not be amiss to suggest, however, that in order to have a good starting point and so
that the most immediate relevant public records may be kept in order, the following
observations in Opinion No. 38, series of 1958, of then Acting Secretary of Justice Jesus G.
Barrera, may be considered as the most appropriate initial step by the interested parties:

Regarding the steps that should be taken by an alien woman married to a


Filipino citizen in order to acquire Philippine citizenship, the procedure followed
in the Bureau of Immigration is as follows: The alien woman must file a petition
for the cancellation of her alien certificate of registration alleging, among other
things, that she is married to a Filipino, citizen and that she is not disqualified
from acquiring her husband's citizenship pursuant to section 4 of
Commonwealth Act No. 473, as amended. Upon the filing of said petition, which
should be accompanied or supported by the joint affidavit of the petitioner and
her Filipino husband to the effect that the petitioner does not belong to any of
the groups disqualified by the cited section from becoming naturalized Filipino
citizen (please see attached CEB Form 1), the Bureau of Immigration conducts
an investigation and thereafter promulgates its order or decision granting or
denying the petition.

Once the Commissioner of Immigration cancels the subject's registration as an alien, there
will probably be less difficulty in establishing her Filipino citizenship in any other proceeding,
depending naturally on the substance and vigor of the opposition.

Before closing, it is perhaps best to clarify that this third issue We have passed upon was not
touched by the trial court, but as the point is decisive in this case, the Court prefers that the
matter be settled once and for all now.

IN VIEW OF ALL THE FOREGOING, the judgment of the Court a quo dismissing appellants'
petition for injunction is hereby reversed and the Commissioner of Immigration and/or his
authorized representative is permanently enjoined from causing the arrest and deportation
and the confiscation of the bond of appellant Lau Yuen Yeung, who is hereby declared to have
become a Filipino citizen from and by virtue of her marriage to her co-appellant Moy Ya Lim
Yao alias Edilberto Aguinaldo Lim, a Filipino citizen on January 25, 1962. No costs.

Dizon, Castro, Teehankee and Villamor, JJ., concur.


[G.R. No. 163256. November 10, 2004]

CICERON P. ALTAREJOS, petitioner, vs. COMMISSION ON ELECTIONS, JOSE ALMIE


and VERNON VERSOZA, respondents.

DECISION
AZCUNA, J.:

This is a petition for certiorari, with prayer for the issuance of a temporary restraining
order and/or a writ of prohibitory and mandatory injunction, to set aside the Resolution
promulgated by the Commission on Elections (COMELEC), First Division, on March 22, 2004
disqualifying petitioner Ciceron P. Altarejos from running as mayor of San Jacinto, Masbate,
and another resolution of the COMELEC en banc promulgated on May 7, 2004 denying
petitioners motion for reconsideration.
The factual antecedents are as follows:
Petitioner Altarejos was a candidate for mayor in the Municipality of San Jacinto, Masbate
in the May 10, 2004 national and local elections.
On January 15, 2004, private respondents Jose Almie Altiche and Vernon Versoza,
registered voters of San Jacinto, Masbate, filed with the COMELEC, a petition to disqualify and
to deny due course or cancel the certificate of candidacy of petitioner on the ground that he
is not a Filipino citizen and that he made a false representation in his certificate of candidacy
that [he] was not a permanent resident of or immigrant to a foreign country.
Private respondents alleged that based on a letter[1] from the Bureau of Immigration
dated June 25, 2001, petitioner was a holder of a permanent U.S. resident visa, an Alien
Certificate of Registration No. E139507 issued on November 3, 1997, and an Immigration
Certificate of Residence No. 320846 issued on November 3, 1997 by the Bureau of
Immigration.[2]
On January 26, 2004, petitioner filed an Answer[3] stating, among others, that he did not
commit false representation in his application for candidacy as mayor because as early as
December 17, 1997, he was already issued a Certificate of Repatriation by the Special
Committee on Naturalization, after he filed a petition for repatriation pursuant to Republic Act
No. 8171. Thus, petitioner claimed that his Filipino citizenship was already restored, and he
was qualified to run as mayor in the May 10, 2004 elections. Petitioner sought the dismissal
of the petition.
On the date of the hearing, the parties were required to submit their Memoranda within
three days. Private respondents filed their Memorandum, while petitioner did not file one
within the required period.[4] Petitioner, however, filed a Reply Memorandum[5] subsequently.
Atty. Zacarias C. Zaragoza, Jr., regional election director for Region V and hearing officer
of this case, recommended that petitioner Altarejos be disqualified from being a candidate for
the position of mayor of San Jacinto, Masbate in the May 10, 2004 national and local elections.
He found, thus:

xxx
The provisions of law governing the qualifications and disqualifications of elective local officials
are found in Sections 39 and 40 of Republic Act No. 7160 otherwise known as the Local
Government Code of 1991, which provide as follows:

SEC. 39. Qualifications. (a) An elective local official must be a citizen of the
Philippines; a registered voter in the barangay, municipality, city or province or, in the case
of member of the sangguniang panlalawigan, sangguniang panlungsod, or sangguniang
bayan, the district where he intends to be elected; a resident therein for at least one (1) year
immediately preceding the day of the election; and able to read and write Filipino or any other
local language or dialect.

xxx.

(c) Candidates for the position of mayor or vice-mayor of independent component cities,
component cities or municipalities must be at least twenty-one (21) years of age on election
day.

[SEC. 40. Disqualifications. The following persons are disqualified from running for any
elective position:]

xxx.

(d) Those with dual citizenship.

xxx.

(f) Permanent residents in a foreign country or those who have acquired the right to reside
abroad and continue to avail of the same right after the effectivity of this Code; xxx

Under the terms of the above quoted statutory provisions, it is required that an elective local
official must be a citizen of the Philippines, and he must not have a dual citizenship; must not
be a permanent resident in a foreign country or must not have acquired the right to reside
abroad.

In the present case, it has been established by clear and convincing evidence that respondent
is a citizen of the United States of America. Such fact is proven by his Alien Certificate of
Registration (ACR) No. E139507 issued on 3 November 1997 and Immigration Certificate of
Residence (ICR) with No. 320846 issued on 3 November 1997 by the Alien Registration
Division, Bureau of Immigration and Deportation. This was further confirmed in a letter dated
25 June 2001 of then Commissioner ANDREA D. DOMINGO of the Bureau of Immigration and
Deportation.

Although respondent had petitioned for his repatriation as a Filipino citizen under Republic Act
No. 8171 on 17 December 1997, this did not restore to respondent his Filipino citizenship,
because Section 2 of the aforecited Republic Act No. 8171 specifically provides
that repatriation shall be effected by taking the necessary oath of allegiance to the
Republic of the Philippines and registration in the proper civil registry and in the
Bureau of Immigration.

It appears from the records of this case that respondent failed to prove that he has fully
complied with requirements of the above-quoted Section 2 of Republic Act 8171 to perfect his
repatriation and reacquire his Filipino citizenship. Respondent has not submitted any
document to prove that he has taken his oath of allegiance to the Republic of the Philippines
and that he has registered his fact of repatriation in the proper civil registry and in the Bureau
of Immigration. In fact, in a letter date 25 June 2001, Commissioner ANDREA DOMINGO
stated that RESPONDENT is still a holder of visa under Section 13 (g) of the Philippine
Immigration Act of 1940 as amended, with an indefinite authorized stay in the Philippines,
implying that respondent did not register his supposed Certificate of Repatriation with the
Bureau of Immigration otherwise his Alien Visa would have already been cancelled. The rule
is that in case of doubt concerning the grant of citizenship, such doubt should be resolved in
favor of the State and against the applicant (Cheng vs. Republic, L-16999, 22 June 1965).

xxx

Not having been able to prove that he has fully reacquired his Filipino citizenship after being
naturalized as a citizen of the United States, it is clear that respondent is not qualified to be
candidate for the position of Mayor of San Jacinto, Masbate, in the 10 May 2004 National and
Local Elections, pursuant to the aforequoted Sections 39 and 40 of the Local Government
Code of 1991.

As a further consequence of his not being a Filipino citizen, respondent has also committed
false representation in his certificate of candidacy by stating therein that he is a natural-born
Filipino citizen, when in fact, he has not yet even perfected the reacquisition of Filipino
citizenship. Such false representation constitutes a material misrepresentation as it relates to
his qualification as a candidate for public office, which could be a valid ground for the
cancellation of his certificate of candidacy under Section 78 of the Omnibus Election Code x x
x. [6]

In its Resolution promulgated on March 22, 2004, the COMELEC, First Division, adopted
the findings and recommendation of Director Zaragoza. The dispositive portion of said
Resolution stated, thus:

WHEREFORE, premises considered, respondent CICERON PEREZ ALTAREJOS is hereby


disqualified to run as Mayor of San Jacinto, Masbate. Accordingly, his certificate of candidacy
for the position of Municipal Mayor of San Jacinto, Masbate is denied due course and cancelled
and his name deleted from the certified list of candidates for the May 10, 2004 elections. [7]

On March 25, 2004, petitioner filed a motion for reconsideration and attached the
following documents to prove that he had completed all the requirements for repatriation
which thus entitled him to run for an elective office, viz:
(1) Oath of Allegiance dated December 17, 1997;
(2) Identification Certificate No. 116543 issued by the Bureau of Immigration on March
1, 2004;
(3) Certification from the City Civil Registration Office, Makati City, that the Certificate of
Repatriation and Oath of Allegiance of petitioner was received by said office and registered,
with the corresponding fee paid, on February 18, 2004;
(4) A letter dated December 17, 1997 from the Special Committee on Naturalization to
the Bureau on Immigration and Deportation that it was furnishing said office with the Oath of
Allegiance and Certificate of Repatriation of petitioner for the cancellation of petitioners
registration in said office as an alien, and the issuance to him of the corresponding
Identification Card as Filipino citizen;
(5) A letter dated December 17, 1997 from the Special Committee on Naturalization to
the Local Registrar of San Jacinto, Masbate that it was sending petitioners Oath of Allegiance
and Certificate of Repatriation for registration in their records and for petitioners reacquisition
of his former Philippine citizenship.
On May 7, 2004, the COMELEC en banc promulgated a resolution denying the motion for
reconsideration, the dispositive portion of which reads:

WHEREFORE, premises considered, the Commission (En Banc) RESOLVED as it


hereby RESOLVES to DENY the Motion for Reconsideration for UTTER LACK OF
MERIT and AFFIRMS the Resolution of the First Division.[8]

The Comelec en banc held, thus:

The Comelec Rules of Procedure provides that insufficiency of evidence to justify the decision
is a ground for a motion for reconsideration (Rule 19, Section 1). The evidence referred to
in the above provision and to be considered in the Motion for Reconsideration are those which
were submitted during the hearing and attached to the respective Memoranda of the parties
which are already part of the records of the case. In this regard, the evidence of the
respondent were not able to overcome the evidence of the petitioners.

When the entire records of the case was forwarded to the Commission (First Division) the
respondents only evidence was his Certificate of Repatriation dated 17 December 1977 and
marked as Annex 1 of his answer. This piece of evidence was not enough to controvert the
evidence of the petitioners which consist of the letter of the then Bureau of Immigration
Commissioner Andrea Domingo dated 25 June 2001 which stated that as of the even date
respondent is a holder of permanent resident visa (page 15 of the records) and the
certification of Josephine C. Camata dated 28 January 2004 certifying, that the name of the
respondent could not be found in the records of repatriation. (page 42 of the records) The
questioned resolution, is therefore, in order as the evidence submitted by the respondent
were insufficient to rebut the evidence of the petitioner.

Now, the respondent, in his Motion for Reconsideration, attempted to introduce to the record
new pieces of evidence, which introduction is not anymore allowed in a Motion for
Reconsideration. These are the following a) Annex 2 Oath of Allegiance; b) Annex 3 Bureau
of Immigration Identification Certificate; c) Annex 4 Certification of the City Civil Registrar of
Makati City; d) Annex 5 Letter addressed to the Local Civil Registrar of San Jacinto,
Masbate by Aurora P. Cortes of Special Committee on Naturalization; and e) Annex 6 Letter
addressed to the Bureau of Immigration and Deportation by Aurora P. Cortes of Special
Committee on Naturalization.

Assuming that the new evidence of the respondent are admitted, with more reason should we
cancel his certificate of candidacy for his act of [misrepresenting] himself as a Filipino citizen
when at the time he filed his certificate of candidacy, he has not yet perfected the process of
repatriation. He failed to comply with the requirements under Section 2 of [Republic Act No.]
8171 which provides that repatriation shall be effected by taking the necessary oath of
allegiance to the Republic of the Philippines and registration in the proper civil
registry and in the Bureau of Immigration.

The certification was issued by the same Ms. Josephine C. Camata, City Civil Registrar, dated
February 18, 2004. This time, she certifies that Ciceron Perez Altarejos was registered under
Registry No. 1, Page 19, Book No. 1, Series of 2004 and paid under OR nos. 88325/8833256
dated February 18, 2004. (page 65 of the records). Obviously, he was able to register in
the proper civil registry only on February 18, 2004.

The respondent was able to register with the Bureau of Immigration only on March 1, 2004
as evidenced by the Bureau of Immigration Identification Certificate attached to the Motion
as Annex 3.

This fact confirms the finding of the Commission (First Division) that at the time respondent
filed his certificate of candidacy he is yet to complete the requirement under section two (2)
of RA 8171.

As a consequence of not being a Filipino citizen, he has committed false representation in his
certificate of candidacy. Such false representation constitutes a material misrepresentation
as it relates to his qualification as a candidate. As such the certificate of candidacy may be
cancelled on such ground. (Ycain vs. Caneja, 18 Phil. 778)[9]

On May 10, 2004, the election day itself, petitioner filed this petition praying that: (1)
The petition be given due course and a temporary restraining order and/or writ of preliminary
injunction be issued ex parte restraining the respondents and all persons acting on their
behalf, from fully implementing the questioned COMELEC Resolutions promulgated on March
22, 2004 and May 7, 2004; (2) a writ of preliminary mandatory injunction be issued ordering
the COMELEC and all persons acting on its behalf to allow petitioner to run as Mayor of San
Jacinto, Masbate in the May 10, 2004 elections, and to count and canvass the votes cast in
his favor and to proclaim him as the winning mayor of San Jacinto, Masbate; and (3) after
proper proceedings, judgment be rendered declaring null and void and setting aside the
COMELEC Resolutions promulgated on March 22, 2004 and May 7, 2004 and other related
Orders of the COMELEC or its representatives which have the effect of illegally preventing
petitioner from running as Mayor of San Jacinto, Masbate.
In its Comment,[10] the Office of the Solicitor General stated that, based on the
information relayed to it by the COMELEC, petitioners name, as a mayoralty candidate in San
Jacinto, Masbate, was retained in the list of candidates voted upon by the electorate in the
said municipality. Hence, the cancellation of petitioners certificate of candidacy was never
implemented. The COMELEC also informed the Office of the Solicitor General that petitioners
opponent, Dr. Emilio Aris V. Espinosa, was already proclaimed duly elected Mayor of San
Jacinto, Masbate.
The Office of the Solicitor General contends that said supervening event has rendered the
instant petition moot and academic, and it prayed for the dismissal of the petition.
In his Reply,[11] petitioner opposed the dismissal of his petition. He claims that the
COMELEC resolutions disqualifying him from running as a mayoralty candidate adversely
affected his candidacy, since his supporters were made to believe that his votes would not be
counted. Moreover, he stated that said COMELEC resolutions cast a doubt on his Philippine
citizenship.
Petitioner points out that he took his Oath of Allegiance to the Republic of the Philippines
on December 17, 1997. In view thereof, he ran and was even elected as Mayor of San Jacinto,
Masbate during the 1998 elections. He argues that if there was delay in the registration of his
Certificate of Repatriation with the Bureau of Immigration and with the proper civil registry,
the same was brought about by the inaction on the part of said offices since the records of
the Special Committee on Naturalization show that his Certificate of Repatriation and Oath of
Allegiance have long been transmitted to said offices.
Petitioner also asserts that the subsequent registration of his Certificate of Repatriation
with the Bureau of Immigration and with the Civil Registry of Makati City prior to the May 10,
2004 elections has the effect of curing the defect, if any, in the reacquisition of his Filipino
citizenship as his repatriation retroacted to the date of his application for repatriation as held
in Frivaldo v. Comelec.
The pertinent issues raised are the following: (1) Is the registration of petitioners
repatriation with the proper civil registry and with the Bureau of Immigration a prerequisite
in effecting repatriation; and (2) whether or not the COMELEC en banc committed grave
abuse of discretion amounting to excess or lack of jurisdiction in affirming the Resolution of
the COMELEC, First Division.
As stated by the Office of the Solicitor General, where the issues have become moot and
academic, there is no justiciable controversy, thereby rendering the resolution of the same of
no practical use or value.[12] Nonetheless, courts will decide a question otherwise moot and
academic if it is capable of repetition, yet evading review.[13]

First Issue: Is the registration of petitioners repatriation


with the proper civil registry and with the Bureau of
Immigration a prerequisite in effecting repatriation?

The provision of law applicable in this case is Section 2 of Republic Act No. 8171, [14] thus:

SEC. 2. Repatriation shall be effected by taking the necessary oath of allegiance to the
Republic of the Philippines and registration in the proper civil registry and in the Bureau of
Immigration. The Bureau of Immigration shall thereupon cancel the pertinent alien certificate
of registration and issue the certificate of identification as Filipino citizen to the repatriated
citizen.

The law is clear that repatriation is effected by taking the oath of allegiance to the
Republic of the Philippines and registration in the proper civil registry and in the Bureau of
Immigration. Hence, in addition to taking the Oath of Allegiance to the Republic of the
Philippines, the registration of the Certificate of Repatriation in the proper civil registry and
the Bureau of Immigration is a prerequisite in effecting the repatriation of a citizen.
In this case, petitioner took his Oath of Allegiance on December 17, 1997, but his
Certificate of Repatriation was registered with the Civil Registry of Makati City only after six
years or on February 18, 2004, and with the Bureau of Immigration on March 1, 2004.
Petitioner, therefore, completed all the requirements of repatriation only after he filed his
certificate of candidacy for a mayoralty position, but before the elections.
When does the citizenship qualification of a candidate for an elective office apply?
In Frivaldo v. Commission on Elections,[15] the Court ruled that the citizenship
qualification must be construed as applying to the time of proclamation of the elected official
and at the start of his term. The Court, through Justice Artemio V. Panganiban, discussed,
thus:
Under Sec. 39 of the Local Government Code, (a)n elective local official must be:
* a citizen of the Philippines;
* a registered voter in the barangay, municipality, city, or province x x x where he
intends to be elected;
* a resident therein for at least one (1) year immediately preceding the day of the
election;
* able to read and write Filipino or any other local language or dialect.
* In addition, candidates for the position of governor x x x must be at least twenty-
three (23) years of age on election day.

From the above, it will be noted that the law does not specify any particular date or time
when the candidate must possess citizenship, unlike that for residence (which must consist
of at least one years residency immediately preceding the day of election) and age (at least
twenty three years of age on election day).

Philippine citizenship is an indispensable requirement for holding an elective public office, and
the purpose of the citizenship qualification is none other than to ensure that no alien, i.e., no
person owing allegiance to another nation, shall govern our people and our country or a unit
of territory thereof. Now, an official begins to govern or to discharge his functions only upon
his proclamation and on the day the law mandates his term of office to begin. Since Frivaldo
re-assumed his citizenship on June 30, 1995the very day the term of office of governor (and
other elective officials) beganhe was therefore already qualified to be proclaimed, to hold such
office and to discharge the functions and responsibilities thereof as of said date. In short, at
that time, he was already qualified to govern his native Sorsogon. This is the liberal
interpretation that should give spirit, life and meaning to our law on qualifications consistent
with the purpose for which such law was enacted. x x x Paraphrasing this Courts ruling
in Vasquez v. Giap and Li Seng Giap & Sons, if the purpose of the citizenship requirement is
to ensure that our people and country do not end up being governed by aliens, i.e., persons
owing allegiance to another nation, that aim or purpose would not be thwarted but instead
achieved by construing the citizenship qualification as applying to the time of
proclamation of the elected official and at the start of his term.[16](Emphasis supplied.)

Moreover, in the case of Frivaldo v. Commission on Elections, the Court ruled that the
repatriation of Frivaldo RETROACTED to the date of the filing of his application. In said case,
the repatriation of Frivaldo was by virtue of Presidential Decree No. 725, which took effect on
June 5, 1975. The Court therein declared that Presidential Decree No. 725 was a curative
statute, which is retroactive in nature. The retroactivity of Frivaldos repatriation to the date
of filing of his application was justified by the Court, thus:

xxx

The reason for this is simply that if, as in this case, it was the intent of the legislative authority
that the law should apply to past eventsi.e., situations and transactions existing even before
the law came into beingin order to benefit the greatest number of former Filipinos possible
thereby enabling them to enjoy and exercise the constitutionally guaranteed right of
citizenship, and such legislative intention is to be given the fullest effect and expression,
then there is all the more reason to have the law apply in a retroactive or retrospective
manner to situations, events and transactions subsequent to the passage of such law. That
is, the repatriation granted to Frivaldo x x x can and should be made to take effect as of date
of his application. As earlier mentioned, there is nothing in the law that would bar this or
would show a contrary intention on the part of the legislative authority; and there is no
showing that damage or prejudice to anyone, or anything unjust or injurious would result
from giving retroactivity to his repatriation. Neither has Lee shown that there will result the
impairment of any contractual obligation, disturbance of any vested right or breach of some
constitutional guaranty.
xxx

Another argument for retroactivity to the date of filing is that it would prevent prejudice to
applicants. If P.D. 725 were not to be given retroactive effect, and the Special Committee
decides not to act, i.e., to delay the processing of applications for any substantial length of
time, then the former Filipinos who may be stateless, as Frivaldohaving already renounced
his American citizenshipwas, may be prejudiced for causes outside their control. This should
not be. In case of doubt in the interpretation or application of laws, it is to be presumed that
the law-making body intended right and justice to prevail.[17]

Republic Act No. 8171[18] has impliedly repealed Presidential `Decree No. 725. They cover
the same subject matter: Providing for the repatriation of Filipino women who have lost their
Philippine citizenship by marriage to aliens and of natural-born Filipinos. The Courts ruling
in Frivaldo v. Commission on Elections that repatriation retroacts to the date of filing of ones
application for repatriation subsists for the same reasons quoted above.
Accordingly, petitioners repatriation retroacted to the date he filed his application in 1997.
Petitioner was, therefore, qualified to run for a mayoralty position in the government in the
May 10, 2004 elections. Apparently, the COMELEC was cognizant of this fact since it did not
implement the assailed Resolutions disqualifying petitioner to run as mayor of San Jacinto,
Masbate.

Second Issue: Whether or not the COMELEC en banc


gravely abused its discretion in affirming the
Resolution of the COMELEC, First Division?

The Court cannot fault the COMELEC en banc for affirming the decision of the COMELEC,
First Division, considering that petitioner failed to prove before the COMELEC that he had
complied with the requirements of repatriation. Petitioner submitted the necessary documents
proving compliance with the requirements of repatriation only during his motion for
reconsideration, when the COMELEC en banc could no longer consider said evidence. As the
COMELEC en banc correctly stated:

The Comelec Rules of Procedure provides that insufficiency of evidence to justify the decision
is a ground for a motion for reconsideration (Rule 19, Section 1). The evidence referred to
in the above provision and to be considered in the Motion for Reconsideration are those which
were submitted during the hearing and attached to the respective Memoranda of the parties
which are already part of the records of the case. In this regard, the evidence of the
respondent were not able to overcome the evidence of the petitioners.[19]

It is, therefore, incumbent upon candidates for an elective office, who are repatriated
citizens, to be ready with sufficient evidence of their repatriation in case their Filipino
citizenship is questioned to prevent a repetition of this case.
WHEREFORE, the petition seeking the nullification of the Resolution of the COMELEC en
banc of May 7, 2004, affirming the Resolution of its First Division dated March 22, 2004, is
hereby DENIED. No costs.
SO ORDERED.
Davide, Jr., C.J., Panganiban, Ynares-Santiago, Sandoval-Gutierrez, Carpio, Austria-
Martinez, Carpio-Morales, Callejo, Sr., Chico-Nazario, and Garcia, JJ., concur.
Puno, and Tinga, JJ., on official leave.
Quisumbing, J., in the result.
Corona, J., on leave.

You might also like